You are on page 1of 321

Engineering

Mathematics-II
Engineering
Mathematics-II

E. Rukmangadachari
Professor of Mathematics,
Department of Humanities and Sciences,
Malla Reddy Engineering College,
Secunderabad
Associate Acquisitions Editor: Sandhya Jayadev
Associate Production Editor: Jennifer Sargunar
Composition: MIKS Data Services, Chennai
Printer: Print Shop Pvt. Ltd., Chennai

Copyright © 2011 Dorling Kindersley (India) Pvt. Ltd

This book is sold subject to the condition that it shall not, by way of trade or otherwise, be lent, resold,
hired out, or otherwise circulated without the publisher’s prior written consent in any form of binding or
cover other than that in which it is published and without a similar condition including this condition being
imposed on the subsequent purchaser and without limiting the rights under copyright reserved above, no
part of this publication may be reproduced, stored in or introduced into a retrieval system, or transmitted in
any form or by any means (electronic, mechanical, photocopying, recording or otherwise), without the prior
written permission of both the copyright owner and the publisher of this book.

ISBN 978-81-317-5584-6

10 9 8 7 6 5 4 3 2 1

Published by Dorling Kindersley (India) Pvt. Ltd, licensees of Pearson Education in South Asia.

Head Office: 7th Floor, Knowledge Boulevard, A-8(A), Sector 62, Noida 201 309, UP, India.
Registered Office: 11 Community Centre, Panchsheel Park, New Delhi 110 017, India.
To
my beloved grandchildren,
Nikhil Vikas,
Abhijna Deepthi,
Dhruvanth
About the Author
E. Rukmangadachari is former head of Computer Science and Engineering
as well as Humanities and Sciences at Malla Reddy Engineering College,
Secunderabad. Earlier, he was a reader in Mathematics (PG course) at
Government College, Rajahmundry. He is an M.A. from Osmania University,
Hyderabad, and an M.Phil. and Ph.D. degree holder from Sri Venkateswara
University, Tirupathi.
A recipient of the Andhra Pradesh State Meritorious Teachers’ Award in
1981, Professor Rukmangadachari has published over 40 research papers in
national and international journals. With a rich repertoire of over 45 years’
experience in teaching mathematics to undergraduate, postgraduate and engi-
neering students, he is currently the vice-president of the Andhra Pradesh
Society for Mathematical Sciences. An ace planner with fine managerial
skills, he was the organising secretary for the conduct of the 17th Congress of
the Andhra Pradesh Society for Mathematical Sciences, Hyderabad.
Contents
About the Author vi 2 Eigenvalues and Eigenvectors 2-1
Preface xi
2.1 Introduction 2-1
2.2
1 Matrices and Linear Systems
Linear Transformation 2-1
2.3 Characteristic Value Problem 2-1
of Equations 1-1
Exercise 2.1 2-6
1.1 Introduction 1-1
2.4 Properties of Eigenvalues
1.2 Algebra of Matrices 1-3 and Eigenvectors 2-7
1.3 Matrix Multiplication 1-4 2.5 Cayley–Hamilton Theorem 2-9
1.4 Determinant of a Square Matrix 1-5 Exercise 2.2 2-12
1.5 Related Matrices 1-8 2.6 Reduction of a Square Matrix
1.6 Determinant-related Matrices 1-11 to Diagonal Form 2-14
1.7 Special Matrices 1-12 2.7 Powers of a Square Matrix A—
Finding of Modal Matrix P
Exercise 1.1 1-15 and Inverse Matrix A−1 2-18
1.8 Linear Systems of Equations 1-16 Exercise 2.3 2-23
1.9 Homogeneous (H) and
Nonhomogeneous (NH) Systems
of Equations 1-16 3 Real and Complex Matrices 3-1
1.10 Elementary Row and Column 3.1 Introduction 3-1
Operations (Transformations) 3.2 Orthogonal /Orthonormal System
for Matrices 1-17 of Vectors 3-1
Exercise 1.2 1-20 3.3 Real Matrices 3-1
1.11 Inversion of a Nonsingular Matrix 1-21 Exercise 3.1 3-6
Exercise 1.3 1-24 3.4 Complex Matrices 3-7
1.12 Rank of a Matrix 1-25 3.5 Properties of Hermitian,
1.13 Methods for Finding the Rank Skew-Hermitian and Unitary
of a Matrix 1-26 Matrices 3-8
Exercise 1.4 1-32 Exercise 3.2 3-14
1.14 Existence and Uniqueness
of Solutions of a System
of Linear Equations 1-33
4 Quadratic Forms 4-1
1.15 Methods of Solution of NH 4.1 Introduction 4-1
and H Equations 1-34 4.2 Quadratic Forms 4-1
1.16 Homogeneous System 4.3 Canonical Form (or) Sum
of Equations (H) 1-39 of the Squares Form 4-3
Exercise 1.5 1-40 4.4 Nature of Real Quadratic Forms 4-3
viii    Contents

4.5 Reduction of a Quadratic Form 6.3 Origin of Partial Differential


to Canonical Form 4-5 Equation 6-2
4.6 Sylvestor’s Law of Inertia 4-6 6.4 Formation of Partial
4.7 Methods of Reduction of a Differential Equation by
Quadratic Form to a Canonical Form 4-6 Elimination of Two Arbitrary
Constants 6-3
Exercise 4.1 4-9
Exercise 6.1 6-4
5 Fourier Series 5-1 6.5 Formation of Partial
Differential Equations by
5.1 Introduction 5-1 Elimination of Arbitrary
5.2 Periodic Functions, Properties 5-1 Functions 6-5
5.3 Classifiable Functions—Even Exercise 6.2 6-7
and Odd Functions 5-2 6.6 Classification of
5.4 Fourier Series, Fourier First-order Partial
Coefficients and Euler’s Differential Equations 6-7
Formulae in (a, a + 2p) 5-3
6.7 Classification of Solutions
5.5 Dirichlet’s Conditions for of First-order Partial
Fourier Series Expansion Differential Equations 6-8
of a Function 5-4
6.8 Equations Solvable by Direct
5.6 Fourier Series Expansions: Integration 6-9
Even/Odd Functions 5-5
Exercise 6.3 6-10
5.7 Simply-defined and
Multiply-(Piecewise) defined 6.9 Quasi-linear Equations
Functions 5-7 of First Order 6-11
Exercise 5.1 5-18 6.10 Solution of Linear,
5.8 Change of Interval: Fourier Semi-linear and
Series in Interval (a, a + 2l ) 5-19 Quasi-linear Equations 6-11
Exercise 5.2 5-23 Exercise 6.4 6-17
5.9 Fourier Series Expansions 6.11 Nonlinear Equations
of Even and Odd Functions of First Order 6-18
in (−l, l ) 5-24 Exercise 6.5 6-22
Exercise 5.3 5-26 6.12 Euler’s Method of Separation
5.10 Half-range Fourier Sine/ of Variables 6-22
Cosine Series: Odd and Even Periodic Exercise 6.6 6-25
Continuations 5-26
6.13 Classification of Second-
Exercise 5.4 5-33 order Partial Differential
5.11 Root Mean Square (RMS) Equations 6-25
Value of a Function 5-34 Exercise 6.7 6-33
Exercise 5.5 5-36 6.14 One-dimensional Wave
Equation 6-34
6 Partial Differential Equations 6-1 Exercise 6.8 6-42
6.1 Introduction 6-1 6.15 Laplace’s Equation or Potential
Equation or Two-dimensional
6.2 Order, Linearity and
Steady-state Heat Flow Equation 6-42
Homogeneity of a Partial
Differential Equation 6-1 Exercise 6.9 6-46
Contents    ix

7 Fourier Integral Transforms 7-1 9 Wavelets 9-1


7.1 Introduction 7-1 9.1 Introduction 9-1
7.2 Integral Transforms 7-1 9.2 Characteristic Function of an
7.3 Fourier Integral Theorem 7-1 Interval I 9-2
7.4 Fourier Integral in Complex 9.3 Vector Space of Functions
Form 7-2 with Finite Energy 9-2
7.5 Fourier Transform of f (x) 7-3 9.4 Norm of a Vector 9-3
7.6 Finite Fourier Sine Transform 9.5 Field 9-3
(FFST) and Finite Fourier 9.6 n-Vector Space 9-3
Cosine Transform (FFCT) 7-4 9.7 Scaling and Translation
7.7 Convolution Theorem Functions 9-3
for Fourier Transforms 7-5 9.8 Haar Scaling Function f(t) 9-4
7.8 Properties of Fourier 9.9 Scaling and Translation of f(t) 9-5
Transforms 7-6
9.10 Haar Wavelet Functions 9-5
Exercise 7.1 7-18
9.11 Scaling Factors of the Form 2m 9-7
7.9 Parseval’s Identity for Fourier
Transforms 7-19 9.12 A Wavelet Expansion 9-7
7.10 Parseval’s Identities for Fourier 9.13 Multiresolution Analysis with
Sine and Cosine Transforms 7-20 Haar Wavelets 9-8
Exercise 7.2 7-21 9.14 Subspaces of L2(R) 9-8
9.15 Closed subspace S 9-8
8 Z-Transforms and Solution 9.16 Generation of a Sequence of
of Difference Equations 8-1 Closed Subspaces of L2(R) by
Haar Wavelets 9-8
8.1 Introduction 8-1
9.17 General Construction of
8.2 Z-Transform: Definition 8-1
Wavelets and Multiresolution
8.3 Z-Transforms of Some Analysis 9-9
Standard Functions
9.18 Shannon Wavelets 9-10
(Special Sequences) 8-4
Exercise 9.1 9-11
8.4 Recurrence Formula
for the Sequence of a Power
of Natural Numbers 8-5 Question Bank A-1
8.5 Properties of Z-Transforms 8-6 Multiple Choice Questions A-1
Exercise 8.1 8-11 Fill in the Blanks A-23
8.6 Inverse Z-Transform 8-11 Match the Following A-35
Exercise 8.2 8-16 True or False Statements A-41
8.7 Application of Z-Transforms: Solved Question Papers A-45
Solution of a Difference
Equation by Z-Transform 8-17
Bibliography B-1
8.8 Method for Solving a Linear
Difference Equation with
Constant Coefficients 8-18 Index I-1
Exercise 8.3 8-21
Preface
I am pleased to present this book on Engineering Mathematics-II to the second-year B.Tech. students of
Jawaharlal Nehru Technological Universities (JNTU) at Hyderabad, Anantapur and Kakinada. Written in a
simple, lucid and easy-to-understand manner, the book conforms to the syllabus prescribed for JNTU.
The concepts have been discussed with a focus on clarity and coherence, supported by illustrations for
better comprehension. Over 240 well-chosen examples are worked out in the book to enable students understand
the fundamentals and the principles governing each topic.
The exercises given at the end of each chapter—more than 290 in all—with answers and hints wherever
necessary, provide students an insight into the methods of solving the problems with ingenuity. Model questions
from past University Examinations have been included in examples and exercises. A vast, answer-appended
Question Bank comprising Multiple Choice Questions, Fill in the Blanks, Match the Following and True or
False Statements serves to help the student in effortless recapitulation of the subject. In addition to helping
students to enhance their knowledge of the subject, these pedagogical elements also help them to prepare for
their mid-term examinations.
Suggestions for the improvement of the book are welcome and will be gratefully acknowledged.

Acknowledgements
I express my deep sense of gratitude to Sri Ch. Malla Reddy, Chairman, and Sri Ch. Mahender Reddy,
Secretary, Malla Reddy Group of Institutions (MRGI), whose patronage has given me an opportunity to write
this book.
I am also thankful to Prof. R. Madan Mohan, Director (Academics); Col G. Ram Reddy, Director
(Administration), MRGI; and Dr M. R. K. Murthy, Principal, Malla Reddy Engineering College, Secunderabad,
for their kindness, guidance, and encouragement.

E. RUKMANGADACHARI
Matrices and Linear
Systems of Equations 1
1.1 INTRODUCTION (called rows) and n vertical lines (called columns)
The concept of a matrix was introduced in 1850 by is called a matrix of order (m, n) or m × n (read as
the English mathematician James Joseph Sylvestor.1 m by n). The numbers or functions are called the
Two other English mathematicians namely William elements or entries of the matrix and are enclosed
Rowan Hamilton2 (1853) and Arthur Cayley3 (1858) within brackets [ ] or ( ) or || · ||.
used matrices in the solution of systems of equations. The matrix itself is called an m × n matrix. The
Elementary transformations were used by German rows of a matrix are counted from top to bottom and
mathematicians Hermann Grassmann4 (1862) and the columns are counted from left to right.
Leopold Kronecker5 (1866) in the solution of systems
of equations. The Theory of Matrices is important in ⎡2 1 0 ⎤
⎢1 0 −7⎥ is a matrix of order 2 × 3. In it [2 1 0]
engineering studies while dealing with systems of ⎣ ⎦
linear equations and in the study of linear transfor- is the first row or Row-1. [1 0 −7] is the second
mations and in the solution of eigenvalue problems. ⎡ 2⎤ ⎡ 1 ⎤ ⎡ 0 ⎤
row or Row-2 and ⎢ ⎥ , ⎢ ⎥ , ⎢ ⎥ are first column,
⎣1⎦ ⎣0⎦ ⎣ −7⎦
1.1.1 Matrix: Denition second column and third column, respectively.
A set of mn real or complex numbers or func- Capital letters A, B, C, …, P, Q, … are used to
tions displayed as an array of m horizontal lines denote matrices and small letters a, b, c, … to denote
elements. Letters i and j are used as suffxes on the
1
SYLVESTOR, James Joseph (1814–1897), English algebraist, letters a, b, c, … to denote the row position and
combinatorist, geometer, number theorist and poet; cofounder column position, respectively, of the corresponding
with Cayley of the theory of invariants (anticipated to some entry. Thus,
extent by Boole and Lagrange); spent two periods in the U.S.
where he was a stimulant to mathematical research. In 1850 he
introduced for the first time the word ‘matrix’, in the sense of ‘the col. 1 j th col.
mother of determinants’.
2
↓ ↓
HAMILTON, William Rowan (1805–1865), Great Irish ⎡ a11 a12 ! a1 j ! a1n ⎤ → Row 1
algebraist, astronomer and physicist.
3
⎢a a22 ! a2 j ! a2n ⎥
CAYLEY, Arthur (1821–1895), English algebraist, geometer ⎢ 21 ⎥
and analyst; contributed especially to theory of algebraic ⎢! ! ! ! ⎥
invariants and higher-dimensional geometry. A = [aij ] = ⎢ ⎥
4
GRASSMANN, Hermann Gunterr (1809–1877), Born in ⎢ ai1 ai 2 ! aij ! ain ⎥ → ith Row
Stettin, Prussia, now Szczecin in Poland, a mathematician chiefly ⎢! ! ! ! ⎥
remembered for the development of a general calculus for ⎢ ⎥
vectors. ⎢⎣ am1 am2 ! amj ! amn ⎥⎦
5
KRONECKER, Leopold (1823–1891), German algebraist, [1 ≤ i ≤ m]
algebraic number theorist and intuitionist, rejected irrational [1 ≤ j ≤ n]
numbers insisting that mathematical reasoning be based on the
integers and finite processes. is a matrix with m rows and n columns.
1-2    Engineering Mathematics-II

1.1.2 Types of Matrices ⎡ 5 −3⎤


⎡ 2 0 1⎤ ⎢ ⎥
1. Real Matrix E.g. ⎢ −1 3 5⎥ , ⎢ 0 1 ⎥
⎣ ⎦ ⎢
A matrix whose elements are all real numbers or ⎣12 4 ⎥⎦
functions is called a real matrix.
7. Square Matrix
⎡ −1 0 ⎤
⎡ e −1 2 p ⎤ ⎢ ⎥ ⎡e y⎤ A matrix in which the number of rows and the number
x
E.g. ⎢ ⎥ , ⎢ 2 −2⎥ , ⎢ ⎥ of columns are equal is called a square matrix.
⎣7 sin p / 3 0 ⎦ ⎢13 5 ⎥ ⎣ 0 −1⎦
⎣ ⎦
⎡ 0 5 3⎤
2. Complex Matrix ⎡1 −2⎤ ⎢ ⎥
E.g. ⎢0 5 ⎥ , ⎢ 7 6 4⎥
A matrix which contains at least one complex num- ⎣ ⎦ ⎢
⎣ −3 0 2⎥⎦
ber or function as an element is called a complex
matrix. A square matrix of order n × n is simply
x +i described as an n-square matrix.
⎡ −2 7 3 + i ⎤ ⎡1 −i ⎤ ⎡e y⎤
E.g. ⎢ p 13 8 ⎥ , ⎢0 2 ⎥ , ⎢ ⎥
⎣ ⎦ ⎣ ⎦ ⎣ 0 ix ⎦
Principal or Main Diagonal
In a square matrix [aij], the line of entries for which
3. Row Matrix or Row Vector i = j, i.e., a11, a22, a33, …, ann is called the principal
A matrix with only one row is called a row matrix or main diagonal of the matrix. In the square matrix
or row vector. It is a matrix of order 1 × n for some ⎡1 3 4⎤
positive integer n. ⎢ 0 0 6 ⎥ the line of elements [1 0 −7] is
⎢ ⎥
E.g. [−3 7 0 2 11], [7 4 8], ⎢⎣14 12 −7⎥⎦
[sin p /3 i] the principal or main diagonal of the matrix.

4. Column Matrix or Column Vector 8. Upper Triangular Matrix


A matrix with only one column is called a column A square matrix A = [aij] in which aij = 0 for i > j is
matrix or column vector. It is a matrix of order m × 1 called an upper triangular matrix.
for some positive integer m.
⎡ 2 −3 6⎤
⎡5⎤ ⎡0⎤ ⎢0 4 5⎥ , ⎡6 −2⎤
⎢12⎥ , ⎢21⎥ E.g. ⎢ ⎥ ⎢0 5 ⎥
E.g. ⎢ ⎥ ⎢ ⎥ ⎢⎣0 0 1⎥⎦ ⎣ ⎦
⎢⎣ 6 ⎥⎦ ⎢⎣16⎥⎦

5. Zero or Null Matrix 9. Lower Triangular Matrix


A matrix in which every entry is zero is called a zero A square matrix A = [aij]n×n in which aij = 0 for i < j
matrix or null matrix and is denoted by 0. is called a lower triangular matrix.
⎡0 0⎤ ⎡ −1 0 0 ⎤
E.g. 0 3× 2 = ⎢⎢0 0⎥⎥ , 01× 2 = [0 0] ⎢ ⎥ ⎡ −11 0⎤
E.g. ⎢ 3 4 0⎥ ⎢ 6 8⎥
⎢⎣0 0⎥⎦ ⎢ 2 5 3⎥ ⎣ ⎦
⎣ 2⎦
6. Rectangular Matrix
A matrix in which the number of rows and the 10. Triangular Matrix
number of columns may not be equal is called a A matrix which is either upper triangular or lower
rectangular matrix. triangular is called a triangular matrix.
Matrices and Linear Systems of Equations    1-3

11. Diagonal Matrix 13. Unit or Identity Matrix


A square matrix [aij] with aij = 0 for i ≠ j is called a A square matrix [dij] where dij is the Kronecker delta
diagonal matrix. is called a unit matrix or identity matrix.
⎡3 0 0 ⎤ ⎡1 0 0⎤
⎢0 1 0 ⎥ = diag[3 1 −2] ⎡1 0⎤ ⎢ ⎥
E.g. ⎢ ⎥ E.g. ⎢0 1⎥ , ⎢0 1 0⎥
⎣ ⎦ ⎢
⎢⎣0 0 −2⎥⎦ ⎣0 0 1⎥⎦
That is, a square matrix with all its off-diagonal are identity matrices of orders 2 and 3, respectively.
elements as zeros is called a diagonal matrix.
Note 1 An identity matrix is a scalar matrix with
Note 1 Some of the diagonal elements may be the scalar 1.
zeros.
⎡11 0 0⎤ ⎡ −10 0 0⎤ 1.2 ALGEBRA OF MATRICES
E.g. ⎢ 0 0 0⎥ , ⎢ 0 0 0⎥ 1. Equality of Matrices
⎢ ⎥ ⎢ ⎥
⎢⎣ 0 0 1⎥⎦ ⎢⎣ 0 0 0⎥⎦ Two matrices A and B are equal, denoted by A = B, if
(a) A and B are of the same type (i.e.,) A and B
Note 2 A diagonal matrix is both upper triangular are of the same order and
as well as lower triangular. (b) each entry of A is equal to the correspond-
Trace of a square matrix: The sum of the ing entry of B.
elements along the main diagonal of a square matrix Thus, if A = [aij]m×n, B = [bij]p×q then A = B if
A is called the trace of A and is written as tr A, i.e.,
n
(a) m = p, n = q and (b) aij = bij for all i, j.
tr A = a11 + a22 + ! + ann = ∑ aii
⎡a b ⎤ ⎡ −1 7⎤
i =1 E.g. 1. Let A = ⎢ ⎥ , B= ⎢ ⎥
⎣c d ⎦ ⎣ 3 4⎦
Properties of Trace of A a = −1 b = 7
(i) tr kA = k tr A (k scalar); (ii) tr (A + B) = A= B ⇔
c=3 d=4
tr A + tr B; (iii) tr AB = tr BA
⎡ p p⎤
Kronecker delta: Kronecker delta, denoted by ⎢ sin 2 cos ⎥
3 ⎡1 −1⎤
dij, is defined by 2. If A = ⎢ ⎥, B = ⎢ ⎥
⎢ tan p cos p ⎥ ⎣1 −1⎦
⎧0 if i ≠ j ⎣⎢ 4 ⎦⎥
δ ij = ⎨
⎩1 if i = j then A ≠ B since cos
p
≠ −1
3
12. Scalar Matrix
Note 1 The relation of inequality ‘<’ (less than) is
A square matrix [k dij] where k is a scalar (real or not defined among matrices.
complex number) and dij is the Kronecker delta is
called a scalar matrix.
2. Addition of Matrices
⎡ 3 0 0⎤ Let ᏹ denote the set of m × n matrices with real or
E.g. ⎢0 3 0⎥ , ⎡ −16 0 ⎤ complex entries.
⎢ ⎥ ⎢ 0 −16⎥
⎢⎣0 0 3⎥⎦ ⎣ ⎦ Two matrices in ᏹ are of the same type and
are said to be conformable with respect to matrix
Note 1 A scalar matrix is a diagonal matrix with addition. The sum of two matrices A = [aij]m×n and
the same element k along its main diagonal. B = [bij]m×n in ᏹ is the matrix [(aij + bij)]m×n obtained
1-4    Engineering Mathematics-II

by adding the corresponding entries of A and B and Two matrices A and B are said to be conform-
is denoted by A + B. able for matrix multiplication if the number of
⎡11 −2 7 ⎤ ⎡ −5 2 9⎤ columns of A is same as the number of rows of B.
E.g. A = ⎢ ⎥ B=⎢ ⎥ If A = [aij]m×n and B = [bij]n×p are two matrices
⎣ 5 3 −4⎦ 23 ⎣ −3 0 5⎦ 23 then the product AB of the matrices A and B, in this
⎡11 + ( −5) −2 + 2 7 + 9 ⎤ order, is the matrix C = [cij]m×p where cij is defined
A+ B = ⎢ ⎥ by (1.1).
⎣ 5 + ( −3) 3 + 0 −4 + 5⎦ 2× 3
⎡6 0 16⎤
=⎢ ⎥ 1.3.1 Properties
⎣2 3 1 ⎦ 2× 3
1. Matrix Multiplication is Associative
Negative of a matrix: Let B = [bij]m×n be a matrix If A, B and C are any matrices conformable for
in ᏹ. Then the negative of B, denoted by (−B) is the matrix multiplication, then
matrix [−bij], which is obtained by changing the sign
of each entry of B. A(BC) = (AB)C (1.2)
Subtraction of B from A: Let A, B, ∈ᏹ. If
A = [aij]m×n and B = [bij]m×n then (−B) = [−bij]m×n. 2. Matrix Multiplication Distributes
The matrix obtained by subtracting B from A is over Addition
defined by A − B = [(aij − bij)]m×n .
The ordered pair 〈 ᏹ, + 〉 where ᏹ is the set of If B and C are any matrices of the same type and A is
matrices and +, the addition of matrices forms an any matrix, which is conformable for multiplication
abelian group. by B and C then

A(B + C) = AB + AC (1.3)
3. Scalar Multiplication
Let A = [aij]m×n be a matrix. Then kA is a matrix of Proof
the same order as A and is defined by kA = [kaij] 1. Let A = [aik], B = [bkl], C = [clj] be any three
where k ∈ F (field of real or complex numbers) matrices of orders m × n, n × p and p × q,
kA is called a scalar multiple of A. Scalar multi- respectively then
plication of matrices obeys the following laws.
⎡ n ⎤
Associative law k (lA) = kl (A), 1· A = A AB = [uil ]m× p = ⎢ ∑ aik bkl ⎥
⎣ k =1 ⎦ m× p
Distributive law k (A + B) = kA + kB,
⎡ p ⎤
(k + l )A = kA + lA BC = [vkj ]n × q = ⎢ ∑ bkl clj ⎥
1 is the unity of F ; A, B ∈ᏹ ; k , l ∈ F ⎣⎢ l =1 ⎦⎥ n× q
⎡ n ⎤ ⎡ n ⎛ p ⎞⎤
1.3 MATRIX MULTIPLICATION ⇒ A( BC ) = ⎢ ∑ akl vkj ⎥ = ⎢ ∑ aik ⎜ ∑ bkl clj ⎟ ⎥
Let (ai1, ai2, …, ain) be a row matrix (or row vector) ⎣ k =1 ⎦ ⎢⎣ k =1 ⎝ l =1 ⎠ ⎥⎦
and (b1j, b2j, …, bnj)T a column matrix (or column ⎡ p ⎛ n ⎞ ⎤
vector). The inner product or dot product of these is = ⎢ ∑ ⎜ ∑ aik bkl ⎟ clj ⎥
⎣⎢ l =1 ⎝ k =1 ⎠ ⎦⎥
n p
cij = ∑ aik bkj = ∑ uil clj = ( AB)C
k =1 l =1
= ai1b1 j + ai 2 b2 j + ! + aik bkj + ! + ain bnj (1.1) ⇒ A( BC ) = ( AB)C (1.2)
Matrices and Linear Systems of Equations    1-5

Here the order of summation has been changed 1.4 DETERMINANT OF A SQUARE MATRIX
since they involve a finite number of terms. Determinants were originally introduced for solving
2. Let A = [aik], B = [bkl], C = [ckl] be any three systems of linear equations. More than their initial
matrices of orders m × n, n × p, n × p, use in this respect, the determinants have important
respectively. applications in differential equations, in eigenvalue
Then B + C = [bkl + ckl]. problems, vector algebra and other branches of
applied mathematics.
Left distributive law With each n-square matrix A = [aij ], we associate
⎡ n ⎤ a unique expression called ‘the determinant of
A( B + C ) = ⎢ ∑ aik (bkl + ckl )⎥ matrix A of order n’ denoted by det A or |A| or Δ as
⎣ k =1 ⎦ defined below:
⎡ n ⎤ ⎡ m ⎤
= ⎢ ∑ aik bkl ⎥ + ⎢ ∑ aik ckl ⎥ = AB + AC Note 1 The elements of a determinant are written
⎣ k =1 ⎦ ⎣ k =1 ⎦ (1.3) as in its matrix between two vertical bars while in the
case of a matrix they are enclosed between brackets
Right distributive law [] or ( ) or two pairs of vertical bars ||·||.
If A = [a11], a single element matrix, then
⎡ n ⎤
( B + C ) A = ⎢ ∑ (bik + cik )akl ⎥ det A = |A| = a11
⎣ k =1 ⎦
⎡ n ⎤ ⎡ m ⎤ ⎡ a11 a12 ⎤
= ⎢ ∑ bik akl ⎥ + ⎢ ∑ cik aki ⎥ = BA + CA If A = ⎢
a22 ⎥⎦
, a 2-square matrix, then
⎣ a21
⎣ k =1 ⎦ ⎣ k =1 ⎦ (1.4)
⎡ a11 a12 ⎤
det A = A = ⎢ = a11a22 − a21a12
If A is a matrix of order m × n then AIn = ⎣ a21 a22 ⎥⎦
ImA = A. If A is n square then AIn = In A = A.
Thus, the triple 〈 ᏹ, +, · 〉 where ᏹ is the nonempty The expansion of determinants of higher order
set of matrices of order m × n, + is the operation of is through minors or cofactors of an element of the
matrix addition and ‘.’ is the scalar multiplication of matrix. So we introduce the concepts of minor and
matrices forms a vector space over a field F, which cofactor.
may be the field of real or complex numbers. Minor Let A = [aij] be a square matrix of order
The triple 〈 V, +, · 〉 where V is a nonempty n. Then the minor of the element aij of A is the
set, + is addition operation on V and ‘.’ is multipli- determinant of order (n − 1) obtained from A by
cation with the set of scalars F satisfying the above deleting the row and column in which aij appears.
properties is called a vector space V over F denoted
by V(F). The elements of V are called vectors.
Example 1.1
In particular, an n-tuple of numbers is called
an n-vector and the set of n-tuples forms a vector ⎡ −3 0 7 11⎤
space. ⎢ 0 5 −4 6 ⎥
Let A = ⎢ ⎥
⎢ 3 2 −1 4 ⎥
3. Power of square matrix A ⎢ ⎥
⎣ 8 −3 0 2 ⎦
If A is a square matrix of order n and if p and q are
positive integers. The minor of element −4 in Row-2 and Column-3
−3 0 11
A1 = A is M 23 = 3 2 4
A = Ap · A
p+1

ApAq = Ap+q = AqAp (1.5) 8 −3 2


1-6    Engineering Mathematics-II

Example 1.2 Then det A = |A| = a11A11 + a12A12 + a13A13


⎡2 0 1⎤ expanding by Row-1 where
Let B = ⎢ −1 5 −1⎥ a22 a23
⎢ ⎥ A11 = = a22 a33 − a32 a23 ,
⎢⎣ 8 2 −2⎥⎦ a32 a33

The minor of element 8 in Row-3 and Column-1 a21 a23


A12 = − = −( a21a33 − a31a23 ),
0 1 a31 a33
is M 31 = 5 −1 = −5
a21 a22
A13 = = a21a32 − a31a22
a31 a32
Cofactor of an Element
Let A = [aij] be a square matrix of order n. Then the are the cofactors of a11, a12 and a13, respectively,
cofactor of the element aij is (−1)i+j times the minor in A so that det A = |A| = a11(a22a33 − a32 a23) −
of aij. a12(a21a33 − a31a23) + a13(a21a32 − a31a22).
That is, if Mij is the minor and Aij is the cofactor
of aij in A then Example 1.4
−3 0 7
Aij = (−1)i+j Mij (1.6)
Let det A = 4 3 6
Remark Although a square matrix is a square array 5 8 2
of elements (real or complex numbers or functions) a11 = −3 a12 = 0 a13 = 7
its determinant is a number or a function. a21 = 4 a22 = 3 a23 = 6
Example 1.3 a31 = 5 a32 = 8 a33 = 2
⎡ 2 −3⎤ 3 6 4 6
A= ⎢ ⎥; A11 = = −42, A12 = − = 22,
⎣4 5 ⎦ 8 2 5 2
2 −3 4 3 0 7
det A = A = = 2(5) − 4( −3) = 22 A13 = = 17, A21 = − = 56,
4 5 5 8 8 2
⎡ e x sin x ⎤ −3 7 −3 0
B=⎢ ⎥; A22 = = −41, A23 = − = 24,
−x 5 2 5 8
⎣⎢sin x e ⎥⎦
0 7 −3 7
ex sin x x −x A31 = = −21, A32 = − = 46,
det B = B = = e .e − sin x(sin x ) 3 6 4 6
−x
sin x e
−3 0
= 1 − sin 2 x = cos2 x A33 = = −9
4 3

1.4.1 Expansion of a Determinant Expanding the determinant by R1, R2 and R3 we


of Third Order obtain
Let A = [aij ], 1 ≤ i ≤ 3; 1 ≤ j ≤ 3 or det A = a11A11 + a12A12 + a13A13
⎡ a11 a12 a13 ⎤ = (−3)(−42) + 0.22 + 7.17 = 245
A = ⎢⎢ a21 a22 a23 ⎥⎥ = a21A21 + a22A22 + a23A23
= 4 × 56 + 3 × (−41) + 6 × 24 = 245
⎢⎣ a31 a32 a33 ⎥⎦
= a31A31 + a32A32 + a33A33
be a square matrix of third order. = 5 × (−21) + 8 × 46 + 2 × (−9) = 245
Matrices and Linear Systems of Equations    1-7

respectively. Similarly we can expand by any 4. If the elements of a row of a square matrix
column and get the same value for det A. We can are multiplied by a number k then the value
easily verify the fact that if a row of elements of A of its determinant is k times that of the
is multiplied by the cofactors of the corresponding original matrix.
elements of another row the result is always zero. From (3) and (4) it follows that if the
A similar result holds for columns also. In fact, from elements of a row of a square matrix are
the above example, we have k times the corresponding elements of
another row then the value of the determi-
det A = a11A21 + a12A22 + a13A23
nant of the matrix is zero.
= (−3) × 56 + 0(−41) + 7 × 24 = 0
5. The determinant of a square matrix A can
be expressed as the sum of the determinants
1.4.2 Expansion of the Determinant of two square matrices B and C such that
of a Matrix of any Order n one identified row of A is the sum of the
As explained above a determinant of order n is a corresponding rows of B and C while the
scalar associated with an n × n matrix A = [aij] which others remain the same.
is expressed as
Let A = [aij], B = [bij], C = [cij] be n-square
a11 a12 ! a1n matrices such that
a21 a22 ! a2n aij = bij = cij for all i ≠ r (r fixed)
D = det A = (1.7)
! ! ! ! = bij + cij for i = r
an1 an2 ! ann Then |A| = |B| + |C|

For n = 1 it is defined by D = a11 and for n ≥ 2 by 6. The value of the determinant of a square
matrix remains unaltered if a constant
D = ai1Ai1 + ai2Ai2 + … + ainAin (i = 1, 2, …, n) multiple of another row is added to one of
or D = a1j A1j + a2j A2j + … + anj Anj ( j = 1, 2, …, n) its rows.
(1.8)
Note 1 Ri → Ri + kRj indicates that the
where Aij = (−1)i+j Mij, Mij being a determinant of ith row of a matrix is replaced by the sum
order (n − 1). of the ith row and k times the jth row. The
Here, D is defined in terms of n determinants above property implies that the value of
of order (n − 1) each of which is defined in terms of the determinant of a square matrix remains
(n − 1) determinants of order (n − 2) and so on. unalterd under such an operation. k is any
scalar including zero.
1.4.3 Properties of Determinant 7. The sum of the products of the elements of
of a Matrix A a row of a square matrix and their corres-
1. For every matrix A, det A = det (AT ). ponding cofactors is equal to the determinant
(This implies that if any property holds for of the matrix.
rows it holds for columns of a determinant. Let A = [aij], 1 ≤ i, j ≤ n and Aij be the
AT is the matrix obtained from A by inter- cofactor of aij in A then
changing rows and columns.) ai1Ai1 + ai2Ai2 + … + ainAin = |A|
2. If any two rows of a square matrix are inter- (i = 1, 2, …, n)
changed then the sign of its determinant is 8. The sum of the products of the elements of
changed. a row of a square matrix and the cofactors
3. The value of the determinant of a square of the corresponding elements of another
matrix with identical rows is zero. row is zero.
1-8    Engineering Mathematics-II

ai1Ak1 + ai2Ak2 + … + ainAkn = 0, rows is called the transpose of A and is denoted by


i ≠ k (i = 1, 2, …, n) AT or A¢.
9. Let the elements of a square matrix A be If A = [aij]m×n is the given matrix then its
polynomials in x. If two rows become transpose A¢ or AT = [bij]n×m , where bij = aji .
identical when x = a then (x − a)| Example 1.5
det A or (x − a) is a factor of |A|; and if n
rows become identical then (x − a)n−1 |det A ⎡ 1 7⎤
⎡1 −1 3⎤
or (x − a)n−1 is a factor of |A|. If A = ⎢ ⎥ then AT = ⎢⎢ −1 0⎥⎥
⎣7 0 5⎦ 23 ⎢⎣ 3 5⎥⎦ 32
10. If A and B are n-square matrices then
det AB = det A · det B.
(The determinant of the product of Properties of Transposition of Matrices
matrices = The product of the determinants 1. If A is any matrix then (AT)T = A (the trans-
of matrices.) pose of the transpose of a matrix is the
matrix itself).
⎡ a11 a12 ⎤ ⎡ b11 b12 ⎤ 2. If A and B are two matrices of the same type
Let A= ⎢ ⎥ , B=⎢ ⎥
⎣ a21 a22 ⎦ ⎣b21 b22 ⎦ then (A + B)T = AT + BT (Transpose of sum =
then A = a11a22 − a21a12 , B = b11b22 − b21b12 sum of the transposes).
a11b11 + a12 b12 a11b21 + a12 b12 3. If A is any matrix and k is any scalar then
AB = (kA)T = kAT (The transpose of scalar times
a21b11 + a22 b12 a21b21 + a22 b22 a matrix = scalar times the transpose of the
a11b11 a11b21 a11b11 a12 b22 matrix).
= +
a21b11 a21b21 a21b11 a22 b22 4. If A and B are two matrices which are
a12 b12 a11b21 a12 b12 a12 b22 conformable for matrix multiplication then
+ + , (AB)T = BT AT (The transpose of the product
a22 b12 a21b21 a22 b12 a22 b22 of matrices = The product of the transposes
by property 5 in the reverse order).
a11 a11 a11 a12 5. If I is an identity matrix then IT = I (the
= b11b21 + b11b22 transpose of an identity matrix is itself).
a21 a21 a21 a22
a11 a12 a12 a12 Corollary Property (2) and (4) hold for any finite
−b12 b21 + b11b22 ,
a21 a22 a22 a22 number of matrices
by property 4 (A1 + A2 + … + An)T = A1T + A2T + … + AnT
b11 b12 a11 a12
= 0+ +0 (A1 · A2 … An−1An)T = AnT · ATn−1 … A2T · A1T
b21 b22 a21 a22
= B A= A B, by property 3 Note 1 The transpose of a diagonal matrix is
itself
The method can be used for higher-order matrices.
[diag(a11, a22, …, ann)]T = diag(a11, a22, …, ann)
1.5 RELATED MATRICES
Note 2 The transpose of a scalar matrix is itself
1.5.1 The Transpose of a Matrix: T
⎡k 0 0⎤ ⎡k 0 0⎤
Properties ⎢0 k 0⎥ = ⎢0 k 0⎥
The matrix obtained from a given matrix A by ⎢ ⎥ ⎢ ⎥
interchanging rows into columns and columns into ⎢⎣ 0 0 k ⎥⎦ ⎢⎣ 0 0 k ⎥⎦
Matrices and Linear Systems of Equations    1-9

Note 3 If A is any m × n matrix then AT the Theorem 1.1 For any square matrix A of order n
transpose of A is of order n × m. Now AAT and AT A we have A(Adj A) = (Adj A)A = |A|. (1.9)
are both defined and are square matrices of orders
m × m and n × n, respectively. Proof Let
⎡ a11 a12 .. a1 j .. a1n ⎤
Example 1.6 ⎢a a22 .. a2 j .. a2n ⎥
⎢ 21 ⎥
⎡ −1 2 ⎤ ⎢ .. .. .. .. .. .. ⎥
⎡ −1 0 7⎤
If A = ⎢ ⎥ then AT = ⎢⎢ 0 −4⎥⎥ A= ⎢ ⎥
⎣ 2 −4 3⎦ 23 ⎢ ai1 ai 2 .. aij .. ain ⎥
⎢⎣ 7 3 ⎥⎦ 32 ⎢ .. .. .. .. .. .. ⎥
⎢ ⎥
and ⎢⎣ an1 an2 .. anj .. ann ⎥⎦
⎡ 5 −8 −1 ⎤
⎡50 19 ⎤
AAT = ⎢ ⎥ ; AT A = ⎢⎢ −8 16 −12⎥⎥ If Aij is the cofactor of aij in |A| then
⎣19 29⎦ 22 ⎢⎣ −1 −12 58 ⎥⎦ 33 ⎡ A11 A21 .. Ak1 .. An1 ⎤
⎢A A22 .. Ak 2 .. An2 ⎥
1.5.2 Adjoint of a Square Matrix Adj A = ⎢ ⎥
12
⎢ .. .. .. .. .. .. ⎥
Let A = [aij] be an n-square matrix. Then the trans- ⎢ ⎥
pose of the cofactor matrix [Aij] where Aij is the ⎣ A1n A2n .. Akn .. Ann ⎦
cofactor of aij in A is called the adjoint of A and is
The ij element in the product matrix A(Adj A)
denoted by Adj A or adj A.
n
∑ aik Akj = ai1 Ak1 + ai 2 Ak 2 + !
T
⎡ A11 A12 .. A1n ⎤ + ain Akn
⎢A A22 .. A2n ⎥ j =1
Adj A = ⎢ ⎥
21
Thus, ⎧ A if i = k
⎢ .. .. .. .. ⎥ =⎨
⎢ ⎥
⎣ An1 An2 .. Ann ⎦ ⎩ 0 if i ≠ k
⎡ A11 A21 .. An1 ⎤ Thus, in the product A(Adj A) each diagonal element
⎢A A22 .. An2 ⎥ is |A| and each nondiagonal element is 0.
=⎢ ⎥
12
⎢ .. .. .. .. ⎥ ⎡A 0 .. .. 0⎤
⎢ ⎥ ⎢ ⎥
⎣ A1n A2n .. Ann ⎦
⎢0 A .. .. 0⎥
For a third-order matrix A the cofactors and the A (Adj A) = ⎢ 0 0 .. .. 0⎥
⎢ ⎥
adj A are given below: ⎢0 0 .. A 0⎥
⎢0 0 .. .. A ⎥⎦
⎡ a1 b1 c1 ⎤ ⎣
Let A = ⎢⎢ a2 b2 c2 ⎥⎥ ⎡1 0 .. 0⎤
⎢0 0⎥
⎢⎣ a3 b3 c3 ⎥⎦ 1 ..
= A⎢ ⎥= AI
n
⎢ .. .. .. ..⎥
and let A1, B1, C1 … be the cofactors of elements ⎢ ⎥
a1, b1, c1 in A. ⎣0 0 .. 1⎦

Now Example 1.7


T
⎡ A1 B1 C1 ⎤ ⎡ A1 A2 A3 ⎤ ⎡ a11 a12 a13 ⎤ ⎡ −1 4 0 ⎤
Adj A = ⎢⎢ A2 B2 C2 ⎥ = ⎢⎢ B1 B2
⎥ B3 ⎥⎥ Let A = ⎢⎢ a21 a22 a23 ⎥⎥ = ⎢⎢ 5 2 3 ⎥⎥
⎢⎣ A3 B3 C3 ⎥⎦ ⎢⎣C1 C2 C3 ⎥⎦ ⎢⎣ a31 a32 a33 ⎥⎦ ⎢⎣ 8 0 −5⎥⎦
1-10    Engineering Mathematics-II

Hence, we have verified Theorem 1.1: A(Adj A) = The inverse through the adjoint
(Adj A)A = |A|In
If A is invertible then A−1 = (Adj A) A where | A| ≠ 0.
1.5.3 Invertible Matrix Properties of the adjoint of a matrix A
A square matrix A is said to be invertible if there 1. A is invertible (nonsingular)
exists a matrix B such that
A( Adj A) = A
AB = BA = I
⇒ ⎡⎣ A−1 ⎤⎦ [ A( Adj A)]
B is called an inverse of A.
= Adj A = A−1 A = A A−1
Example 1.8
⎡ 1 2⎤
2. ( Adj A) −1 = ( A ( A−1 )) −1
Let A = ⎢ ⎥ , A is invertible because if we take 1 −1 −1
⎣ −1 3⎦ = (A ) = A A
A
1 ⎡3 −2⎤
B= ⎢ 3. (Adj I ) = I
5 ⎣1 1 ⎥⎦
4. (Adj 0) = 0
⎡ 1 2⎤ ⎡3 5 −2 5⎤ ⎡ 1 0⎤
then AB = ⎢ ⎥⎢ ⎥=⎢ ⎥ ⎡0 0 1⎤
⎣ −1 3⎦ ⎣1 5 1 5 ⎦ ⎣ 0 1⎦
5. A = ⎢⎢0 1 0⎥⎥ ⇒ A−1 = A
⎡3 5 −2 5⎤ ⎡ 1 2⎤
=⎢ ⎥⎢ ⎥ = BA ⎢⎣1 0 0⎥⎦
⎣1 5 1 5 ⎦ ⎣ −1 3⎦
Adj(k A) = kn−1(Adj A), where n is the order
The inverse of an invertible matrix A is unique and is of A.
denoted by A−1. Let B and C be inverses of A. Then
6. Let A be an invertible (nonsingular) matrix
C = CI = C(AB) = (CA)B = IB = B of order n and k be a nonzero scalar. Then
Matrices and Linear Systems of Equations    1-11

1 1 submatrix of A obtained by deleting R2, R3, C1, C2


( kA) −1 = Adj ( kA) = n k n −1 ( Adj A) ⎡0⎤
kA k A and C4 from A; and 021 = ⎢ ⎥ is a 2 × 1 submatrix
⎣0⎦
1 ⎛ Adj A ⎞ 1 −1
= = A of A obtained by deleting R2, C1, C2 and C4 from A.
k ⎜⎝ A ⎟⎠ k
1.6 DETERMINANT-RELATED MATRICES
⎛ Adj A ⎞ 1
7. Adj ( A−1 ) = Adj ⎜ = n −1 ( Adj A) 1.6.1 Singular Matrix
⎝ A ⎟⎠ A
A square matrix whose determinant vanishes is
8. Adj(AB) = (Adj B)(Adj A). This follows called a singular matrix.
from the following results.
Example 1.10
AB( Adj AB) = |AB| I = | A ||B| I
AB( Adj B · Adj A) = A(B Adj B)( Adj A) ⎡1 2 ⎤ 1 2
= A( |B|) I Adj A ⎢2 4⎥ is singular since A = = 1.4 − 22 = 0
⎣ ⎦ 2 4
= | B |(A Adj A)= | B || A | I
(1.10) ⎡1 −2 0 1⎤
⎢2 −1 1 0⎥
1.5.4 Submatrix of a Matrix Also, B = ⎢ ⎥ is singular (verify).
⎢3 −3 1 1⎥
Let A be an m × n matrix. If we retain any r rows ⎢ ⎥
⎣ −1 −1 −1 1⎦
and any s columns deleting (m − r) rows and (n − s)
columns from A we obtain a new matrix of order 1.6.2 Nonsingular Matrix
r × s (r ≤ m, s ≤ n) which we call a submatrix of A A square matrix whose determinant does not vanish
of order r × s. is called a nonsingular matrix.
Thus, a submatrix of matrix A is a matrix
obtained from A by deleting some rows and/or some Example 1.11
columns of A. ⎡1 3⎤
A= ⎢ is nonsingular since
The relation between a submatrix and a matrix
⎣2 4⎥⎦
may appear to be similar to that of a subset and a set.
But it is not. ⎡1 3⎤
A=⎢ = 1.4 − 2.3 = −2 ≠ 0
Though every matrix is a submatrix of itself, ⎣2 4⎥⎦
a null matrix need not be a submatrix of a given
⎡ −1 0 4⎤
matrix. Note that no zero matrix is a submatrix of
B = ⎢⎢ 5 2 3⎥⎥ is nonsingular (verify)
⎡1 −1⎤
A= ⎢ ⎥. ⎢⎣ 0 7 2⎥⎦
⎣2 3 ⎦
Example 1.9 Theorem 1.2 A is invertible ⇔ A is nonsingular.

⎡ 1 −1 0 7⎤ Proof Assume that A is invertible. Then AA−1 =


Let A = ⎢⎢ 4 3 2 8⎥⎥ A−1A = I.
Taking determinants of the two sides 1 = det I =
⎢⎣ −6 11 0 5⎥⎦ 34
det (AA−1) = det A det A−1 ⇒ det A ≠ 0.
⎡ 1 0 7⎤ This shows that A is nonsingular. Conversely
⎡ 1 −1⎤ ⎢ ⎥
Suppose A1 = ⎢ ⎥ and A2 = ⎢ 4 2 8⎥ . assume that det A ≠ 0
⎣4 3 ⎦ ⎢⎣ −6 0 5⎥⎦ Then A( Adj A) = ( Adj A) A = (det A) I
Then A1 is a 2 × 2 submatrix of A obtained by deleting ⎛ Adj A ⎞ ⎛ Adj A ⎞
⇒ A⎜ = A= I
R3, C3, C4 from A; and A2 is a 3 ×3 submatrix of A ⎝ det A ⎟⎠ ⎜⎝ det A ⎟⎠
obtained by deleting C2 from A. Also, [0] is a 1 × 1 ⇒ A is invertible and A−1 = Adj A/(det A)
1-12    Engineering Mathematics-II

1.6.3 Properties of Invertible In other words, a square matrix A is called idempotent


Matrices (Nonsingular if A2 = A
Matrices) Example 1.12
A nonsquare matrix has no inverse. Even among Trivial examples of idempotent matrices are the zero
square matrices only invertible (nonsingular) matrices and the unit matrices.
matrices, that is, matrices whose deteminants are
nonzero, have inverses. ⎡0 0 0⎤
⎡0 0⎤ ⎢
Further, AB = AC ⇒⁄ B = C. But if A is 02 = ⎢ ⎥ , 03 = ⎢ 0 0 0⎥⎥ ;
⎣ 0 0 ⎦
invertible (nonsingular) then ⎢⎣0 0 0⎥⎦
AB = AC ⇒ A−1(AB) = A−1(AC)
⎡1 0 0⎤
⇒ (A−1A)B = (A−1A)C ⎡1 0⎤ ⎢
I2 = ⎢ ⎥ , I 3 = ⎢0 1 0⎥⎥
⇒ IB = IC ⇒ B = C ⎣0 1⎦ ⎢⎣0 0 1⎥⎦
Properties
1. (A−1)−1 = A (the inverse of the inverse of a Example 1.13
matrix is the matrix itself). ⎡1 0 ⎤ ⎡1 0⎤ ⎡1 0⎤ ⎡1 0⎤
A= ⎢ ⎥; A2 = ⎢ ⎥⎢ ⎥=⎢ ⎥=A
2. (kA)−1 = k−1A−1 (k ≠ 0). ⎣0 0 ⎦ ⎣0 0⎦ ⎣0 0⎦ ⎣0 0⎦
−1 −1 −1
3. (AB) = B A (reversal law for the
inverses of the product). Example 1.14

4. (AT)−1 = (A−1)T. ⎡1 0 0⎤ ⎡1 0 0⎤
5. (A1A2 … Am)−1 = Am−1 A−1 −1 −1 B = ⎢0 1 0⎥ ; B = ⎢⎢0
⎢ ⎥ 2
1 0⎥⎥
m−1 … A2 A1 .
⎣⎢1 0 0⎥⎦ ⎣⎢1 0 0⎥⎦
Properties of the Product of Matrices
⎡1 0 0⎤ ⎡1 0 0⎤
Let ᏹ be the set of all n-square matrices and suppose
A, B, C … e ᏹ. Then the following laws hold for = ⎢⎢0 1 0⎥⎥ ⎢⎢0 1 0⎥⎥ = B
matrix multiplication. ⎢⎣1 0 0⎥⎦ ⎢⎣1 0 0⎥⎦
1. Closure law: AB e ᏹ for all A, B e ᏹ.
Example 1.15
2. Associative law: (AB)C = A(BC) for all
A, B, C e ᏹ. ⎡ 2 −2 −4⎤
C = ⎢⎢ −1 3 4 ⎥⎥ ;
3. Existence of identity: There exists identity
matrix I e ᏹ such that AI = IA = A for every ⎣⎢ 1 −2 −3⎥⎦
A e ᏹ. ⎡ 2 −2 −4⎤ ⎡ 2 −2 −4⎤
4. Existence of inverse: There exists A−1 C = ⎢⎢ −1 3 4 ⎥⎥ ⎢⎢ −1 3 4 ⎥⎥
2
e ᏹ such that AA−1 = A−1A = I for every ⎢⎣ 1 −2 −3⎥⎦ ⎢⎣ 1 −2 −3⎥⎦
invertible A e ᏹ.
⎡ 2 −2 −4⎤
The above laws show that the set of invertible (nonsin-
gular) matrices ᏹ form a nonabelian (noncommuta- = ⎢⎢ −1 3 4 ⎥⎥ = C
tive) group with respect to matrix multiplication. ⎢⎣ 1 −2 −3⎥⎦

1.7 SPECIAL MATRICES A is idempotent ⇒ AT is idempotent (A2 = A)


T 2 2 T T
⇒ (A ) = (A ) = A )
1.7.1 Idempotent Matrix
A square matrix which remains the same under A is idempotent and A is nonsingular
multiplication by itself is called an idempotent matrix. ⇒ A−1 is idempotent because (A−1)2 = (A2)−1 = A−1
Matrices and Linear Systems of Equations    1-13

1.7.2 Nilpotent Matrix Examples of Nilpotent Complex


A square matrix which vanishes when it is raised to Matrices
some positive integral power m is called a nilpotent Example 1.20
matrix. In other words, a square matrix A which is ⎡0 3 + i ⎤ 2 ⎡0 3 + i ⎤ ⎡0 3 + i ⎤
P=⎢ ⎥ ; P = ⎢0
such that Am = 0 for some m ∈ N is called a nilpotent
⎣ 0 0 ⎦ ⎣ 0 ⎥⎦ ⎢⎣0 0 ⎥⎦
matrix.
The least possible integer for which this holds ⎡0 0⎤
=⎢ ⎥ ; I (P) = 2
is called the index of the nilpotent matrix and is ⎣0 0⎦
denoted by I(A).
Example 1.21

Examples of Nilpotent Real Matrices ⎡ a ia ⎤ 2 ⎡ a ia ⎤ ⎡ a ia ⎤


Q=⎢ ⎥ ; Q = ⎢ia − a ⎥ ⎢ia − a ⎥
Example 1.16 ⎣ ia − a ⎦ ⎣ ⎦⎣ ⎦
⎡ 0 2⎤ ⎡0 2⎤ ⎡ 0 2⎤ ⎡ 0 0 ⎤
A= ⎢ A2 = ⎢ =⎢ ⎥ ; I (Q ) = 2
⎥; 0⎥⎦ ⎢⎣0 0⎥⎦ 0 0
⎣0 0⎦ ⎣0 ⎣ ⎦
⎡0 0⎤
=⎢ ; I ( A) = 2 1.7.3 Involutory Matrix
⎣0 0⎥⎦
A square matrix which is its own inverse is called an
Example 1.17
involutory matrix. In other words, a square matrix A
is involutory if A2 = I.
⎡1 1 3⎤ Unit matrices are trivial examples of involutory
B = ⎢⎢ 2 2 6 ⎥⎥ ; matrices. Some of the other 2-square real involutory
⎢⎣ −1 −1 −3⎥⎦ matrices are the following:
⎡ 1 1 3 ⎤⎡ 1 1 3 ⎤ Examples 1.22–1.29
B 2 = ⎢⎢ 2 2 6 ⎥⎥ ⎢⎢ 2 2 6 ⎥⎥ ⎡ −1 0 ⎤ ⎡ −1 0⎤ ⎡1 0 ⎤ ⎡0 1⎤
⎢ 0 −1⎥ ; ⎢ 0 ; ;
1⎥⎦ ⎢⎣ 0 −1⎥⎦ ⎢⎣ 1 0⎥⎦
;
⎢⎣ −1 −1 −3⎥⎦ ⎢⎣ −1 −1 −3⎥⎦ ⎣ ⎦ ⎣
⎡0 0 0⎤ ⎡ 0 1⎤ ⎡ 0 −1⎤ ⎡ 0 −1⎤ ⎡ 6 5 ⎤
⎢ −1 0⎥ ; ⎢ −1 ; ;
0 ⎥⎦ ⎢⎣ −1 0 ⎥⎦ ⎢⎣ −7 −6⎥⎦
= ⎢⎢0 0 0⎥⎥ ; I ( B ) = 2 ⎣ ⎦ ⎣
⎣⎢0 0 0⎦⎥ Example 1.30
⎡ 3 −4 4 ⎤
Example 1.18
Show that A = ⎢⎢ 0 −1 0 ⎥⎥ is an involutory
⎡a −a⎤ ⎡a −a⎤ ⎡a −a⎤
C=⎢ ⎥ ; C2 = ⎢ matrix. ⎢⎣ −2 2 −3⎥⎦
⎣a −a⎦ ⎣a − a ⎥⎦ ⎢⎣ a − a ⎥⎦
Solution A square matrix A is involutory if A2 = I
⎡0 0⎤
=⎢ ; I (C ) = 2
⎣0 0⎥⎦ ⎡3 −4 4 ⎤ ⎡ 3 −4 4 ⎤
A = ⎢⎢ 0
2
−1 0 ⎥⎥ ⎢⎢ 0 −1 0 ⎥⎥
Example 1.19 ⎢⎣ −2 2 −3⎥⎦ ⎢⎣ −2 2 −3⎥⎦
⎡ ab b2 ⎤ 2
⎡ ab b2 ⎤ ⎡ ab b2 ⎤
⎡1 0 0⎤
D=⎢ ⎥ ; D = ⎢ 2 ⎥⎢ 2 ⎥
= ⎢⎢0 1 0⎥⎥ = I ( Unit matrix )
2
⎣⎢ − a ab ⎦⎥ ⎢⎣ − a ab ⎦⎥ ⎣⎢ − a ab ⎦⎥
⎡0 0⎤ ⎢⎣0 0 1⎥⎦
=⎢ ⎥ ; I ( D) = 2
⎣0 0⎦ Hence A is involutory.
1-14    Engineering Mathematics-II

Example 1.31 If A is nonsingular and periodic with period n


⎡0 0 1⎤ then A−1 is periodic with period n.
Show that B = ⎢⎢0 1 0⎥⎥ is involutory.
Example 1.33
⎢⎣1 0 0⎥⎦
⎡ 0 1⎤
Show that A = ⎢ ⎥ is periodic with period 4.
Solution A square matrix A is involutory if A2 = I ⎣ −1 0⎦
⎡0 0 1⎤ ⎡0 0 1⎤ ⎡1 0 0⎤ Solution
Here B = ⎢⎢0 1 0⎥⎥ ⎢⎢0 1 0⎥⎥ = ⎢⎢0 1 0⎥⎥ = I
2
⎡ 0 1⎤ ⎡ 0 1⎤ ⎡ −1 0 ⎤
⎢⎣1 0 0⎥⎦ ⎢⎣1 0 0⎥⎦ ⎢⎣0 0 1⎥⎦ A2 = ⎢ ⎥⎢ ⎥=⎢ ⎥;
⎣ −1 0⎦ ⎣ −1 0⎦ ⎣ 0 −1⎦
Hence B is involutory. ⎡ −1 0 ⎤ ⎡ −1 0 ⎤
A4 = ⎢ ⎥⎢ ⎥ = I;
A is involutory ⇒ AT is involutory A2 = I ⇒ (AT)2 = ⎣ 0 −1⎦ ⎣ 0 −1⎦
(A2)T = IT = I.
A5 = A4 ⋅ A = IA = A
−1
A is involutory and nonsingular ⇒ A is
involutory (A2 = I ⇒ (A−1)2 = (A2)−1 = I −1 = I) Hence A is periodic and P(A) = 4.

Example 1.34
1.7.4 Periodic Matrix
Write A = U + L where U is upper triangular and L is
If A is a square matrix and is such that An+1 = A for
lower triangular matrix with zero diagonal elements
some positive integer n then A is called a periodic
matrix. ⎡2 0 −1⎤
The least positive integer p for which Ap+1 = A
if A = ⎢⎢ 3 1 2 ⎥⎥
holds is called the period of A and is denoted by P(A).
⎢⎣1 2 1 ⎥⎦
Note 1 A periodic matrix of period one is an
idempotent matrix. Solution The entries below the main diagonal are
put in L and the others in U.
Example 1.32
⎡ 2 0 −1⎤ ⎡0 0 0⎤
⎡ 1 1⎤
⎢ 2 − ⎥
2 ∴ U = ⎢0 1 2 ⎥ ; L = ⎢⎢ 3 0 0⎥⎥
⎢ ⎥
Show that A = ⎢ ⎥ is a periodic matrix of ⎢⎣0 0 1 ⎥⎦ ⎢⎣1 2 0⎥⎦
⎢− 1 1 ⎥
⎢⎣ 2 2 ⎥⎦
period one. Example 1.35

Solution ⎡u11 u12 u13 ⎤


⎡ 1 1⎤ ⎡ 1 1⎤ ⎡ 1 1⎤ Write A = LU where U = ⎢⎢ 0 u22 u23 ⎥⎥ and
⎢ 2 − ⎥ ⎢ − ⎥⎢ − ⎥
2 2 2 2 2 ⎢⎣ 0 0 u33 ⎥⎦
A= ⎢ ⎥ ⇒ A2 = ⎢ ⎥⎢ ⎥
⎢− 1 1 ⎥ ⎢− 1 1 ⎥ ⎢− 1 1 ⎥ ⎡ 1 0 0⎤ ⎡ −2 3 0 ⎤
⎣⎢ 2 2 ⎦⎥ ⎣⎢ 2 2 ⎦⎥ ⎣⎢ 2 2 ⎦⎥
L = ⎢⎢l21 1 0⎥⎥ if A = ⎢⎢ −4 7 6 ⎥⎥
⎡ 1 1⎤
⎢⎣ l31 l32 1⎥⎦ ⎢⎣ −6 13 29⎥⎦
⎢ 2 − 2⎥
=⎢ ⎥= A
⎢− 1 1 ⎥ Solution
⎢⎣ 2 2 ⎥⎦
⎡1 0 0⎤ ⎡u11 u12 u13 ⎤ ⎡ −2 3 0 ⎤
Properties ⎢l
⎢ 21 1 0⎥⎥ ⎢⎢ 0 u22 u23 ⎥⎥ = ⎢⎢ −4 7 6 ⎥⎥
If A is periodic with period n then AT is periodic with
⎢⎣ l31 l32 1⎥⎦ ⎢⎣ 0 0 u33 ⎥⎦ ⎢⎣ −6 13 29⎥⎦
period n.
Matrices and Linear Systems of Equations    1-15

u11 = −2, u12 = 3, u13 = 0 ⎡ 1 −2 1 ⎤


1
l21u11 = −4 ⇒ l21 = −4 (− 2) = 2 (b) Adj A = ⎢⎢ 2 −1 −5⎥⎥ ; A−1 =
15
l31u11 = −6 ⇒ l31 = (−6) (−2) = 3 ⎢⎣ −3 3 9 ⎥⎦
l21u12 + u22 = 7 ⇒ u22 = 7 − 6 = 1 1
A−1 = ( Adj A) A
l21u13 + u23 = 6 ⇒ u23 = 6 − 0 = 6; 15
l31u12 + l32u22 = 13 ⇒ l32 = 13 − 3 × 3 = 4 = ( −9 − 12) − 213 − 15 − 1(6 − 3) = 6 + = −3

l31u13 + l32u23 + u23 = 29 ⇒ u33 = 29 − 0 − 4.6 = 5


⎡ ab b2 ⎤
⎡ −2 3 0⎤ ⎡1 0 0 ⎤ 5. Show that ⎢ 2 ⎥ is nilpotent of index 2.
⎢⎣ b − ab ⎥⎦
U = ⎢ 0 1 6⎥ ; L = ⎢⎢2 1 0⎥⎥
⎢ ⎥
⎢⎣ 0 0 5⎥⎦ ⎢⎣ 3 4 1⎥⎦ ⎡ 1 0⎤
6. Write the submatrices of (a) A = ⎢ ⎥
⎡ 2 0 1⎤ ⎣ −2 3⎦
EXERCISE 1.1 (b) A = ⎢⎢ 1 5 7⎥⎥ which do not contain row 1 and
1. Find the matrices AB and BA if ⎢⎣ −1 2 0⎥⎦
⎡ −1 2⎤ ⎡5 1 ⎤ column 2 of A
(a) A = ⎢ ⎥ B = ⎢0 −2⎥
⎣ 7 0 ⎦ ⎣ ⎦ ⎡ 1 7⎤
⎡ 0 6 −1⎤ ⎡5 0 1⎤ Ans:(a) [2]; (b) ⎢ ⎥
⎣ −1 0⎦
(b) A = ⎢⎢ 5 2 1 ⎥⎥ B = ⎢⎢2 4 6⎥⎥ .
⎢⎣ 4 0 −1⎥⎦ ⎢⎣ 3 −3 0⎥⎦ ⎡ 2 3 4⎤
7. Find Adj A and A (if exists) when A = ⎢⎢ 4 3 1 ⎥⎥ .
−1

⎡ 9 27 36⎤ ⎢⎣ 1 2 4⎥⎦
⎡ −5 −5⎤ ⎢32 5 17 ⎥ ;
Ans: AB = (a) ⎢ ⎥ (b) ⎢ ⎥ Ans:
⎣ 35 7 ⎦
⎣⎢17 3 4 ⎥⎦ A = 2(10) − 3(15) + 4(5)⎤
⎥ ⎡ 10 −4 −9⎤
⎡ 4 30 −6⎤ = 20 − 45 + 20 ⎥ Adj A = ⎢ −15 4 14 ⎥ ;
⎡ 2 10⎤ ⎢ 44 20 −4⎥ ⎥ ⎢ ⎥
BA = (a) ⎢ ⎥ (b) = −5 ≠ 0
⎢ ⎥ ⎥ ⎢⎣ 5 −1 − 6 ⎥⎦
⎣ −14 0 ⎦ ⎢⎣ −15 12 −6⎥⎦ A−1exists ⎥⎦
⎡ 1 −1 2⎤ 1
A−1 = − Adj A.
2. If A = ⎢⎢ 3 1 7⎥⎥ is singular then x = ? 5
⎢⎣ x 1 0⎥⎦ ⎡ 3 −3 4⎤
⎢ ⎥
Ans: 0 = |A| = 1.(0.7) − (−1)(0 − 7x) + 2(3 − x) 8. If A = ⎢ 2 −3 4⎥ then prove that A3 = A−1.
1 ⎢⎣0 −1 1 ⎥⎦ [JNTU 2004 (4)]
= −9x − 1 ⇒ x = −
9 ⎡1 0⎤
9. Show that ⎢ ⎥ is an idempotent matrix.
3. For what value of x is the matrix AB singular if ⎣0 0⎦
⎡4 8⎤ ⎡ −1 2⎤ [Hint: A2 = A.]
A= ⎢ ⎥ and B = ⎢ 3 1⎥ ?
⎣ 2 x ⎦ ⎣ ⎦
⎡ 1 1⎤
Ans: x = 4 ⎢ − ⎥
10. Show that ⎢ 2 2 is periodic with period 1.

4. Find Adj A and A−1 if ⎢− 1 1 ⎥
⎡2 5 3⎤ ⎣⎢ 2 2 ⎦⎥
⎡1 −1⎤ ⎢ −1
(a) A = ⎢ ⎥ (b) A = ⎢ 2 1⎥⎥.
⎣1 1 ⎦ ⎢⎣ 1
⎡ −1 1 −1⎤
1 1⎥⎦
11. Show that ⎢⎢ 3 −3 3 ⎥⎥ is idempotent.
⎡1 −1⎤ −1 1 ⎡1 −1⎤ ⎢⎣ 5 −5 5 ⎥⎦
Ans: (a) A = ⎢ ⎥; A =
⎣1 1 ⎦ 2 ⎢⎣1 1 ⎥⎦
[Hint: A = A.]
2
1-16    Engineering Mathematics-II

⎡1 1 3⎤ decomposition, I.U. decomposition from Gauss’s


⎢ ⎥ elimination, tridiagonal system and rank method.
12. Show that ⎢ 5 2 6 ⎥ is nilpotent of index 3.
⎣⎢ −2 −1 −3⎦⎥
1.9 HOMOGENEOUS (H) AND
[Hint: A3 = 0.]
NONHOMOGENEOUS (NH) SYSTEMS
OF EQUATIONS
⎡ 4 −2⎤
13. Show that ⎢ ⎥ is nilpotent of index 2. A linear system of m equations in n unknowns
⎣ 8 −4⎦
x1, x2, …, xn is a set of equations of the type
[Hint: A2 = 0.]
a11 x1 + a12 x2 + ! + a1n xn = b1 ⎤
⎡ −1 0 ⎤ ⎡ 6 5 ⎤
14. Prove that ⎢ ⎥; ⎢ ⎥ are involutory a21 x1 + a22 x2 + ! + a2n xn = b2 ⎥⎥
⎣ 0 −1⎦ ⎣ −7 −6⎦
matrices. ! ! ! ⎥

am1 x1 + am2 x2 + ! + amn xn = bm ⎦
⎡ 2 −2 −4⎤
⎢ ⎥ m
15. Show that ⎢ −1 3 4 ⎥ is idempotent.
⎢⎣ 1 −2 −3⎥⎦
or ∑ aij x j = bi , (1 ≤ i ≤ m) (1.11)
j =1

⎡ a − b −( a + b)⎤ Here aij are given numbers called the coefficients


16. If a ≠ 0, b ≠ 0 then ⎢−a b ( a + b) ⎥⎥ is nilpotent of the linear system. The numbers bi (1 ≤ i ≤ m) on the

of index 2. ⎢⎣ a − b −( a + b)⎥⎦ RHS of (1.11) are also given numbers. If bi = 0 for
all i then the system (1.11) is called a homogeneous
[Hint: A2 = 0.]
system (H ) and if bi ≠ 0 for at least one i then (1.11)
⎡ −4 −3 −3⎤ is called a nonhomogeneous system (NH ).
17. Show that if A = ⎢⎢ 1 0 1 ⎥⎥ then (a) adj A = A; A set of numbers x1, x2, …, xn which simultane-
ously satisfies the system (1.11) is called a solution
⎣⎢ 4 4 3 ⎥⎦
(b) find A−1 and show that A is involutory.
set or a solution vector for the system (1.11). Also, if
such a set of numbers exists for a given system then
⎡ −1 −2 −2⎤ the system itself is said to be a consistent system;
18. If A = ⎢⎢ 2 1 −2⎥⎥ then show that adj A = 3A.
otherwise, it is called inconsistent.
A solution vector of (1.11) is a vector x =
⎢⎣ 2 −2 1 ⎥⎦
(x1, x2, …, xn) whose components x1, x2, …, xn satisfy
the system of equations (1.11). We notice the fact
1.8 LINEAR SYSTEMS OF EQUATIONS that the homogeneous system (H ) always has a
solution and is always consistent. If no other solution
1.8.1 Introduction
exists it has at least the solution x1 = 0, x2 = 0, … ,
The most important practical use of matrices is in xn = 0 which is called the trivial solution. We will
the solution of linear equations, which appear as study the conditions under which the system of
models in many engineering and other problems as, equations (1.11) has a solution in Section 1.13
for instance, in electrical networks, statistics, traffic below.
flows, growth of population, assignment of jobs
to workers, numerical methods for the solution of
differential equations and so on. 1.9.1 Matrix Form of the Linear
We study in this chapter the following methods System
of solution of linear system of equations: Matrix We see that the m equations of (1.13) can be put in
inversion, Cramer’s rule, Gauss’s elimination, LU matrix form AX = B
Matrices and Linear Systems of Equations    1-17

⎡ a11 a12 ! a1n ⎤ Elementary Matrices


⎢a a22 ! a2n ⎥ The matrix obtained from a unit matrix by the appli-
=⎢ ⎥ (1.12)
21
where A = [aij ]m× n cation of a single elementary row or column transfor-
⎢! ! ! ! ⎥
⎢ ⎥ mation is called an elementary matrix or E-matrix.
⎣ am1 am2 ! amn ⎦ By applying R12, R12(3) and R1(3) (or C12,
C12(3) and C1(3)) transformations on I2 we obtain
is the coeffcient matrix while X = [x1, x2, …, xn]Tn×1 E-matrices
and B = [b1, b2, …, bm]m×1 are the column vectors
of n unknowns xj (1 ≤ j ≤ n) and of given constants ⎡0 1⎤ ⎡1 3⎤ ⎡ 3 0⎤
⎢1 0⎥ ⎢0 1⎥ ⎢0 1⎥
bi(1 ≤ i ≤ m), respectively. Here A ≠ 0 and X has n ⎣ ⎦ ⎣ ⎦ ⎣ ⎦
components and B has m components. The matrix
which are denoted by E12, E12(3) and E1(3),
⎡ a11 a12 ! a1n b1 ⎤ respectively.
⎢ ⎥ Again, by the transformations R12(3), R1(3) and
a21 a22 ! a23 b2 ⎥
[ A | B] = ⎢ (1.13) R13 (or equivalently by C12(3), C1(3) and C13) on I3
⎢! ! ! ! ! ⎥
⎢ ⎥ we obtain the E-matrices
⎢⎣ am1 am2 ! amn bm ⎥⎦
⎡1 3 0⎤ ⎡ 3 0 0⎤ ⎡0 0 1⎤
obtained by appending the elements bi(1 ≤ i ≤ m) on ⎢0 1 0⎥ ⎢0 1 0⎥ ⎢0 1 0⎥
⎢ ⎥ ⎢ ⎥ ⎢ ⎥
the right side of A as the last column, is called the ⎢⎣0 0 1⎥⎦ ⎢⎣0 0 1⎥⎦ ⎢⎣1 0 0⎥⎦
augmented matrix of the system (1.10).
which are denoted by E12(3), E1(3) and E13,
respectively.
1.10 ELEMENTARY ROW AND COLUMN
OPERATIONS (TRANSFORMATIONS) Notation As shown above, we use the same
FOR MATRICES notations for E-matrices as for row or column
In solving linear system of equations, the following transformations of a matrix with the letter E in place
elementary operations (transformations) are applied. of R or C.
When the equations are written in matrix notation,
they correspond to the following elementary row Note 1 (a) |Eij| = −1; (b) |Ei(k)| = k ≠ 0;
operations on the augmented matrix. The notation (c) |Eij(k)| = 1.
we use in this context is given below. Every elementary matrix is nonsingular and so,
invertible.
1. Interchange of rows (Interchange of the
ith row and jth row) Ri ↔ Rj or Rij. Properties
2. Addition of a constant multiple of one 1. Let A and B be conformable for matrix
row to another row (Addition of k times multiplication. Then
the jth row to ith row) Ri → Ri + kRj , or (a) s (AB) = (sA)B and (b) s (AB) = A(sB)
Rij (k) or simply Ri + kRj. where s is a row or column transformation.
3. Multiplication of a row by a nonzero Example 1.36
constant k (Multiplication of the row Ri
by k) Ri → kRi or Ri(k) or simply kRi. ⎡ −1 2⎤ ⎡ 5 1 7⎤
Let A = ⎢ ⎥ ; B = ⎢ 0 3 2⎥ .
⎣ 3 4 ⎦ ⎣ ⎦
In the same way we may perform column
operations (or transformations) which are denoted ⎡ −5 5 −3⎤
Then AB = ⎢ ⎥
similarly with the letter ‘C ’ instead of ‘R ’. ⎣15 15 29 ⎦
1-18    Engineering Mathematics-II

Applying R12 on A we get Over-determined and Under-


determined Linear Systems
⎡ 3 4⎤
s ( A) = A12 = ⎢ ⎥, A system is called over-determined if it has more
⎣ −1 2⎦ equations than unknowns (m > n); determined if
the number of equations is equal to the number
⎡15 15 29 ⎤ of unknowns (m = n) and under-determined if the
s ( A) B = A12 B = ⎢ ⎥ = s ( AB)
⎣ −5 5 −3⎦ number of equations is less than unknowns (m < n).

Similarly we can verify for a column transformation.


1.10.1 Equivalence of Matrices
2. Multiplication by E-matrices A linear system S2 is called row-equivalent to a
Elementary row/column transformations on a matrix linear system S1 if S2 can be obtained from S1 by
can be effected by pre-/post-multiplication, respec- finite sequence of elementary row operations.
tively, by the corresponding E-matrices. A similar definition can be given for column
This property is of theoretical use but for equivalence of matrices.
problems the usual row-/column-transformations Thus, if a matrix Q is obtained from a given
are preferable. matrix P by a finite chain of elementary transforma-
tions then P is said to be equivalent to Q and we
3. Inverse E-matrices denote it by P ∼ Q.
Two equivalent matrices are of the same order
(a) Eij−1 = Eij ; and same rank.
⎛ 1⎞ −1 We observe that a system of equations may
(b) [ Eij ( k )]−1 = Ei ⎜ ⎟ ; ⎣⎡ Eij ( k )⎦⎤ = Eij ( − k ). have no solution at all, a unique solution or infinitely
⎝ k⎠
many solutions. To find the solution to the question
Remark The above-mentioned elementary trans- of existence and uniqueness of solutions of a linear
formations or equivalently multiplications of a system of equations we may have to introduce the
matrix by elementary matrices (E-matrices) do key concept of rank of a matrix. But now we need
not alter the order or the rank of a matrix, which is the following concepts.
defined below. While the value of a minor may get
affected by transformations 1 and 2 their vanishing 1.10.2 Vectors: Linear Dependence
or nonvanishing character remains unaffected. The and Independence
elementary transformations help us in simplifying
Ordered Set of Numbers as Vectors
the method for finding the inverse of a matrix
or finding the rank of a matrix, which help in the Vector: An ordered n-tuple
solution of systems of linear equations. (ai1, ai2, …, ain)
The above operations (transformations) have
useful applications in of n numbers is called an n-vector or simply a
vector.
1. deciding the question of existence of E.g. An ordered pair (1, −2) is a two-dimensional
solutions of a system of equations; vector.
2. solving a system of linear equations; An ordered triple (−3, 0, 4) is a three-dimensional
3. determining the rank of a matrix; vector.
The numbers are called the components of the
4. finding the inverse of an invertible matrix;
vector. If the numbers are written in a horizontal
5. determining linear dependence (L.D.) or line it is called a row–vector and if they are written
linear independence (L.I.) of a given set of vertically it is called a column–vector; they are also
vectors. called row matrix and column matrix, respectively,
Matrices and Linear Systems of Equations    1-19

since a vector can be taken as a special case of a


⎡ v1 ⎤ ⎡3 6 2 ⎤
matrix. A column vector
A = ⎢⎢ v2 ⎥⎥ = ⎢⎢1 7 4 ⎥⎥ and
⎡3⎤ ⎢⎣ v3 ⎥⎦ ⎢⎣3 −9 −8⎥⎦
⎢ −1⎥
⎢ ⎥ ⎡ v1 ⎤ ⎡3 6 2⎤
⎢⎣ 2 ⎥⎦ ⎢ ⎥ = ⎢1
B=⎢ v2 ⎥ ⎢ 7 4⎥⎥
can be written as the transpose of row vector namely, ⎢⎣2v1 − 3v2 − v3 ⎥⎦ ⎣⎢0 0 0 ⎥⎦
[3 −1 2]T and vice versa.
are equivalent and since the rank of B (number of
Linear dependence of vectors: A set of vectors independent rows) is 2 the rank of A is also 2.
vi | (i = 1, 2, …, n), is said to be linearly dependent If a given matrix A has r linearly independent
(L.D.) if there exist scalars l1, l2, …, ln, not all vectors (rows/columns) and the remaining vectors
zero, such that are linear combination of these r vectors then the
rank of A is r. Conversely, if a matrix A is of rank r
λ1v1 + λ 2 v2 + ! + λ n vn = 0 (1.14)
it contains r linearly independent vectors, and the
A set of vectors vi | (i = 1, 2, …, n) is linearly remaining vectors, if any, can be expressed as a
independent (L.I.) if it is not linearly dependent. In linear combination of these vectors.
such a case every relation of the form (1.14) implies
⎡1 0 −1 2 ⎤
l1 = l2 = … = ln = 0.
A = ⎢⎢2 4 0 12 ⎥⎥
⎢⎣ 3 −4 −5 −2⎥⎦
1.10.3 Rank of a Matrix: Denition 1
The maximum number of L.I. row vectors of a we can easily check that 5v1 = v2 + v3.
matrix A = [aij] is called the rank of A and is denoted So, v1 and v2 are L.I. while all the three row
by r(A) or r (A). vectors are L.D.
Hence r(A) = no. of L.I. row vectors = 2
Example 1.37
Test the vectors v1 = (3, 6, 2), v2 = (1, 7, 4), v3 = Note 1 It follows from the definition that r(A) = 0
(3, −9, −8) for linear dependence. ⇔ A = 0.

Solution The relation l1v1 + l2v2 + l3v3 = 0


Theorem 1.3 The rank of a matrix A equals the
implies that maximum number of L.I. column vectors of A.
Hence A and its transpose AT have the same rank.
l1(3, 6, 2) + l2(1, 7, 4) + l3(3, −9, −8) = (0, 0, 0)
1.10.4 Methods for Determining
This is equivalent to the system of equations.
Linear Dependence (L.D.)
3l1 + l2 + 3l3 = 0, 6l1 + 7l2 − 9l3 = 0, and Linear Independence
2l1 + 4l2 − 8l3 = 0 (L.I.) of Vectors
These are satisfied by the values l1 = 2, l2 = −3, Consider m vectors each with n components. Add suit-
l3 = −1. able constant multiples of one vector to all other (m − 1)
So, the vectors v1, v2 and v3 are linearly vectors so that we obtain (m − 1) vectors with zero
dependent. first components. Repeat this process with the (m − 1)
Also, we have the relation 2v1 − 3v2 − v3 = 0 vectors and obtain (m − 2) vectors with zero first and
which shows that any of the vectors can be expressed second components. Proceeding in this way after n
as a linear combination of the others. steps (if m > n) we arrive at (m − n) vectors with the n
Applying elementary row operations to the zero components, i.e., with their sum equal to the zero
vectors v1, v2, v3 we see that the matrices. vector and hence the given vectors are L.D.
1-20    Engineering Mathematics-II

Otherwise, the vectors are L.I. If m < n then after ⎡ 1 −1⎤ R2 − R1 ⎡1 −1⎤
m steps if we arrive at zero vectors on both sides the Solution ⎢ −1 1 ⎥ ! ⎢0 2 ⎥ is nonsingular
vectors are L.D. If there are nonzero components in the ⎣ ⎦ ⎣ ⎦
vector on the RHS then the system of vectors is L.I. and hence the vectors are L.I.
Example 1.43
Example 1.38
Show that a = (1 1 1 3), b = (1 2 3 4), Show that the vectors (1, −1, 0), (2, 3, 1) and (3, 2,
c = (2 3 4 8) are L.I. 1) are L.D.
Solution
Solution Vectors b − a = (0, 1, 2, 1) and
c − 2a = (0, 1, 2, 2) have zero first component. ⎡1 −1 0⎤ R2 − 2 R1 ⎡1 −1 0⎤ ⎡1 −1 0⎤
Now, subtracting the first vector from the ⎢2 3 1⎥ ! ⎢0 5 1⎥ R3 − R2 ⎢0 5 1⎥
⎢ ⎥ ⎢ ⎥ ! ⎢ ⎥
second c − b − a = (0, 0, 0,1) ≠ (0, 0, 0, 0) . ⎣⎢ 3 2 1⎥⎦ R3 − 3R1 ⎣⎢0 5 1⎥⎦ ⎣⎢0 0 0⎥⎦
So, the given set of vectors is L.I.
is singular and hence the vectors are L.D.
Example 1.39 Note 1 In the above examples we have applied
Show that a = (1 1 1 3), b = (1 2 3 4), elementary operations (transformations) Ri → Ri +
c = (2 3 4 7) are L.D. kRj (addition of k times Rj to Ri).
R
A " B means that the matrices A and B are row
Solution b − a = (0, 1, 2, 1), c − 2a = (0, 1,
equivalent.
2, 1), b − c + a = 0; a , b , c are L.D.
EXERCISE 1.2
Example 1.40
1. Solve the system of equations by Gauss’s elimination
Show that a = (1, − 2, 6), b = (3, 2, 7), c = (2, 4, 1) method:
are linearly dependent. x + y + z = 1, 2x − y + 3z = 6, 3x + 2y + 2z = 3
Solution b − 3a = (0, 8, − 11); c − 2a = (0, 8, Ans: x = 1, y = − 1, z = 1
–11). Now, subtracting the first vector from the
2. Solve the linear nonhomogeneous system of equa-
second
tions by Gauss’s elimination method:
c − b + a = (0, 0, 0) = 0 x + z = 3, 2x + y − z = 0, x − 3y + 2z = 5
So, the given set of vectors is linearly Ans: x = 1, y = 0, z = 2
dependent.
3. Show that the system in Problem 1 is equivalent to
Remarks If m = n (the number of vectors = the the system
number of components in each vector) then the set x + y = 0, y + z = 0 and z + x =2
of vectors is linearly dependent (L.D.) or linearly [Hint: Show that the solutions are same.]
independent (L.I.) according as the matrix of their
components is singular or nonsingular. 4. Show that the system in Problem 2 is equivalent to
the system
Example 1.41 x − y = 1, y − z = −2 and z − x =1
Show that the vectors (1, −1)(−1, 1) are L.D. [Hint: Show that the solutions are same.]
5. Show that the matrices
⎡ 1 −1⎤ R2 + R1 ⎡1 −1⎤
Solution ⎢ −1 1 ⎥ ⎢0 0 ⎥ ⎡1 1 2 3 ⎤ ⎡1 1 2 3⎤
! ⎢0 1 2 2 ⎥ ⎢0
⎣ ⎦ ⎣ ⎦ 1 2 2⎥⎥
is singular and hence the vectors are L.D. A= ⎢ ⎥ and B=⎢
⎢ 3 4 8 11⎥ ⎢0 0 0 0⎥
⎢ ⎥ ⎢ ⎥
Example 1.42 ⎣1 3 6 7 ⎦ ⎣0 0 0 0⎦
Show that the vectors (1, −1) and (1, 1) are L.I. are row-equivalent.
Matrices and Linear Systems of Equations    1-21

6. Show that the vectors v1 = (3, 2, 7), v2 = (2, 4, 1) and 1 1 1 1


v3 = (1, −2, 6) are linearly dependent. A31 = = 2; A32 = − = 1;
−1 1 2 1
[Hint: Scalars k, l exist such that kv1 + lv2 = v3;
1 1
k = 1, l = −1.] A33 = = −3;
2 −1
7. Show that the vectors v1 = (1, 1, 1, 5), v2 = (1, 2, 3, 4)
and v3 = (2, 3, 4, 9) are linearly dependent. A = a11 A11 + a12 A12 + a13 A13
[Hint: scalars k, l exist such that kv1 + lv2 = v3; k =1,
= 1⋅ 5 + 1⋅ 7 + 1⋅ ( −3) = 9 ≠ 0 ⇒ A−1 exists
l =1.]
⎡ A11 A21 A31 ⎤
8. Show that the vectors v1 = (1, −1, 0), v2 = (1, 1, −1) Adj A 1 ⎢
A −1
= = ⎢ A12 A22 A32 ⎥⎥
and v3 = (2, 0, 1) are linearly independent. A 9
⎢⎣ A13 A23 A33 ⎥⎦
[Hint: k1v1 + k2v2 + k3v3 = 0 ⇒ k1 = k2 = k3 = 0.]
⎡5 1 2⎤
= ⎢⎢ 7 −4 1 ⎥⎥
1
1.11 INVERSION OF A NONSINGULAR 9
⎢⎣ −3 3 −3⎥⎦
MATRIX
We now consider the methods for finding the inverse
of an invertible matrix. Example 1.45
⎡1 1 3⎤

Find the inverse of 1 3 −3⎥⎥. [Andhra 1998]
1.11.1 Method 1: Adjoint Method ⎢
(or Determinants Method) ⎢⎣ −2 −4 −4⎥⎦
Example 1.44 Solution Let
Compute the adjoint and inverse of the matrix ⎡ a1 b1 c1 ⎤ ⎡ 1 1 3⎤
⎡1 1 1 ⎤ A = ⎢⎢ a2 b2 ⎥ ⎢
c2 ⎥ = ⎢ 1 3 −3⎥⎥
A = ⎢⎢2 −1 1 ⎥⎥ ⎢⎣ a3 b3 c3 ⎥⎦ ⎢⎣ −2 −4 −4⎥⎦
⎢⎣1 −2 −3⎥⎦
If Ai, Bi, Ci (i = 1, 2, 3) are the cofactors of
Solution Let ai, bi, ci (i = 1, 2, 3), respectively, then

⎡ a11 a12 a13 ⎤ ⎡1 1 1 ⎤ 3 −3 1 3


A1 = + = −24; A2 = − = −8;
A = ⎢⎢ a21 a22 a23 ⎥⎥ = ⎢⎢2 −1 1 ⎥⎥ −4 −4 −4 −4
⎢⎣ a31 a32 a33 ⎥⎦ ⎢⎣1 −2 −3⎥⎦ 1 3 1 −3
A3 = + = −12; B1 = − = 10;
3 −3 −2 −4
If Aij denotes the cofactor of entry aij in the
matrix A then 1 3 1 3
B2 = + = 2; B3 = − = 6;
−2 −4 1 −3
−1 1 2 1
A11 = = 5; A12 = − = 7; 1 3 1 1
−2 −3 1 −3 C1 = + = 2; C2 = − = 2;
−2 −4 −2 −4
2 −1 1 1
A13 = = −3; A21 = − = 1; 1 1
1 −2 −2 −3 C3 = + = 2;
1 3
1 1 1 1 Δ = det A = a1 A1 + a2 A2 + a3 A3
A22 = = −4; A23 = − = 3;
1 −3 1 −2 = 1( −24) + 1( −8) + ( −2)( −12) = −8 ≠ 0
1-22    Engineering Mathematics-II

A1 A2 A3 ⎡ −24 −8 −12⎤ ⎡1 0 0⎤ ⎡ a1 b1 c1 ⎤
adj A = B1 B2 B3 = ⎢⎢ 10 2 6 ⎥⎥ E1 ⋅ A = ⎢⎢0 0 1⎥⎥ ⎢⎢ a2 b2 c2 ⎥⎥
C1 C2 C3 ⎢⎣ 2 2 2 ⎥⎦ ⎢⎣0 1 0⎥⎦ ⎢⎣ a3 b3 c3 ⎥⎦
⎡ a1 b1 c1 ⎤
= ⎢⎢ a3 b3 c3 ⎥⎥
Then the inverse of the given matrix A is

⎢⎣ a2 b2 c2 ⎥⎦
⎡ −24 −8 −12⎤
1⎢
6 ⎥⎥
−1 adj A
A = = − ⎢ 10 2 So, pre-multiplication by E1 has interchanged
det A 8
⎢⎣ 2 2 2 ⎥⎦ the second and third rows of A. Similarly pre-
multiplication by E2 will multiply the second row
⎡ 3 ⎤ of A by k and pre-multiplication by E3 will result in the
⎢ 3 1
2 ⎥
⎢ ⎥ addition of p times the second row of A to its first row.
5 1 3⎥
= ⎢− − −
⎢ 4 4⎥
⎢ 1
4 1.11.3 Method 2: Gauss–Jordan6–7
⎢− 1 1⎥ Method of Finding the
− − ⎥
⎢⎣ 4 4 4 ⎥⎦ Inverse of a Matrix
Those elementary row transformations which reduce
We have already defined elementary matrices.
a given square matrix A to the unit matrix when
We consider now their properties and then the Gauss–
applied to the unit matrix I give the inverse of A.
Jordan method of finding the inverse of a matrix.
Let the successive row transformations which
reduce A to I result from pre-multiplication by the
1.11.2 Elementary Matrices elementary matrices R1, R2, …, Rm so that
An elementary matrix is that which is obtained from RmRm−1 … R2R1A = I
a unit matrix by subjecting it to any one of the ele-
mentary transformations. RmRm−1 … R2R1AA−1 = IA−1,
Examples of elementary matrices obtained post-multiplying by A−1
from I3 are RmRm−1 … R2R1I = A−1  AA−1 = I
⎡1 0 0⎤ ⎡ 1 0 0⎤ Hence the result.
by R23,
E1 = ⎢⎢0 0 1⎥⎥ E2 = ⎢⎢ 0 k 0⎥⎥ by kR2 ; Let A be a given n-square matrix. Suppose
or C23 ; |A| ≠ 0. Then A−1 exists. The method of Gauss−Jordan
⎣⎢0 1 0⎥⎦ ⎣⎢ 0 0 1⎥⎦
for inverting A consists in writing the nth order unit
⎡1 p 0⎤ matrix In alongside A and then applying row trans-
E3 = ⎢⎢ 0 1 0⎥⎥ R1 + pR2 formations on both A and I until A gets transformed
to In so that in the place of In we will have A−1.
⎢⎣ 0 0 1⎥⎦

Elementary row (column) transformations of a 6


Named after the great German mathematician Carl Friedrick
matrix A can be obtained by pre-multiplying (post- Gauss (1777–1855) who made his first great discovery as a student
multiplying) A by the corresponding elementary at Gottingen. His important contributions are to algebra, number
matrices. theory, mechanics, complex analysis, differential equations,
differential geometry, noneuclidean geometry, numerical
⎡ a1 b1 c1 ⎤ analysis, astronomy and electromagnetism. He became director
If A = ⎢⎢ a2 b2 c2 ⎥⎥ then of the observatory of Gottingen in 1807.
7
Named after another German mathematician and geodesist
⎢⎣ a3 b3 c3 ⎥⎦ Wiehelm Jordan (1842–1899).
Matrices and Linear Systems of Equations    1-23

Example 1.46 Example 1.47


⎡1 1 1 ⎤ Use Gauss–Jordan method to find the inverse of
Find the inverse of A = ⎢⎢2 −1 1 ⎥⎥ by Gauss– ⎡8 4 3⎤
⎢⎣1 −2 −3⎥⎦ B = ⎢⎢2 1 1⎥⎥ [Mangalore 1997]
Jordan method. ⎢⎣1 2 1⎥⎦
Solution We write
Solution Writing the matrix and the unit matrix
⎡1 1 1 1 0 0 ⎤ side by side

[A I ] = ⎢⎢2 −1 1 0 1 0⎥ ⎡8 4 3 : 1 0 0 ⎤
⎢⎣1 −2 −3 0 0 1⎥
⎦ [B I ] = ⎢⎢2 1 1 : 0 1 0⎥⎥
⎢⎣1 2 1 : 0 0 1 ⎥⎦
⎡1 1 1 1 0 0⎤
R2 − 2 R1 ⎢ ⎥ ⎡1 2 1 : 0 0 1 ⎤
! ⎢0 −3 −1 −2 1 0⎥
R3 − R1 ⎢⎣0 −3 −4 −1 0 1⎥ ∼ ⎢⎢2 1 1 : 0 1 0 ⎥⎥ by R13

⎢⎣8 4 3 : 1 0 0 ⎦⎥
⎡1 1 1 1 0 0⎤ ⎡1 2 1 : 0 0 1 ⎤ by
⎢ ⎥ R2 − 2 R1
⎢0 −3 −1 −2 1 0⎥ ∼ ⎢⎢0 −3 −1 : 0 1 −2⎥⎥
!
R3 − R2
⎢⎣0 0 −3 1 −1 1⎥ R3 − 4 R2
⎦ ⎢⎣0 0 −1 : 1 −4 0 ⎥⎦

⎡ 2 1 1 ⎤ ⎡1 2 1 : 0 0 1 ⎤
by
⎢1 0 3 3 0 ⎥ ∼ ⎢⎢0 −3 −1 : 0 1 −2⎥⎥
⎢ 3 ⎥ ( −1) R3
! ⎢⎣0 0 1 : −1 4 0 ⎥⎦
R1 + 13 R2 ⎢0 −3 −1 −2 1 0 ⎥
− 13 R3 ⎢0 0 1 1 1 1⎥ ⎡1 2 1 : 0 0 1 ⎤
⎢ − − ⎥ by
⎣ 3 3 3 ⎥⎦ ∼ ⎢⎢0 −3 0 : −1 5 −2⎥⎥
R + R3
⎢⎣0 0 1 : −1 4 0 ⎥⎦ 2
⎡ 5 1 2 ⎤
⎢ ⎥ ⎡1 2 1 : 0 0 1⎤
9 9 9 ⎥
⎢1 0 0 ⎢ 1 5 2⎥ 1
! ⎢ 7 4 1⎥ ∼ ⎢0 1 0 : − ⎥ by − R2
R1 − 23 R3 ⎢0 −3 0 − 3 − ⎥ ⎢ 3 3 3⎥ 3
3 3 ⎢0 0 1 : −1 4
R2 + R3 ⎢0 0 1 ⎥ ⎣ 0 ⎥⎦
⎢ 1 1 1⎥
− −
⎢⎣ 3 3 3 ⎥⎦ ⎡ 1 2 1⎤
⎢1 0 0 : 3 − 3 − ⎥
3
⎢ ⎥
⎡ 5 1 2⎤ 1 5 2 ⎥ by
⎢ + ⎥ ∼ ⎢0 1 0 : −
9 9 9⎥ ⎢ 3 3 3 ⎥ R1 − 2 R2 − R3
⎢1 0 0 ⎢ ⎥
! ⎢0 1 0 7 −
4 1 ⎥ ⎢ 0 0 1 : −1 4 0 ⎥
1
− R2 ⎢ 9 9 9 ⎥ ⎢⎣ ⎥⎦
3 ⎢0 0 1 ⎥
⎢ 3 3 3⎥ ⎡1 2 1⎤
− −
⎢⎣ 9 9 9 ⎥⎦ ⎢3 −3 − ⎥
3
⎢ ⎥
1 5 2 ⎥
⎡5 1 2⎤ A−1 =⎢ −
⎢3 3 3 ⎥
1⎢
∴ A−1 = ⎢ 7 −4 1 ⎥⎥ ⎢
−1 4 0 ⎥

9 ⎢
⎢⎣ −3 3 −3⎥⎦ ⎢⎣ ⎥⎦
1-24    Engineering Mathematics-II

Example 1.48 Operate C2 + C3


⎡ 3 −3 4⎤ ⎡1 0 0 ⎤ ⎡1 −1 0⎤ ⎡1 0 0⎤
If A = ⎢⎢ 2 −3 4⎥⎥ find A−1. Also find two nonsin- ⎢ 2 1 4⎥ = ⎢0 1 0⎥ A ⎢⎢0 1 0⎥⎥
⎢ ⎥ ⎢ ⎥
⎢⎣0 −1 1 ⎥⎦ ⎢⎣0 0 1 ⎥⎦ ⎢⎣0 0 1⎥⎦ ⎢⎣0 1 1⎥⎦
gular matrices P and Q such that P AQ = I, where I
is the unit matrix; verify that A−1 = QP. Operate R2 − 2R1 − 4R3
⎡1 0 0⎤ ⎡ 1 −1 0 ⎤ ⎡1 0 0⎤
Solution We find A−1 by the Gauss–Jordan ⎢0 1 0⎥ = ⎢ −2 3 −4⎥ A ⎢⎢0 1 0⎥⎥
method. We write A and I side by side ⎢ ⎥ ⎢ ⎥
⎣⎢0 0 1⎥⎦ ⎣⎢ 0 0 1 ⎦⎥ ⎣⎢0 1 1⎥⎦
So, I = P AQ, where
⎡ 1 −1 0 ⎤ ⎡1 0 0⎤
P = ⎢ −2 3 −4⎥ and Q = ⎢⎢0 1 0⎥⎥
⎢ ⎥
⎢⎣ 0 0 1 ⎥⎦ ⎢⎣0 1 1⎥⎦

⎡1 0 0⎤ ⎡ 1 −1 0 ⎤
Also, QP = ⎢⎢0 1 0⎥⎥ ⎢⎢ −2 3 −4⎥⎥
⎢⎣0 1 1⎥⎦ ⎢⎣ 0 0 1 ⎥⎦
⎡ 1 −1 0 ⎤
= ⎢⎢ −2 3 −4⎥⎥ = A−1
⎢⎣ −2 3 −3⎥⎦

EXERCISE 1.3
⎡1 1 3⎤
 1. Find the inverse of A = ⎢⎢ 1 3 −3⎥⎥ .
⎢⎣ −2 −4 −4⎥⎦
[Andhra, 1998]
⎡ −24 −8 −12⎤
Ans: − ⎢⎢ 10 6 ⎥⎥
1
2
8
⎢⎣ 2 2 2 ⎥⎦

Write A = IAI ⎡1 3 3⎤
⎡ 3 −3 4⎤ ⎡1 0 0⎤ ⎡1 0 0⎤ 2. Find the inverse of the matrix A = ⎢⎢1 4 3⎥⎥ .
⎢ 2 −3 4⎥ = ⎢0 1 0⎥ ⎢⎣1 3 4⎥⎦
⎢ ⎥ ⎢ ⎥ A ⎢⎢0 1 0⎥⎥
[Andhra 1991, Kuvempu 1996]
⎢⎣0 −1 1 ⎥⎦ ⎢⎣0 0 1⎥⎦ ⎢⎣0 0 1⎥⎦
⎡ 7 −3 −3⎤
Ans: ⎢⎢ −1 1 0 ⎥⎥
Operate R1 − R2
⎢⎣ −1 0 1 ⎥⎦
⎡1 0 0 ⎤ ⎡1 −1 0⎤ ⎡1 0 0⎤
⎢ 2 −3 4⎥ = ⎢0 1 0⎥ ⎡2 5 3⎤
A ⎢⎢0 1 0⎥⎥
⎢ ⎥ ⎢ ⎥ 3. Find the inverse of the matrix A = ⎢⎢ 3 1 2⎥⎥ .
⎢⎣0 −1 1 ⎥⎦ ⎢⎣0 0 1⎥⎦ ⎢⎣0 0 1⎥⎦ ⎢⎣1 2 1⎥⎦
Matrices and Linear Systems of Equations    1-25

⎡ −3 1 7 ⎤ 10. Using the Gauss–Jordan method find the inverse of


Ans: ⎢⎢ −1 −1 5 ⎥⎥ ⎡7 −3 3⎤
⎢⎣ 5 1 −13⎥⎦ A = ⎢⎢ −1 1 0⎥⎥ .
⎢⎣ −1 0 1⎥⎦
4. Find, by the Gauss–Jordan method, the inverse of the ⎡1 3 3⎤
⎡2 2 4⎤ Ans: ⎢⎢1 4 3⎥⎥
matrix A = ⎢⎢1 3 2⎥⎥ . ⎢⎣1 3 4⎥⎦
⎢⎣ 3 1 3⎥⎦

⎡ 7 −2 −8⎤ 1.12 RANK OF A MATRIX


1
Ans: − ⎢⎢ 3 −6 0 ⎥⎥ We have defined the rank of a matrix earlier. We
12
⎢⎣ −8 4 4 ⎥⎦ give below another definition and discuss different
5. Using the Gauss–Jordan method, find the inverse of methods of determination of the rank of a matrix.
the matrix in Ex. 1 above.
1.12.1 Rank of a Matrix: Denition 2
⎡1 2 3⎤
6. Find the inverse of the matrix A = ⎢⎢2 4 5⎥⎥ using With each matrix A of order m × n we associate a
the Gauss–Jordan method. ⎢⎣ 3 5 6⎥⎦
unique nonnegative integer r such that

⎡ 1 −3 2 ⎤ (a) every (r + 1)-rowed minor, if exists, is of zero


value or there is no such minor in A and
Ans: ⎢⎢ −3 3 −1⎥⎥
(b) there is at least one r-rowed minor which
⎣⎢ 2 −1 0 ⎥⎦
does not vanish.
⎡1 3 3⎤
Thus, the rank of an m × n matrix A is the order r
7. Find the inverse of the matrix A = ⎢⎢1 4 3⎥⎥ by
of the largest nonvanishing minor of A. It is denoted
using the Gauss–Jordan method. ⎢⎣1 3 4⎥⎦
by r(A) or r(A).
⎡ 7 −3 −3⎤
Ans: ⎢⎢ −1 1 0 ⎥⎥ Note 1 r(A) = r(AT). (The rank of a matrix is the
same as that of its transpose.)
⎣⎢ −1 0 1 ⎦⎥
Note 2 By definition r(0) = 0. (The rank of a null
8. Use the Gauss–Jordan method and find out the inverse matrix is zero.)
⎡0 1 3⎤
Note 3 If In is the nth-order unit matrix r(In) = n.
of the matrix A = ⎢⎢1 2 3⎥⎥ . [Andhra, 1998]
⎢⎣ 3 1 1⎥⎦ Note 4 If A is a nonsingular matrix of order n then
r(A) = n.
⎡ 1 −1 1 ⎤
1⎢
Ans: ⎢ −8 6 −2⎥⎥ Note 5 If A is a singular matrix of order n then
2 r(A) < n.
⎢⎣ 5 −3 1 ⎥⎦
Note 6 If B is a submatrix of matrix A then
9. By the Gauss–Jordan method find the inverse of the
r(A) ≥ r(B)
⎡ 4 −1 1 ⎤
matrix A = ⎢⎢ 2 0 −1⎥⎥ . r(A) ≤ min(m, n) (A is an m × n matrix)
⎢⎣ 1 −1 3 ⎥⎦ r(AB) ≤ r(A) or r(B) (proved below)
⎡ −1 2 1⎤ (The rank of the product of two matrices cannot
Ans: ⎢ −7 11 6⎥ exceed the rank of either matrix)
⎢ ⎥
⎢⎣ −2 3 2⎥⎦ Def. 1 ⇔ Def. 2
1-26    Engineering Mathematics-II

Important Note Clearly a = b = c = 0 is the only solution for


The following points help in determining the rank these equations which shows that the vectors are
of a matrix linearly independent.

(a) r(A) ≤ r if all minors of A of order (r + 1) ∴ r(A) = Number of linearly independent


vanish. vectors = 3.
(b) r(A) ≥ r if at least one r-rowed minor of A Example 1.51
is nonzero. ⎡1 1 −1 ⎤
(c) If a matrix B is obtained from A by a finite Find the rank of the matrix A = ⎢⎢ −2 −2 2 ⎥⎥ .
sequence of elementary row/column ⎢⎣16 16 −16⎥⎦
transformations on A then B is said to be
equivalent to A. We write B ∼ A. Then Solution Clearly every pair of vectors is linearly
r(A) = r(B). dependent. If we write
a = (1, 1, − 1); b = ( −2, − 2, 2); c = (16, 16, − 16)
If B is the echelon form of A then r(A) = r(B) =
Number of nonzero rows. 16a = −8b = c
∴ r(A) = Number of linearly independent
1.13 METHODS FOR FINDING THE RANK vectors = 1.
OF A MATRIX

1.13.1 Method 1: Maximum Number 1.13.2 Method 2: Method of Minors


of Linearly Independent Rows (Enumeration Method)
The rank of a matrix A can be determined by finding In this method, we list out square submatrices of
the maximum number of linearly independent row the given matrix, starting from the largest ones and
vectors of matrix A. check if any of them is nonsingular. If we succeed
This is useful when we can easily find the linear in finding a nonsingular submatrix then the rank of
independence of row vectors in a matrix, as the the matrix is equal to the order of that submatrix. If
following examples will illustrate. all of them are singular then we consider the next
largest submatrices and so on.
Example 1.49 This procedure is laborious and is not advis-
⎡1 −2 6 0 ⎤ able especially when the given matrix has more than
Find the rank of A = ⎢⎢ 3 2 7 −2⎥⎥ . 3 rows/columns.
The following examples will illustrate the points.
⎢⎣2 4 1 −2⎥⎦
⎡a a a13 ⎤
Solution Here R1 + R3 = R2 so three rows are linearly The matrix ⎢ 11 12
a a a ⎥ has one 2 × 3 sub-
⎣ 21 22 23 ⎦
dependent and any two rows are linearly independent,
as one cannot be expressed as scalar times another. matrix, that is, itself and has three 2 × 2 submatrices,
r(A) = number of linearly independent rows = 2. ⎡ a11 a12 ⎤ ⎡ a11 a13 ⎤ ⎡ a12 a13 ⎤
namely, ⎢ ⎥⎢ ⎥⎢ ⎥ ; two 1 × 3
Example 1.50 ⎣ a21 a22 ⎦ ⎣ a21 a23 ⎦ ⎣ a22 a23 ⎦
submatrices, i.e., two row vectors [a11, a12, a13] and
⎡ 1 −1 1 ⎤
[a21, a22, a23]; three 2 × 1 submatrices, (i.e., three
Find the rank of A = ⎢⎢ −1 1 1 ⎥⎥ .
⎡ a11 ⎤ ⎡ a21 ⎤ ⎡ a13 ⎤
⎢⎣ 2 3 4⎥⎦ column vectors) ⎢ ⎥ ⎢ ⎥ ⎢ ⎥ and six 1 × 2
⎣ a21 ⎦ ⎣ a22 ⎦ ⎣ a23 ⎦
Solution If we write a(1, −1, 1) + b(−1, 1, 1) + submatrices, [a11 a12] [a11 a13] [a12 a13] [a21 a22]
c(2, −3, 4) = (0, 0, 0) we have [a21 a23] [a22 a23] and six 1 × 1 submatrices (a11)
a − b + 2c = 0; −a + b − 3c = 0; a + b + 4c = 0 (a12) (a13) (a21) (a22) (a23)
Matrices and Linear Systems of Equations    1-27

Determine the rank of each of the matrices in Solution Since A is a third-order submatrix
Examples 1.52–1.54. r(A) ≤ 3.
Example 1.52 |A| = 4(−6 + 2) − 2(−12 + 12) + 3(−8 + 8) = 0
⎡ 1 −2 −1⎤ ∴ r(A) < 3 i.e., r(A) ≤ 2
A = ⎢⎢ −3 3 0 ⎥⎥ . The following are the nine two-rowed
⎢⎣ 2 2 4 ⎥⎦ submatrices of A
Solution A is singular since ⎡4 2⎤ ⎡ 4 3⎤ ⎡ 2 3⎤ ⎡ 4 2 ⎤ ⎡ 4 3 ⎤
⎢8 4⎥⎦ ⎢⎣ 8 6⎥⎦ ⎢⎣ 4 6⎥⎦ ⎢⎣ −2 −1⎥⎦ ⎢⎣ −2 −1.5⎥⎦
3 0 −3 0 −3 3 ⎣
A = 1⋅ +2 −1
2 4 2 4 2 2 ⎡2 3 ⎤⎡8 4⎤⎡8 6 ⎤⎡4 6 ⎤
⎢ −1 −1.5⎦ ⎣ −2 −1⎦ ⎣ −2 −1.5⎦ ⎣ −1 −1.5⎥⎦
⎥ ⎢ ⎥ ⎢ ⎥ ⎢
= 1⋅12 + 2( −12) − 1( −6 − 6) = 0 ⎣

Since A contains a two-rowed nonsingular submatrix all of these have vanishing determinants. So
⎡3 0⎤ r(A) ≠ 2.
⎢2 4⎥ whose determinant is 12 Since A is a nonnull matrix r(A) ≠ 0. Hence
⎣ ⎦
p(A) = 1.
∴ r(A) = 2.
Example 1.53 1.13.3 Method 3: Reduction
⎡ 4 2 1 3⎤ to Normal or Canonical
A = ⎢⎢ 6 3 4 7⎥⎥ . Form by Elementary
Transformations
⎢⎣ 2 1 0 1⎥⎦
Every m × n matrix A whose rank is r can be transformed
Solution A has the following three-rowed submatrices, by the application of a finite number of elementary
B1 B2 B3 B4 transformations to a sequence of equivalent matrices,
consequently assuming the normal form N where
⎡ 4 2 1 ⎤ ⎡ 4 2 3 ⎤ ⎡ 4 1 3 ⎤ ⎡ 2 1 3⎤
⎢ 6 3 4 ⎥ ⎢ 6 3 7⎥ ⎢ 6 4 7⎥ ⎢ 3 4 7⎥ ⎡Ir 0⎤ ⎡Ir ⎤
⎢ ⎥⎢ ⎥⎢ ⎥⎢ ⎥ N=⎢ [I r 0] or [I r ]
⎢⎣ 2 1 0 ⎥⎦ ⎢⎣ 2 1 1⎥⎦ ⎢⎣ 2 0 1⎥⎦ ⎢⎣1 0 1⎥⎦ ⎣0 0⎥⎦ ⎢0⎥
⎣ ⎦
obtained by deleting C4, C3, C2 and C1, respectively. where Ir is the r-rowed identity matrix, and 0 denotes
Expanding by R3, we have a zero matrix of an appropriate order.
|B1| = 2(2.4 − 3.1) − 1(4.4 − 6.1) = 0 Theorem 1.4 Every m × n matrix of rank r can be
|B2| = 2(2.7 − 3.3) − 1(4.7 − 6.3) + 1.(4.3 − 6.2) = 0 reduced to the normal form
|B3| = 2(1.7 − 4.3) + 0 + 1(4.4 − 6.1) = 0
⎡Ir 0⎤
|B4| =1.(1.7 − 4.3) + 0 + 1(2.4 − 3.1) = 0 ⎢0
⎣ 0⎥⎦
Since all the submatrices of order 3 are
singular r(A) < 3. r(A) = 2 since there is a two- by a sequence of elementary operations, where Ir is
⎡ 4 7⎤ the r-rowed unit matrix.
rowed submatrix ⎢ ⎥ whose determinant is
nonvanishing. ⎣ 0 1⎦ Corollary 1 The rank of an m × n matrix A is r if
and only if it can be reduced to the normal form
Example 1.54
⎡Ir 0⎤
⎡4 2 3 ⎤ ⎢0
⎢ ⎣ 0⎥⎦
A= ⎢ 8 4 6 ⎥⎥ .
by a finite sequence of elementary transformations.
⎢⎣ −1 −1 −1.5⎥⎦
1-28    Engineering Mathematics-II

Corollary 2 Two matrices A and B are equivalent, 1.13.4 Rank of a Product


denoted by A ∼ B, if and only if A and B are of the of Matrices
same size and same rank.
Theorem 1.7 The rank of a product of two
Corollary 3 If A is an m × n matrix of rank r then matrices cannot exceed the rank of either matrix.
there exist elementary matrices P1 · P2 … Ps and
Proof Let A and B be matrices of order m × n and
Q1 … Q2 … Qt such that
n × p, respectively. Suppose r(A) = r1, r(B) = r and
( Ps ⋅ Ps −1 " P1 ) A(Q1 " Q2 " Qt ) such that r(AB) = r.
Now, there exists a nonsingular matrix P such
⎡Ir 0⎤
Ps ⋅ Ps −1 " P1 AQ1 " Q2 " Qt = ⎢ that
⎣0 0⎥⎦
⎡G ⎤
PA = ⎢ ⎥ where G is an r1 × n matrix of rank r1.
Corollary 4 If A is an m × n matrix of rank r then ⎣O ⎦
⎡G ⎤
there exist nonsingular matrices P and Q such that PAB = ⎢ ⎥ B
⎣O ⎦
⎡Ir 0⎤
PAQ = ⎢
⎣0 0⎥⎦ Since the rank of a matrix is unaltered on
multiplication by a nonsingular matrix.
Corollary 5 If A is a nonsingular matrix of order
⎛ ⎡G ⎤ ⎞
n then A ∼ In. r ( PAB) = r ( AB) = r ⎜ ⎢ ⎥ B⎟ = r
⎝ ⎣O ⎦ ⎠
Corollary 6 Every nonsingular matrix can be
expressed as a product of elementary matrices. Since G has only r1 nonzero rows, the matrix
⎡G ⎤
Row/Column Equivalence of Matrices ⎢O ⎥ B cannot have more than r1 nonzero rows
⎣ ⎦
Denition An m × n matrix A is said to be
row/column equivalent to an m × n matrix B, ⎛ ⎡G ⎤ ⎞
R C ∴ r = r ⎜ ⎢ ⎥ B⎟ ≤ r1 ⇒ r( AB) ≤ r( A) (i)
denoted by A ∼ B/A ∼ B if B can be obtained from ⎝ ⎣O ⎦ ⎠
A by a finite number of elementary row/column
transformations. Also, r = r(AB) = r((AB)T) r(BT AT) ≤ P(BT)
by (i)
Theorem 1.5 If A is an m × n matrix of rank r, = r(B) = r2
then there exists a nonsingular matrix P such that
Hence r ≤ min (r1, r2).
⎡G ⎤
PA = ⎢ ⎥
⎣O ⎦ Example 1.55
where G is an r × n matrix of rank r and O is ⎡ 8 1 3 6⎤
(m − r) × n zero matrix. Reduce the matrix A = ⎢⎢ 0 3 2 2⎥⎥ to the
⎢⎣ −8 −1 −3 4⎥⎦
Corollary 7 The rank of a matrix does not change
by pre- or post-multiplication by a nonsingular normal form and find its rank. [JNTU 1999, 2002]
matrix. Solution
Theorem 1.6 A is an m × n matrix of rank r, then ⎡ 1 1 3 6⎤
there exists a nonsingular matrix Q such that AQ = 1
A ! ⎢⎢ 0 3 2 2⎥⎥ , C1
[H 0] where H is an m × r matrix of rank r and O 8
is the m × (n − r) zero matrix. ⎢⎣ −1 −1 −3 4⎥⎦
Matrices and Linear Systems of Equations    1-29

⎡1 1 3 6 ⎤ ⎡1 0 0 0 ⎤
1
! ⎢⎢0 3 2 2 ⎥⎥ R3 + R1 ; ! ⎢⎢0 −3 −2 −5⎥⎥ , − R3 ;
12
⎣⎢0 0 0 10⎦⎥ ⎣⎢0 0 0 1 ⎥⎦
⎡1 0 0 0 ⎤ C2 − C1 , ⎡1 0 0 0⎤
! ⎢⎢0 3 2 2 ⎥⎥ C3 − 3C1 , ! ⎢⎢0 −3 −2 0⎥⎥ , R2 + 5R3 ;
⎢⎣0 0 0 10⎥⎦ C4 − 6C1 ; ⎢⎣0 0 0 1⎥⎦

⎡1 0 0 0⎤ ⎡1 0 0 0⎤
! ⎢⎢0 1 1 1⎥⎥ ;
1 1 1
C2 , C3 , C4 ; ! ⎢⎢0 −3 1 0⎥⎥ , C3 − C1 ;
3 2 2 ⎢⎣0 0 0 1⎥⎦
⎢⎣0 0 0 5⎥⎦
⎡1 0 0 0⎤
⎡1 0 0 0⎤
! ⎢⎢0 0 1 0⎥⎥ C2 + 3C3 ;
! ⎢⎢0 1 0 0⎥⎥ C3 − C2 , C4 − C2 ;
⎣⎢0 0 0 1⎦⎥
⎣⎢0 0 0 5⎦⎥
⎡1 0 0 0⎤
⎡1 0 0 0⎤
! ⎢⎢0 1 0 0⎥⎥ , C2 ↔ C3 , C3 ↔ C4 ! [I3 0]
! ⎢⎢0 1 0 0⎥⎥ C3 ↔ C4 ; ⎢⎣0 0 1 0⎥⎦
⎢⎣0 0 5 0⎥⎦

⎡1 0 0 0⎤ Hence r(A) = 3.
1
! ⎢⎢0 1 0 0⎥⎥ C3 ; ! [ I 3 0]
5 1.13.5 Method 4: Reduction of an
⎢⎣0 0 1 0⎥⎦
m × n Matrix to a
Hence r(A) = 3. Normal Form by Finding
Nonsingular Matrices P and
Example 1.56 Q Such That PAQ = N
⎡1 2 3 4 ⎤ Write Am×n = Im×m A In×n .
⎢ ⎥ Now apply row and column transformations on
By reducing the matrix A ⎢2 1 4 3 ⎥ into a
the LHS matrix A to transform it to a normal form,
⎢⎣ 3 0 5 −10⎥⎦ carrying out every row transformation on the pre-
normal form find its rank. [JNTU 2002] factor Im×m and every column transformation on
the post-factor In×n on the RHS. Then Im×m and In×n
Solution
reduce to nonsingular matrices P and Q such that
⎡1 0 0 0 ⎤
⎡Ir 0⎤
A ! ⎢2 −3 −2 −5 ⎥⎥ , C2 − 2C1 , C3 − 3C1 , C4 − 4C1 ;

PAQ = N = ⎢
⎣0 0⎥⎦
⎢⎣ 3 −6 −4 −22⎥⎦

⎡1 0 0 0 ⎤
Example 1.57
! ⎢⎢0 −3 −2 −5 ⎥⎥ , R2 − 2 R1 , R3 − 3R1 ; ⎡1 1 2⎤
⎢⎣0 −6 −4 −22⎥⎦ For the matrix A = ⎢1 2 3 ⎥⎥ find nonsingular

⎡1 0 0 0 ⎤ ⎣⎢0 −1 −1⎥⎦
! ⎢⎢0 −3 −2 −5 ⎥⎥ , R3 − 2 R1 ; matrices P and Q such that PAQ is in the normal
⎢⎣0 0 0 −12⎥⎦ form. [JNTU 2002/S]
1-30    Engineering Mathematics-II

Solution Write A = I3 AI3 Solution Write


⎡−1 1 2 ⎤ ⎡1 0 0 ⎤ ⎡1 0 0 ⎤ ⎡2 1 −3 −6⎤
A = ⎢⎢ 1 2 3 ⎥⎥ = ⎢⎢0 1 0 ⎥⎥ A ⎢⎢0 1 0 ⎥⎥ A = I 3 AI 4 ⇒ ⎢2 −3 1 2 ⎥
⎢⎣ 0 −1 −1⎥⎦ ⎢⎣0 0 1 ⎥⎦ ⎢⎣0 0 1 ⎥⎦ ⎢ ⎥
⎢⎣1 1 1 2 ⎥⎦
Transform A into normal form applying elemen-
⎡1 0 0 0⎤
tary transformations on the left hand side matrix A ⎡1 0 0⎤ ⎢
0 1 0 0⎥
simultaneously effecting row transformations on the = ⎢⎢0 1 0⎥⎥ A ⎢ ⎥
prefactor I3 and column transformations on the post- ⎢0 0 1 0⎥
⎢⎣0 0 1⎥⎦ ⎢ ⎥
factor I3 on the right side. ⎣0 0 0 1⎦

⎡1 0 0⎤ ⎡1 0 0⎤ ⎡1 −1 −1⎤
⎢1 1 0⎥ = ⎢0 1 0⎥ A ⎢0 1 −1⎥ C2 − C1
⎢ ⎥ ⎢ ⎥ ⎢ ⎥ C −C −C We will transform A on the LHS into the normal
3 1 2
⎣⎢0 −1 0⎥⎦ ⎢⎣0 0 1⎦⎥ ⎣⎢0 0 1 ⎦⎥ form by applying elementary transformations
simultaneously carrying out row transformations on
⎡1 0 0⎤ ⎡ 1 0 0⎤ ⎡1 −1 −1⎤ the prefactor I3 and column transformations on the
⎢0 1 0⎥ = ⎢ −1 1 0⎥ A ⎢0 1 −1⎥ R2 − R1 post factor I4 on the RHS
⎢ ⎥ ⎢ ⎥ ⎢ ⎥
⎢⎣0 −1 0⎥⎦ ⎢⎣ 0 0 1⎥⎦ ⎢⎣0 0 1 ⎥⎦

⎡1 0 0⎤ ⎡ 1 0 0⎤ ⎡1 −1 −1⎤
⎢0 1 0⎥ = ⎢ −1 1 0⎥ A ⎢0 1 −1⎥ ⎡1 1 1 2 ⎤
⎢ ⎥ ⎢ ⎥ ⎢ ⎥ R3 + R2 ⎢2 −3 1 2 ⎥
R1 ↔ R3 ⎢ ⎥
⎢⎣0 0 0⎥⎦ ⎢⎣ −1 1 1⎥⎦ ⎢⎣0 0 1 ⎥⎦
⎣⎢2 1 −3 −6⎦⎥
⎡I2 0⎤
or = PAQ ⎡1 0 0 0⎤
⎢0 0⎥⎦ ⎡0 0 1⎤ ⎢0
⎣ 1 0 0⎥
= ⎢⎢0 1 0⎥⎥ A ⎢ ⎥
⎡ 1 0 0⎤ ⎡1 −1 −1⎤ ⎢0 0 1 0⎥
⎢⎣1 0 0⎥⎦ ⎢ ⎥
where P = ⎢⎢ −1 1 0⎥⎥ , Q = ⎢⎢0 1 −1⎥⎥ ⎣0 0 0 1⎦
⎢⎣ −1 1 1⎥⎦ ⎢⎣0 0 1 ⎥⎦ C2 − C1 ⎡1 0 0 0 ⎤
C3 − C1 ⎢2 −5 −1 −2 ⎥
|P| = 1 ≠ 0. |Q| = 1 ≠ 0. So, P and Q are nonsingular. ⎢ ⎥
Thus, we have found nonsingular matrices P and Q C4 − 2C1 ⎢⎣2 −1 −5 −10⎥⎦
⎡ I 2 0⎤ ⎡1 −1 −1 −2⎤
such that PAQ is in the normal form ⎢ ⎥ ⎡0 0 1⎤ ⎢
⎣ 0 0⎦ 0 1 0 0⎥
= ⎢⎢0 1 0⎥⎥ A ⎢ ⎥
∴ r(A) = 2 ⎢0 0 1 0 ⎥
⎢⎣1 0 0⎥⎦ ⎢ ⎥
Note The nonsingular matrices P and Q obtained ⎣0 0 0 1 ⎦
above are not unique. ⎡1 0 0 0 ⎤
R2 − 2 R1 ⎢
Example 1.58 0 −5 −1 −2⎥⎥
R3 − R2 ⎢
⎢⎣0 4 −4 −8⎥⎦
⎡2 1 −3 −6⎤
⎡1 −1 −1 −2⎤
If A = ⎢⎢2 −3 1 2 ⎥⎥ find nonsingular matrices ⎡0 0 1 ⎤ ⎢
0 1 0 0⎥
⎢⎣1 1 1 2 ⎥⎦ = ⎢⎢0 1 −2⎥⎥ A ⎢ ⎥
⎢0 0 1 0 ⎥
such that PAQ is in the normal form. ⎣⎢1 −1 0 ⎦⎥ ⎢0 0 0 1 ⎥
[JNTU 2002] ⎣ ⎦
Matrices and Linear Systems of Equations    1-31

⎡1 0 0 0⎤ Nonzero row: By a nonzero row of a matrix


C2 + C3 ⎢
0 −6 −1 0 ⎥⎥ we mean a row which contains at least one nonzero
C4 − 2C3 ⎢ entry.
⎢⎣0 0 −4 0 ⎥⎦
Echelon Form: The echelon form of matrix A is
⎡1 −2 −1 0 ⎤
⎡0 0 1 ⎤ ⎢ an equivalent matrix, obtained by a finite sequence
0 1 0 0⎥
= ⎢⎢0 1 −2⎥⎥ A ⎢ ⎥ of elementary operations on A which has the follow-
⎢0 1 1 −2⎥ ing properties,
⎣⎢1 −1 0 ⎦⎥ ⎢0 0 0 1 ⎥
⎣ ⎦ 1. The zero rows, if any, are below a nonzero row.
⎡1 0 0 0⎤ 2. The first nonzero entry in each nonzero row
C3 − 16 C2 ⎢
0 1 0 0 ⎥⎥ is 1.
− 16 C2 ⎢
⎢⎣0 0 −4 0 ⎥⎦ 3. The number of zeros before the first nonzero
1 2 entry in a row is less than the number of such
⎡ ⎤
⎢1 3 −
3
0⎥ zeros in the next row immediately below it.
⎢ ⎥
⎡0 0 1 ⎤ ⎢0 − 1 1 Note 1 Condition (2) is optional.
− 0⎥
= ⎢⎢0 1 −2⎥⎥ A⎢ 6 6 ⎥
Note 2 The rank of A is equal to the number of
⎢ ⎥
⎢⎣1 −1 0 ⎥⎦ ⎢0 − 1 5
−2⎥ nonzero rows in its echelon form.
⎢ 6 6 ⎥
⎢0 0 Determine the ranks of the following matrices.
⎣ 0 1 ⎥⎦
Example 1.59
⎡1 0 0 0 ⎤
Determination of the rank of a matrix
− 4 R3 ⎢⎢0 1 0 0 ⎥⎥
1

⎢⎣0 0 1 0 ⎥⎦ ⎡ 1 2 3⎤
1 2 A = ⎢⎢1 4 2⎥⎥ .
⎡ ⎤
1 − 0⎥ ⎢⎣2 6 5⎥⎦
⎡ ⎤ ⎢ 3 3
⎢ 0 ⎢ ⎥
0 1⎥ ⎢ 1 1
⎢ ⎥ 0 − − 0⎥ Solution
=⎢ 0 1 −2⎥ A ⎢ 6 6 ⎥
⎢ 1 1 ⎥ ⎢ 1 5
⎥ ⎡1 2 3 ⎤
R2 − R1 ⎢
⎡1 2 3 ⎤
⎢− 0 ⎥ ⎢0 − −2⎥ A ! ⎢⎢0 2 −1⎥⎥ by ! ⎢0 2 −1⎥⎥ , R3 − R2 .
⎣ 4 4 ⎦ ⎢ 6 6 ⎥ R3 − 2 R1
⎢0 0 0 1 ⎥⎦ ⎢⎣0 2 −1⎥⎦ ⎢⎣0 0 0 ⎥⎦

[I3 0] = P A Q
∴ r (A ) = 3 Clearly |A| = 0. But the following two-rowed minor
⎡1 3 ⎤
1.13.6 Method 5: Reduction of ⎢0 −1⎥ = −1 ≠ 0
⎣ ⎦
Matrix A to Echelon Form
By the application of a finite number of elementary ∴ r(A) = 2 (number of nonzero rows)
row transformations on the given m × n matrix A
we can reduce it to an equivalent matrix in echelon Example 1.60
form which is defined below. First we define zero ⎡ 2 3 −1 −1⎤
and nonzero rows of a matrix. ⎢ 1 −1 −2 −4 ⎥
Zero row: By a zero row of a matrix we mean a
A= ⎢ ⎥. [JNTU 2000 S]
⎢3 1 3 −2 ⎥
row which contains only zero entries. ⎢ ⎥
⎣6 3 0 −7 ⎦
1-32    Engineering Mathematics-II

Solution 3. Find the rank of the matrix


R2 − 2 R1 , R3 − 3 R1
⎡ 1 3 4 −3⎤
R1 ↔ R2 R4 − R3 − R2 − R1
⎡1 −1 −2 −4⎤ ⎡1 −1 −2 −4⎤ A = ⎢⎢ 3 9 12 −9⎥⎥ . [University Exam]
⎢2 3 −1 −1⎥ ⎢0 5 3 7 ⎥ ⎢⎣ −1 −3 −4 3 ⎥⎦
A!⎢ ⎥!⎢ ⎥
⎢ 3 1 3 −2⎥ ⎢0 4 9 10 ⎥ Ans: 1; R1, R2, R3 pairwise L.D. (method 1 or 2)
⎢ ⎥ ⎢ ⎥
⎣6 3 0 −7⎦ ⎣0 0 0 0 ⎦ 4. Find the rank of the matrix
R2 − R3 R3 − 4 R2
⎡1 −1 −2 −4⎤ ⎡1 −1 −2 −4⎤ ⎡1 3 4 −3⎤
⎢0 1 −6 −3⎥ ⎢0 1 −6 −3⎥ A = ⎢⎢3 9 12 −9⎥⎥ . [University Exam]
!⎢ ⎥!⎢ ⎥ ⎢⎣1 3 4 1 ⎥⎦
⎢0 4 9 10 ⎥ ⎢0 0 33 22 ⎥
⎢ ⎥ ⎢ ⎥
⎣0 0 0 0 ⎦ ⎣0 0 0 0 ⎦ Ans: 2; R1, R2 are L.D. R1, R3 are L.I.
(method 1, 2 or 3)
The final matrix is in echelon form with three nonzero
5. Determine the rank of the matrix
rows ∴ r(A) = 3.
⎡6 1 3 8⎤
⎢ 4 2 6 −1⎥
Example 1.61 A= ⎢ ⎥.
⎢10 3 9 7 ⎥
⎡4 2 3 ⎤ ⎢ ⎥
⎢ ⎣16 4 12 15 ⎦
A= ⎢ 8 4 6 ⎥⎥ .
Ans: 2; R1 + R2 = R3 2R1 + R2 = R4
⎢⎣ −2 −1 −1.5⎥⎦
(method 1 or 2 or 5)
6. Determine the rank of the matrix
⎡ 4 2 3⎤ ⎡ 1 −2 1 −1⎤
⎛ 1⎞
Solution A = ⎢⎢ 0 0 0⎥⎥ R21 ( −2), R31 ⎜ ⎟ ⎢ 1 1 −2 3 ⎥
⎝ 2⎠ A= ⎢ ⎥.
⎣⎢ 0 0 0⎥⎦ ⎢ 4 1 −5 8 ⎥
⎢ ⎥
Since A contains only one nonzero row, r(A) = 1. ⎣ 5 −7 2 −1⎦
Ans: 2 (method 5)
EXERCISE 1.4 Find the rank of each of the following matrices:
1. Find the rank of the matrix
⎡1 2 3⎤
⎡ ⎤ 7. A = ⎢⎢1 4 2⎥⎥ .
p
⎢cos 4 1 ⎥ [Bhopal 1991, Ranchi 1989]
A= ⎢ ⎥. ⎣⎢2 6 5⎥⎦
⎢ sin p sin p ⎥
⎢⎣ 4 2 ⎥⎦ Ans: 2 (method 1 or 2)
Ans: 1 (method 1 or 2) ⎡1 2 3 0⎤
⎢2 4 3 2⎥⎥
2. Find the rank of the matrix
8. A = ⎢ . [Karnataka 1990, Delhi 1997]
⎡ p p⎤ ⎢3 2 1 3⎥
⎢ 1 2 cos 2tan ⎥ ⎢ ⎥
4 4 ⎣6 8 7 5⎦
⎢ ⎥
p p p⎥
A = ⎢sin 2 sin 2cos . Ans: 3 (method 2 or 3)
⎢ 2 4 4⎥
⎢ ⎥ ⎡2 3 −1 −1⎤
⎢ esin p p ⎥
− cos p sec
⎢⎣ 2 ⎥⎦ 9. A = ⎢⎢1 −1 −2 −4⎥⎥ . [Mysore 1997,
Ans: 1 (method 1 or 2) ⎣⎢6 3 0 −7⎥⎦ Bangalore 1988]
Ans: 3 (method 3 or 5)
Matrices and Linear Systems of Equations    1-33

⎡ 2 3 4 −1⎤ ⇒ x1C1 + x2C2 + … + xnCn = B (1.18)


10. A = ⎢ 5 2 0 −1⎥
⎢ ⎥ Let r(A) = r ⇒ r of the n columns of A
⎢⎣ −4 5 12 −1⎥⎦
are L.I. and each of the remaining (n − r) columns
Ans: 2 (method 5) can be expressed as a linear combination of these r
column vectors.
Let (1.17) be consistent. Then there exist n
1.14 EXISTENCE AND UNIQUENESS scalars K1, K2, …, Kn such that
OF SOLUTIONS OF A SYSTEM
OF LINEAR EQUATIONS K1C1 + K2C2 + … + KnCn = B (1.19)
We now take up the question of existence and ⇒ B is a linear combination of Ci (1.20)
uniqueness of solutions of a system of linear
equations (1.12) and deal with different methods of (i = 1, 2, …, n)
solution of both NH and H systems.
The system (1.17) has a solution (i.e., the Since Cr+1, Cr+2, …, Cn can be expressed as a
system is consistent) iff the rank of the coeffcient linear combination of Ci (i = 1, 2, …, r).
matrix A is equal to the rank of the augmented The augmented matrix [A, B] has precisely r
matrix A = [ A | B ]. L.I. columns,
That is, if r(A) = r and r(Ā ) = r¢ and if r ≠ r¢ ∴ r(A, B) = r = r(A) (1.21)
then the system (1.17) is inconsistent and it has no
solution Conversely, let
If r = r¢ = n, unique solution exists r(A, B) = r(A) = r (1.22)
If r = r¢ < n, infinitely many solutions exist.
If m = n then (H) has a nontrivial solution iff the Thus, the maximum number of L.I. columns of
coeffcient matrix A is singular. [A, B] is r.
Since C1, C2, …, Cr are L.I. columns of
1.14.1 The System of NH [A, B], B can be expressed as a linear combination of
C1, C2, …, Cr.
Equations: Consistency
⇒ There exist scalars K1, K2, …, Kr, not all
and Inconsistency
zero, such that
Theorem 1.8 The NH system of equations
K1C1 + K2C2 + … + KrCr = B
AX = B (1.15) ⇒ K1C1 + K2C2 + … + KrCr + 0.Cr+1
is consistent (possesses at least one solution) iff + 0·Cr+2 + … + 0·Cn = B
r(A, B) = r(A) (1.16) x1 = K1, x2 = K2, …, xr = Kr, xr+1 = 0, …, xn = 0 is a
solution of (1.17).
(The rank of the augmented matrix = The rank of the Hence the NH system (1.17) is consistent.
coeffcient matrix).
Proof Let C1, C2, …, Cn be the column vectors of 1.14.2 Existence of a Unique
the coeffcient matrix A. Solution for NH System of
Then AX = B is equivalent to
n Equations in n Unknowns
⎡ x1 ⎤ Theorem 1.9 The system of n equations in n
⎢x ⎥ unknowns
[C1 , C2 ! Cn ] ⎢ ⎥ = B
2
(1.17)
⎢ "⎥
⎢ ⎥ AX = B (1.15)
⎣ xn ⎦
has a unique solution if A is nonsingular.
1-34    Engineering Mathematics-II

Proof Let A be a nonsingular matrix of order The rank of matrix A = r(A) = r. The number of
n. Then r(A) = r(A, B) = r. ⇒ The system has a equations = m; The number of unknowns = n.
solution (NH = Nonhomogeneous system of equations;
A is nonsingular ⇒ A−1 exists. H = Homogeneous system of equations.)
A−1(AX) = A−1B 1. NH has a solution ⇔ r(A) = r(A|B).
The rank of the coeffcient matrix A = The
⇒ X = A−1B is a solution of (1.17). range of the augmented matrix (A|B).
If X1, X2 are two solutions then
2. r = m: NH has a solution for any B ∈ Vm.
AX1 = B, AX2 = B
H has a solution which is unique
⇒ AX1 = AX2 (m < n) (i.e., trivial solution).
⇒ A (AX1) = A−1(AX2)
−1
⇒ X1 = X2 3. r = m = n: NH has a unique solution for any
B ∈ Vm. H has a unique solution (i.e., trivial
Hence the solution is unique. solution).
NH⎫
1.14.3 Existence of a Solution for 4. r =m<n ⎬ have an infinite number
NH System of m Equations N⎭
in n Unknowns of solutions r of the unknowns can be deter-
mined in terms of the remaining (n − r)
Theorem 1.10 The NH system AX = B of m
unknowns whose values can be chosen
equations in n unknowns has
arbitrarily.
(i) no solution if r(A) < r(A, B), r < m = n⎫
(ii) a unique solution if r(A) = r(A, B) = n and 5. ⎪
r < m < n⎬ If NH has a solution then it has
(iii) infinitely many solutions if r(A) = r(A, B) < n. r < n < m⎪⎭
an infinite number of solutions. r of the
1.14.4 Solutions of NH and unknowns can be determined in terms of
H Systems of Equations the remaining (n − r) unknowns. H has an
infinite number of solutions.
Summary of the results 6. r = n < m. If NH has a solution it is unique.
Consider m linear equations in n unknowns H has a unique (trivial) solution.
Am×n Xn×1 = Bm×1 (NH) 7. m = n. H has a nontrivial solution if and
only if it is singular.
Fundamental theorem Let r(A) = r and r(Ã) = r¢.
If r ≠ r¢ the systems are inconsistent so that no
1.15 METHODS OF SOLUTION OF NH
solution exists.
AND H EQUATIONS
If r = r¢ the system is consistent; and
There are two methods of solution of a system of n
(i) if r = r¢ = n then a unique solution exists equations in n unknowns.
(The solution may be obtained by Cramer’s
rule or by the matrix inversion method) 1. Method of determinants of matrices:
Cramer’s rule.8
(ii) if r = r¢ < n then infinitely many solutions
exist. 2. Method of inversion of matrices: In
(Rewrite x1, x2, …, xr variables (whose both the cases, the coeffcient determinant
coefficient submatrix has rank r) in terms of should be different from zero, i.e., the
the remaining (n − r) variables and solve by coeffcient matrix should be nonsingular.
Gaussian elimination or by Gauss–Jordan
8
elimination method.) CRAMER, Gabriel (1704–1752) was a Swiss mathematician.
Matrices and Linear Systems of Equations    1-35

1.15.1 Method 1: Method of Solution The determinant of the coeffcient


Determinants (Cramer’s Rule) matrix is
Consider the system of three equations in three 1 1 1 1 1 1
unknowns
D = 2 −6 −1 = 0 −8 −3
a1x + b1y + c1z = d1
3 4 2 0 1 −1
a1x + b2y + c1z = d2 or AX = B (1.23)
a3x + b3y + c3z = d3 by R2 − 2 R1 , R3 − 3R1
= 11 ≠ 0
⎡ a1 b1 c1 ⎤ ⎡ x⎤ ⎡ d1 ⎤
A = ⎢a b2 c2 ⎥⎥ ; X = ⎢⎢ y ⎥⎥ ; B = ⎢⎢ d2 ⎥⎥ Hence Cramer’s rule can be applied
where ⎢ 2
⎢⎣ a3 b3 c3 ⎥⎦ ⎢⎣ z ⎥⎦ ⎢⎣ d3 ⎥⎦ 11 1 1
D1 = 0 −6 −1 = 11( −6.2 + 1.4) = −88
(1.24)
0 4 2
If we denote the determinant of the coeffcients
by D then 1 11 1
xa1 b1 c1 xa1 + yb1 + zc1 b1 c1 D2 = 2 0 −1 = ( −11)(2.2 + 1.3) = −77
xD = xa2 b2 c2 = xa2 + yb2 + zc2 b2 c2 , 3 0 2
xa3 b3 c3 xa3 + yb3 + zc3 b3 c3 1 1 11
D3 = 2 −6 0 = 11(2.4 + 6.3) = 286
by C1 + yC2 + zC3,
3 4 0
d1 b1 c1
D1 −88 D2 −77
= d2 b2 c2 x= = = −8; y= = = −7;
D 11 D 11
d3 b3 c3 D 286
z= 3 = = 26
D1 D 11
∴ x= where D1 is the determinant D with
D
C1 replaced by the constant matrix B ( D ≠ 0). Cramer’s Rule
D2 D Example 1.63
Similarly y= , z= 3
D D Solve x + y + z = 6, x − y + 2z = 5, 2x − 2y + 3z = 7.
[JNTU 2000]
where D2 and D3 are determinants in which C2 and
C3 are replaced, respectively, by B. Solution
x y z 1 1 1 1
Equations = = = giving the
D1 D2 D3 D
D = 1 −1 2 = 1( −3 + 4) − 1(3 − 4)
values x, y, z constitute Cramer’s rule. This reduces
2 −2 3
the solution of linear equations to a problem of eval-
uation of determinants. (1.25) +1( −2 + 2) = 2 ≠ 0
Example 1.62 6 1 1
Solve by Cramer’s rule D1 = 5 −1 2 = 6( −3 + 4) − 5(3 + 2)
x + y + z = 11 7 −2 3
2x − 6y − z = 0
3x + 4y + 2z = 0 + 7(2 + 1) = 6 − 25 + 21 = 2
1-36    Engineering Mathematics-II

1 6 1 ⎡ a1 b1 c1 ⎤
D2 = 1 5 2 = 1(15 − 14) − 1(18 − 7) Let A = ⎢⎢ a2 b2 c2 ⎥⎥
2 7 3 ⎢⎣ a3 b3 c3 ⎥⎦

+2(12 − 5) = 1 − 11 + 14 = 4 and Ai, Bi, Ci denote the cofactors of elements ai, bi,
ci (i = 1, 2, 3), respectively.
1 1 6
D3 = 1 −1 5 = 1( −7 + 10) − 1(7 + 12) 5 −2 1 2
A1 = + = −21; A2 = − = +21
2 −2 7 7 −7 7 −7
+ 2(5 + 6) = 3 − 19 + 22 = 6 1 2 2 −2
A3 = + = −12; B1 = − = 12;
D1 2 D2 4 5 −2 1 −7
x= = = 1; y= = = 2;
D 2 D 2 1 2 1 2
B2 = + = −9; B3 = − = 6;
D3 6 1 −7 2 −2
z= = =3
D 2 2 5 1 1
C1 = + = 9; C2 = − = −6;
1 7 1 7
1.15.2 Method 2: Method of Matrix 1 1
Inversion (or Adjoint C3 = + = 3;
2 5
Method)
If the coeffcient matrix A is nonsingular, then AX = B det A = Aa1A1 + a2A2 + a3A3 = 1· (−21) + 2 · 21 + 1
can be multiplied by A−1 to get · (−12) = 9 ≠ 0
A−1 (AX) = (A−1A)X = IX = X = A−1B So, A−1 =
Adj A
exists and is equal to
det A
⎡x⎤ ⎡ A1 A2 A3 ⎤ ⎡ d1 ⎤
⎢ y⎥ = 1 ⎢ B B B3 ⎥⎥ ⎢⎢ d2 ⎥⎥ ⎡ −21 21 −12⎤
⎢ ⎥ D⎢ 1 2 1⎢
⎢⎣ z ⎥⎦ ⎢⎣C1 C2 C3 ⎥⎦ ⎢⎣ d3 ⎥⎦ 12 −9 6 ⎥⎥
9⎢
⎢⎣ 9 −6 3 ⎥⎦
where Ai, Bi, Ci are the cofactors of ai, bi, ci of A.
⎡ −7 7 −4⎤
1⎢
Example 1.64
−1
A = ⎢4 −3 2 ⎥⎥
3
Solve by calculating the inverse by the adjoint ⎣⎢ 3 −2 1 ⎥⎦
method
Solution is
x1 + x2 + 2x3 = 4
⎡ x1 ⎤ ⎡ −7 7 −4 ⎤ ⎡ 4 ⎤ ⎡ −9 ⎤
2x1 + 5x2 − 2x3 = 3 1
X = ⎢⎢ x2 ⎥⎥ = A−1 B = ⎢ 4 −3 2 ⎥ ⎢ 3 ⎥ = ⎢17 3⎥
⎢ ⎥⎢ ⎥ ⎢ ⎥
x1 + 7x2 − 7x3 = 5 3
⎢⎣ x3 ⎥⎦ ⎢⎣ 3 −2 1 ⎥⎦ ⎢⎣ 5 ⎥⎦ ⎢⎣11 3 ⎥⎦
Solution The given system can be written in
matrix notation as i.e., x1 = −9; x2 = 17/3; x3 = 11/3.

AX = B
1.15.3 Method 3: Gauss’s
⎡1 1 2 ⎤ ⎡ x1 ⎤ ⎡ 4⎤ Elimination Method
where A = ⎢⎢ 2 5 −2⎥⎥ ; ⎢ ⎥
X = ⎢ x2 ⎥ ; B = ⎢⎢ 3⎥⎥ This method is simple and general. It consists of
⎣⎢1 7 −7⎥⎦ ⎢⎣ x3 ⎥⎦ ⎣⎢ 5⎦⎥ two steps:
Matrices and Linear Systems of Equations    1-37

Step 1: Reduction of the coeffcient matrix to the assuming that a22 ≠ 0.


upper triangular or echelon form.
The above augmented matrix corresponds to an
Step 2: Finding the values of the unknown vari- upper triangular system which is solved by backward
ables by back substitution. substitution.
This method applies to all types of nonhomo- Example 1.65
geneous systems of equations (NH) whether the Solve the equations 2x + y + z = 10; 3x + 2y + 3z = 18;
coeffcient matrix A is nonsingular or singular. We x + 4y + 9z = 16
explain the method for n = 3 to be simple.
It can be extended to the case when n > 3. Solution The given system of equations can be
The augmented matrix of the system is written in a matrix form as
AX = B (1)
⎡ a11 a12 a13 b1 ⎤
[ A, B] = ⎢⎢ a21 b2 ⎥⎥
where
a22 a23
⎡ 2 1 1⎤ ⎡ x⎤ ⎡10 ⎤
⎣⎢ a31 a32 a33 b3 ⎦⎥
A = ⎢⎢ 3 2 3⎥⎥ ; X = ⎢ y ⎥ ; B = ⎢⎢18⎥⎥
⎢ ⎥ (2)
Assume that a11 ≠ 0. Otherwise, by interchang- ⎢⎣1 4 9⎥⎦ ⎢⎣ z ⎥⎦ ⎢⎣16⎥⎦
ing rows we can achieve this. The first equation
is called the pivotal equation and a11 is called the The augmented matrix of the system is
pivot. ⎡2 1 1 10 ⎤
a
Performing R2 → R2 − 21 R1 and R3 → R3 − 31 R3
a [ A, B] = ⎢⎢ 3 2 3 18⎥⎥
a11 a11 ⎢⎣1 4 9 16⎥⎦
on [ A B] we get
Performing R2 − 3 R1 , R3 −
1 we get
R1
⎡ a11 a12 a13 b1 ⎤ 2 2
⎡2 1 1 10 ⎤
[ A, B] ~ ⎢⎢ 0 a 22 a 23 b2 ⎥⎥
[ A, B] ~ ⎢⎢0 1/ 2 3 / 2 3 ⎥⎥
⎢⎣ 0 a 32 a 33 b3 ⎥⎦
⎣⎢0 7 / 2 17 / 2 11⎦⎥
where
⎡2 1 1 10 ⎤
⎛a ⎞ ⎛a ⎞ R3 − 7 R2 ⎢ ⎥
a 22 = a22 − a12 ⎜ 21 ⎟ ; a 23 = a23 − a13 ⎜ 21 ⎟  ⎢0 1/ 2 3 / 2 3 ⎥
⎝a ⎠ ⎝a ⎠
11 11
⎢⎣0 0 −2 −10 ⎥⎦
⎛a ⎞ ⎛a ⎞
a 32 = a32 − ⎜ 31 ⎟ a12 ; a 33 = a33 − ⎜ 31 ⎟ a13 The system of equations, equivalent to the
⎝ a11 ⎠ ⎝ a11 ⎠
given system and corresponding to the above upper
⎛a ⎞ ⎛a ⎞ triangular matrix, is
b2 = b2 − ⎜ 21 ⎟ b1 ; b3 = b3 − ⎜ 31 ⎟ b1
⎝ a11 ⎠ ⎝ a11 ⎠ 2 x + y + z = 10
a 32 1 3
Now performing R3 → R3 − R1 we get y+ z =3
a 22 2 2
−2 z = −10
⎡ a11 a12 a13 b1 ⎤
[ A, B] ~ ⎢⎢ 0 a 22 a 23 b2 ⎥⎥ By backward substitution
⎢⎣ 0 0 g 33 d 3 ⎥⎦ 10
z=− =5
( −2)
a 32 a
where g 33 = a 33 − a 23 ; d 3 = b3 − 32 b2 y = 6 − 3z = 6 − 3 × 5 = −9
a 22 a 22
1-38    Engineering Mathematics-II

10 − y − z Example 1.67
x=
2 Solve x + y − z = 3, 3x + 2y − 2z = 8, 2x − y − 3z = 7
10 − ( −9) − 5 using the Gauss–Jordan method
= =7
2 Solution

⎡1 1 −1 3⎤ ⎡1 1 −1 3 ⎤
1.15.4 Method 4: Gauss–Jordan ⎢ ⎥ R !− 3R ⎢ ⎥
[ A B] = ⎢ 3 2 −2 8⎥ R2 − 2 R1 ⎢0 −1 1 −1⎥
Elimination Method 3 1
⎢⎣2 −1 −3 7⎥⎦ ⎢⎣0 −3 −1 1 ⎥⎦
The first part of the Gauss–Jordan method consists of
Gauss’s elimination method, which transforms the aug- ⎡1 1 −1 3 ⎤
mented matrix [AB] into [UH] where U is an upper tri- ⎢ ⎥
0 −1 1 −1⎥
R3 − 3R2 ⎢
!
angular matrix and H is the transformed column vector ⎢⎣0 0 −4 4 ⎥⎦
obtained as a result of application of transformations.
The second part consists of reducing the upper R!
1 + R2 ⎡1 0 0 2 ⎤
triangular matrix to a diagonal matrix which in 1
R2 + R3 ⎢ ⎥
turn is reduced to the unit matrix. The matrix now 4 ⎢0 −1 0 0 ⎥
becomes the solution matrix X. 1 ⎢⎣0 0 1 −1⎥⎦
− R3
4
Example 1.66
Solve x + y + z = 1, 2x − y + z = 4, x − 2y − 3z = 0 ⎡1 0 0 2 ⎤
using the Gauss–Jordan method ⎢ ⎥
! ⎢ 0 1 0 0⎥
( −1) R2
⎢⎣0 0 1 −1⎥⎦
Solution
⎡1 1 1 1 ⎤ ⎡1 1 1 1 ⎤ The solution vector is
⎢ ⎥ ⎢ ⎥
[ A B ] = ⎢2 −1 1 4 ⎥ R2 − 2 R1 ⎢0 −3 −1 2 ⎥
⎢⎣1 −2 −3 0 ⎥⎦ R3 − R1 ⎢⎣0 −3 −4 −1⎥⎦ ⎡x⎤ ⎡ 2 ⎤
X = ⎢⎢ y ⎥⎥ = ⎢⎢ 0 ⎥⎥
⎡1 1 1 1 ⎤
 ⎢0 −3 −1 2 ⎥ ⎢⎣ z ⎥⎦ ⎢⎣ −1⎥⎦
R3 − R2 ⎢ ⎥
⎢⎣0 0 −3 −3⎥⎦
Example 1.68
1 ⎡1 0 2 3 5 3⎤ Solve 2x − y + z = 2, x + 2y − z = 3, x + 3z = 5 by
R1 + R2 ⎢ ⎥ the Gauss–Jordan method.
13 ⎢0 −3 −1 2 ⎥
− R3 ⎢⎣0 0 1 1 ⎥⎦
3 Solution The system can be written in a matrix
form AX = B where
 ⎡1 0 0 1 ⎤ ⎡1 0 0 1 ⎤
2
R1 − R3 ⎢ ⎥ 
1 ⎢ ⎥
3 ⎢0 −3 0 3 ⎥ − R2 ⎢0 1 0 −1⎥ ⎡2 −1 1 ⎤ ⎡ x⎤ ⎡ 2⎤
R2 + R3 ⎢⎣0 0 1 1 ⎥⎦ 3 ⎢⎣0 0 1 1 ⎥⎦ A = ⎢⎢1 2 −1⎥⎥ ; X = ⎢ y ⎥ ; B = ⎢⎢ 3⎥⎥
⎢ ⎥
⎢⎣1 0 3 ⎥⎦ ⎢⎣ z ⎥⎦ ⎢⎣ 5⎥⎦
The solution vector is
⎡x⎤ ⎡ 1 ⎤ We find the inverse of the coeffcient matrix by
⎢ y ⎥ = ⎢ −1⎥ the Gauss–Jordan method. Writing A|I and applying
⎢ ⎥ ⎢ ⎥ transformations so that A is transformed into I we
⎢⎣ z ⎥⎦ ⎢⎣ 1 ⎥⎦
obtain A−1 on the RHS
Matrices and Linear Systems of Equations    1-39

⎡2 −1 1 1 0 0⎤ 2x − y + 4z = 0 [JNTU 2002]
⎢ ⎥ x − 11y + 14z = 0
[ A I ] = ⎢1 2 −1 0 1 0 ⎥
⎢⎣1 0 3 0 0 1 ⎥⎦ Solution The system of equations can be written
in matrix form as
⎡1 0 3 0 0 1⎤
R13 ⎢ ⎥ AX = 0
1 2 −1 0 1 0 ⎥
 ⎢
⎢⎣2 −1 1 1 0 0 ⎥⎦ where
R2 − R1 ⎡1 0 3 0 0 1 ⎤ ⎡1 3 −2⎤ ⎡x⎤ ⎡0⎤
⎢ ⎥
R3 − 2 R1 ⎢0 2 −4 0 1 −1⎥ A = ⎢⎢2 −1 4 ⎥⎥ ; X = ⎢⎢ y ⎥⎥ ; O = ⎢⎢0⎥⎥
 ⎢⎣0 −1 −5 1 0 −2⎥⎦ ⎢⎣1 −11 14 ⎥⎦ ⎢⎣ z ⎥⎦ ⎢⎣0⎥⎦
1 ⎡1 0 3 0 0 1 ⎤
R2 ⎢ ⎥ The homogeneous system is always consistent
2 ⎢0 1 −2 0 1 2 − 1 2⎥ (i.e., has a solution) since the rank of the coeffi-
 ⎢⎣0 −1 −5 1 0 −2 ⎥⎦ cient matrix A = the rank of the augmented matrix
R3 + R2 (A|O).
⎡1 0 3 0 0 1 ⎤
⎢ ⎥ By application of elementary transformations

⎢0 1 −2 0 12 − 1 2⎥ on A we can reduce it to echelon form
⎢⎣0 0 −7 1 12 −5 2 ⎥⎦
⎡1 3 −2⎤
⎡1 0 3 0 0 1 ⎤ A ~ ⎢⎢0 −7 8 ⎥⎥ , R2 − 2 R1 , R3 − R1
⎢ ⎥
⎢0 1 −2 0 1 2 −1 2 ⎥ ⎢⎣0 −14 16 ⎥⎦
1
− R3 ⎢0 0 1 − 1 7 1 14 −5 14 ⎥⎦
7 ⎣
⎡1 3 −2⎤
R1 − 3R3 ⎡1 0 0 3 7 ~ ⎢⎢0 −7 8 ⎥⎥ , R3 − 2 R2
3 14 − 1 14 ⎤
 ⎢ ⎥ ⎢⎣0 0 0 ⎥⎦
⎢0 1 0 − 4 14 5 14 3 14 ⎥
R2 + 2 R3 ⎢⎣0 0 1 − 1 7 − 1 14 5 14 ⎥⎦
r(A) = number of nonzero rows = 2 = r(A|O) =
rank of the augmented matrix
⎡ 6 3 −1⎤
< 3 = number of unknowns
= ⎢ −4 5 3 ⎥⎥
−1 1 ⎢
∴ A ∴ The system is consistent and has an infinite
14
⎣⎢ −2 −1 5 ⎥⎦ number of solutions. Now, n − r = 3 − 2 = 1 variable
can be chosen arbitrarily and the others can be written
The solution is
in terms of this.
⎡ 6 3 −1⎤ ⎡2⎤ ⎡ 8 7 The equivalent system of equations is
X = A B = ⎢ −4 5 3 ⎥⎥ ⎢⎢ 3⎥⎥ = ⎢⎢11 7
−1 1 ⎢
14 x + 3y − 2z = 0
⎣⎢ −2 −1 5 ⎥⎦ ⎢⎣5⎥⎦ ⎢⎣ 9 7 −7y + 8z = 0

Taking z = k and by back substitution


1.16 HOMOGENEOUS SYSTEM
OF EQUATIONS (H)
−8 z 8
y= = k
Example 1.69 −7 7
8 10
Solve the system of equations x = 2 z − 3 y = 2k − 3 ⋅ k = − k
x + 3y − 2z = 0 7 7
1-40    Engineering Mathematics-II

Example 1.70 The number of variables n − r = 3 − 1 = 2 can


Show that the system of equations be chosen arbitrarily. The third variable depends on
these values.
2x1 − 2x2 + x3 = lx1
∴ x1 = k1, x2 = k2 and x3 = −k1 + 2k2 where k1,
2x1 − 3x1 + 2x3 = lx2 k2 ∈ ! is a solution set giving an infinite number of
−x1 + 2x2 = lx3 solutions of the system.

possesses a nontrivial solution only if l = 1 and


Case 2
l = −3 and obtain the general solution in each case.
[JNTU 2004] l = −3
The equivalent system of equations is AX = 0
Solution The given system of equations can be
written in matrix form as
⎡ 5 −2 1⎤ ⎡ 5 −2 1 ⎤ R2 − 2 R1 ,
AX = 0 5
A = ⎢ 2 0 2⎥ ~ ⎢0 4 / 5 8 / 5 ⎥⎥
⎢ ⎥ ⎢
(1)
where 1
⎢⎣ −1 2 3⎥⎦ ⎢⎣0 8 / 5 16 / 5⎥⎦ R3 + R1
5
⎡2 − l −2 1⎤ ⎡ x1 ⎤ ⎡0⎤
⎢ ⎢ ⎥ ⎡ 5 −2 1⎤ 5
A= ⎢ 2 −(3 + l ) 2 ⎥ , X = ⎢ x2 ⎥ , O = ⎢0⎥⎥
⎥ ⎢
R , R3 − 2 R2
~ ⎢⎢0 1 2⎥⎥ 4 2
⎢⎣ −1 2 − l ⎦⎥ ⎢⎣ x3 ⎦⎥ ⎣⎢0⎦⎥
(2) ⎣⎢0 0 0⎥⎦ (echelon form of A)

The system has nonzero (nontrivial) solutions if r = r(A) = 2 (number of nonzero rows) < 3
r(A) < 3 = number of unknowns. So, the coefficient (n = number of unknowns); n − r = 3 − 2 = 1; one
matrix A is singular. variable can be chosen arbitrarily.
C1 + C2 + C3 The reduced equivalent system is
⎡2 − l −2 1 ⎤ ⎡1 − l −2 1⎤ ⎡1 −2 1⎤
A = ⎢⎢ 2 −3 − l 2 ⎥⎥ = ⎢⎢1 − l −3 − l 2 ⎥⎥ = (1 − l ) ⎢⎢1 −3 − l 2 ⎥⎥ 5 x1 − 2 x2 + x3 = 0
⎢⎣ −1 2 − l ⎥⎦ ⎢⎣1 − l 2 -l ⎥⎦ ⎢⎣1 2 -l ⎥⎦
x2 + 2 x3 = 0
R2 − R1 , R3 − R1
⎡1 −2 1 ⎤
= (1 − l )[(l + 1) 2 − 4] = − (l − 1) 2 (l + 3) = 0
= (1 − l ) ⎢⎢0 −1 − l 1 ⎥⎥ Choosing x3 = k we get
⇒ l = 1, 1, − 3
⎢⎣0 4 −1 − l ⎥⎦
x2 = −2k ,
Case 1 1 1
x1 = (2 x2 − x3 ) = ( −4k − k ) = − k
l=1 5 5
The equivalent system of equation is (1) AX = 0
⎡ 1 −2 1 ⎤ The solution set is (−k, −2k, k)T for all k ∈ ! .
where A = ⎢⎢ 2 −4 2 ⎥⎥
⎢⎣ −1 2 −1⎥⎦ EXERCISE 1.5
NH System
i.e., x1 − 2x2 + x3 = 0
1. Find for what values of l the equation x + y + z = 1,
2x1 − 4x2 + 2x3 = 0 x + 2y + 4z = l, x + 4y + 10z = l2 have a solution and
−x1 + 2x2 −x3 = 0 solve them in each case.
Ans: (a) l = 1, (x, y, z)T = (2k −1, −3, 1)T
r(A) = 1, one linearly independent row and the
(b) l = 2, (x, y, z)T = (2k −1, −3, 1)T
number of unknowns = n = 3.
Matrices and Linear Systems of Equations    1-41

2. If a + b + c ≠ 0 then show that the system of equations 10. Solve the system of equations x + 3y + 2z = 0,
−2x + y + z = a, x − 2y + z = b, x + y − 2z = c is con- 2x − y + 3z = 0, 3x − 5y + 4z = 0, x + 17y + 4z = 0
sistent. If a + b + c = 0 then show that it has infinitely Ans: (x, y, z) T = k(−11, −1, 7)T
many solutions.
11. Solve the system of equations x + 2y + (2 + k)z = 0;
3. Show that the system of equations x − 3y − 8z = − 10, 2x + (2 + k)y + 4z = 0; 7x + 13y + (18 + k) = 0
3x + y − 4z = 0, 2x + 5y + 6z = 13 is consistent and 4
solve the system. Ans: (a) k ≠ 1, ≠ : x = y = z = 0 (trivial solution)
3
Ans: x = −1 + 2k, y = 3 − 2k, z = k. (k ∈ R) (b) k = 1, ( x, y, z )T = k (1, −2,1)T
4
(c) k = , ( x, y, z )T = k ( −14, −12,3)T
4. For what values of k the equations 4x + y + 10z = k2, 3
2x + y + 4z = k have a solution? Solve them com- 12. Find the values of l for which the equations
pletely for those values. [JNTU 1993]
(l − 1)x + (3l +1)y + 2lz = 0
Ans: k = 0, 2 (l − 1)x + (4l − 2)y + (l + 3)z = 0
(a) k = 0: (x, y, z)T = k(−3, 2, 1)T 2x + (3l + 1)y + 3(l − 1)z = 0
(b) k = 2: (x, y, z)T = (1−3k, 2k, k)T
are consistent and find the ratio x : y : z where l has
5. Solve the equations x + y + z = 6, 3x + 3y + 4z = 20, the smallest of these values. What happens when l
2x + y + 3z = 13 using Gauss’s elimination method has the greatest of these values? [JNTU 2004 S(2)]
Ans: (x, y, z)T = (3, 1, 2)T Ans: (a) l = 0, x = y = z = 0
6. Solve by Gauss’s elimination method 2x + y + z = 10; (b) l = 3, (x, y, z)T = (−5l − 3k, l, k)T
3x + 2y + 3z = 18; x + 4y + 9z = 16
13. Solve completely the following systems of equa-
Ans: (x, y, z)T = (7, −9, 5) tions:

7. Solve by the Gauss–Jordan method 10x + y + z = 12, (a) 4x + 2y + z + 3w = 0, 6x + 3y + 4z + 7w = 0,


x + 10y − z = 10, x − 2y + 6z = 5 2x + y + w = 0 [JNTU 2004 S(2)]

Ans: (x, y, z) T = (1, 1, 1) (b) 3x + 4y − z − w = 0, 2x + 3y + 2z − 3w = 0,


2x + y − 12z − 9w = 0 [JNTU 2002 S(3)]
8. Solve by the method of factorization x + 3y + 8z = 4,
Ans:
x + 4y + 3z = −2, x + 3y + 4z = 1 1 1
1 (a ) ( x, y, z )T = k1 ( − , 0, −1, 1)T + k2 ( − , 1, 0, 0)T
Ans: ( x, y, z )T = (19, −9, 3) 2 2
4 (b) ( x, y, z )T = k ( −119, 87, − 10, 1)T
H System
14. Show that the system of equations 2x1 − 2x2 + x3 =
9. Find the values of l for which the following set of lx1, 2x1 − 3x2 + 2x3 = lx2, −x1 + 2x2 = lx3 can possess
equation may possess a nontrivial solution. 3x + y − nontrivial solutions only if l = 1 and l = −3. Obtain
lz = 0, 4x − 2y − 3z = 0, 2lx + 4y + lz = 0 the general solution in each case.
Ans: l = − 9: (x, y, z)T = k(3, 9, −2)T Ans: l = 1, x1 = 2k − l, x2 = k, x3 = l
l = 1: (x, y, z)T = k(1, −1, 2)T l = −3, x1 = −k, x2 = −2k, x3 = k

 &%#
 
 &$!"# 2
      
  
In this chapter we will consider polynomials whose An n × n real matrix can be considered as a linear
coefficients are not real numbers but n-square transformation from vector space Vn to Vn. The
matrices, find eigenvalues and eigenvectors of a simplest type of linear transformation is the one
matrix, state and prove an important theorem known that merely multiplies all vectors X in Vn by a fixed
as the Cayley–Hamilton Theorem and see how it scalar, i.e., AX = lX for some fixed l. We will now
can be used to find powers of square matrices and deal with complex vector spaces and complex
inverses of nonsingular matrices. scalars; consequently the matrices may have complex
entries also.
 $"'!( ! Consider an n-square matrix
If P0, P1, ..., Pm (≠ 0) are n-square matrices, then an ⎡ a11 a12 ! a1n ⎤
expression of the form ⎢a a22 ! a2n ⎥
A = [aij ] = ⎢ ⎥
21
(2.3)
m ⎢! ! ! ! ⎥
∑ Pr x r 2
= P0 + P1 x + P2 x + ! + Pm x m
(2.1) ⎢
⎣ an1 an2 !

ann ⎦
r=0

is called a matrix polynomial of degree ‘m’. Our aim is to find out whether for some nonzero
vectors X = [x1, x2, …, xn]T it is possible to have
⎡ −1 1⎤ ⎡0 7⎤
E .g . Let P0 = ⎢ ⎥, P1 = ⎢ ⎥, AX = lX (2.4)
⎣ 0 3 ⎦ ⎣ 3 2⎦
⎡ 5 −4 ⎤ for some suitable scalar l, i.e., the linear transformation
P2 = ⎢ ⎥ Y = AX transforms X into a scalar multiple of X, i.e.,
⎣ −3 6 ⎦ X is invariant.
⎡ −1 1⎤ ⎡0 7⎤
Then P0 + P1 x + P2 x 2 = ⎢ ⎥+⎢ ⎥x
⎣ 0 3⎦ ⎣ 3 2⎦
  
  


⎡ 5 −4 ⎤ 2
+⎢ ⎥x Given a square matrix A = [aij]n×n the problem
⎣ −3 6 ⎦
of finding scalars l and nonzero vectors
⎡ −1 + 5 x 2 1 + 7 x − 4 x 2 ⎤
=⎢ ⎥ X = [ x1 , x2 ,! xn ] Tn ×1 which satisfy equation (2.4)
⎢⎣3x − 3x 2 3 + 2 x + 6 x 2 ⎥⎦ is known as the characteristic value Problem. We
(2.2) will now take up the solution of the problem.
is a matrix polynomial of degree 2. From equation (2.4) we have AX − lIX = 0
2-2    Engineering Mathematics-II

⎡ a11 − l a12 ! a1n ⎤ ⎡ x1 ⎤    


⎢ a a22 − l ! a2n ⎥ ⎢ x2 ⎥  
or ⎢ ⎥ ⎢ ⎥ = 0 (2.5)
21
⎢ ! ! ! ! ⎥ ⎢ "⎥  
⎢ ⎥⎢ ⎥ 1. Solve the characteristic equation P(l) =
⎣ an1 an2 ! ann − l ⎦ ⎣ xn ⎦
det(A − lI) = 0 for eigenvalues li. If A is an
This is a system of n homogeneous linear equations n-square matrix then the eigenvalues will
which has a nontrivial solution if be n; repeated roots being counted accord-
P(l) = |A − l I| = 0 ing to their multiplicity.
a11 − l a12 ! a1n 2. Substituting a specific value li solve the
system of linear homogeneous equations
a21 a22 − l ! a2n
or =0 (2.6) (A − li I)Xi = 0 to obtain the corresponding
! ! ! ! nonzero eigenvector Xi .
an1 an2 ! ann − l
Orthogonal matrix: A real square matrix A is called
an orthogonal matrix if AT A = I = AAT or equiva-
  lently AT = A−1.

!A Since 1 = |I| = |AT | = |AT ||A| = |A|2, ∴ |A| = ±1.
Expanding the determinant in (2.6) we get a A is nonsingular and A−1 exists. The linear transfor-
polynomial P(l) of degree n, called the characteristic mation given by Y = AX is called an orthogonal
polynomial of A and therefore (2.6) has n roots, l 1, transformation if the matrix A is an orthogonal
l 2, ..., l n. For each of these l’s equation (2.4) has matrix.
a nontrivial solution; l i are called the characteristic
values or eigenvalues or proper values or latent values !
of matrix A; and each vector is called an eigenvec- Find the eigenvalues and eigenvectors of
tor, characteristic vector or proper vector or latent ⎡ −5 2 ⎤
A= ⎢ ⎥.
vector. Equation (2.6) is called the characteristic ⎣ 2 −2⎦
equation of A.  The characteristic equation of A is
Deg P (l) = order of A −5 − l 2
A − lI = 0 ⇒ =0
 A 2 −2 − l
The set of all eigenvalues of A is called the ⇒ ( −5 − l ) ( −2 − l ) − 4 = 0
spectrum of A. ⇒ l 2 + 7l + 6 = ( l + 1) ( l + 6) = 0
  If the eigenvalues of A are distinct then ⇒ l1 = −1, l2 = −6 (1), (2)
the corresponding eigenvectors are a set of n are the eigen-values of A
linearly independent vectors.
 In the case of repeated eigenvalues we may Let X = [x1, x2]T be the eigenvector corresponding to
not have a linearly independent set of n eigenvectors. eigenvalue l. Then we have to solve
  The algebraic multiplicity of an eigen- ⎡ −5 − l 2 ⎤ ⎡ x1 ⎤ ⎡0⎤
( A − lI ) X = 0 ⇒ ⎢ = (3)
value l is the order of the eigenvalue as a root of the ⎣ 2 −2 − l ⎦⎥ ⎣⎢ x2 ⎦⎥ ⎣⎢ 0⎦⎥
characteristic polynomial (i.e., if l is a double root
then its algebraic multiplicity is 2). For l1 = −1
  The geometric multiplicity of l is the ⎡ −5 +1 2 ⎤ ⎡ x1 ⎤ ⎡0 ⎤ −4 x1 + 2 x2 = 0⎫
number of linearly independent eigenvectors ⎢ 2 ⎥ ⎢ ⎥ =⎢ ⎥ ⇒ ⎬
⎣ −2 +1⎦ ⎣ x2 ⎦ ⎣0 ⎦ 2 x1 − x2 = 0 ⎭
corresponding to l.
Eigenvalues and Eigenvectors    2-3

x1 x2 ⎡ x1 ⎤ ⎡1⎤ 
  From Examples 2.1 and 2.2 we observe
⇒ = = c1 (say ) ⇒ X 1 = ⎢ ⎥ = c1 ⎢ ⎥ (4) that B = 3A. The eigenvalues of B are three times the
1 2 x
⎣ 2⎦ ⎣ 2⎦
eigenvalues of A. But the eigenvectors are same.
is the corresponding eigenvector of A Also, tr. A = sum of eigenvalues and | A | =
Product of eigenvalues.
For l2 = −6

⎡ −5 + 6 2 ⎤ ⎡ x1 ⎤ ⎡0⎤ x1 + 2 x2 = 0 ⎫  

⎢ 2 ⎥ ⎢ x ⎥ = ⎢ 0 ⎥ ⇒ 2 x + 4 x = 0⎬ ⎡1 2⎤
⎣ −2 + 6 ⎦ ⎣ 2⎦ ⎣ ⎦ 1 2 ⎭ Find the eigenvalues and eigenvectors of A = ⎢ ⎥.
⎣1 0⎦
⎡2 4⎤ ⎡3 2 ⎤
x1 x2 ⎡ x1 ⎤ ⎡2⎤
Also find those of B = ⎢
2 0 ⎥ and C = ⎢1 .2⎥ . What
⇒ = = c2 (say ) ⇒ X 2 = ⎢ ⎥ = c2 ⎢ ⎥ (5) ⎣ ⎦ ⎣ ⎦
2 −1 x
⎣ 2⎦ ⎣ −1⎦ do you observe?
is the corresponding eigenvector of A
   The characteristic equation of A is
 
 1− l 2
A − lI = 0 ⇒ =0
Find the eigenvalues and eigenvectors of 1 0−l
⎡ −15 6 ⎤ ⇒ l 2 − l − 2 = ( l + 1)( l − 2) = 0
B=⎢ ⎥.
⎣ 6 −6⎦
⇒ l1 = −1, l2 = 2 are the eigenvalues of A.
   The characteristic equation of B is
Let X = [x1, x2]T be the eigenvector correspond-
B − lI = 0
ing to the eigenvalue l.
= l 2 + 21l + 54 = 0 We have to solve
−15 − l 6
⇒ ⇒ ( l + 3)( l + 18) = 0 ⎡1 − l 2 ⎤ ⎡ x1 ⎤ ⎡0⎤
6 −6 − l ( A − lI ) X = 0 ⇒ ⎢ =
⇒ l1 = −3, l 2 = −18 ⎣ 1 0 − l ⎥⎦ ⎢⎣ x2 ⎥⎦ ⎢⎣0⎥⎦
Let X = [x1, x2]T be the eigenvector corresponding
to eigenvalue l. We have to solve For l1 = −1
⎡ −15 − l 6 ⎤ ⎡ x1 ⎤ ⎡0⎤ ⎡1 + 1 2⎤ ⎡ x1 ⎤ ⎡ 0 ⎤
( B − lI ) X = 0 ⇒ ⎢ = ( A − λI ) X = 0 ⇒ ⎢ ⎥⎢ ⎥=⎢ ⎥
⎣ 6 −6 − l ⎦⎥ ⎣⎢ x2 ⎦⎥ ⎣⎢ 0⎦⎥ ⎣ 1 1⎦ ⎣ x2 ⎦ ⎣ 0 ⎦
2 x1 + 2 x2 = 0
For l = −3 ⇒
x1 + x2 = 0
⎡ −15 + 3 6 ⎤ ⎡ x1 ⎤ ⎡0⎤ ⎧ −12 x1 + 6 x2 = 0
⎢ 6 ⎥ ⎢ ⎥ =⎢ ⎥ ⇒ ⎨ x1 x2 ⎡ x1 ⎤ ⎡1⎤
⎣ −6 + 3⎦ ⎣ x2 ⎦ ⎣0⎦ ⎩ 6 x1 − 3x2 = 0 ⇒ = = c1 ⇒ X1 = ⎢ ⎥ = c1 ⎢ ⎥
x1 x2 ⎡ x1 ⎤ ⎡1⎤
1 −1 ⎣ x2 ⎦ ⎣ −1⎦
⇒ = = c1 (say ) ⇒ X 1 = ⎢ ⎥ = c1 ⎢ ⎥
1 2 ⎣ x2 ⎦ ⎣ 2⎦ is the corresponding eigenvector of A.

For l = −18 For l2 = 2


⎡1 − 2 2 ⎤ ⎡ x1 ⎤ ⎡0⎤ − x1 + 2 x2 = 0
⎡ −15 +18 6 ⎤ ⎡ x1 ⎤ ⎡0⎤ ⎧ 3x1 + 6 x2 = 0 ⎢ = ⇒
⎢ 6 ⎥ ⎢ x ⎥ = ⎢0⎥ ⇒ ⎨6 x + 12 x = 0 ⎣ 1 −2⎦⎥ ⎣⎢ x2 ⎦⎥ ⎣⎢0⎦⎥ x1 − 2 x2 = 0
⎣ −6 +18 ⎦ ⎣ 2⎦ ⎣ ⎦ ⎩ 1 2
x1 x2 ⎡ x1 ⎤ ⎡ 2⎤
x1 x2 ⎡ x1 ⎤ ⎡2⎤ ⇒ = = c2 ⇒ X 2 = ⎢ ⎥ = c2 ⎢ ⎥
⇒ = = c2 (say ) ⇒ X 2 = ⎢ ⎥ = c2 ⎢ ⎥ 2 1 ⎣ x2 ⎦ ⎣1⎦
2 −1 x
⎣ 2⎦ ⎣ −1⎦ is the corresponding eigenvector of A.
2-4    Engineering Mathematics-II

The characteristic equation of B is For l1 = 1


⎡3 −1 2 ⎤ ⎡ x1 ⎤ ⎡0 ⎤ 2 x1 + 2 x2 = 0
2−l 4 ⎢ 1 ⎥ ⎢ ⎥ =⎢ ⎥ ⇒
B − lI = 0 ⇒ =0 ⎣ 2 −1⎦ ⎣ x2 ⎦ ⎣0 ⎦ x1 + x2 = 0
2 0−l
x1 x2
⇒ l 2 − 2l − 8 = ( l + 2)( l − 4) = 0 ⇒ = = c1 (say)
1 −1
T
⇒ l1 = −2, l2 = 4 are the eigenvalues of B. ⇒ X 1 = [x1, x2 ] = c1 [1, −1]T
Let X = [x1, x2]T be the eigenvector correspond-
ing to the eigenvalue l of B. is the corresponding eigenvector of C.
We have to solve (B − lI)X = 0
For l2 = 4
⎡2 − l 4 ⎤ ⎡ x1 ⎤ ⎡0⎤
⎡3 − 4 2 ⎤ ⎡ x1 ⎤ ⎡0 ⎤ − x1 + 2 x2 = 0
⇒ ⎢ 2 =
⎣ − l ⎥⎦ ⎢⎣ x2 ⎥⎦ ⎢⎣0⎥⎦ ⎢ 1 ⎥ ⎢ ⎥ =⎢ ⎥ ⇒
2 − 4 ⎦ ⎣ x2 ⎦ ⎣0 ⎦ x1 − 2 x2 = 0

For l1 = −2
x x
⇒ 1 = 2 = c2 (say)
⎡2 + 2 4⎤ ⎡ x1 ⎤ ⎡0⎤ 4 x1 + 4 x2 = 0 2 1
⎢ 2 ⎥ ⎢ ⎥ =⎢ ⎥ ⇒
⎣ 2⎦ ⎣ x2 ⎦ ⎣0⎦ 2 x1 + 2 x2 = 0 T
⇒ X 2 = [x1, x2 ] = c2 [2, 1]T
x1 x2
⇒ = = c1 (say) ⇒ X 1 = [ x1 , x2 ]T is the corresponding eigenvector of C.
1 −1
Observation
= c1 [1, − 1]T is the corresponding eigenvector
We observe that B = 2A, C = A2.
of B.
The eigenvalues of B are (2l1, 2l2) = (2(−1),
For l2 = 4 2(2)) = (−2, 4).
⎡2 − 4 4 ⎤ ⎡ x1 ⎤ ⎡0 ⎤ −2 x1 + 4 x2 = 0 The eigenvalues of C are (l12, l22) = ((−1)2, 22) =
⎢ 2 ⎥ ⎢ ⎥ =⎢ ⎥ ⇒ (1, 4), where l1, l2 are the eigenvalues of A, while
⎣ −4 ⎦ ⎣ x2 ⎦ ⎣0 ⎦ 2 x1 − 4 x2 = 0
the eigenvectors of A, B, C are the same.
x x
⇒ 1 = 2 = c2 (say)  

2 −1
T Find the eigenvalues and eigenvectors of
⇒ X 2 = [x1, x2 ] = c2 [2, 1]T
⎡2 −2 2 ⎤
is the eigenvector corresponding to eigenvalue A = ⎢⎢1 1 1 ⎥⎥.
l = 4 of A.
⎣⎢1 3 −1⎥⎦
The characteristic equation of C is

  The characteristic equation of A is
3− l 2
C − lI = 0 ⇒ =0 2 − l −2 2
1 2−l
P ( l ) = A − lI = 1 1− l 1
⇒ l 2 − 5l + 6 − 2 = 0 ⇒ ( l − 1) ( l − 4) = 0 1 3 −1 − l
l 1 = 1, l 2 = 4 are the eigenvalues of C. = (2 − l )( l 2 − 4) + 2( −2 − l ) + 2(2 + l ) = 0
Let X = [x1, x2]T be the eigenvector corresponding
to the eigenvalue l of C. = −( l + 2)( l − 2)2 = 0
We have to solve ⇒ l1 = −2, l2 = 2, l3 = 2
⎡3 − l 2 ⎤ ⎡ x1 ⎤ ⎡0⎤ are the eigenvalues of A. To find the eigenvectors we
(C − lI ) X = 0 ⇒ ⎢ 1 =
⎣ 2 − l ⎥⎦ ⎢⎣ x2 ⎥⎦ ⎢⎣0⎥⎦ have to solve (A − lI )X = 0 where X = [x1, x2, x3]T.
Eigenvalues and Eigenvectors    2-5

For l = −2 = (3 − l )( l 2 + 4l − 2)
⎡ 4 −2 2⎤ ⎡ x1 ⎤ ⎡0⎤ + 2( −22 − 4l ) − 5( −6 − 2l )
⎢ 1 3 1⎥ ⎢ x ⎥ = ⎢0⎥ !
⎢ ⎥ ⎢ 2⎥ ⎢ ⎥ = − l 3 − l 2 + 16l − 20 = 0
⎢⎣ 1 3 1⎥⎦ ⎢⎣ x3 ⎥⎦ ⎢⎣0⎥⎦ ⇒ l1 = −5, l2 = 2, l3 = 2
e1 : 2 x1 − x2 + x3 = 0 (1)
e2 : x1 + 3x2 + x3 = 0 (2) 2 −1 −1 16 −20
(e1 ) − (e2 ) : x1 − 4 x2 = 0 (3) −2 −6 20
2 −1 −3 10 0
x1 x2 x3
⇒ = = = c1 (say) −2 −10
4 1 −7 −1 −5 0
X1 = c1 [4 1 −7]
T

For l1 = −5
R1 − R2 , R2 + 2 R3
For l2 = 2
⎡ 8 −2 −5⎤ ⎡ 4 −6 0 ⎤
⎡0 −2 2 ⎤ ⎡ x1 ⎤ ⎡0⎤ ( A − l1I ) = ⎢⎢ 4 4 −5⎥⎥ ∼ ⎢⎢ 0 2 −1⎥⎥
⎢1 −1 1 ⎥ ⎢ x ⎥ = ⎢0⎥
⎢ ⎥ ⎢ 2⎥ ⎢ ⎥ ⎢⎣ −2 −1 2 ⎥⎦ ⎢⎣ −2 −1 2 ⎥⎦
⎢⎣1 3 −3⎥⎦ ⎢⎣ x3 ⎥⎦ ⎢⎣0⎥⎦
− x2 + x3 = 0 (1) The system is equivalent to
⇒ x1 − x2 + x3 = 0 (2)
2 x1 − 3x2 = 0
x1 + 3x2 − 3x3 = 0 (3) x1 x2 x3
2 x2 − x3 = 0 ⇒ = = = c1 (say)
3 2 4
x1 x2 x3 −2 x1 − x2 + 2 x3 = 0
⇒ = = = c2 (say)
0 1 1 ⇒ X 1 = c1 [3 2 4]T
⇒ X 2 = c2 [0 1 1]T
For l2 = 2
Two eigenvalues are equal. ∴ algebraic multi-
R3 − R1 , R2 + 2 R3
plicity = 2; The geometric multiplicity = 1, since there
⎡ 1 −2 −5⎤ ⎡ 1 −2 −5 ⎤
is one eigenvector only corresponding to l2 = l3 = 2.
( A − l2 I ) = ⎢⎢ 4 −3 −5⎥⎥ ! ⎢⎢ 0 −5 −15⎥⎥
 
 ⎢⎣ −2 −1 −5⎥⎦ ⎢⎣ −3 1 0 ⎥⎦
⎡ 3 −2 −5⎤ ⎧ x1 − 2 x2 − 5 x3 = 0⎫
If A = ⎢⎢ 4 −1 −5⎥⎥ , find the eigenvalues and ! ⎪⎨ x2 + 3x3 = 0⎬

⎢⎣ −2 −1 −3⎥⎦ ⎪
⎩ −3x1 + x2 = 0⎪⎭
eigenvectors of A.
x1 x2 x3
⇒ = = = c2 (say)

  The characteristic equation of A is 1 3 −1
P(l) = |A− lI| = 0 ⇒ X 2 = c2 [1, 3, −1]T

3−l −2 −5 The algebraic multiplicity = 2  two eigen-


values are equal and the geometric multiplicity = 1
⇒ 4 −1 − l −5
since there is only one eigenvector corresponding to
−2 −1 −3 − l the eigenvalues l2 = l3 = 2.
2-6    Engineering Mathematics-II

   ⎡1⎤ ⎡2⎤


Find the eigenvalues and eigenvectors of Ans: l1 = 4, l2 = 6; X 1 = ⎢1⎥ ; X 2 = ⎢1 ⎥
⎣⎦ ⎣ ⎦
⎡1 0 0⎤
A = ⎢⎢0 1 0⎥⎥. Find the algebraic and geometric 3. Find the eigenvalues and eigenvectors of matrix
[JNTU 1995, 1998]
⎣⎢0 0 1⎦⎥
multiplicity. ⎡ −2 2 −3⎤ ⎡3 −6 3 ⎤
(a) A = ⎢ 2 1 −6⎥ (b) B = ⎢1 0 −1⎥

 The characteristic equation of A is ⎢ ⎥ ⎢ ⎥


⎢⎣ −1 −2 0 ⎥⎦ ⎢⎣1 2 −3⎥⎦
1− λ 0 0
A − λI = 0 1− λ 0 = (1 − λ )3 = 0 ⎡ 3 1 4⎤ ⎡0 1 0⎤
0 0 1− λ (c) C = ⎢⎢0 2 6⎥⎥ (d) D = ⎢⎢0 0 1⎥⎥
⎢⎣0 0 5⎥⎦ ⎢⎣1 −3 3⎥⎦
⇒ λ 1, λ 2, λ 3 = 1
Algebraic multiplicity = 3 ⎡ 2 0 0⎤
(e) E = ⎢⎢ 3 1 0⎥⎥
l=1
⎢⎣ −1 2 3⎥⎦
⎡1−1 0 0 ⎤ ⎡ x1 ⎤ ⎡0 0 0 ⎤ ⎡ x1 ⎤
⎢ 0 1−1 0 ⎥ ⎢x ⎥ = 0 ⇒ ⎢0 0 0 ⎥ ⎢ x ⎥ = 0
⎢ ⎥ ⎢ 2⎥ ⎢ ⎥ ⎢ 2⎥ Ans: (a) l1 = −3, l2 = −3, l3 = 5;
⎢⎣ 0 0 1−1⎥⎦ ⎢⎣ x3 ⎥⎦ ⎢⎣0 0 0 ⎥⎦ ⎢⎣ x3 ⎥⎦ Algebraic multiplicity = 2;
Geometric multiplicity = 2
⇒ x1, x2, x3 are arbitrary.
⎡ −2⎤ ⎡ 3⎤ ⎡1⎤
⎡1 ⎤ ⎡0 ⎤ ⎡0 ⎤ X 1 = ⎢ 1 ⎥ ; X 2 = ⎢0⎥ ; X 3 = ⎢⎢ 2 ⎥⎥
⎢ ⎥ ⎢ ⎥
X 1 = k1 ⎢⎢0 ⎥⎥ ; X 2 = k2 ⎢⎢1 ⎥⎥ ; X 3 = k3 ⎢⎢0 ⎥⎥ are ⎢⎣ 0 ⎥⎦ ⎢⎣1⎥⎦ ⎢⎣ −1⎥⎦
⎢⎣0 ⎥⎦ ⎢⎣0 ⎥⎦ ⎢⎣1 ⎥⎦ (b) l1 = 0, l2 = 2, l3 = −2;
three L. 1 nonzero eigenvectors corresponding to
⎡1⎤ ⎡3⎤ ⎡0 ⎤
X 1 = ⎢⎢1⎥⎥ ; X 2 = ⎢⎢1⎥⎥ ; X 3 = ⎢⎢1 ⎥⎥
eigenvalue l = 1. ⎢⎣1⎥⎦ ⎢⎣1⎥⎦ ⎢⎣ 2⎥⎦
∴ The geometric multiplicity = 3.
(c) l1 = 2, l2 = 3, l3 = 5;
  
  ⎡1⎤ ⎡1 ⎤ ⎡3⎤
1. Find the eigenvalues and eigenvectors of X 1 = ⎢⎢ −1⎥⎥ ; X 2 = ⎢⎢0 ⎥⎥ ; X 3 = ⎢⎢2 ⎥⎥
⎡1 3 4⎤ ⎣⎢ 0 ⎦⎥ ⎣⎢0 ⎦⎥ ⎢⎣1 ⎦⎥
A = ⎢⎢0 2 5⎥⎥ and those of its inverse. ⎡1⎤
⎢⎥
⎢⎣0 0 3⎥⎦ [JNTU 2000] (d) l1 = 1 = l2 = l3, X 1 = X 2 = X 3 = ⎢1⎥
⎢⎣1⎥⎦
Ans: Eigenvalues of A are 1, 2, 3; those of A−1 (e) l1 = 1, l2 = 2, l3 = 3,
1 1
are 1, , ; eigenvectors of A, A−1 are same
2 3 ⎡0⎤ ⎡1⎤ ⎡0⎤
⎡1 ⎤ ⎡ 3⎤ ⎡19 ⎤ X 1 = ⎢ −1⎥ ; X 2 = ⎢ 3 ⎥ ; X 3 = ⎢⎢0⎥⎥
⎢ ⎥ ⎢ ⎥

X 1 = c1 ⎢⎢0 ⎥⎥ , X 2 = c2 ⎢⎢1 ⎥⎥ , X 3 = c3 = ⎢⎢10 ⎥⎥ ⎢⎣ 1 ⎥⎦ ⎢⎣ −5⎥⎦ ⎢⎣1⎥⎦


⎢⎣0 ⎥⎦ ⎢⎣0 ⎥⎦ ⎢⎣ 2 ⎥⎦ 4. Find the eigenvalues and eigenvectors of
⎡ −3 −7 −5⎤
A = ⎢⎢ 2 4 3 ⎥⎥.
2. Find the eigenvalues and eigenvectors of
⎡8 −4⎤ ⎢⎣ 1 2 2 ⎥⎦
A= ⎢ ⎥. [JNTU 2004 S (2)]
⎣2 2 ⎦
Eigenvalues and Eigenvectors    2-7

Ans: Eigenvalues l1 = l2 = l3 =1; eigenvector The roots of the characteristic equation


⎡ −3⎤ Pn(l) = |A − lI| = 0 are called characteristic roots,
X = ⎢⎢ 1 ⎥⎥ ; Algebraic multiplicity = 3; latent roots, eigenvalues or proper values of the
⎢⎣ 1 ⎥⎦ matrix A.
Geometric multiplicity = 1 1. If A is real then the eigenvalues will be real
5. Find the eigenvalues and eigenvectors of or complex conjugate pairs.
⎡2 2 1⎤ Let l1, l2, …, ln be the eigenvalues of A
A = ⎢⎢ 1 3 1 ⎥⎥ i.e., the roots of the characteristic polynomial
⎢⎣ 1 2 2 ⎥⎦ Pn(l). Then
Ans: l1 = 1, l2 = 1, l3 = 5 Pn(l) = (l1 − l) (l2 − l) … (ln − l)
⎡2⎤ ⎡1⎤ ⎡1⎤ = (−1)n (l − l1) (l − l2) … (l − ln)
X 1 = c1 ⎢⎢ −1⎥⎥ ; X 2 = c2 ⎢⎢ 0 ⎥⎥ ; X 3 = c3 ⎢⎢1⎥⎥ = (−1)n [ln + (−1) (l1 + l2 + … + ln) ln−1
+ … + (−1)n l1⋅l2 … ln]
⎢⎣ 0 ⎥⎦ ⎢⎣ −1⎥⎦ ⎢⎣1⎥⎦
2. The sum of the eigenvalues of a matrix is
the sum of the elements of the principal
6. Find the eigenvalues and eigenvectors of
diagonal.
⎡ 3 10 5 ⎤
A = ⎢⎢ −2 −3 −4⎥⎥ 3. The product of the eigenvalues of a matrix
A is equal to its determinant
⎣⎢ 3 5 7 ⎦⎥
det A = |A| = Pn(0) = l1l2 … ln = Product of
⎡5⎤ ⎡1⎤ the eigenvalues of A
Ans: l1 = 2, l2 = 2, l3 = 3; X 1 = c1 ⎢⎢ 2 ⎥⎥ ; X 3 = c3 ⎢⎢ 1 ⎥⎥
⎢⎣ −5⎥⎦ ⎢⎣ −2⎥⎦ 4. Eigenvalues of AT: Since the diagonal
elements of A and AT are the same and
|A| = |AT|, det | A −lI | = det |AT − lI| and
        hence the eigenvalues of AT and A are same.

   
5. Singular matrix A: If at least one of the
 !#!"# $  Pnl
eigenvalues of a matrix A is zero then
Let A be an n-square matrix. Then the characteristic det A = 0 so that A is a singular matrix.
polynomial of A is Conversely, if A is singular then |A| = 0
which implies that at least one eigenvalue
a11 − l a12 ! a1n of A is zero.
a21 a22 − l ! a2n 6. Nonsingular matrix A: If every eigenvalue
Pn ( l ) = A − lI =
! ! ! ! of a matrix A is nonzero then the matrix A is
an1 an2 ! ann − l nonsingular since det A ≠ 0. Conversely, if A
is nonsingular then |A| ≠ 0 so that none of the
= ( − l )n + p1 ( − l )n −1 + ! + pn −1 ( − l ) + pn eigenvalues of A is zero.
(2.7)
7. Inverse of matrix A: If every eigenvalue of
This is an nth degree polynomial in l, called a matrix A is not zero then A is nonsingular
the characteristic polynomial of matrix A. The so that A−1 exists.
coefficients pi are expressible in terms of ai. In 1 1 1
particular, p1 = coefficient of (−l)n−1 in the expan- Eigenvalues of A−1 are , ,! , if the
l1 l2 ln
sion of |A−lI| = a11+ a22 + … + ann and pn = |A|,
obtained by putting l = 0. eigenvalues of A are l1, l2, …, ln., i.e.,
2-8    Engineering Mathematics-II

the eigenvalues of A−1 are the reciprocals of = (k2 l2 + k1 l + k0) X


those of A.
12. An eigenvector cannot correspond to two
AX = lX ⇒ A−1 ( AX ) = A−1 ( lX ) distinct eigenvalues:
⇒ ( A−1 A) X = l ( A−1 X ) Let X ≠ 0 be the eigenvector corresponding
to two eigenvalues l1, l2 (l1 ≠ l2). Then
1
⇒ IX = l ( A−1 X ) ⇒ A−1 X =
X
l (A − l1I) X = 0 and (A − l2 I) X = 0
8. Spectral shift: If the eigenvalues of A ⇒ (A − l1I) X − (A − l2 I) X = (l1 − l2) IX = 0
are l1, l2, …, ln then the eigenvalues of ⇒ X=0  l1 ≠ l2
(A  kI) are li ± k : and the eigenvectors of
But X ≠ 0. Hence there is a contradiction.
(A  kI) are same as those of A.
This proves that an eigenvector cannot
9. Scalar multiple of A: If the eigenvalues correspond to two distinct eigenvalues of
of A are l1, l2, …, ln then the eigenvalues matrix A.
of a scalar multiple of matrix kA are kli
(i = 1, 2, …, n) 13. Eigenvalues of a diagonal matrix and
triangular matrix:
k(A − lI)X = 0 ⇒ |kA − klI| = 0 Let A = diag [a11, a22, … , ann] (A − lI) =
diag [a11 − l, a22 − l, …, ann − l]
∴ Matrix kA has eigenvalues kli The characteristic polynomial
(i = 1, 2, … ln)
Pn(l) = det (A − lI ) = |A − lI|
10. Powers of A: If the eigenvalues of A are = |diag [a11 − l, a22 − l, …, ann − l]|
l1, l2, … ln then the eigenvalues of Am ∴ characteristic eqn. Pn (l) = 0
(m ∈ N) are lim(i = 1, 2, …, n) ⇒ (a11 − l) (a22 − l) … (ann − l) = 0
Assume that Ak X i = lik X i is true Hence the eigenvalues are the diagonal
elements a11, a22, …, ann.
⇒ A( Ak X i ) = A( lik X i )
⇒ Ak +1 X i = lik ( AX i ) = lik +1 X i 14. Orthogonal matrix and its eigenvalues:
If l is an eigenvalue of an orthogonal
! AX i = li X i . matrix A then its reciprocal 1/l is also an
For m = 1, A1 xi = li1 X i is true eigenvalue of A. By definition, an orthogo-
m nal matrix A is a square matrix such that
By induction A X i = lim X i is true
ATA = I = AAT or AT = A−1. But A and AT
for all m ∈ N . have the same eigenvalues. Also, the eigen-
11. Spectral mapping theorem: If l1, l2, …, ln values of A−1 are the reciprocals of those of
are the eigenvalues of a matrix A then A. Hence if the eigenvalue of A is l then
P ( li ) = km lim + km−1lim−1 + " + k1li + k0 1/l will be the reciprocal of l and will be the
(i = 1, 2, # , n are the eigenvalues of the eigenvalue of AT.
polynomial matrix 15. Orthogonal vectors: Two vectors X and Y
P(A) = kmAm + km−1Am−1 + … + k1A + k0 are said to be orthogonal if X TY = Y T X = 0.

By 9 and 10 above, kAn has eigenvalues 16. Orthonormal set of vectors: A set of
kln, n = 1, 2, …, m. vectors Xi (i = 1, 2, …, n) is said to be an
orthonormal set
For m = 2, (k2A2 + k1A + k0I)X
⎧1 if i = j ⎫
= k2 A2X + k1 A X + k0 I X if X iT X j = d ij = ⎨ ⎬
= k2 l X + k1 l X + k0 I X
2 ⎩0 if i ≠ j ⎭
Eigenvalues and Eigenvectors    2-9

 #  Column or row vectors of an orthogonal B0 − AB1 = p1 I


matrix form an orthogonal system − AB0 = p0 I
Multiplying these equations by An, An−1, …, A; I,
  '
 
  respectively, and adding we get
We now state and prove an important theorem called
the Cayley–Hamilton Theorem. 0 = An + pn−1 An−1 + pn−2 An−2 + p1A + p0 I (2.11)

 !  Every square matrix satisfies its This proves the theorem.
own characteristic equation.
  $!"  #!%&
!  Let A be an n-square matrix and let D(l) be &&'
#  !
the characteristic polynomial of A so that By the Cayley–Hamilton Theorem every square
D(l) = |lI − A| = ln + pn−1ln−1 + pn−2 ln−2 matrix A satisfies its characteristic equation
+ … + p1l + p0 (2.8) An + pn−1 An−1 + pn−2 An−2 + … + p1A + p0 I = 0
Let B(l) be the adjoint of (lI −A). The elements (2.12)
−1
of B(l) are the cofactors of the elements of the A exists ⇔ A is nonsingular
matrix (lI −A) and are polynomials in l of degree ⇔ det A = |A| = D(0) = p0 ≠ 0
not exceeding (n −1). Thus Multiplying (2.12) by A−1
B(l) = Bn−1ln−1 + Bn−2ln−2 + … + B1l + B0 (2.9) A−1 ( An + pn −1 An −1 + pn − 2 An − 2 + ! + p1 A + p0 I ) = 0
where B’s are n-square matrices whose elements are ⇒ An −1 + pn −1 An − 2 + ! + p1 I + p0 A−1 = 0
functions of the elements of A and are independent 1
⇒ A−1 = − ⎡ An −1 + pn −1 An − 2 + ! + p1 I ⎤
of l. We know that p0 ⎣ ⎦
Product of a matrix and its adjoint = determi-
nant of the matrix × Unit matrix (2.13)

(lI − A). adj (lI − A) = |lI − A|I This result gives us the inverse of A in terms
⇒ (lI − A).B(l)= |lI −A|.I of (n −1) powers of A. It is considered as a practical
method for the computation of the inverses of large
From (2.8) and (2.9) we have matrices. As a by-product of the computation, the
(lI − A) (Bn−1ln−1 + Bn−2ln−2 + … + B1l + B0) characteristic equation and the determinant of the
= I (ln + pn−1ln−1 + … + p1l + p0) (2.10) matrix are obtained.

Equating the like powers of l on both sides of &## #  For verification of the
(2.10) we get Cayley–Hamilton Theorem and finding A−1, the char-
Bn−1 = I acteristic equation (2.12) can be written in the form
Bn−2 − ABn−1 = pn−1 I ((… (A + pn−1 I ) A + pn−2 I ) A + … + p1 I )A
+ p0 I = 0 (p0 ≠ 0) (2.14)
Bn−3 − ABn−2 = pn−2 I
or B A = −p0 I (2.15)
… … …
where
B = (… (A + pn−1 I )A + pn−2 I )A + … + p1 I
1
CAYLEY, Arthur (1821–1895) was a professor at Cambridge
⎛ 1⎞
and is known for his important contributions to algebra, matrices ⇒ A−1 = ⎜ − ⎟ B (2.16)
and differential equations. He contributed to the theory of ⎝ p0 ⎠
algebraic invariants and higher dimension and geometry.
2
HAMILTON, William Rowan (1805–1865), a neat Irish math- Equation (2.14) is more convenient for the
ematician, is known for his work in dynamics, is an astronomer verification of the Cayley–Hamilton Theorem; and
and also a physicist. Equation (2.16) for finding A−1.
2-10    Engineering Mathematics-II

     


⎡1 4 ⎤
Verify the Cayley–Hamilton Theorem for A = ⎢ ⎥. ⎡ 2 1 1⎤
⎣ 2 3⎦
Find A−1 and B = A5 − 4A4 − 7A3 + 11A2 − A − 10I. If A = ⎢⎢0 1 0⎥⎥ find A−1 and B = A8 − 5A7 + 7A6
⎢⎣1 1 2⎥⎦
 
  The characteristic equation of A is
− 3A + A4 − 5A3 + 8A2 − 2A + I.
5
⎡1 − l 4 ⎤ 2
P (l ) = ⎢ ⎥ = l − 4l − 5 = 0
⎣ 2 3 − l ⎦  
  The characteristic equation of A is

2−l 1 1
⎡1 4⎤ ⎡1 4⎤ ⎡9 16⎤
A2 = ⎢ = ; P (l ) = 0 1− l 0 = (1 − l )( l 2 − 4l + 3),
⎣2 3⎥⎦ ⎢⎣2 3⎥⎦ ⎢⎣8 17⎥⎦
1 1 2 − l expanding by R2
⎡9 16⎤ ⎡ 4 16⎤ ⎡ 5 0⎤ ⎡0 0⎤
A2 − 4 A − 5I = ⎢ − − = (1)
⎣8 17⎥⎦ ⎢⎣ 8 12⎥⎦ ⎢⎣0 5⎥⎦ ⎢⎣0 0⎥⎦
The Cayley–Hamilton Theorem is verified. = −l3 + 5l2 − 7l + 3 = 0 (2)
−1
det A = P (0) = −5 ≠ 0 ∴ A exists. By the Cayley–Hamilton Theorem, A satisfies
its characteristic equation so that we have
A2 − 4A− 5I = 0
⇒ A (A − 4A − 5I) = A − 4I − 5A−1 = 0
−1 2 A3 − 5A2 + 7A − 3I = 0 (3)
−1
1 1 ⎧ ⎡1 4 ⎤ ⎡ 4 0 ⎤ ⎫ Since det A= P(0) = 3 ≠ 0, A exists.
⇒ A−1 = ( A − 4 I ) = ⎨ ⎢ − ⎬
5 5 ⎩ ⎣ 2 3⎥⎦ ⎢⎣ 0 4⎥⎦ ⎭ Multiplying (3) by A−1

1 ⎡ −3 4 ⎤ ∴ A−1( A3 − 5A2 + 7A − 3I ) = 0
= ⎢ ⇒ A2 − 5A + 7I − 3A−1 = 0
5 ⎣ 2 −1⎥⎦
⇒ 3A−1 = A2 − 5A + 7I
A3 (A2 − 4A − 5I) = A5 − 4A4 − 5A3;
⎡ 2 1 1⎤ ⎡ 2 1 1⎤ ⎡ 5 4 4⎤
−2A(A2 − 4A − 5I) = −2A3 + 8A2 + 10A
3(A2 − 4A − 5I) = (3A2 − 12A − 15I); Now, A = ⎢⎢0 1 0⎥⎥ ⎢⎢0 1
2
0⎥⎥ = ⎢⎢ 0 1 0 ⎥⎥
A + 5I = A + 5I ⎢⎣1 1 2⎥⎦ ⎢⎣1 1 2⎥⎦ ⎢⎣ 4 4 5⎥⎦
Adding we get A3 .0 − 2A.0 + 3.0 + A + 5I = B ⎡ 5 4 4⎤ ⎡10 5 5 ⎤ ⎡ 7 0 0⎤
⎡1 4 ⎤ ⎡ 5 0 ⎤ ⎡6 4 ⎤ ∴ 3 A−1 = ⎢⎢ 0 1 0 ⎥⎥ − ⎢⎢ 0 5 0 ⎥⎥ + ⎢⎢ 0 7 0⎥⎥
∴ B = A + 5I = ⎢ ⎥+⎢ ⎥=⎢ ⎥
⎣ 2 3⎦ ⎣ 0 5⎦ ⎣ 2 8 ⎦ ⎢⎣ 4 4 5⎥⎦ ⎢⎣ 5 5 10⎥⎦ ⎢⎣0 0 7⎥⎦
⎡ 2 −1 −1⎤
Synthetic method For verification of the Cayley–
Hamilton Theorem we have to show that = ⎢⎢ 0 3 0 ⎥⎥
⎣⎢ −1 −1 2 ⎥⎦
5I = A2 − 4 A = ( A − 4 I ) A = BA
⎡ 2 −1 −1⎤
⎡1 4⎤ ⎡1 0⎤ ⎡ −3 4 ⎤ 1⎢
where B = A − 4 I = ⎢ ⎥ − 4 ⎢0 1⎥ = ⎢ 2 −1⎥ A = ⎢ 0 3 0 ⎥⎥
−1
(4)
⎣ 2 3 ⎦ ⎣ ⎦ ⎣ ⎦ 3
⎢⎣ −1 −1 2 ⎥⎦
⎡ −3 4 ⎤ ⎡1 4 ⎤ ⎡ 5 0 ⎤
BA = ⎢ ⎥⎢ ⎥=⎢ ⎥ = 5 I ; and B = A8 − 5 A7 + 7 A6 − 3 A5 + A4 − 5 A3 + 8 A2 − 2 A + I
⎣ 2 −1⎦ ⎣2 3 ⎦ ⎣0 5⎦
the Cayley–Hamilton Theorem is verified = A5 ( A3 − 5 A2 + 7 A − 3I ) + A( A3 − 5 A2 + 7 A − 3I )
1 1 ⎡ −3 4 ⎤ + A2 + A + I
Also, A−1 = B= ⎢
5 5 ⎣ 2 −1⎥⎦ = A5 (0) + A(0) + A2 + A + I
Eigenvalues and Eigenvectors    2-11

⎡ 5 4 4 ⎤ ⎡ 2 1 1 ⎤ ⎡ 1 0 0 ⎤ ⎡ 8 5 5⎤ By the Cayley Hamilton Theorem, A satisfies


= ⎢⎢ 0 1 0 ⎥⎥ + ⎢⎢0 1 0⎥⎥ + ⎢⎢0 1 0⎥⎥ = ⎢⎢0 3 0⎥⎥ its characteristic equation so that
⎢⎣ 4 4 5⎥⎦ ⎢⎣1 1 2⎥⎦ ⎢⎣0 0 1⎥⎦ ⎢⎣ 5 5 8⎥⎦ A3 − 5A2 + 7A − 3I = 0 (3)
(5)

  
  For verification of the Also, det A = P (0) = 3 ≠ 0 and hence A−1 exists.
Cayley–Hamilton Theorem we have to show that Multiplying (1) by A−1

3I = A3 − 5 A2 + 7 A = (( A − 5I ) A + 7 I ) A = BA A−1 (A3 − 5A2 + 7A − 3I ) = 0


⇒ A2 − 5A + 7I − 3A−1 = 0
where B = (( A − 5I ) A + 7 I
⇒ 3A−1 = A2 − 5A + 7I (4)
⎡ −3 1 1 ⎤ ⎡2 1 1⎤ ⎡1 0 0⎤
= ⎢⎢ 0 −4 0 ⎥⎥ ⎢⎢0 1 0⎥⎥ + 7 ⎢⎢0 1 0⎥⎥
⎡7 2 −2⎤ ⎡ 7 2 −2⎤
⎢⎣ 1 1 −3⎥⎦ ⎢⎣1 1 2⎥⎦ ⎢⎣0 0 1⎥⎦
Now, A2 = ⎢⎢ −6 −1 2 ⎥⎥ ⎢⎢ −6 −1 2 ⎥⎥
⎡ −5 −1 −1⎤ ⎡1 0 0⎤
⎢⎣ 6 2 −1⎥⎦ ⎢⎣ 6 2 −1⎥⎦
= ⎢⎢ 0 −4 0 ⎥⎥ + 7 ⎢⎢0 1 0⎥⎥
⎡ 25 8 −8⎤
⎣⎢ −1 −1 −5⎦⎥ ⎣⎢0 0 1⎥⎦
= ⎢⎢ −24 −7 8 ⎥⎥
⎡ 2 −1 −1⎤
⎢⎣ 24 8 −7⎥⎦
= ⎢⎢ 0 3 0 ⎥⎥
⎢⎣ −1 −1 2 ⎥⎦
⎡ 2 −1 −1⎤ ⎡ 2 1 1⎤ ⎡ 3 0 0⎤
⎡ 25 8 −8⎤ ⎡ 35 10 −10⎤
BA = ⎢ 0 3 0 ⎥⎥ ⎢⎢0 1 0⎥⎥ = ⎢⎢0 3 0⎥⎥ = 3I
⎢ 3A−1
= ⎢⎢ −24 −7 8 ⎥⎥ − ⎢⎢ −30 −5 10 ⎥⎥
⎢⎣ −1 −1 2 ⎦⎥ ⎢⎣1 1 2⎥⎦ ⎢⎣0 0 3⎥⎦ ⎢⎣ 24 8 −7⎥⎦ ⎢⎣ 30 10 −5 ⎥⎦
∴ The Cayley–Hamilton Theorem is verified ⎡ 7 0 0⎤ ⎡ −3 −2 2⎤
⎡ 2 −1 −1⎤ + ⎢⎢0 7 0⎥⎥ = ⎢⎢ 6 5 −2⎥⎥ ;
B = ⎢⎢ 0 3 0 ⎥⎥
1 1
Also, A−1 = ⎢⎣0 0 7⎥⎦ ⎢⎣ −6 −2 5 ⎥⎦
3 3
⎢⎣ −1 −1 2 ⎥⎦
⎡ −3 −2 2 ⎤
1⎢
A−1 = 6 5 −2⎥⎥ (5)

 3⎢
⎢⎣ −6 −2 5 ⎥⎦
⎡ 7 2 −2⎤
Find A if A = ⎢⎢ −6 −1 2 ⎥⎥.
−1

 
   For verification of the Cayley–
⎢⎣ 6 2 −1⎥⎦ Hamilton Theorem we have to show that

   The characteristic equation of A is 3I = A3 − 5 A2 + 7 A = (( A − 5I ) A + 7 I ) A = BA


where B = ( A − 5I ) A + 7 I
7−l 2 −2 ⎡2 2 −2⎤ ⎡ 7 2 −2
P ( l ) = −6 −1 − l 2 = ⎢⎢ −6 −6 2 ⎥⎥ ⎢⎢ −6 −1 2 +7I
6 2 −1 − l ⎢⎣ 6 2 −6⎥⎦ ⎢⎣ 6 2 −1
= (7 − l )( l 2 + 2l − 3) + 6( −2 − 2l + 4) −10 −2 2 1 0 0 ⎤ ⎡ −3 −2 2 ⎤
+ 6 {4 + ( −2 − 2l )} (1) = 6 −2 −2 +7 0 1 0 ⎥⎥ = ⎢⎢ 6 5 −2⎥⎥
= −l + 5 l − 7l + 3 = 0
3 2
(2) −6 −2 −2 0 0 1 ⎥⎦ ⎢⎣ −6 −2 5 ⎥⎦
2-12    Engineering Mathematics-II

⎡ −3 −2 2 ⎤ ⎡ 7 2 −2⎤ 5. Show that Adj (A − lI) = B0 + lB1 + l2I where


BA = ⎢⎢ 6 5 −2⎥⎥ ⎢⎢ −6 −1 2 ⎥⎥ = 3I ⎡ −4 4 −2⎤ ⎡ −1 0 2 ⎤
⎢⎣ −6 −2 5 ⎥⎦ ⎢⎣ 6 B0 = ⎢⎢ −2 −2 −2⎥⎥ and B1 = ⎢⎢ 0 1 2 ⎥⎥
2 −1⎥⎦
⎢⎣ 1 −2 1 ⎥⎦ ⎢⎣ −1 2 −2⎥⎦
⎡ −3 −2 2⎤
1 1⎢ ⎡ 1 0 2⎤
−1
Also, A = B = ⎢ 6 5 −2⎥⎥
3 3 if A = ⎢⎢0 1 2⎥⎥
⎢⎣ −6 −2 5 ⎥⎦ ⎢⎣1 2 0⎥⎦

⎡ 2 1 1⎤ 6. State the Cayley–Hamilton Theorem and verify for



  The matrix (Example 2.8) 0 1 0
⎢ ⎥ ⎡3 0 0⎤
⎢ ⎥
⎢⎣1 1 2⎥⎦ A = ⎢⎢5 4 0⎥⎥. [JNTU 1999]
⎢⎣3 6 1⎥⎦
⎡ 7 2 −2⎤
and the matrix (Example 2.9) ⎢⎢ −6 −1 2 ⎥⎥ have 7. Verify the Cayley–Hamilton Theorem for
⎢⎣ −6 2 −1⎥⎦ ⎡3 1 1⎤
the same characteristic equation and the same A = ⎢⎢ −1 5 −1⎥⎥. Find A−1. [JNTU 2002]
eigenvalues. Such matrices are called ‘similar’. We ⎢⎣ 1 −1 3 ⎥⎦
consider similarity of matrices below. ⎡ 7 2 −3⎤
1 ⎢
Ans: A−1 = 10 4 1 ⎥⎥
20 ⎢
   ⎢⎣ −2 2 8 ⎥⎦
1. Using the Cayley–Hamilton Theorem find A8 if
8. Verify the Cayley–Hamilton Theorem for
⎡1 2 ⎤
A= ⎢ ⎥. [JNTU 2003] ⎡1 1 3⎤
⎣2 −1⎦ ⎢
A= ⎢ 1 3 −3⎥⎥. [JNTU November 2003]
Ans: A8 = 625I
⎢⎣ −2 −4 −4⎥⎦
2. Use the Cayley–Hamilton Theorem to find A3 and
9. Verify the Cayley–Hamilton Theorem for
−3 ⎡2 4⎤ ⎡ 2 −1 1 ⎤
A if A = ⎢ ⎥. [JNTU 2002 S]
⎣1 1 ⎦ A = ⎢⎢ −1 2 −1⎥⎥. Find A−1. [JNTU 2000 S, 2002]
⎡28 44⎤ 1 ⎡ −13 44 ⎤ ⎢⎣ 1 −1 1 ⎥⎦
Ans: A3 = ⎢ ⎥ , A−3 = ⎢
⎣11 17 ⎦ 8 ⎣ 11 −28⎥⎦ ⎡ 3 1 −1⎤
1 ⎢
Ans: A = ⎢ 1 3 1 ⎥⎥
−1
3. Use the Cayley–Hamilton Theorem to express 4
⎢⎣ −1 1 3 ⎥⎦
⎡ 3 1⎤
2A5 − 3A4 + A2 − 4I = aA + bI if A = ⎢ ⎥.
⎣ −1 2⎦ 10. Verify the Cayley–Hamilton Theorem for
Ans: a = 138, b = −403 ⎡1 0 3 ⎤
A = ⎢⎢2 1 −1⎥⎥. [JNTU 2002, 1999 S]
⎡1 −2 2⎤ ⎢⎣1 −1 1 ⎥⎦
4. Show that A = ⎢⎢1 2 3⎥⎥ satisfies its characteristic ⎡0 3 3 ⎤
1 ⎢
⎣⎢0 −1 2⎥⎦ Ans: A−1 = 3 2 −7⎥⎥
9 ⎢
equation. Find A−1. [JNTU 2002] ⎢⎣ 3 −1 −1⎥⎦
⎡7 2 −10⎤ ⎡0 c −b⎤
1 ⎢ −c 0 a ⎥⎥.
Ans: A = ⎢ −2 2 −1 ⎥
−1
11. Find the characteristic equation of A = ⎢
9 ⎢ ⎥
⎢⎣ −1 1 4 ⎥⎦ ⎣⎢ b − a 0 ⎥⎦
Characteristic equation is l3  5l2 + 13l  = 0
Eigenvalues and Eigenvectors    2-13

Ans: Characteristic equation is l3 + l (a2 + b2 + c2) = 0; ⎡ 4 1 −1⎤


1 ⎢
⎡ 0 −c b ⎤ Ans: A−1 = −12 2 3 ⎥⎥
5 ⎢
A3 = (a2 + b2 + c2) ⎢⎢ c 0 − a ⎥⎥ ⎢⎣ 5 0 0 ⎥⎦
⎢⎣ − b a 0 ⎥⎦
17. Show that in addition to A, Adj A also satisfies
⎡ 0 3 −2⎤
⎡ −4 −3 −3⎤
12. Find the characteristic equation of A = ⎢⎢ −3 0 1 ⎥⎥
the characteristic equation for A = ⎢⎢ 1 0 1 ⎥⎥.
⎢⎣ 2 −1 0 ⎥⎦
⎢⎣ 4 4 3 ⎥⎦
. Verify that A satisfies it. [Hint: Adj A = A.]
Ans: Characteristic equation is l3 + 14l = 0; ⎡ a 0 0⎤
⎡ 0 −3 2 ⎤ 18. Show that A = ⎢⎢ 0 b 0⎥⎥ satisfies its characteristic
A3 = 14 ⎢⎢ 3 0 −1⎥⎥ ⎢⎣ d 0 c ⎥⎦
⎢⎣ −2 1 0 ⎥⎦ equation
13. By applying the Cayley–Hamilton Theorem to p(l) = s0s1 − (s0 + s1s2)l + (s1 + s2)l2 − l3 = 0
⎡7 −1 3⎤ where s0 = bc, s1 = a + d and s2 = b + c.
A = ⎢⎢6 1 4 ⎥⎥, find A−1.
19. Verify the Cayley–Hamilton Theorem for
⎣⎢2 4 8 ⎦⎥
⎡ 2 −1 1 ⎤
⎡ −1.60 0.40 −0.14 ⎤ A = ⎢⎢ −1 2 −1⎥⎥. Hence compute A−1.
Ans: A−1 = ⎢ − 0.80 1.00 −0.20 ⎥
⎢ ⎥ ⎣⎢ 1 −1 2 ⎥⎦
⎢⎣ 0.44 −0.60 0.26⎥⎦ ⎡ 3 1 −1⎤ ; Characteristic equation is
1
Ans: A−1 = ⎢⎢ 1 3 1 ⎥⎥ l 6l + 9l 4 = 0
3 2
14. Verify the Cayley–Hamilton Theorem and hence find
4
⎡0 1 0 0⎤ ⎢⎣ −1 1 3 ⎥⎦
⎢0 0 0 1⎥
A−1 if A = ⎢ ⎥
⎡ 1 3 7⎤
⎢0 0 1 0⎥
⎢ ⎥ 20. Find the characterstic equation of A = ⎢⎢ 4 2 3⎥⎥.
⎣1 0 0 0⎦
⎢⎣ 1 2 1⎥⎦
Ans: Characteristic equation is l4 − l3 − l + 1 = 0
−1
⎡0 0 0 1⎤ Show that it is satisfied by A. Find A .
⎢1 0 0 0⎥
Ans: l 3 − 4l 2 − 20l − 35 = 0;
A−1 = ⎢ ⎥
⎢0 0 1 0⎥ ⎡ −4 11 −5 ⎤
⎢ ⎥ 1 ⎢
⎣0 1 0 0⎦ A−1 = −1 −6 25 ⎥⎥
35 ⎢
15. Using the Cayley–Hamilton Theorem find A−1 and A4 ⎢⎣ 6 1 −10⎥⎦
⎡ 7 2 −2⎤
A = ⎢ −6 −1 2 ⎥ 21. Verify the Cayley–Hamilton Theorem for A if
if ⎢ ⎥. [JNTU 2000]
⎢⎣ 6 2 −1⎥⎦ ⎡1 2 3 ⎤
−1
A = ⎢⎢2 −1 4 ⎥⎥; and find A
⎡ −3 −2 2 ⎤ ⎡ 241 80 −80 ⎤
1 ⎢ ⎥ ; A4 = ⎢ −240 −79 80 ⎥ ⎣⎢ 3 1 −1⎦⎥
Ans: A −1 = 6 5 −2
3 ⎢ ⎥ ⎢ ⎥ Ans: Characteristic equation is l3 + l2 − 18l − 40 = 0
⎣⎢ −6 −2 5 ⎥⎦ ⎣⎢ 240 80 −79⎦⎥
⎡ −3 5 11 ⎤
16. By the Cayley–Hamilton Theorem find A−1 if 1 ⎢
A−1 = 14 −10 2 ⎥⎥
⎡0 0 1⎤ 40 ⎢
⎢⎣ 5 5 −5⎥⎦
A = ⎢⎢ 3 1 0 ⎥⎥. [JNTU 1993]
⎢⎣ −2 1 4⎥⎦
2-14    Engineering Mathematics-II

22. Using the Cayley–Hamilton Theorem find A−1 if ⎡ x11 x12 ! x1 j ! x1n ⎤ ⎡ l1 0 ! 0⎤
⎡1 2 1 ⎤ ⎢x x2n ⎥ ⎢ 0 l2
x22 ! x2 j ! ! 0⎥
=⎢ ⎥ ⎢
21
A = ⎢⎢ 2 3 5 ⎥⎥ . ⎥
⎢! ! ! ! ! ! ⎥ ⎢! ! ! ! ⎥
⎣⎢0 −1 4 ⎦⎥ ⎢ ⎥ ⎢
xn2 ! xnj ! xnn ⎦⎥ ⎣ 0 0

⎣⎢ xn1 ! ln ⎦
⎡ −17 9 −7⎤
Ans: A−1 = ⎢ 8 = PD where D = diag [l1, l2, … ln]
−4 3 ⎥⎥
⎢ ⇒ P−1 AP = P−1 PD = ID = D
⎢⎣ 2 −1 1 ⎥⎦
23. Using the Cayley–Hamilton Theorem find A−1 if
 %"*(!-$&*("
⎡ 4 1 2⎤
*(!-%'+(*(!-A
A = ⎢⎢ 1 2 0⎥⎥.
⎢⎣ 0 −1 3⎥⎦
The matrix P which diagonalises A is called a modal
matrix of A and the resulting diagonal matrix D is
⎡ 6 −5 −4⎤ known as the spectral matrix of A.
1 ⎢
Ans: A−1 = −3 12 2 ⎥⎥
19 ⎢ The matrix P is found by grouping the eigen-
⎢⎣ −1 4 7 ⎥⎦ vectors of A into a square matrix and the diagonal
⎡ 0 w 2 −w ⎤ matrix has the eigenvalues of A as the principal
⎢ 2 ⎥ diagonal elements. If n = 1 then P does not exist.
24. If A = ⎢ −w 0 1 ⎥ where w is a cube root of
⎢ ⎥
w −1 0 ⎥
⎣⎢ ⎦  !#!"(!*.%*(!)
unity, by using the Cayley–Hamilton Theorem show
that A is nilpotent of index 3. Let A be a square matrix of order n. A square
matrix B of order n is said to be similar to A if there
exists a nonsingular matrix P such that AP = PB or
 
   
B = P−1 AP. In such a case we say that A and B are

  
similar matrices.
 
!%$"!)*!%$/%,() This relation defines a transformation of a
%'+(*(!-A matrix A into another matrix B and the transforma-
tion is called a similarity transformation.
 %(#  If a square matrix A of order n Similarity of matrices is an equivalence relation
has n (≥2) linearly independent eigenvectors, then on the set of n-square matrices.
a nonsingular matrix P can be found such that
P−1 AP is a diagonal matrix.  %(#  Similar matrices have the same
eigenvalues.
(%% Let X1, X2, …, Xn be the n ≥ 2 linearly
independent eigenvectors of A corresponding to its (%% Let B be similar to matrix A. Then there
n eigenvalues l1, l2, …, ln. exists a nonsingular matrix P such that B = P−1 AP.
Construct matrix P with X1, X2, …, Xn as n Consider the characteristic polynomial of B
column vectors.
Let Pn×n = [X1, X2 … Xn] where Xj = [xij], |B − lI| = |P−1AP − lI| = |P−1AP − lP−1IP|
i = 1, 2, …, n. Since X1, X2, …, Xn are linearly indepen- = |P −1(A − lI)P | = |P −1|| A − lI ||P|
dent P is nonsingular and therefore P−1 exists. | A − lI |,
Consider AP = A [X1, X2 … Xn] = [AX1, AX2 … since |P −1||P | = |P −1P| = |I| =1
AXn] = [l1X1, l2X2 … lnXn]
Thus A and B have the same characteristic
⎡ l1 x11 l2 x12 ! l j x1 j ! ln x1n ⎤ polynomial and hence the same eigenvalues.
⎢l x l2 x22 ! l j x2 j ! ln x2n ⎥
=⎢ ⎥
1 21  %(# If X is an eigenvector of a matrix A
⎢ ! ! ! ! ! ! ⎥ corresponding to an eigenvalue l of A then the eigen-
⎢ ⎥
⎢⎣ l1 xn1 l 2 xn 2 ! l j xnj ! ln xnn ⎥⎦ vector of B, which is similar to A, is Y = P−1X.
Eigenvalues and Eigenvectors    2-15

 Let X be an eigenvector corresponding to l=1


an eigenvalue l of A so that AX = lX.
We have to solve (A − I)X = 0
Since B is similar to A we have B = P−1AP for
some nonsingular matrix P. Now ⎡0 1⎤ ⎡ x1 ⎤ ⎡0⎤
BP−1 = (P−1AP) P−1 = P−1A (PP−1) = P−1 A; ⇒ ⎢0 0⎥ ⎢ x ⎥ = ⎢0⎥ ⇒ 0.x1 + 1.x2 = 0
post-multiplying this by X, ⎣ ⎦ ⎣ 2⎦ ⎣ ⎦
x x ⎡1⎤
(BP−1)X = (P−1A) X = P−1(AX) = P−1(lX) Solving, we get = 2 ⇒ X1 = c1 ⎢ ⎥
= l(P−1X) ⇒ B(P−1X) = l(P−1X)
1 0 ⎣0⎦
We cannot find a second L.I. eigenvector.
Thus P−1X is an eigenvector of B corresponding
to the eigenvalue l. Algebraic multiplicity of (l = 1) = 2; Geometric
Multiplicity of (l = 1) = 1
    ∴ The matrix is not diagonalisable.
 


A
⎡ −2 2 −3⎤
The existence of distinct eigenvalues is only a ⎢ ⎥
Diagonalise the matrix A = ⎢ 2 1 −6⎥.
sufficient condition but not a necessary condition for
a square matrix A to be diagonalisable. ⎢⎣ −1 −2 0 ⎥⎦
The real condition for diagonalisability of
a square matrix A of order n is the existence of n  The characteristic equation of A is
linearly independent eigenvectors, which may exist |A − lI| = 0
even if some of the eigenvalues of a matrix are −2 − l 2 −3
repeated (algebraic multiplicity of an eigenvalue is ⇒ 2 1− l −6
greater then 1).
We know that, in general, −1 −2 0 − l
Algebraic multiplicity (A.M.) ≥ Geometic 1− l −6 2 −3 2 −3
multiplicity (G.M.) of eigenvalue l of a matrix A. = ( − 2 − l) −2 −1
−2 −l −2 − l 1− l −6
If G.M. < A.M. of an eigenvalue in respect of
a square matrix A then we cannot find n linearly = − l 3 − l 2 + 21l + 45 = 0
independent eigenvectors and hence A is not Characteristic roots are l1 = −3, l2 = −3, l3 = 5.
diagonalisable. To find eigenvectors we have to solve
If G.M. and A.M. of an eigenvalue in respect (A − lI )X = 0.
of a square matrix A are equal then we can find n
l = −3
linearly independent eigenvectors and hence A is
diagonalisable. ⎡ −2 + 3 2 −3⎤

(A − lI ) = ⎢ 2 1 + 3 −6⎥⎥


⎢⎣ −1 −2 3 ⎥⎦
⎡1 1⎤
Prove that A = ⎢ ⎥ is not diagonalisable. ⎡ 1 2 −3⎤ ⎡1 2 −3⎤
⎣0 1⎦ R2 − 2 R1
= ⎢⎢ 2 4 −6⎥⎥ ! ⎢⎢0 0 0 ⎥⎥
 The characteristic equation is |A − lI| R + R1
⎢⎣ −1 −2 3 ⎥⎦ ⎢⎣0 0 0 ⎥⎦ 3
⎡1 − l 1 ⎤
=⎢ = (l −1)2 = 0 ⇒ λ = 1, 1 Equivalent equation x1 + 2x2 − 3x3 = 0
⎣ 0 1 − l ⎥⎦
Since number of unknowns n = 3⎫
To find the eigenvectors we have to solve ⎬ n − r = 3 −1 = 2
and rank r = 1 ⎭
(A − lI) X = 0 for l = 1, 1 unknowns can be chosen arbitrarily.
2-16    Engineering Mathematics-II

We choose the eigenvectors so that they are L.I. Such a matrix is called an orthogonal matrix. It

x1 x x x
= 2 = 3 ; 1 = 2 = 3
x x satisfies PTP = I or P−1 = PT. The diagonalisation
2 −1 0 3 0 1 of a square matrix A by an orthogonal matrix P is
X1 = [ 2 −1 0 ] ; X 2 = [3 0 1]T
T called orthogonalisation of A.
taking c1 = c2 = 1   
l=5 Find a modal matrix (orthogonal matrix) P and
⎡ −2 − 5 2 −3⎤ ⎡1 1⎤
(A − lI ) = ⎢⎢ 2 1 − 5 −6⎥⎥ diagonalise the symmetric matrix A = ⎢ ⎥.
⎣1 1⎦
⎢⎣ −1 −2 −5⎥⎦

 The characteristic equation of A is
R2 + 2 R3 , R1 − 7 R3
⎡ −7 2 −3⎤ ⎡ 0 16 32 ⎤ 1− l 1
P (l ) = = (1 − l )2 − 1
= ⎢⎢ 2 −4 −6⎥⎥ ! ⎢⎢ 0 −8 −16⎥⎥ 1 1− l
⎣⎢ −1 −2 −5⎥⎦ ⎢⎣ −1 −2 −5 ⎦⎥ = l ( l − 2) = 0 ⇒ l1 = 0, l2 = 2
⎧ x2 + 2 x3 = 0 x x x l1 = 0
⇒ ( A − 5I ) X = 0 ⎨ ⇒ 1 = 2 = 3
x
⎩ 1 3+ x = 0 1 2 − 1
⎡1 1⎤ ⎡ x1 ⎤ ⎡0⎤
⇒ X 3 = [1 2 − 1]T ⎢1 1⎥ ⎢ x ⎥ = ⎢0⎥ ! x1 + x2 = 0
⎣ ⎦ ⎣ 2⎦ ⎣ ⎦
Modal matrix:
⎡ 2 3 1 ⎤ ⎡ a1 b1 c1 ⎤ x1 x2 ⎡1⎤
⇒ = = c1 ; X 1 = c1 ⎢ ⎥
P = ⎢⎢ −1 0 2 ⎥⎥ = ⎢⎢ a2 b2 c2 ⎥⎥ (say); 1 1 ⎣ −1⎦
⎣⎢ 0 1 −1⎦⎥ ⎢⎣ a3 b3 c3 ⎥⎦
l2 = 2
⎡ A1 A2 A3 ⎤ ⎡ −2 4 6 ⎤
Adj P = ⎢⎢ B1 B2 B3 ⎥⎥ = ⎢⎢ −1 −2 −5⎥⎥ ⎡ −1 1 ⎤ ⎡ x1 ⎤ ⎡ 0 ⎤
⎢ 1 −1⎥ ⎢ x ⎥ = ⎢0 ⎥ ∼ − x1 + x2 = 0
⎢⎣C1 C2 C3 ⎥⎦ ⎢⎣ −1 −2 3 ⎥⎦ ⎣ ⎦ ⎣ 2⎦ ⎣ ⎦
P = a1 A1 + a2 A2 + a3 A3 = 2( −2) + ( −1)4 + 0 = −8 x x ⎡1⎤
⇒ 1 = 2 = c2 ; X 2 = c2 ⎢ ⎥
1 1 ⎣1⎦
⎡2 −4 −6⎤
1 ⎢
∴ P = ⎢1 2 5 ⎥⎥
−1
8
⎢⎣1 2 −3⎥⎦ ⎡1⎤ ⎡1⎤
The eigenvectors X1 = ⎢ ⎥ and X 2 = ⎢ ⎥ (c1 = c2 = 1)
⎡ −3 0 0⎤ ⎣ −1⎦ ⎣1⎦
⎡1⎤
⇒ P −1 AP = D = ⎢⎢ 0 −3 0⎥⎥ are orthogonal.  X1T . X 2 = [1, − 1] ⎢ ⎥ = 0
⎢⎣ 0 0 5⎥⎦ ⎣1⎦
We construct modal matrix P with orthonormal
X X2
      vectors 1 and .
X1 X2

   
Let A be a real symmetric matrix with n distinct ⎡ 1 1 ⎤ ⎡ 1 −1 ⎤
eigenvalues l1, l2, …, ln. Then the correspond- ⎢ 2 2 ⎥ ⎢ 2 2⎥
ing eigenvectors are pairwise orthogonal. One can P=⎢ ⎥ ; P −1 = P T = ⎢ ⎥
⎢ −1 1 ⎥ ⎢ 1 1 ⎥
construct modal matrix P with the normalised eigen- ⎢ 2
⎣ 2 ⎥⎦ ⎢ 2
⎣ 2 ⎥⎦
⎛ X X X ⎞
vectors, ⎜ 1 , 2 , " , n ,⎟ as its columns. T
(P is an orthogonal matrix satisfying P P = I or
⎝ X1 X 2 Xn ⎠ P−1 = PT.)
Eigenvalues and Eigenvectors    2-17

Now 1x1 + 0. x2 + 0. x3 = 0
⎡ 1 −1 ⎤ ⎡ 1 1 ⎤
∼ 0. x1 + 1x2 − x3 = 0
⎢ 2 ⎥
2 ⎡1 1⎤ 2 ⎢ 2⎥
P −1 AP = ⎢ ⎥⎢ ⎥ ⎢ ⎥ 0. x1 − x2 + x3 = 0
⎢ 1 1 ⎥ ⎣1 1⎦ ⎢ −1 1 ⎥
⎢ ⇒ x1 = 0, x2 = x3 = c2
⎣ 2 2 ⎥⎦ ⎢ 2
⎣ 2 ⎥⎦
−1⎤ ⎡1 1⎤ ⎡ 1 1⎤ ⇒ X2 = c2 [ 0 1 1 ]T
1 ⎡1
= ⎢
2 ⎣1 1 ⎥⎦ ⎢⎣1 1⎥⎦ ⎢⎣ −1 1⎥⎦
1 ⎡0 0⎤ ⎡ 1 1⎤ 1 ⎡0 0 ⎤ ⎡0 0⎤ For l = 4
= ⎢ 2 2⎥ ⎢ −1 1⎥ = 2 ⎢0 4⎥ = ⎢0 2⎥ We have to solve (A − 4I)X = 0
2 ⎣ ⎦⎣ ⎦ ⎣ ⎦ ⎣ ⎦
=D ⎡ −3 0 0 ⎤ ⎡ x1 ⎤ ⎡0 ⎤
Along the main diagonal of D we find the eigen- ⇒ ⎢⎢ 0 −1 −1⎥⎥ ⎢⎢ x2 ⎥⎥ = ⎢⎢0 ⎥⎥
values of A. ⎢⎣ 0 −1 −1⎥⎦ ⎢⎣ x3 ⎥⎦ ⎢⎣0 ⎥⎦
  
Find a modal matrix (orthogonal matrix) P and −3 x1 + 0. x2 + 0. x3 = 0

⎡1 0 0⎤ 0. x1 − x2 + x3 = 0
diagonalise the symmetric matrix A = ⎢⎢0 3 −1⎥⎥.
⇒ x1 = 0, x2 = −x3 = c3
⎣⎢0 −1 3 ⎦⎥
⇒ X3 = c3 [ 0 1 −1 ]T
 
  The characteristic equation of A is
⎡1 − l 0 0 ⎤
A − l I = ⎢⎢ 0 3− l −1 ⎥⎥ = (1 − l )( l 2 − 6l + 8) Normalising these mutually orthogonal vectors
⎡ X X2 X3 ⎤
⎣⎢ 0 −1 3 − l ⎦⎥
we write P = ⎢ 1 ⎥
= (1 − l )( l − 2)( l − 4) = 0 ⎣⎢ X1 X 2 X 3 ⎦⎥
⇒ l = 1, 2, 4
To find the eigenvectors corresponding to the ⎡ ⎤
eigenvalue l we have to solve (A − lI)X = 0. ⎢1 0 0 ⎥
⎢ ⎥
For l = 1 ⎢ 1 1 ⎥
or P = ⎢0
We have to solve (A − I)X = 0
⎢ 2 2 ⎥⎥
⎡0 0 0 ⎤ ⎡ x1 ⎤ ⎡0 ⎤ ⎢ 1 1 ⎥

⇒ ⎢⎢0 2 −1⎥⎥ ⎢⎢ x2 ⎥⎥ = ⎢⎢0 ⎥⎥ ⎢0
⎣ 2 2 ⎥⎦
⎢⎣0 −1 2 ⎥⎦ ⎢⎣ x3 ⎥⎦ ⎢⎣0 ⎥⎦
P −1 = P T = P
0. x1 + 0. x2 + 0. x3 = 0
∼ 0. x1 + 2 x2 − x3 = 0 ( P is symmetric) P1 AP
0. x1 + ( −1) x2 + 2 x3 = 0 ⎡ ⎤
⎡ ⎤
⇒ x2 = 0, x3 = 0 and x1 = c1 (arbitrary) ⎢1 0 0 ⎥ ⎢1 0 0 ⎥
⇒ X1 = c1 [ 1 0 0 ]T ⎢ ⎥ ⎡1 0 0 ⎤ ⎢ ⎥
⎢ 1 ⎥⎢ ⎢ 1 1 ⎥
0 3 −1⎥⎥ ⎢0
1
For l = 2 = ⎢0
2 ⎥
2 ⎥ ⎢ 2 2 ⎥⎥
We have to solve (A − 2I)X = 0 ⎢ ⎢0 −1 3 ⎥⎦ ⎢
⎢ 1 1 ⎥⎣ ⎢ 1 1 ⎥
⎡ −1 0 0 ⎤ ⎡ x1 ⎤ ⎡0 ⎤ ⎢0 − ⎢0 −
⎥ 2 ⎥⎦
⇒ ⎢⎢ 0 1 −1⎥⎥ ⎢⎢ x2 ⎥⎥ = ⎢⎢0 ⎥⎥ ⎣ 2⎦ ⎣
2 2

⎣⎢ 0 −1 1 ⎦⎥ ⎣⎢ x3 ⎦⎥ ⎣⎢0 ⎦⎥ = diag [1, 2, 4] = D


2-18    Engineering Mathematics-II

 
 
 A& ∴ The modal matrix which diagonalises A is
     P ⎡ 1 1⎤.
 P=⎢ ⎥
  

 A  ⎣ −2 1⎦
Let A be an n-square matrix and D, the diagonal
matrix similar to A. So, D = P−1 AP Verification

Then D2 = (P−1AP) (P−1AP) = P−1A (PP−1) AP 1 ⎡ 1 1⎤


= P−1AI AP = P−1AAP = P−1A2P P −1 =
3 ⎢⎣ −2 1⎥⎦
⇒ Dn = P−1 An P, by induction
1 ⎡ 1 1⎤ ⎡ 4 1⎤ ⎡ 1 1⎤
∴ PDnP−1 = P(P−1AnP)P−1 = (PP−1) An (PP−1) ⇒ P −1 AP =
3 ⎢⎣ −2 1⎥⎦ ⎢⎣ 2 3⎥⎦ ⎢⎣ −2 1⎥⎦
= IAnI = An
or A = PDnP−1
n
1 ⎡ 2 −2⎤ ⎡ 1 1⎤
= ⎢
3 ⎣0 5 ⎥⎦ ⎢⎣ −2 1⎥⎦

%"
1 ⎡6 0 ⎤ ⎡ 2 0⎤
Find a matrix P which diagonalises the matrix = ⎢ = = D,
⎡ 4 1⎤ 3 ⎣0 15⎥⎦ ⎢⎣0 5⎥⎦
−1
A= ⎢ ⎥. Verify that P AP = D, where D is a
⎣ 2 3⎦ the diagonal matrix containing the eigenvalues 2 and
diagonal matrix. Hence find A6. 5 of A as the principal diagonal elements.
!$#!  The matrix A can be diagonalised by
P whose columns are linearly independent eigen- To find A6
vectors of A, if A has at least two linearly indepen-
1 ⎡ 1 1⎤ ⎡2 0 ⎤ ⎡1 −1⎤
6
dent eigenvectors. A6 = PD 6 P −1 = ⎢ ⎥
To find the eigenvectors of A we write the 3 ⎣ −2 1⎦ ⎢⎣ 0 56 ⎥⎦ ⎢⎣2 1 ⎥⎦
⎢ ⎥

characteristic equation of A. 1 ⎡ 1 1⎤ ⎡64 0 ⎤ ⎡1 −1⎤


= ⎢
4−l 1 3 ⎣ −2 1⎦ ⎣ 0 6250⎥⎦ ⎢⎣2 1 ⎥⎦
⎥ ⎢
P ( l ) = A − lI =
2 3− l 1 ⎡ 64 15625 ⎤ ⎡1 −1⎤
= ⎢
= l 2 − 7l + 10 = ( l − 2)( l − 5) = 0; 3 ⎣ −128 115625⎥⎦ ⎢⎣2 1 ⎥⎦
l = 2, 5 are two distinct eigenvalues of A. 1 ⎡31314 15561⎤ ⎡10438 5187⎤
= =
3 ⎢⎣31122 15753⎥⎦ ⎢⎣10374 5251⎥⎦
⎡2 1⎤ ⎡ x1 ⎤
l=2:⎢ ⎥⎢ ⎥=0 ⇒ 2 x1 + x2 = 0
⎣2 1⎦ ⎣ x2 ⎦
%"
x1 x2 ⎡1⎤
⇒ = ⇒ X 1 = c1 ⎢ ⎥ ⎡1 6 1⎤
1 −2 ⎣ −2⎦ Diagonalise A = ⎢⎢1 2 0⎥⎥ and hence find A8. Find
⎡ −1 1 ⎤ ⎡ x1 ⎤ ⎢⎣0 0 3⎥⎦
l=5:⎢ ⎥⎢ ⎥ = 0 ⇒ x1 + x2 = 0 the modal matrix.
⎣ 2 −2⎦ ⎣ x2 ⎦
x1 x2 ⎡1⎤ !$#! The matrix A can be diagonalised by
⇒ = ⇒ X 2 = c2 ⎢ ⎥
1 1 ⎣1⎦ constructing the modal matrix P whose column
vectors are three linearly independent eigenvectors
⎡1⎤ ⎡1⎤ of A.
X 1 = ⎢ ⎥ and X 2 = ⎢ ⎥ are two linearly
⎣ −2⎦ ⎣1⎦ To find the eigenvalues of A we write the
independent eigenvectors of A. characteristic equation of A.
Eigenvalues and Eigenvectors    2-19

⎡1 − l 6 1 ⎤ Diagonalisation of matrix A

P (l ) = ⎢ 1 2−l 0 ⎥⎥ D = P −1 AP
⎣⎢ 0 0 3 − l ⎦⎥ ⎡ −4 8 1 ⎤ ⎡1 6 1 ⎤ ⎡ −3 1 2 ⎤
1 ⎢
2
= (3 − l )( l − 3l − 4) = 0 0 −5⎥⎥ ⎢⎢1 2 0 ⎥⎥ ⎢⎢ 1 1 1 ⎥⎥
20 ⎢
= (3 − l )( l + 1)( l − 4) = 0; ⎣⎢ 4 12 4 ⎥⎦ ⎢⎣0 0 3⎥⎦ ⎢⎣ 0 −4 0 ⎥⎦
⎡4 −8 −1 ⎤ ⎡ −3 1 2⎤
1 ⎢
l = −1, 3, 4 are the eigenvalues of A. = 0 0 −15⎥⎥ ⎢⎢ 1 1 1 ⎥⎥
20 ⎢
⎢⎣16 48 16 ⎥⎦ ⎢⎣ 0 −4 0 ⎥⎦
l = −1
⎡ −20 0 0 ⎤ ⎡ −1 0 0 ⎤
1 ⎢
⎡ 2 6 1 ⎤ ⎡ x1 ⎤ 2 x1 + 6 x2 + x3 = 0 = 0 60 0 ⎥⎥ = ⎢⎢ 0 3 0 ⎥⎥
⎢1 3 0⎥ ⎢ x ⎥ = 0 x1 + 3x2 = 0 20 ⎢
⎢ ⎥ ⎢ 2⎥ ⎢⎣ 0 0 80 ⎥⎦ ⎢⎣ 0 0 4 ⎥⎦
⎢⎣0 0 4⎥⎦ ⎢⎣ x3 ⎥⎦ 4 x3 = 0
Note that the diagonal elements are the eigenvalues
x1 x2 x3
= = X1 = c1 [−3 1 0]T of A.
−3 1 0
To find A8

l=3 ⎡ −3 1 2⎤ ⎡1 0 0 ⎤
8
A = PD P 8 −1 ⎢ ⎥ ⎢
= ⎢ 1 1 1⎥ ⎢0 6561 0 ⎥⎥
⎡ −2 6 1⎤ ⎡ x1 ⎤ ⎢⎣ 0 −4 0⎥⎦ ⎢⎣0 0 65536⎥⎦
⎢ 1 −1 0⎥ ⎢ x ⎥ = 0 −2 x1 + 6 x2 + x3 = 0
⎢ ⎥ ⎢ 2⎥ x1 − x2 = 0 ⎡ −4 8 1 ⎤
⎢⎣ 0 0 0⎥⎦ ⎢⎣ x3 ⎥⎦ 1 ⎢
0 0 −5⎥⎥
x1 x2 x3 20 ⎢
= = X 2 = c2 [1 1 −4]T ⎢⎣ 4 12 4 ⎥⎦
1 1 −4
⎡ −3 6561 131072⎤ ⎡ −4 8 1 ⎤
1 ⎢
= 1 6561 65536 ⎥⎥ ⎢⎢ 0 0 −5⎥⎥
l=4 20 ⎢
⎣⎢ 0 −26244 0 ⎥⎦ ⎢⎣ 4 12 4 ⎦⎥
⎡ −3 6 1 ⎤ ⎡ x1 ⎤ −3 x1 + 6 x2 + x3 = 0
⎢1 ⎥ ⎢ ⎥ ⎡26215 78642 24574⎤
−2 0 ⎥ ⎢ x2 ⎥ = 0 x1 − 2 x2 = 0
⎢ = ⎢⎢13107 39322 11467 ⎥⎥
⎣⎢ 0 0 −1⎦⎥ ⎣⎢ x3 ⎦⎥ − x3 = 0
⎢⎣ 0 0 6561 ⎥⎦
x1 x2 x3
= = X 3 = c3 [2 1 0 ]T
2 1 0
  
Find matrix P which diagonalises
∴ The modal matrix which diagonalises A is
⎡ 7.3 0.2 −3.7⎤
⎡ −3 1 2⎤ ⎡ a1 b1 c1 ⎤
A = ⎢⎢ −11.5 1.0 5.5 ⎥⎥ .
P = ⎢⎢ 1 1 1⎥⎥ = ⎢⎢ a2 b2 c2 ⎥⎥ (say)
⎢⎣ 17.7 1.8 −9.3⎥⎦
⎢⎣ 0 −4 0⎥⎦ ⎢⎣ a3 b3 c3 ⎥⎦
 
 The characterstic equation is
To find P−1 7.3 − l 0.2 −3.7
To find the inverse of P we write [P |I] and apply
P ( l ) = −11.5 1.0 − l 5.5 = 0
elementary transformations to reduce it to the form
[I | P−1]. 17.7 1.8 −9.3 − l
2-20    Engineering Mathematics-II

It can be verified that for l = 0, 3, −4 the above l=3


determinant vanishes and hence the characteristic
equation is l3 + l2 − 12l = 0. ⎡ 4.3 0.2 −3.7 ⎤ ⎡ x1 ⎤
7.3 0.2 −3.7
⎢ −11.5 −2
⎢ 5.5 ⎥⎥ ⎢⎢ x2 ⎥⎥ = 0
P (0) = −11.5 1.0 5.5 ⎢⎣ 17.7 1.8 −12.3⎥⎦ ⎢⎣ x3 ⎥⎦
17.7 1.8 −9.3 ⎧4.3x1 + 0.2 x2 − 3.7 x3 = 0 (i)
!⎨
R2 − 5 R1 , R3 − 9 R1 ⎩ − x1 + x3 = 0 (ii)
7.3 0.2 −3.7 Eliminating x3 from (i) and (ii) we get
= −48 0 24 = 0; ∴ l = 0 is a root 3x1 + x2 = 0 (iii)
−48 0 24 Solving (ii) and (iii) we get
4.3 0.2 −3.7 x1 x2 x3
= = ⇒ X 2 = c2 [1 −3 1] T
P (3) = −11.5 −2 5.5 1 −3 1
17.7 1.8 −12.3
l = −4
R2 +10 R1 , R3 − 9 R1
4.3 0.2 −3.7
⎡ 11.3 0.2 −3.7⎤ ⎡ x1 ⎤
= 31.5 0 −31.5 = 0; ∴ l = 3 is a root ⎢ −11.5 5.0 5.5 ⎥ ⎢ x ⎥ = 0
⎢ ⎥ ⎢ 2⎥
−21.0 0 21.0 ⎢⎣ 17.7 1.8 −5.3⎥⎦ ⎢⎣ x3 ⎥⎦
11.3 0.2 −3.7 ⎧11.3x1 + 0.2 x2 − 3.7 x3 = 0 (i)
!⎨
P ( −4) = −11.5 −2 5.5 ⎩ −3x1 + x3 = 0 (ii)
17.7 1.8 −5.3 x
Eliminating 3 from (i) and (ii)
R2 +10 R1 , R3 − 9 R1 [11.3 + 3(−3.7)]x1 + 0.2 x2 = 0 x1 + x2 = 0 (iii)
11.3 0.2 −3.7 Solving (ii) and (iii) we get
= −294 0 98 = 0; ∴ l = −4 is a root x1 x2 x3
−84 0 28 = = ⇒ X 3 = c3 [1 −1 3] T
1 −1 3
⎡2 1 1 ⎤
l=0
∴ P = ⎢⎢ 1 −3 −1⎥⎥
⎡ 7.3 0.2 −3.7⎤ ⎡ x1 ⎤
⎢ −11.5 1.0 5.5 ⎥ ⎢ x ⎥ = 0 ⎣⎢ 4 1 3 ⎥⎦
⎢ ⎥ ⎢ 2⎥
⎢⎣ 17.7 1.8 −5.3⎥⎦ ⎢⎣ x3 ⎥⎦ To find P−1 we write [P |I] and transform it to [I |
⎧ 7.3x1 + 0.2 x2 − 3.7 x3 = 0 (i) P−1] form by elementary transformations.
!⎨
⎩ −2 x1 + x3 = 0 (ii), by R2 − 5R1
⎡2 1 1 1 0 0⎤
Since (ii) ⇒ x3 = 2 x, ⎢ ⎥
[ P | I ] = ⎢ 1 −3 −1 0 1 0⎥
(i) ⇒ 7.3x1 − 2(3.7) x1 + 0.2 x2 = 0
⎢⎣ 4 1 3 0 0 1⎥⎦
⇒ − x1 + 2 x2 = 0 (iii)
Solving (ii) and (iii) we get ⎡1 −3 −1 0 1 0⎤
R12 ⎢ ⎥
2 1 1 1 0 0⎥
x1 x2 x3
= = ⇒ X1 = c1[2 1 4] T " ⎢
2 1 4 ⎢⎣ 4 1 3 0 0 1⎥⎦
Eigenvalues and Eigenvectors    2-21

R2 − 2 R1 , R3 − 2 R2  
 The characteristic equation is
⎡1 −3 −1 0 1 0⎤ ⎡1 − l 1 3 ⎤
! ⎢0 7 3 1 −2 0⎥ ⎢
P (l ) = ⎢ 1 5−l 1 ⎥⎥
⎢⎣0 −1 1 −2 0 1⎥⎦ ⎢⎣ 3 1 1 − l ⎥⎦
R2 − 3 R3 = (1 − l )( l 2 − 6l + 4) − 1(1 − l − 3)
⎡1 −3 −1 0 1 0 ⎤ + 3(1 − 15 + 3l )
! ⎢0 10 0 7 −2 −3⎥ = − l 3 + 7l 2 − 36 = −( l + 2)( l − 3)( l − 6) = 0
⎢⎣0 −1 1 −2 0 1 ⎥
⎦ −2 −1 7 0 −36
1 2 −18 36
R2
10 3 −1 9 −18 0
⎡1 −3 −1 0 1 0 ⎤ −3 18
! ⎢0 1 0 7 10 −2 10 −3 10⎥ −1 6 0
⎢⎣0 −1 1 −2 0 1 ⎥⎦ l = −2
R1 + 4 R2 + R3 , R3 + R2
⎡3 1 3⎤ ⎡ x1 ⎤
⎡1 0 0 45 15 −1 5 ⎤ ⎢1 7 1⎥ ⎢ x ⎥ = 0;
! ⎢0 1 0 7 10 −2 10 −3 10⎥ = [ I | P ]
−1
⎢ ⎥ ⎢ 2⎥
⎢⎣3 1 3⎥⎦ ⎢⎣ x3 ⎥⎦
⎢⎣0 0 1 −13 10 −2 10 7 10 ⎥
⎦ 3x1 + x2 + 3x3 = 0⎫ x1 x2 x3
⎡ 8 2 −2⎤ ! ⎬ = =
1 ⎢ x1 + 7 x2 + x3 = 0 ⎭ 1 0 1
P −1
= ⎢ 7 −2 −3⎥⎥
10
⎢⎣ −13 −2 7 ⎥⎦ l=3
⎡ −2 1 3 ⎤ ⎡ x1 ⎤
D = P −1 AP ⎢ 1 2 1 ⎥ ⎢ x ⎥ = 0;
⎢ ⎥ ⎢ 2⎥
⎡ 8 2 −2⎤ ⎡ 7.3 0.2 −3.7⎤ ⎡2 1 1 ⎤ ⎢⎣ 3 1 −2⎥⎦ ⎢⎣ x3 ⎥⎦
1 ⎢ ⎢ 1 −3 −1⎥
= ⎢ 7 −2 −3⎥⎥ ⎢⎢ −11.5 1.0 5.5 ⎥⎥ ⎢ ⎥ ⎧ −2 x1 + x2 + 3x3 = 0 (i) (iii) − (i)
10 ⎪
⎢⎣ −13 −2 7 ⎥⎦ ⎢⎣ 17.7 1.8 −9.3⎥⎦ ⎢⎣ 4 1 3 ⎥⎦ ! ⎨ x1 + 2 x2 + x3 = 0 (ii) 5 x1 − 5 x3 = 0 = ⇒ x3 = x1 ;

⎩ 3x1 + x2 − 2 x3 = 0 (iii) using in (ii), x1 + x2 = 0
⎡ 8 2 −2⎤ ⎡0 3 −4 ⎤
1 ⎢ x1 x2 x3
= 7 −2 −3⎥⎥ ⎢⎢0 −9 4 ⎥⎥ = =
10 ⎢ 1 −1 1
⎢⎣ −13 −2 7 ⎥⎦ ⎢⎣0 3 −12⎥⎦
l=6
⎡0 0 0 ⎤ ⎡0 0 0⎤
1 ⎢
= ⎢0 30 0 ⎥⎥ = ⎢⎢0 3 0 ⎥⎥ ⎡ −5 1 3 ⎤ ⎡ x1 ⎤
10 ⎢ ⎥⎢ ⎥
⎢⎣0 0 −40 ⎥⎦ ⎢⎣0 0 −4 ⎥⎦ ⎢ 1 −1 1 ⎥ ⎢ x2 ⎥ = 0;
⎣⎢ 3 1 −5⎦⎥ ⎣⎢ x3 ⎦⎥
  
⎧ 8 x1 − 8 x3 = 0, by R3 − R1 ⎫
!⎨ ⎬
⎡1 1 3⎤ ⎩2 x1 − x2 = 0, by R3 + 5R2 ⎭
A = ⎢⎢1 5 1⎥⎥ and hence find A4. x x x
⎢⎣3 1 1⎥⎦ Solving we get 1 = 2 = 3
1 2 1
2-22    Engineering Mathematics-II

⎡ −1 1 1⎤ ⎡ a1 b1 c1 ⎤  
 Characteristic equation of A is
∴ P = ⎢⎢ 0 −1 2⎥⎥ = ⎢⎢ a2 b2 c2 ⎥⎥ 8−l −6 2
⎢⎣ 1 1 1⎥⎦ ⎢⎣ a3 b3 c3 ⎥⎦ P ( l ) = −6 7 − l −4
⎡ A1 A2 A3 ⎤ ⎡ −3 0 3⎤ 2 −4 3 − l

⇒ Adj P = ⎢⎢ B1 B2 B3 ⎥⎥ = ⎢⎢ 2 −2 2⎥⎥ = (8 − l )( l 2 − 10l + 5) + 6( −10 + 6l ) + 2(10 + 2l )


⎢⎣C1 C2 C3 ⎥⎦ ⎢⎣ 1 2 1⎥⎦ = − l 3 + l 2 (8 + 10) + l ( −80 − 5 + 36 + 4) + 40 − 60 + 20
= − l 3 + 18l 2 − 45l = −l ( l − 3)( l − 15) = 0
P = a1 A1 + a2 A2 + a3 A3 = ( −1)( −3) + 0 + 1⋅ 3 = 6
To find the eigenvector X = [x1, x2, x3]T corre-
D = P −1 AP sponding to eigenvalue l we have to solve (A − lI)
X = 0. Denote the equations by e1, e2, e3.
⎡ −3 0 3⎤ ⎡1 1 3⎤ ⎡ −1 1 1⎤
1⎢
= 2 −2 2⎥⎥ ⎢⎢1 5 1⎥⎥ ⎢⎢ 0 −1 2⎥⎥ l=0
6⎢
⎢⎣ 1 2 1⎥⎦ ⎢⎣3 1 1⎥⎦ ⎢⎣ 1 1 1⎥⎦ ⎡ 8 −6 2 ⎤ ⎡ x1 ⎤
⎢ −6 7 −4⎥ ⎢ x ⎥ = 0
⎡ −3 0 3⎤ ⎡ 2 3 6 ⎤ ⎢ ⎥ ⎢ 2⎥
1⎢ ⎢⎣ 2 −4 3 ⎥⎦ ⎢⎣ x3 ⎥⎦
= ⎢ 2 −2 2⎥⎥ ⎢⎢ 0 −3 12⎥⎥
6
⎣⎢ 1 2 1⎥⎦ ⎢⎣ −2 3 6 ⎥⎦ ⎧e1 + e2 − e3 ⇒ 5 x2 − 5 x3 = 0 ⎫ Solving we get
⎪ ⎪
⎨ ⇒ x2 = x3 ⎬ x1 x2 x3
⎡ −12 0 0 ⎤ ⎡ −2 0 0⎤ = =
1⎢ ⎪ Now e ⇒ 2 x − x = 0 ⎪
= ⎢ 0 18 0 ⎥⎥ = ⎢⎢ 0 3 0⎥⎥ ⎩ 3 1 2 ⎭ 1 2 2
6
⎢⎣ 0 0 36⎥⎦ ⎢⎣ 0 0 6⎥⎦ l=3
4
A = PD P 4 −1 ⎡ 5 −6 2 ⎤ ⎡ x1 ⎤
⎢ −6 4 −4⎥ ⎢ x ⎥ = 0
4
⎡ −1 1 1⎤ ⎡( −2) 0 0 ⎤ ⎡ −3 0 3⎤ ⎢ ⎥ ⎢ 2⎥
⎢ ⎥1 ⎢⎣ 2 −4 0 ⎥⎦ ⎢⎣ x3 ⎥⎦
= ⎢⎢ 0 −1 2⎥⎥ ⎢ 0 34
0 ⎥ ⎢⎢ 2 −2 2⎥⎥
⎢⎣ 1 1 1⎥⎦ ⎢ 0 ⎥6 Solving we get
⎢⎣ 0 64 ⎥⎦ ⎢⎣ 1 2 1⎥⎦ ⎧ e3 ⇒ x1 = 2 x2 ⎫
⎨ ⎬ x1 x2 x3
⎡ −1 1 1⎤ ⎡16 0 0 ⎤ ⎡ −3 0 3⎤ ⎩ Now e1 ⇒ 2 x2 + x3 = 0⎭ = =
2 1 −2
1
= ⎢⎢ 0 −1 2⎥⎥ ⎢⎢ 0 81 0 ⎥⎥ ⎢⎢ 2 −2 2⎥⎥
6 l = 15
⎢⎣ 1 1 1⎥⎦ ⎢⎣ 0 0 1296⎥⎦ ⎢⎣ 1 2 1⎥⎦
⎡ −16 81 1296 ⎤ ⎡ −3 0 3⎤ ⎡ −7 −6 2 ⎤ ⎡ x1 ⎤
1⎢ ⎢ −6 −8 −4 ⎥ ⎢ x ⎥ = 0
= ⎢ 0 −81 2592⎥⎥ ⎢⎢ 2 −2 2⎥⎥ ⎢ ⎥⎢ 2⎥
6 ⎢⎣ 2 −4 −12⎥⎦ ⎢⎣ x3 ⎥⎦
⎢⎣ 16 81 1269 ⎥⎦ ⎢⎣ 1 2 1⎥⎦
⎡ 251 405 235⎤ Solving we get
⎧e 2 + 2 e 1 ⇒ x1 = − x2⎫
= ⎢⎢ 405 837 405⎥⎥ ⎨
⎩ e2 − 2e3 ⇒ x1 = 2 x3 ⎭
⎬ x1 x2 x3
= =
2 −2 1
⎣⎢ 235 405 251⎦⎥
We can easily see that the eigenvectors
   ⎡ ⎡1⎤ ⎡ 2 ⎤ ⎡ 2 ⎤ ⎤
⎢ ⎥
⎡ 8 −6 2 ⎤ [ X1 , X 2 , X 3 ] = ⎢ ⎢⎢2⎥⎥ , ⎢⎢ 1 ⎥⎥ , ⎢⎢ −2⎥⎥ ⎥ are pairwise
A = ⎢⎢ −6 7 −4⎥⎥ . ⎢⎣ ⎣⎢ 3⎦⎥ ⎣⎢ −2⎦⎥ ⎣⎢ 1 ⎦⎥ ⎥⎦
⎢⎣ 2 −4 3 ⎥⎦ orthogonal, each with magnitude = 3.
Eigenvalues and Eigenvectors    2-23

Normalised modal matrix l2 = 1 − i


⎡1 2 2 ⎤ ⎡ i 1 ⎤ ⎡ x1 ⎤
∴ P = ⎢2 1 −2⎥⎥ and its inverse P−1 = PT = P
1⎢ = 0; ix1 + x2 = 0
⎢ −1 i ⎥⎦ ⎢⎣ x2 ⎥⎦
3 ⎣
⎢⎣2 −2 1 ⎥⎦
x1 x2 ⎡i ⎤
since P is orthogonal and symmetric. ⇒ = ; X 2 = c2 ⎢ ⎥
i 1 ⎣1⎦
⎡ 8 −6 2 ⎤ ⎡1 2 2 ⎤
1⎢ ⎡1 i ⎤ 1 ⎡ 1 −i ⎤
Now, AP = −6 7 −4 ⎥⎥ ⎢⎢2 1 −2⎥⎥  P=⎢ ⎥ and P −1 = ⎢ −i 1 ⎥
3⎢ ⎣i 1⎦ 2 ⎣ ⎦
⎢⎣ 2 −4 3 ⎥⎦ ⎢⎣2 −2 1 ⎥⎦
1 ⎡1 −i ⎤ ⎡ 1 1 ⎤ ⎡1 i ⎤
D = P −1 AP = ⎢
⎡0 6 30 ⎤ 2 ⎣ −i 1 ⎥⎦ ⎢⎣ −1 1 ⎥⎦ ⎢⎣i 1⎥⎦
= ⎢⎢0 3 −30 ⎥⎥
1
3 1 ⎡ 1 + i 1 − i ⎤ ⎡1 i ⎤ ⎡1 + i 0 ⎤
⎢⎣0 −6 15 ⎥⎦ = ⎢ −1 − i 1 − i ⎥ ⎢i 1⎥ = ⎢ 0 1 − i ⎥
2 ⎣ ⎦⎣ ⎦ ⎣ ⎦
⎡1 2 2 ⎤ ⎡0 6 30 ⎤
⇒ D = P AP = ⎢2 1 −2⎥⎥ ⎢⎢0 3 −30 ⎥⎥
1⎢
−1
3     
⎣⎢2 −2 1 ⎦⎥ ⎢⎣0 −6 15 ⎦⎥
Find the eigenvalues and eigenvectors of matrix A =
⎡0 0 0 ⎤
⎡ 4 −2⎤ (Qns. 1–4)
= ⎢⎢0 3 0 ⎥⎥ 1. ⎢ ⎥.
⎣ 1 1 ⎦
⎢⎣0 0 15⎥⎦ X1 X2
⎡1⎤ ⎡2⎤
Ans: 2, 3; ⎢ ⎥ , ⎢1 ⎥
 ! " # ⎣1⎦ ⎣ ⎦

$ " 
  # "# ⎡2 4 ⎤
2. ⎢ ⎥.

 ⎣1 5 ⎦
X1 X2
# ⎡ −4 ⎤ ⎡1⎤
Find the eigenvalues and eigenvectors and Ans: 1, 6; ⎢ ⎥ , ⎢1⎥
⎣1⎦ ⎣⎦
⎡ 1 1⎤
diagonalise A = ⎢ ⎥ ⎡ 6 −2 2 ⎤
⎣ −1 1⎦ ⎢ ⎥
!  The characterstic equation is 3. ⎢ −2 3 −1⎥ . [JNTU 2003 (1) 2005 S(3)]
⎢⎣ 2 −1 3 ⎥⎦
1− l 1
P (l ) = = l 2 − 2l + 2 = 0 X1 X2 X3
−1 1 − l ⎡1⎤ ⎡1 ⎤ ⎡2⎤
⇒ l1,2 = 1 ± i Ans: 2, 2, 8; ⎢⎢ 0 ⎥⎥ , ⎢2⎥
⎢ ⎥
⎢ −1⎥
⎢ ⎥
⎢⎣ −2⎥⎦ ⎢⎣0 ⎥⎦ ⎢⎣ 1 ⎦⎥
There are no real eigenvalues. Hence matrix A
is not diagonalisable over reals.
⎡ 2 0 1⎤
l1 = 1 + i 4. ⎢⎢0 2 0⎥⎥ . [JNTU 2000 S]
⎡ −i 1 ⎤ ⎡ x1 ⎤ ⎢⎣1 0 2⎥⎦
⎢ −1 −i ⎥ ⎢ x ⎥ = 0; − ix1 + x2 = 0 X1 X2 X3
⎣ ⎦ ⎣ 2⎦
⎡1 ⎤ ⎡1⎤ ⎡0 ⎤
x1 x2 ⎡1⎤ Ans: 1, 2, 3; ⎢⎢0 ⎥⎥ , ⎢0⎥ ⎢1 ⎥
⇒ = ; X1 = c1 ⎢ ⎥ ⎢ ⎥ ⎢ ⎥
1 i ⎣i ⎦ ⎢⎣1 ⎥⎦ ⎢⎣ −1⎥⎦ ⎢⎣0 ⎥⎦
2-24    Engineering Mathematics-II

Diagonalise, if possible, the matrix A = 11. 2A.


(Qns. 3–10) Ans: The eigenvalues of A are (2, 3);
⎡ −2 2 −3⎤ those of 2A are 2(2, 3) = (4, 6).
⎢ ⎥
5. ⎢ 2 1 −6⎥ . [JNTU 1995, 2005 S (4)]
⎢⎣ −1 −2 0 ⎥⎦ 12. 8A2 − A + 3I .
X1 X2 X3 [Hint: 8(22, 32) − (2, 3) + (3, 3) = (33, 72).]
⎡2⎤ ⎡ 3⎤ ⎡ 1⎤
Ans: −3, −3, 5; ⎢ −1⎥ , ⎢0 ⎥ ⎢ 2⎥ Ans: (33, 72)
⎢ ⎥ ⎢ ⎥ ⎢ ⎥
⎢⎣ 0 ⎥⎦ ⎢⎣1 ⎥⎦ ⎢⎣−1 ⎥⎦
13. A−1.
1 1
⎡ 4 1⎤ Ans: ,
⎥ . Also, find A .
5
6. ⎢ 2 3
⎣ 3 2 ⎦
⎡ 1 1⎤ 5 ⎡2344 781⎤
Ans: 1, 5; P = ⎢ ⎥ ; A = ⎢ 2343 782⎥ 14. (A + A−1)2.
⎣ −3 1⎦ ⎣ ⎦
Verify the Cayley–Hamilton Theorem. Find A−1 if A =
⎡ 1 2⎤ (Qns. 15–19)
⎥ . Also, find A .
3
7. ⎢
⎣ 4 3⎦
2 −2
⎡ 5 0 ⎤ 3 ⎡ 41 42⎤ Ans: Eigenvalues of A + A + 2 I :
Ans: 5, −1; D = ⎢ ⎥ ; A = ⎢84 83⎥
⎣0 −1⎦ ⎣ ⎦ 1 1
(22 , 32 ) + 2 , 2 + 2 (1,1)
2 3
⎡ −1 2 2⎤
8. ⎢⎢ 1 2 1 ⎥⎥ . ⎛ 1 1⎞
[JNTU 2004] = ⎜ 6 , 11 ⎟
⎝ 4 8⎠
⎢⎣ −1 −1 0 ⎥⎦
⎡2⎤ ⎡0⎤
⎡ 1 2⎤
Ans: −1, 1, 1; X 1 = ⎢ −1⎥ ; X 2 = ⎢⎢ 1 ⎥⎥
⎢ ⎥
15. ⎢ ⎥ . Also find A .
4

⎣ −1 3⎦
⎣⎢ 1 ⎦⎥ ⎢⎣ −1⎦⎥
Geometric multiplicity of l = 1 is 1, not diagonalizable. 1 ⎡3 −2⎤ 4 1 ⎡ −31 48 ⎤
Ans: A−1 = ;A = ⎢
5 ⎢⎣1 1 ⎥⎦ 5 ⎣ −24 17 ⎥⎦
⎡1 6 1⎤
9. ⎢⎢1 2 0⎥⎥ .
⎢⎣0 0 3⎥⎦ ⎡1 2 3⎤
⎢ ⎥
16. ⎢2 4 5⎥ . [JNTU 2005 S (1)]
⎡ −1 0 0 ⎤ ⎡ −3 1 2 ⎤
⎢ ⎥ ⎢ ⎥ ⎢⎣ 3 5 6⎥⎦
Ans: D = ⎢ 0 3 0 ⎥ ; P = ⎢ 1 1 1 ⎥
⎢⎣ 0 0 4 ⎥⎦ ⎢⎣ 0 −4 0 ⎥⎦ ⎡ 1 −3 2 ⎤
Ans: A = ⎢⎢ −3 3 −1⎥⎥
−1

⎡1 0 0 ⎤ ⎢⎣ 2 −1 0 ⎥⎦
10. ⎢⎢0 3 −1⎥⎥ . [JNTU 2003(4)]
⎢⎣0 −1 3 ⎥⎦
⎡ 2 −1 1 ⎤
⎡1 0 0 ⎤ ⎡1 0 0 ⎤ 17. ⎢⎢ −1 2 −1⎥⎥ . [JNTU 2000 S (1), 2001]
Ans: D = ⎢⎢0 2 0 ⎥⎥ ; P = ⎢⎢0 1 1 ⎥⎥ ⎢⎣ 1 −1 2 ⎥⎦
⎢⎣0 0 4⎥⎦ ⎢⎣0 1 −1⎥⎦
⎡ 4 −2⎤ ⎡3 1 −1⎤
Determine the eigenvalues of B if A = ⎢ 1⎢
⎣1 1 ⎦
⎥ Ans: A = ⎢1 3 1 ⎥⎥
−1
4
and B = (Qns. 11–14) ⎢⎣1 1 3 ⎥⎦
Eigenvalues and Eigenvectors    2-25

⎡1 0 −1⎤ 20. Prove that the following matrices are pairwise


18. ⎢⎢1 2 1 ⎥⎥ . similar.
⎢⎣2 2 3 ⎥⎦ ⎡2 0 1⎤ ⎡1 0 −1⎤
⎡1 0 0⎤ A = ⎢⎢0 2 0⎥⎥ B = ⎢⎢1 2 1 ⎥⎥
Ans: D = ⎢⎢0 2 0⎥⎥ ⎢⎣1 0 2⎥⎦ ⎢⎣2 2 3 ⎥⎦
⎢⎣0 0 3⎥⎦ ⎡1 3 4⎤ ⎡ 2 0 0⎤
C = ⎢⎢0 2 5⎥⎥ D = ⎢⎢ 3 1 0⎥⎥
⎡2 2 1 ⎤ ⎢⎣0 0 3⎦⎥ ⎢⎣ −1 2 3⎦⎥
⎢ ⎥
19. ⎢1 3 1 ⎥ .
[Hint: Each of these matrices has the same charac-
⎣⎢1 2 2⎥⎦
terstic equation l3 − 6l2 + 11l − 6 = 0 and the same
⎡ 4 −2 −1⎤ eigenvalues l = 1, 2, 3.]
1⎢
Ans: A −1 = −1 3 −1⎥⎥
5⎢
⎢⎣ −1 −2 4 ⎥⎦
Real and Complex
Matrices 3
3.1 INTRODUCTION 3.2.2 Orthonormal System
A matrix with real entries is called a real matrix. of Vectors
We have studied various types of real matrices and A set of vectors X1, X2, …, Xn is said to form an
their properties. Matrices with complex entries are orthonormal system if
also important in the study of quantum mechanics, ⎧1 if i = j
quadratic forms, etc. We now consider three classes X iT ⋅ X j = δ ij = ⎨ (3.2)
⎩0 if i ≠ j
of real square matrices—symmetric, skew-symmetric
i.e., vectors are mutually orthogonal (perpendicular)
and orthogonal matrices and also their counterparts
and normalised (that is, such that || X || = 1).
with complex elements which are important in appli-
After studying the various properties of
cations. But first we define certain terms.
symmetric and skew-symmetric matrices, we will
define below a matrix whose row and column vectors
3.2 ORTHOGONAL/ORTHONORMAL are mutually orthogonal and study the various
SYSTEM OF VECTORS properties of the matrix.
3.2.1 Norm of a Vector
The norm of a vector X denoted by || X || is defined by 3.3 REAL MATRICES
3.3.1 Symmetric Matrix
! X != X T X (3.1)
A square matrix A = [aij] in which aij = aji for all i, j is
⎡ x1 ⎤ called a symmetric matrix. In other words, a square
matrix A is symmetric if AT = A. (The matrix is equal
If X = ⎢⎢ x2 ⎥⎥ then X T = [x1 x2 x3 ] and
to its transpose.)
⎢⎣ x3 ⎥⎦
⎡ 3 −1 7 ⎤
! X !2 = X T X E.g. ⎢ −1 2 −2⎥ , ⎡11 4⎤ are symmetric.
⎢ ⎥ ⎢ 4 9⎥
⎡ x1 ⎤ ⎢⎣ 7 −2 6 ⎥⎦ ⎣ ⎦ (3.3)
= [x1 x2 x3 ] ⎢ x ⎥ = x2 + x2 + x2
⎢ 2⎥ 1 2 3
⎢⎣ x3 ⎥⎦ ⎡ 0 4 −4⎤
whereas the matrix ⎢⎢ 4 3 5 ⎥⎥ is not symmetric
⇒ ! X != x12 + x22 + x32 (3.4)
⎣⎢ −4 −5 6 ⎥⎦
⎡ 5 ⎤ since a23 = 5 and a32 = −5 and are not equal. For a
E.g. Let X = ⎢⎢ −12⎥⎥ ⇒ X T = [5 −12 13] symmetric matrix a23 = a32.
⎢⎣ 13 ⎥⎦
3.3.2 Skew-Symmetric Matrix
 X  = 52 + ( −12)2 + 132
A square matrix A = [aij] is called a skew-symmetric
= 25 + 144 + 164 = 13 2 matrix if aji = −aij for all i, j.
3-2    Engineering Mathematics-II

In other words, a square matrix A is skew- T


⎛ ( A − AT ) ⎞ 1 T
symmetric if AT = −A. ⎟⎟ = ⎡⎢ A − ( A ) ⎤⎥
T T T
C = ⎜⎜
(The matrix is equal to the negative of its ⎝ 2 ⎠ 2⎣ ⎦
transpose.) 1
= ( AT − A) = −C ⇒ C is skew-symmetric
⎡ 0 −3 5⎤ 2
⎢ ⎥ ⎡0 −7⎤
E.g. ⎢ 3 0 1⎥ , ⎢ are skew-symmetric
7 0 ⎥⎦ A + AT A − AT
⎢⎣ −5 −1 0⎥⎦ ⎣ (3.5) Hence A =
2
+
2
= B + C.

⎡0 2 −1⎤ Thus square matrix A is expressible as the sum of


whereas the matrix ⎢ 2 0 −5⎥ is not skew-symmetric two matrices one symmetric and the other skew-
⎢ ⎥
⎢⎣1 5 1 ⎥⎦ (3.6) symmetric.
Suppose A = P + Q where P is symmetric and
since a12 = 2 and a21 = 2(a12 ≠ −a21) and further main Q skew-symmetric
diagonal element a33 = 1 ≠ 0.
AT = ( P + Q )T = P T + QT
Note 1 The main diagonal elements of a skew-
symmetric matrix should be zero since for all A + AT
= P −Q ⇒ P = = B and
aij = −aij ⇒ aij = 0 (i = j) ⇒ aii = 0 2
A − AT
Q= =C
2
3.3.3 Properties of Symmetric and
Skew-Symmetric Matrices This proves the uniqueness of the representation.
Theorem 3.1 Show that for every square matrix A,
Theorem 3.3 Show that for any matrix A the
the matrix A + AT is symmetric and A − AT is skew-
matrix B = AAT is square and symmetric.
symmetric.

Proof Proof Let A = [aij] be a square matrix of order n.


T T
Then AT = [bij] where bij = aji is also a square matrix
T T T T T
(A+ A ) = A + (A ) = A + A = A+ A of order n. B = AAT is the square of order n.
⇒ A + AT is symmetric BT = (AAT)T = (AT)T AT = AAT = B; ∴ B is symmetric.
T T
( A − AT ) = AT − ( AT ) = AT − A = −( A − AT ) Theorem 3.4 If A and B are symmetric then
T
⇒ A− A is skew-symmetric AB is symmetric if and only if A and B commute,
AB = BA.
Theorem 3.2 Every square matrix is uniquely
expressible as the sum of two matrices one symmet- Proof Let A and B be symmetric matrices of
ric and the other skew-symmetric. order n. Then

Proof Let A be a square matrix and AT = A, BT = B (3.7)


1 1 Suppose A and B commute.
B= ( A + AT ) and C = ( A − AT )
2 2 Then
AB = BA (3.8)
T
⎛ ( A + AT ) ⎞ 1 T
⎟⎟ = ⎡⎢ A + ( A ) ⎤⎥
T T (AB)T = BTAT = BA
BT = ⎜⎜ by (3.7)
⎝ 2 ⎠ 2⎣ ⎦
= AB by (3.8)
1
= ( AT + A) = B ⇒ B is symmetric (AB)T = AB. This implies that AB is symmetric.
2
Real and Complex Matrices    3-3

Suppose AB is symmetric so that Theorem 3.8 The determinant of an orthogonal


T matrix has the value +1 or −1.
(AB) = AB (3.9)
T T T Proof We know that
Then (AB) = B A = BA by (3.7)
= AB by (3.9) det AB = det A · det B and
det AT = det A (3.11), (3.12)
Hence A and B commute and BA = AB.
If A is an orthogonal matrix then AT · A = I
Theorem 3.5 If A is symmetric then show that 1 = det I = det ATA = det AT · det A = (det A)2
B T AB is symmetric. ⇒ det A = ±1.
Proof Let A be symmetric Theorem 3.9 The eigenvalues of an orthogonal
⇒ AT = A (3.10) matrix are of unit modulus.
Now consider(BT AB)T = BTAT (BT)T Proof Let A be an orthogonal matrix ⇒ ATA = I
(Reversal law for transposes) (3.13)
= BTATB  (BT)T = B If l is an eigenvalue and X ≠ 0 is the eigenvector
= BTAB by (3.10) corresponding to l then
T
Hence B AB is symmetric. AX = lX (3.14)

3.3.4 Orthogonal Matrix Taking transposes of both sides of (3.14)


A square matrix A is said to be orthogonal if (AX)T = (lX)T = lXT ⇒ XTAT = lXT (3.15)
AT = A−1, i.e., transpose of a matrix equals its inverse. Multiplying the LH and RH members of equations
(3.15) and (3.14), we have
3.3.5 Properties of Orthogonal
Matrix (XTAT ) (AX) = (lXT ) (lX)
Theorem 3.6 If A is orthogonal then ATA = I. By Associative law
T −1
Proof Since A is orthogonal we have A = A . XT (ATA)X = l2XTX ⇒ XTX (1 − l2) = 0
T −1
A A = A A = I. (3.16)
Theorem 3.7 (a) A is orthogonal ⇒ (i) A−1 is But XT ≠ 0, X ≠ 0 ⇒ |XTX| ≠ 0. Now we obtain
T
orthogonal (ii) A is orthogonal. [JNTU 2002] from (3.16),
(b) A and B are orthogonal of the same type X T X 1− l2 = 0 ⇒ l 2 = 1 or l = ±1
⇒ AB and BA are orthogonal. [JNTU 2003]
⇒ l =1 (3.17)
(c) Any two row (column) vectors of an orthogonal
matrix are orthogonal. Hence the eigenvalues of an orthogonal matrix
T −1 are of unit modulus.
Proof (a) A is orthogonal ⇒ A = A (definition).
Theorem 3.10 The eigenvectors corresponding
(i) Now (A−1)T = (AT)−1 = (A−1)−1. Hence A−1 is
to distinct eigenvalues of a real symmetric matrix
orthogonal.
are orthogonal.
(ii) (AT)T = (A−1)T (AT)−1. Hence AT is orthogonal.
Proof Let l1, l2 be any two different eigenvalues
Let A and B be orthogonal matrices of order n, of a real symmetric matrix A: and let X1, X2 be the
AT = A−1, BT = B−1. Then AB and BA are square corresponding eigenvectors of A.
matrices of order n. (AB)T = BTAT = B−1A−1 = (AB)−1;
this implies that AB is orthogonal. Then AX1 = l1X1, (3.18)
Similarly, (BA)T = ATBT = A−1B−1 = (BA)−1; AX2 = l2X2 (3.19)
which implies that BA is orthogonal. Pre-multiplying both sides of (3.18) by X 2T
3-4    Engineering Mathematics-II

X 2T AX1 = l1 X 2T X1 (3.20) 1
∴ P = ( A + AT )
Taking the transposes of both sides of (3.20) 2
⎡ 3 + 3 −2 + 2 6+5 ⎤
⎢ 2 2 2 ⎥
( X 2T AX1 )T = ( l1 X 2T X1 )T ⎢ ⎥
2 − 2 7 + 7 −1 + 4 ⎥
⇒ X1T AT X 2 = l1 X1T X 2 (! X 2T = X 2 ) =⎢
⎢ 2 2 2 ⎥
⎢ ⎥
⇒ X1T AX 2 = l1 X1T X 2 (! AT = A) ⎢ 5 + 6 4 −1 0+0 ⎥
(3.21) ⎣⎢ 2 2 2 ⎥⎦
⎡ 11⎤
Now, pre-multiplying both sides of (3.19) by X1T ⎢3 0 2⎥
⎢ ⎥
3⎥
X1T AX 2 = λ 2 X1T X 2 (3.22) =⎢0 7 (Symmetric)
⎢ 2⎥
⎢ ⎥
Since LHS members of (3.21) and (3.22) are ⎢11 3 0 ⎥
equal, RHS members must be equal. ⎣⎢ 2 2 ⎦⎥
1
Q = ( A − AT )
l1 X1T X 2 = l2 X1T X 2 ⇒ ( l1 − l2 ) X1T X 2 = 0 , 2
on transposing (3.23) ⎡ 3 − 3 −2 − 2 6 − 5 ⎤
⎢ 2 2 2 ⎥
⎢ ⎥
Since l1, l2 are different, l1 ≠ l2. Equation ⎢ 2 + 2 7 − 7 −1 − 4 ⎥
=
(3.23) ⇒ X1T X 2 = 0. ⎢ 2 2 2 ⎥
⎢5 6 4 1 0 − 0 ⎥⎥
This is the condition to be satisfied for ⎢ − +
orthogonality of vectors X1 and X2. ⎣⎢ 2 2 2 ⎦⎥
∴ The eigenvectors X1 and X2 are orthogonal. ⎡ 1⎤
⎢ 0 −2 2⎥
⎢ ⎥
Example 3.1 −5 ⎥
=⎢ 2 0 (Skew-symmetric)
Express the matrix A as the sum of a symmetric and ⎢ 2⎥
⎢ 1 5 ⎥
⎢−
a skew-symmetric matrix,
0⎥
⎡ 3 −2 6 ⎤ ⎢⎣ 2 2 ⎥⎦
if A = ⎢⎢2 7 −1⎥⎥ .
Example 3.2
⎣⎢5 4 0 ⎥⎦
⎡ 3 −4 −1⎤
Write A = ⎢⎢ 6 0 −1⎥⎥ as the sum of a symmetric
Solution A square matrix A can be expressed
uniquely as the sum of two matrices one symmetric
and the other skew-symmetric. A = P + Q where ⎢⎣ −3 13 −4⎥⎦
1 1
P = ( A + AT ), Q = ( A − AT ) matrix R and a skew-symmetric matrix S.
2 2
Solution
⎡ 3 −2 6 ⎤ ⎡ 3 2 5⎤ ⎡ 3 −4 −1⎤ ⎡ 3 6 −3⎤
Here A = ⎢2 7 −1⎥ , A = ⎢⎢ −2 7 4⎥⎥
⎢ ⎥ T
A = ⎢⎢ 6 0 −1⎥⎥ , A = ⎢⎢ −4 0 13 ⎥⎥
T

⎢⎣5 4 0 ⎥⎦ ⎢⎣ 6 −1 0 ⎥⎦ ⎢⎣ −3 13 −4 ⎥⎦ ⎢⎣ −1 −1 −4 ⎥⎦
Real and Complex Matrices    3-5

⎡ 6 2 −4 ⎤ Solution If A is an orthogonal matrix then A · AT = I


1 1 ⎢
R = ( A + A ) = ⎢ 2 0 12 ⎥⎥
T
2 2 ⎡ 0 2b c ⎤ ⎡ 0 a a⎤
⎢⎣ −4 12 −8⎥⎦
∴ A = ⎢ a b −c ⎥ ⎢2b b −b ⎥⎥
⎢ ⎥ ⎢
⎡ 3 1 −2 ⎤ ⎢⎣ a −b c ⎥⎦ ⎢⎣ c −c c ⎥⎦
= ⎢⎢ 1 0 6 ⎥⎥
⎡ 4b 2 + c 2 2b2 − c 2 −2b2 + c 2 ⎤
⎢⎣ −2 6 −4 ⎥⎦ ⎢ 2 2 ⎥
= ⎢ 2b − c a2 + b2 + c 2 a2 − b2 − c 2 ⎥ = I
⎡ 0 −10 2 ⎤ ⎢ 2 2 2 2 2 2 2 2⎥
1 1⎢ ⎢⎣ −2b + c a − b − c a + b + c ⎥⎦
T
S = ( A − A ) = ⎢10 0 −14 ⎥⎥
2 2
⎢⎣ −2 14 0 ⎥⎦
Equating the nondiagonal elements 2b2 − c2 = 0,
⎡ 0 −5 1 ⎤ a2 − b2 − c2 = 0
= ⎢⎢ 5 0 −7⎥⎥
⇒ c = ± 2 ⋅ b,
⎢⎣ −1 7 0 ⎥⎦
a2 = b2 + c 2 = b2 + 2b2 = 3b2 , a = ± 3 ⋅ b
Equating the diagonal elements 4b2 + c2 = 1,
Example 3.3
4b + 2b2 = 1
2
Show that the eigenvalues of the skew-symmetric 1 1 1
b=± ,c=± ,a=±
⎡0 9 −12⎤ 6 3 2
matrix ⎢⎢ −9 0 20 ⎥⎥ are purely imaginary or
⎢⎣12 −20 0 ⎥⎦ ⎡ 2a 2 0 0 ⎤
⎢ ⎥
zero. T
Note 1 I = A A = ⎢ 0 6b2 0 ⎥ gives the result
⎢ ⎥
Solution The characteristic equation of the given ⎣⎢ 0 0 3c 2 ⎦⎥
matrix is immediately.
0−l 9 −12 Example 3.5
P ( l ) = −9 0 − l 20 If X1 = [2 − 1 2]T and X2 = k[3 − 4 − 5]T where
12 −20 0 − l 1
k= construct an orthogonal matrix A =
= −l ( l 2 + 400) + 9 ( −9l − 240) 50
+12 (180 − 2l ), expanding by c1 [X1 X2 X3].

= −l 3 + l (−400 − 81 − 144) − 2160 + 2160 Solution Let X3 = [a1 a2 a3]T be the undetermined
= −l − 625l = 0 ⇒ l = 0, ± 25i
3 vector. Since A is orthogonal, the column vectors of
A form an orthogonal system X iT X j = δij
Thus, the eigenvalues of A are purely imaginary
or zero. ⎡ 3k ⎤
⎡2 1 2⎤ ⎢
X1T X2 = ⎢ − −4k ⎥⎥
Example 3.4 ⎣ 3 3 3 ⎥⎦ ⎢
⎢⎣ −5k ⎥⎦
Determine a, b, c so that A is orthogonal where
⎡ 0 2b c⎤ 4k 10
= 2k + − k = 0 (True)
A = ⎢⎢ a b −c ⎥⎥ [JNTU 2002, 2004 S(1)] 3 3
⎢⎣ a −b c ⎥⎦ ∴ X1 and X2 are orthogonal.
3-6    Engineering Mathematics-II

⎡ a1 ⎤ ⎡ 4 + 9 +1 8 − 9 + 1 −6 − 3 + 9⎤
⎡2 1 2⎤ ⎢ ⎥ ⎢
X1T X 3 = ⎢ − a2 = ⎢ 8 − 9 +1 16 + 9 + 1 − 12 + 3 − 9⎥⎥
⎣3 3 3 ⎥⎦ ⎢ ⎥
⎢⎣ a3 ⎥⎦ ⎢⎣ −6 − 3 + 9 −12 + 3 + 9 9 + 1 + 81 ⎥⎦
1 ⎡14 0 0 ⎤
= [2a1 − a2 + 2a3 ] = 0
3 = ⎢⎢ 0 26 0 ⎥⎥ ≠ I 3
⇒ 2a1  a2 + 2a3 = 0 (1)
⎢⎣ 0 0 91⎥⎦
⎡ a1 ⎤
X 2T X 3 = [3k − 4k − 5k ] ⎢⎢ a2 ⎥⎥ Hence the matrix A is not orthogonal.
⎢⎣ a3 ⎥⎦ Example 3.7
= [3a1 − 4a2 − 5a3 ] k = 0 ⎡0 1 0⎤
⇒ 3a1  4a2  5a3 = 0 (2) Verify that A = ⎢⎢1 0 0⎥⎥ is orthogonal.
The norm of X3 is given by ⎣⎢0 0 1⎥⎦
⎡ a1 ⎤
Solution We have
X 3 = X 3T X 3 = [a1 a2 a3 ] ⎢⎢ a2 ⎥⎥
2
⎡0 1 0⎤ ⎡0 1 0⎤
⎢⎣ a3 ⎥⎦ ∴ AA = ⎢⎢1 0 0⎥⎥ ⎢⎢1
T
0 0⎥⎥
(3)
= a1 + a2 + a3 = 1 ⎣⎢0 0 1⎦⎥ ⎢⎣0 0 1⎥⎦
2 2 2

By the rule of cross-multiplication, we get from ⎡1 0 0⎤


(1) and (2), = ⎢⎢0 1 0⎥⎥ = I ⇒ A is orthogonal.
2 2
a1 a2 a3 a1 + a2 1 , + a32 by (3)
⎢⎣0 0 1⎥⎦
= = = =
13 16 −5 2 2
13 + 16 + 52 15 2
Example 3.8
The required orthogonal matrix is Prove that the product AB of two symmetric matri-
⎡ 2 3 3k −13 5k1 ⎤ ces A and B is symmetric if they are commutative,
1 i.e., AB = BA.
A = ⎢⎢ −1 3 −4k −16 5k1 ⎥⎥ , k1 =
3 2 Solution A, B are symmetric
⎢⎣ 2 3 −5k k1 ⎥⎦
⇒ AT = A, BT = B (1)
Example 3.6 A, B are commutative
⇒ AB = BA (2)
⎡ 2 −3 1⎤
Now, (AB)T = BTAT = BA, by (1)
Is the matrix ⎢⎢ 4 3 1⎥⎥ orthogonal? = AB, by (2)
⎢⎣ −3 1 9⎥⎦ [JNTU 2003(4)] The product matrix AB is symmetric.

Solution A square matrix A is orthogonal if AAT = I EXERCISE 3.1


⎡ 2 −3 1⎤ ⎡ 2 4 −3⎤ 1. Express each of the following matrices as the sum
Let A = ⎢ 4 3 1⎥ then A = ⎢⎢ −3 3 1 ⎥⎥
⎢ ⎥ T
of two matrices one symmetric and the other skew-
⎣⎢ −3 1 9⎥⎦ ⎣⎢ 1 1 9 ⎦⎥
symmetric.
⎡ 3 −2 6 ⎤ ⎡a a b⎤
⎡ 2 −3 1⎤ ⎡ 2 4 −3⎤ (a) A = ⎢⎢2 7 −1⎥⎥ (b) B = ⎢⎢ c b b ⎥⎥
AA = ⎢⎢ 4 3 1⎥⎥ ⎢⎢ −3 3 1 ⎥⎥
T
⎢⎣ c c b ⎥⎦
⎣⎢5 4 0 ⎥⎦
⎣⎢ −3 1 9⎥⎦ ⎢⎣ 1 1 9 ⎦⎥ [Marathwada, 1993]
Real and Complex Matrices    3-7

Ans: (a) A = P + Q; ⎡ cosq 0 sin q ⎤


⎡ 11 ⎤ 8. ⎢ 0 1 0 ⎥⎥ .
⎢3 0 2⎥ ⎢
⎢ ⎥ ⎢⎣ − sin q 0 cos q ⎥⎦
P=⎢0 7
3⎥
(Symmetric) ⎡cosq 0 − sin q ⎤
⎢ 2⎥
⎢ ⎥ 9. ⎢ 0 1 0 ⎥⎥ .

⎢11 3 0 ⎥ ⎢⎣ sin q 0 cos q ⎥⎦
⎣⎢ 2 2 ⎥⎦
⎡ 1⎤ ⎡cos q − sin q 0 ⎤
⎢ 0 −2 2 ⎥ ⎢ ⎥
10. ⎢ sin q cos q 0 ⎥ .
⎢ ⎥
−5 ⎥ ⎢⎣ 0 0 1 ⎥⎦
Q=⎢ 2 0 (Skew-symmetric)
⎢ 2⎥ ⎡ cos f 0 sin f ⎤
⎢ ⎥
⎢ −1 5 11. ⎢ sin q sin f cosq − sin q cos f ⎥ .
0 ⎥ ⎢ ⎥
⎢⎣ 2 2 ⎥⎦ ⎢⎣ − cosq sin f sin q cosq cos f ⎥⎦
(b) B = P + Q,
3.4 COMPLEX MATRICES
⎡ a+c b + c⎤
⎢ a 2 2 ⎥ We now consider three classes of complex square
⎢ ⎥ matrices Hermitian, skew-Hermitian and unitary
c+a b + c⎥
P=⎢ b matrices which are not only important in applica-
⎢ 2 2 ⎥
⎢ ⎥ tions but are counter-parts, respectively, of real
⎢b + c b+c
b ⎥ square matrices namely, symmetric, skew-symmetric
⎣⎢ 2 2 ⎦⎥
and orthogonal matrices discussed earlier.
⎡ a−c b − c⎤
⎢ 0 2 2 ⎥
But first we define conjugate and conjugate
⎢ ⎥ transpose of a matrix.
c−a b − c⎥
Q=⎢ 0
⎢ 2 2 ⎥ 3.4.1 Conjugate of a Matrix
⎢ ⎥
⎢c − b c−b
0 ⎥ The matrix obtained from a given complex matrix
⎢⎣ 2 2 ⎥⎦
A on replacing each of its elements by the corres-
2. A and B are n-square matrices. If A is skew-symmetric ponding conjugate complex number is called the
then show that BT AB is skew-symmetric. conjugate of A and is denoted by A.
3. If A is a nonsingular symmetric matrix then show that
If A = [aij]m×n then its conjugate is A = [a ij]m×n
A−1 is symmetric.
where a ij is the complex conjugate of aij.
⎡ −1 1 1 1 ⎤ ⎡3 + 4i −7i 13 ⎤
E.g. If A= ⎢

1 1 −1 1 1 ⎥

⎣ 12 0 14 + 3i ⎥⎦
4. Show that the matrix ⎢ is
2 ⎢ 1 1 −1 1 ⎥ ⎡3 − 4i 7i 13 ⎤
⎢ ⎥ then A=⎢
⎣ 1 1 1 −1⎦ ⎣ 12 0 14 − 3i ⎥⎦
orthogonal. [JNTU 2004(4)]
⎡eix 1 − e − ix ⎤ ⎡e − ix 1 − eix ⎤
5. Prove that the following matrices are orthogonal: If B = ⎢ ⎥ then B=⎢ ⎥
⎣1 7 + 2i ⎦ ⎣ 1 7 − 2i ⎦
⎡1 2 2 ⎤ ⎡ −2 1 2⎤
1⎢ 1⎢
(a) A = ⎢2 1 −2⎥⎥ , (b) B = ⎢ 2 2 1⎥⎥ 3.4.2 Properties of Conjugate
3 3
⎢⎣2 −2 1 ⎥⎦ ⎣⎢ 1 −2 2⎥⎦ Matrices
Show that the following matrices are orthogonal: The following properties of a conjugate matrix A
⎡1 0 0 ⎤ could be easily verified
cos q sin q ⎤ ⎢0 cos q − sin q ⎥ .
6. ⎡ 7. 1. ( A) = A (conjugate of conjugate of A is
⎢ − sin q cos q ⎥ . ⎢ ⎥
⎣ ⎦ ⎢⎣0 sin q cos q ⎥⎦ A itself ).
3-8    Engineering Mathematics-II

2. ( A + B) = A + B (Conjugate of sum = sum For i = j, a ii = aii ⇒ aii – a ii = 0 ⇒ m(aii) = 0.


of conjugates). This implies that the main diagonal elements of a
3. ( kA) = k A (Conjugate of k times A is k times Hermitian matrix are purely real or zero.
its conjugate), k is a complex number. ⎡ 1 1+ i 2 − 3i ⎤
4. ( AB ) = A ⋅ B (Conjugate of a product = ⎡ 2 4 − 3i ⎤ ⎢ ⎥
E.g. ⎢ , 1− i 0 2 − i⎥ .
Product of their conjugates). ⎣ 4 + 3i 6 ⎥⎦ ⎢⎢ ⎥
⎣2 + 3i 2 +i 5 ⎦
3.4.3 Transposed Conjugate are Hermitian matrices.
(Tranjugate) of a Matrix
The transpose of the conjugate of a matrix A is called 3.4.6 Skew-Hermitian Matrix
the transposed conjugate (or tranjugate) of A and is A square matrix A = [aij] is called a skew-Hermitian
denoted by Aq or A*. matrix if a ji = −aij for all i, j. i.e., Aq = −A. For
T i = j, a ii = – aii ⇒ aii + a ii = 0 ⇒ Re (aii) =
Aq = ( A) = ( AT ) (Transpose of the conjugate
0. This implies that the main diagonal elements of a
of A = conjugate of the transpose of A)
skew-Hermitian matrix are purely imaginary or zero.
⎡ 2 − i 1 + 2i 2 + 3i ⎤
⎢ ⎥ ⎡ i 0 0⎤
⎡ 2i−1 + i ⎤ ⎢
E.g. If A = ⎢ 4 − 3i 6i 0 ⎥ E.g. ⎢1 + i , 0 0 i ⎥⎥
⎢ 1− i 0 ⎥⎦ ⎢
⎣ 2i 1 + 7i ⎥⎦ ⎣ ⎢⎣0 i 0⎥⎦
⎡ 2 + i 4 + 3i 1 + i ⎤ are skew-Hermitian matrices.
q ⎢ ⎥
then A = ⎢1 − 2i −6i − 2i ⎥
⎢2 − 3i 0 1 − 7i ⎥⎦ 3.4.7 Unitary Matrix

An n-square complex matrix is called unitary if
3.4.4 Properties of Tranjugate AqA = I.
Matrices
⎡ 1+ i ⎤ ⎡1 1 1⎤
1. (Aq)q = A (Tranjugate of tranjugate of A is A ⎢ 0 2⎥ 1⎢ ⎥
itself). E.g. A = ⎢ ⎥ , B = ⎢1 w w 2 ⎥ are
2. (A + B)q = Aq + Bq (Tranjugate of sum = ⎢1 − i ⎥ 3⎢ ⎥
0 ⎥ 2
⎢ 2
⎣ ⎦ ⎣1 w w⎦
sum of tranjugates).
3. (A)q = k Aq (Tranjugate of k times A = k unitary matrices of orders 2 and 3, respectively.
times the tranjugate of A).
4. (AB)q = BqAq (Tranjugate of a product = 3.5 PROPERTIES OF HERMITIAN,
product of the tranjugates in the reverse SKEW-HERMITIAN AND UNITARY
order). MATRICES
Theorem 3.11 If A is any square matrix then prove
3.4.5 Hermitian1 Matrix
that (i) A + Aq is Hermitian; (ii) AAq and AqA are
A square matrix A = [aij] is a Hermitian matrix if both Hermitian; (iii) A − Aq is skew-Hermitian.
a ji = aij for all i, j, i.e., Aq = A.
q q
Proof Since ( A ) = A
1
HERMITE, Charles (1832–1901), French algebraist, analyst
and number theorist. He solved the general quintic equation in (i) ( A + Aq )q = Aq + ( Aq )q = Aq + A = A + Aq ,
one variable by using elliptic functions. He was widely influential
and trained many distinguished mathematicians. ⇒ ( A + Aq ) is Hermitian
Real and Complex Matrices    3-9

(ii) ( AAq )q = ( Aq )q Aq , (Reversal Law) Proof Let P and Q be two given Hermitian
= AA q
⇒ q
AA is Hermitian matrices and A be any square matrix.
1 1
q q q
( A A) = A ( A ) q q Write P = ( A + Aq ) and Q = ( A − Aq )
2 2i
= Aq A ⇒ Aq A is Hermitian Then A = P + iQ; Now
(iii) ( A − Aq )q = Aq − ( Aq )q q
⎡1 ⎤ 1
Pq = ⎢ ( A + Aq )⎥ = ( Aq + A) = P
= Aq − A = − ( A − Aq ) ⎣ 2 ⎦ 2
⇒ A − Aq is skew-Hermitian q
⎡1 ⎤ 1 q
and Qq = ⎢ ( A − Aq )⎥ = ( A − A)
⎣ 2i ⎦ 2i
Theorem 3.12 Show that every square matrix can
1 1
be uniquely expressed as the sum of a Hermitian and =− ( A − Aq ) = ( A − Aq ) = Q
a skew-Hermitian matrix. 2i 2i
! i = −i
Proof Let A be the given square matrix;
∴ P and Q are both Hermitian.
⎡1 ⎤ ⎡1 ⎤ The uniqueness can be established proceeding
A = ⎢ ( A − Aq )⎥ + ⎢ ( A − Aq )⎥ = P + Q (say) on similar lines as in Theorem 3.12.
⎣2 ⎦ ⎣2 ⎦
q Theorem 3.14 Prove that the inverse and trans-
⎡1 ⎤ 1
Now Pq = ⎢ ( A + Aq )⎥ = ( Aq + A) pose of a unitary matrix are unitary.
⎣ 2 ⎦ 2
Proof Let A be unitary ⇒ Aq A = I
1
= ( A + Aq ) = P ⇒ P is Hermitian Taking the inverses of both sides we have
2
( Aq A) −1 = I −1 ⇒ A−1 ( Aq ) −1 = I
q
⎡1 ⎤ 1 (Reversal Law of inverses)
Also, Qq = ⎢ ( A − Aq ) ⎥ = ( A − ( Aq )q )
⎣ 2 ⎦ 2
⇒ A−1 ( A−1 )q = I [! ( Aq ) −1 = ( A−1 )q ]
1
= − ( A − Aq ) = − Q ⇒ A−1 is unitary
2
⇒ Q is skew-Hermitian Again, taking the transposes of both sides of
Thus, square matrix A = P + Q where P is AqA = I we have
Hermitian and Q is skew-Hermitian.
To prove the uniqueness of the representation ( Aq A)T = I T = I ⇒ AT ( Aq )T = I
assume that A = R + S where R is a Hermitian matrix ⇒ AT ( AT )q = I [! ( Aq )T = ( AT )q ]
and hence Rq = R and S is a skew-Hermitian matrix
⇒ AT is unitary
and hence Sq = −S
Theorem 3.15 Prove that the conjugate and
Aq = (R + S)q = Rq + Sq = R − S
conjugate transpose of a unitary matrix are unitary.
A + Aq A − Aq
so that R= = P, S = =Q Proof Let A be unitary ⇒ AqA = I
2 2
Taking conjugates of both sides
This proves the uniqueness of the representation.
( Aq A) = I ⇒ ( Aq )( A) = I
Theorem 3.13 Every square matrix A can be
⇒ ( A)q A = I [! ( Aq ) = ( A)q ]
expressed uniquely as P + iQ where P and Q are
Hermitian matrices. ⇒ A is unitary
3-10    Engineering Mathematics-II

Again, taking conjugate transposes of both Theorem 3.19 Prove that the eigenvalues of a
sides of AqA = I real symmetric matrix are all real.
we have ( Aq A)q = I q ⇒ Aq ( Aq )q = I Proof Let A be a real symmetric matrix. Then
⇒ Aq is unitary A = A (! A is real) and
Theorem 3.16 Prove that the product of two
unitary matrices of the same order is unitary. AT = A (! A is symmetric) (3.29), (3.30)

Proof Let A and B be two unitary matrices of the Now, Aq = (A)T = A, ⇒ A is Hermitian, and its
same order eigenvalues are all real.
⇒ AqA = I, BqB = I Theorem 3.20 Prove that the eigenvalues of a
q q q q skew-Hermitian matrix are purely imaginary or zero.
Now (AB) (AB) = B (A A) B = B B = I ⇒
AB is unitary. [JNTU 2002 S]

Theorem 3.17 Prove that the determinant of a Proof Let A be a skew-Hermitian matrix. Suppose
unitary matrix has absolute value 1. X ≠ 0 is the eigenvector corresponding to an eigen-
value l of A.
Proof Let A be a unitary matrix Then we have AX = lX (3.31)
⇒ Aq A = I ⇒ Aq A = I
⇒ (iA)X = (il)X, (3.32)
q 2
⇒ A A = A A =1 ⇒ A =1 This shows that il is an eigenvalue of the
matrix iA. Now
⇒ The absolute value of |A| is 1
(iA)q = −iAq = −i (−A) = iA [ A is skew-
Theorem 3.18 Prove that the eigenvalues of a
Hermitian] (3.33)
Hermitian matrix are all real.
⇒ (iA) is a Hermitian matrix
Proof Let A be a Hermitian matrix Aq = A

il being an eigenvalue of the Hermitian matrix
(3.24)
iA, must be real.
If X ≠ 0 is the eigenvector corresponding to an
il will be real provided l = 0 or l is purely
eigenvalue l of A then AX = lX (3.25)
imaginary.
Pre-multiplying both sides of (3.25) by X0,
X AX = l XqX
q
(3.26) Theorem 3.21 Prove that the eigenvalues of a
real skew-symmetric matrix are all purely imagi-
Taking conjugate transposes on both sides nary or zero.
of (3.26)
Proof Let A be a real skew-symmetric matrix so
( X q AX )q = l ( X q X )q that A = A and AT = −A.
Taking the transposes of both sides
⇒ X q Aq ( X q )q = l X q ( X q )q
⇒ X q AX = l X q X (! Aq = A) (A)T = AT = −A ⇒ Aq = −A, A is skew-
(3.27) Hermitian.
Consequently, we conclude that the eigenvalues
From (3.26) and (3.27) we obtain of real skew-symmetric matrix are either zero or
(l − l ) X q X = 0 purely imaginary.
⇒ l=l (! X ≠ 0) ⇒ l is real Theorem 3.22 Prove that the eigenvalues of a
(3.28) unitary matrix are of unit modulus. [JNTU 2002 S]
∴ The eigenvalues of a Hermitian matrix are Proof Let A be a unitary matrix so that AqA = I
all real. (3.34)
Real and Complex Matrices    3-11

Suppose that X ≠ 0 is the eigenvector corres- Taking the transposes of both sides
ponding to an eigenvalue l of A ⎡ 2 3 − 4i ⎤
Aq = ( A)T = ⎢ =A
AX = lX (3.35) ⎣3 + 4i 2 ⎥⎦
Taking conjugate transposes of both sides ⇒ A is Hermitian (3)
( AX )q = ( lX )q ⇒ X q Aq = lX q (3.36)
⎡ 2i i (3 − 4i )⎤
Multiplying equations (3.35) and (3.36) Now, iA = ⎢
⎣i (3 + 4i ) 2i ⎥⎦
( X q Aq )( AX ) = ( lX q )( lX ) ⎡ 2i 4 + 3i ⎤
=⎢ ⎥ (4)
⇒ X q ( Aq A) X = ll( X q X ) ⎣ −4 + 3i −2i ⎦
(Associative Law) ⎡ 2i 4 + 3i ⎤
⇒ ( iA) = ⎢
⇒ X q X = ( ll ) X q X by (3.34) ⎣ −4 + 3i 2i ⎥⎦
⇒ (1 − ll ) X q X = 0 (3.37) ⎡ −2i 4 − 3i ⎤
=⎢
⎣ − 4 − 3i −2i ⎥⎦
Since X ≠ 0, XqX ≠ 0
∴ 1 − ll = 0 ⇒ ll = 1 Taking transposes of both sides


2
l = 1⇒ l = 1 ⎡ −2i −4 − 3i ⎤
(3.38) (iA)q = (iA)T = ⎢
⎣ 4 − 3i −2i ⎥⎦
∴ The eigenvalues of a unitary matrix are of
unit modulus. ⎡ 2i 4 + 3i ⎤
=−⎢ = − (iA)
⎣ 4 − 3i 2i ⎥⎦
⇒ iA is skew-Hermitian.
Complex matrix Real matrix
Example 3.10
Im l Skew-Hermitian (Skew-symmetric)
⎡ 1+ i ⎤
Unitary (Orthogonal) ⎢ 0 2⎥
Prove that A = ⎢ ⎥ is a unitary matrix.
Hermitian (Symmetric)
⎢1 − i ⎥
⎢ 2 0 ⎥
⎣ ⎦
Re l
Solution

⎡ 1+ i ⎤ ⎡ 1+ i ⎤
⎢ 0 2 ⎥ ⎢ 0
2⎥
A= ⎢ ⎥ ⇒ Aq = ⎢ ⎥
⎢1 − i ⎥ ⎢1 − i ⎥
⎢ 2 0 ⎥ ⎢ 2 0 ⎥
⎣ ⎦ ⎣ ⎦
Example 3.9 ⎡ 1 + i ⎤ ⎡ 1 + i ⎤
⎢ 0 ⎥ ⎢ 0 ⎥
⎡ 2 3 − 4i ⎤ 2 2
Show that the matrix A = ⎢ is Hermitian Aq A = ⎢ ⎥ ⎢ ⎥
⎣3 + 4i 2 ⎥⎦ ⎢1 − i ⎥ ⎢1 − i
0 ⎥ ⎢ 0 ⎥

and iA is skew-Hermitian. ⎢ 2 2
⎣ ⎦ ⎣ ⎦
Solution ⎡ 1+ i 1− i 1+ i 1+ i ⎤
⎡ 2 3 − 4i ⎤ ⎡ 2 3 + 4i ⎤ ⎢0 ⋅ 0 + 2 ⋅ 2 0 ⋅ 2 + 2 ⋅ 0⎥
A= ⎢ ⎥ , A = ⎢3 − 4i =⎢ ⎥
⎣ 3 + 4i 2 ⎦ ⎣ 2 ⎥⎦ ⎢1 − i 1− i 1− i 1+ i ⎥
⎢ 2 ⋅0 + 0⋅ 2 2

2
+ 0 ⋅ 0⎥
(1), (2) ⎣ ⎦
3-12    Engineering Mathematics-II

⎡ 12 + 12 ⎤ Example 3.13
0 ⎡ 1 i⎤
⎢ ⎥ Show that the matrix A = ⎢ ⎥ is Hermitian and
=⎢
2⋅ 2 ⎥ = ⎡1 0⎤ = I ⎣ −i 1⎦
⎢ 2 2 ⎥ ⎢0 1⎥
1 +1 ⎣ ⎦ ⎡i −1⎤
⎢ 0 ⎥ the matrix B = iA = ⎢ ⎥ is skew-Hermitian. Find
⎣ 2⋅ 2⎦ ⎣1 i ⎦
Example 3.11 the eigenvalues and eigenvectors of A and B.
⎡1 1 1⎤ ⎡i −1⎤
1 ⎢ Solution The given matrix is B = iA =
2⎥ ⎢1
Prove that B = ⎢1 w w ⎥ is a unitary matrix, ⎣ i ⎥⎦
3⎢ 2 ⎥
⎣1 w w⎦ T T
⎡1 i⎤ ⎡1 −i ⎤
if w is a cube root of unity. Aq = ( A)T = ⎢ ⎥ =⎢ ⎥
⎣ −1 1⎦ ⎣i 1 ⎦
Solution w is a cube root of unity
⎡ 1 i⎤
⎧⎪ w3 = 1 =⎢ ⎥= A ⇒ A is Hermitian
⇒ ⎨ 2
⎣ −i 1⎦
⎪⎩1 + w + w = 0 The characteristic equation of A is
⎛ −1 + i 3 2 −1 − i 3 ⎞ 1− l i
⎜w = 2
,w =
2 ⎟ A − lI =
−i 1 − l
=0
⎝ ⎠
⎡1 1 1⎤ ⎡1 1 1⎤ ⇒ ( l 2 − 1)2 − i ⋅ i = 0
1 ⎢ 2⎥ 1 ⎢ 2⎥
q
B B= ⎢1 w w ⎥ ⋅ ⎢1 w w ⎥ ⇒ l 2 − 2l = 0 ⇒ l1 = 0, l2 = 2
3⎢ 2 ⎥ 3⎢ 2 ⎥
⎣1 w w⎦ ⎣1 w w⎦ are the eigenvalues of A, which are real since A is a
Hermitian matrix.
⎡1 1 1 ⎤ ⎡1 1 1⎤
1⎢ ⎥ ⎢ ⎥ To find the eigenvectors corresponding to the
2
= ⎢1 w w ⎥ ⎢1 w w 2 ⎥ eigenvalue l of A we have to solve
3⎢ 2⎥ ⎢ 2 ⎥
⎣1 w w ⎦ ⎣1 w w⎦ ⎡1 − l i ⎤ ⎡ x1 ⎤ ⎡ 0⎤
( A − lI ) X = ⎢ =
⎡ 3 1+ w + w 2 1+ w 2 + w ⎤ ⎣ − i 1 − l ⎥⎦ ⎢⎣ x2 ⎥⎦ ⎢⎣ 0⎥⎦
1⎢ ⎥
= ⎢1 + w 2 + w 1 + w 3 + w 3 1 + w 4 + w 2 ⎥
3⎢ For l1 = 0
2 2 4 3 3⎥
⎣⎢1 + w + w 1 + w + w 1 + w + w ⎦⎥
⎡1 − 0 i ⎤ ⎡ x1 ⎤ ⎡0⎤ ⎡ x1 + ix2 = 0 ⎤
⎡1 0 0 ⎤ ⎢ −i 1 − 0⎥ ⎢ x ⎥ = ⎢ 0⎥ ⇒ ⎢ −ix + x = 0⎥
= ⎢⎢0 1 0 ⎥⎥ = I 3 ⎣ ⎦ ⎣ 2⎦ ⎣ ⎦ ⎣ 1 2 ⎦
Solving we get
⎣⎢0 0 1 ⎥⎦
x1 x2 ⎡ x1 ⎤ ⎡i ⎤
Example 3.12 = = c1 (say ) ⇒ X 1 = ⎢ ⎥ = c1 ⎢ ⎥
i −1 ⎣ x2 ⎦ ⎣ −1⎦
⎡ 0 a + ib ⎤
Show that C = ⎢ is unitary if a2 + b2 =
⎣c + id 0 ⎥⎦
For l2 = 2
c2 + d2 = 1.
Solution ⎡1 − 2 i ⎤ ⎡ x1 ⎤ ⎡ 0⎤ ⎡ − x1 + ix2 = 0⎤
⎡ 0 c − id ⎤ ⎡ 0 a + ib ⎤ ⎢ −i 1 − 2⎥ ⎢ x ⎥ = ⎢ 0⎥ ⇒ ⎢ −ix − x = 0⎥
C q⋅ C = ⎢ ⎣ ⎦ ⎣ 2⎦ ⎣ ⎦ ⎣ 1 2 ⎦
⎥ ⎢ 0 ⎥⎦
⎣ a − ib 0 ⎦ ⎣ c + id Solving we get
⎡c2 + d 2 0 ⎤ x1 x2 ⎡ x1 ⎤ ⎡i ⎤
=⎢ ⎥ = I2 = = c 2 (say ) ⇒ X 2 = ⎢ ⎥ = c2 ⎢ ⎥
⎢⎣ 0 a2 + b2 ⎥⎦ i 1 ⎣ x2 ⎦ ⎣1⎦
Real and Complex Matrices    3-13

⎡ 1 i ⎤ ⎡i −1⎤ B = iA; but the eigenvectors of A and B remain the


(ii) B = iA = i ⎢ ⎥=⎢ ⎥ same (unchanged).
⎣ −i 1⎦ ⎣1 i ⎦
T T
q T ⎡i −1⎤ ⎡ −i −1⎤ Example 3.14
B = ( B) = ⎢ ⎥ = ⎢ 1 −i ⎥
⎣ 1 i ⎦ ⎣ ⎦ ⎡ i 0 0⎤
⎡ −i 1 ⎤ ⎡i −1⎤ Show that A = ⎢⎢0 0 i ⎥⎥ is skew-Hermitian and
=⎢ ⎥ = − ⎢1 i ⎥ = − B
⎣ −1 − i ⎦ ⎣ ⎦ ⎢⎣0 i 0⎥⎦
⇒ B is skew-Hermitian also unitary. Find its eigenvalues and eigenvectors.

The characteristic equation of B is ⎡ i 0 0⎤


i − 1 −1 ⎢
Solution The given matrix A = 0 0 i

B − lI = =0 ⎢ ⎥
1 i−l ⎢⎣0 i 0⎥⎦
⇒ ( l − i )2 + 1 = 0 ⇒ l 2 − 2i l = 0 ⎡ −i 0 0⎤ ⎡ −i 0 0⎤
⇒ l1 = 0, l2 = 2i A = ⎢⎢ 0 0 −i ⎥ and A = ( A) = ⎢⎢ 0
⎥ q T
0 −i ⎥⎥ = − A,
are the eigenvalues of the skew-Hermitian matrix B. ⎢⎣ 0 −i 0 ⎥⎦ ⎢⎣ 0 −i 0 ⎥⎦
The eigenvalues of a skew-Hermitian matrix should
be zero or purely imaginary, and we have them here Hence A is skew-Hermitian.
accordingly. Also
To find the eigenvector corresponding to the ⎡ i 0 0 ⎤ ⎡ −i 0 0 ⎤ ⎡1 0 0⎤
eigenvalue l of B we have to solve
AAq = ⎢⎢0 0 i ⎥⎥ ⎢⎢ 0 0 −i ⎥⎥ = ⎢⎢ 0 1 0⎥⎥ = I
⎡i − 1 −1 ⎤ ⎡ x1 ⎤ ⎡0⎤ ⎢⎣0 i 0⎥⎦ ⎢⎣ 0 −i 0 ⎥⎦ ⎢⎣0 0 1⎥⎦
( A − lI ) X = ⎢ ⎥⎢ ⎥=⎢ ⎥
⎣ 1 i − l ⎦ ⎣ x2 ⎦ ⎣0⎦
∴ A is unitary also.
For l1 = 0 To find eigenvalues and eigenvectors we can
proceed as follows.
⎡i − 0 −1 ⎤ ⎡ x1 ⎤ ⎡0 ⎤ ⎡ix1 − x2 = 0 ⎤
⎢ 1 i − 0 ⎥ ⎢ x ⎥ = ⎢0 ⎥ = ⎢ x + ix = 0 ⎥ ⎡1 0 0⎤
⎣ ⎦ ⎣ 2⎦ ⎣ ⎦ ⎣ 1 ⎦
Write A = iB where B = ⎢0 0 1⎥; the eigen-
2
x1 x2 ⎢ ⎥
Solving we get = = C1 (say) ⎢⎣0 1 0⎥⎦
i −1
⎡x ⎤ ⎡i ⎤ values of A are i times those of B.
= X1 = ⎢ 1 ⎥ = C1 ⎢ ⎥ The characteristic equation of B is
⎣ x2 ⎦ ⎣ −1⎦

For l2 = 2i 1− l 0 0
B − lI = 0 ⇒ 0 0−l 1 =0
⎡i − 2i −1 ⎤ ⎡ x1 ⎤ ⎡ 0⎤ ⎡ −ix1 − x2 = 0⎤
⎢ 1 = ⇒ 0 1 0−l
⎣ i − 2i ⎦⎥ ⎢⎣ x2 ⎥⎦ ⎢⎣0⎥⎦ ⎢ x − ix = 0 ⎥
⎣ 1 2 ⎦
⇒ (1 − l )( l 2 − 1) = 0 ⇒ l1 = − 1, l2 = 1, l3 = 1
Solving we get x1 = x2 = − c2 (say)
i 1 are the eigenvalues of B
x ⇒ i(l1, l2, l3) = i (−1, 1, 1) = (−i, i, i) are
⎡ ⎤ ⎡i ⎤
⇒ X 2 = ⎢ 1 ⎥ = c2 ⎢ ⎥ the eigenvalues of A, which are purely imaginary as
⎣ x2 ⎦ ⎣1⎦ they should be.
Note 1 The eigenvalues of B are (il1, il2) = To find the eigenvectors of B we have to solve
(0, 2i) where l1, l2 are the eigenvalues of A since (B − lI)X = 0
3-14    Engineering Mathematics-II

⎡1 − l 0 0 ⎤ ⎡ x1 ⎤ ⎡0⎤ ⎡ x1 ⎤ EXERCISE 3.2


⎢ 0
⎢ −l 1 ⎥⎥ ⎢⎢ x2 ⎥⎥ = ⎢⎢0⎥⎥ , taking X = ⎢x ⎥
⎡ 4 1 − 3i ⎤
⎢ 2⎥ 1. Prove that A = ⎢ is Hermitian. Find its
⎣⎢ 0 1 − l ⎦⎥ ⎣⎢ x3 ⎦⎥ ⎣⎢0⎦⎥ ⎢⎣ x3 ⎥⎦ ⎣1 + 3i 7 ⎥⎦
eigenvalues.
For l1 = −1 Ans: Characteristic equation is l2 − 11l + 18 = 0;
⎡1 + 1 0 0⎤ ⎡ x1 ⎤ ⎡0⎤ eigenvalues 9, 2
⎢ 0 1 1⎥ ⎢ x ⎥ = ⎢0⎥ 2 x1 = 0
⇒ ⎡ i 3⎤
⎢ ⎥ ⎢ 2⎥ ⎢ ⎥ x2 + x3 = 0 ⎢ ⎥
⎢⎣ 0 1 1⎥⎦ ⎢⎣ x3 ⎥⎦ ⎢⎣0⎥⎦ 2 2 ⎥
2. Prove that U = ⎢ is unitary and find its
⎢ 3 i ⎥
Solving we get ⎢ ⎥
⎣ 2 2 ⎦
eigenvalues.
⎡ x1 ⎤ ⎡0⎤
x1 x2 x3 Ans: Characteristic equation is l2 − il − 1 = 0; eigen-
= = = c1 (say) ⇒ X1 = ⎢⎢ x2 ⎥⎥ = c1 ⎢ ⎥
⎢1⎥
0 1 −1 values: 3 + i , − 3 + i
⎢⎣ x3 ⎥⎦ ⎢⎣ −1⎥⎦ 2 2
is the eigen-vector. ⎡ 0 1 + 2i ⎤
3. If A = ⎢ then show that (I − A)(I + A)−1
⎣ −1 + 2i 0 ⎥⎦
For l2 = 1
is a unitary matrix.
⎡1 − 1 0 0 ⎤ ⎡ x1 ⎤ ⎡ 0 ⎤ 4. Show that the column (and also row) vectors of the
⎢ 0 1 1 ⎥ ⎢x ⎥ = ⎢ 1 ⎥ x1 = arbitrary
⎢ ⎥ ⎢ 2⎥ ⎢ ⎥ ⇒ 1 ⎡1 + i −1 + i ⎤
x2 = x3 unitary matrix A = ⎢ form an ortho-
⎢⎣ 0 1 −1⎥⎦ ⎢⎣ x3 ⎥⎦ ⎢⎣ −1⎥⎦ 2 ⎣1 + i 1 − i ⎥⎦
gonal system.
We choose x1 so that two L.I. vectors are obtained:
⎡0 ⎤ ⎡1 ⎤ 5. Find the eigenvalues and eigenvectors of the unitary
⎡ 1 i ⎤
X 2 = c2 ⎢⎢1 ⎥⎥ , X 3 = c3 ⎢⎢0 ⎥⎥ ⎢ 2
matrix ⎢ 2⎥.

⎣⎢1 ⎦⎥ ⎣⎢0 ⎦⎥ ⎢ −i −1 ⎥
⎢⎣ 2 2 ⎥⎦
The absolute value of each eigenvalue of A is 1.
 |−i| = |i| = 1 as A is unitary. The eigenvectors Ans: Eigenvalues: 1, −1; eigenvectors (1, ±i 2)T
of A are those of B only.
Quadratic Forms
4
4.1 INTRODUCTION Putting cij = 12 ( aij + a ji ) = c ji and cij + c ji = aij + a ji ,
Second-degree homogeneous expressions are a quadratic form can be written as
called quadratic forms. They occur in physics and
n n
in geometry. In analytical geometry, for instance,
a quadratic form has to be transformed into its Q = ∑∑ cij xi x j
i =1 j =1
principal-axes-form so as to determine the nature
of the conic section such as parabola, ellipse or
hyperbola, etc., if it involves two variables, and of 1
where C = ⎡⎣cij ⎤⎦ = ⎡⎣ A + AT ⎤⎦ is a real symmetric
the quadratic surface such as paraboloid, ellipsoid 2
or hyperboloid, etc., if it involves three variables. matrix.
A quadratic form can be represented by Thus, the coeficient matrix in a quadratic form
n which is called the matrix of the quadratic form can
X T AX = ∑ aij xi x j (4.1) always be assumed to be or can be constructed in the
i , j =1 form of a real symmetric matrix.
where X is a column n-vector and A is a symmetric
matrix of the coefficients. We study here the method Note 1 The rank of a quadratic form is the rank
of transformation of a quadratic form (4.1) to sum- of the coefficient matrix. The quadratic form is
of-squares (Canonical) form to determine its nature. singular if n > r where n is the number of variables
and r is the rank.
4.2 QUADRATIC FORMS
4.2.1 Quadratic Form: Denition Example 4.1

A quadratic form in n variables x1, x2, ..., xn is an Express 2x2 − 5xy − 3y2, a quadratic form in two
expression of the form variables x and y in the form X T AX.
n n
Q = X T AX = ∑∑ aij xi x j Solution
i =1 j =1
5 5
= a11 x12 + a12 x1 x2 + ! + a1n x1 xn 2 x 2 − 5 xy − 3 y 2 = 2 x 2 − xy − yx − 3 y 2
2 2
+ a21 x2 x1 + a22 x22 + ! + a2n x2 xn ⎡ 2 −5 / 2⎤ ⎡ x ⎤
= [x y] ⎢
+ ! + an1 xn x1 + an2 xn x2 + ! + ann xn2 ⎣ −5 / 2 −3 ⎥⎦ ⎢⎣ y ⎥⎦
Here A is called the coefficient matrix. Rewriting
Example 4.2
Q = X T AX = a11 x12 + ( a12 + a21 ) x1 x2 + ! +
Write ax2 + by2 + cz2 + 2fyz + 2gzx + 2hxy, a general
( a1n + an1 ) x1 xn + a22 x22 + ( a23 + a32 ) x2 x3 quadratic form in three variables x, y and z in the
+ ! + ( a2n + an2 ) x2 xn + ! + ann xn2 form XT AX.
4-2    Engineering Mathematics-II

Solution Symmetric matrix


⎡a h g⎤ ⎡x⎤ 1
C = ( A + AT )
If A = ⎢⎢ h b f ⎥⎥ X = ⎢⎢ y ⎥⎥ , X T = [ x y z] 2
⎢⎣ g f c ⎥⎦ ⎢⎣ z ⎥⎦ ⎛ ⎡1 2 0⎤ ⎡1 0 0⎤⎞ ⎡1 1 0⎤
1 ⎜⎢
= ⎢ 0 3 6⎥⎥ + ⎢⎢ 2 3 0⎥⎥⎟ = ⎢⎢1 3 3⎥⎥
2⎜⎜ ⎟
then the quadratic form is q = XT AX = ax2 + by2 + ⎝ ⎢⎣0 0 2⎥⎦ ⎢⎣0 6 2⎥⎦⎟⎠ ⎢⎣0 3 2⎥⎦
cz2 + 2fyz + 2gzx + 2hxy, which is given above.
The simplest way to write matrix C is
Example 4.3
1. Put the coefficients of square terms as the
Obtain the matrix of the quadratic form diagonal elements.
x12 − 2 x1 x2 + 2 x22 . 1
2. Place 2
of aij , the coefficient of xi xj at cij and
Solution
the remaining half of aji at cji , i.e., cij = 12 aij so
If XT AX is the quadratic form then the coefficient
⎡ 1 −1⎤ that cij + c ji = 12 ( aij + a ji ) = aij . For example,
matrix is A = ⎢ ⎥ since the quadratic form
⎣ −1 2 ⎦ the coefficient of x2x3 in Example 4.5 is
distributed equally as 3 and 3 to c23 and c32.
⎡ 1 −1⎤ ⎡ x1 ⎤
= x12 − x1 x2 − x2 x1 + 2 x2 = [ x1 x2 ] ⎢ ⎥⎢ ⎥ Example 4.6
⎣ −1 2 ⎦ ⎣ x2 ⎦
Find real symmetric matrix C such that Q = XT CX
2 2
Example 4.4 where Q = 6 x1 − 4 x1 x2 + 2 x2.
Obtain the matrix of the quadratic form x2 + 2y2 +
Solution
3z2 + 4xy + 5yz + 6zx .
Q = 6 x12 − 4 x1 x2 + 0.x2 x1 + 2 x22
Solution
The matrix of the quadratic form is
⎡1 2 3 ⎤ ⎡x⎤
X T AX = [ x y z ] ⎢⎢2 2 5 / 2⎥⎥ ⎢ ⎥
⎢ y⎥ ⎡6 −4⎤ ⎡ 6 0⎤
A= ⎢ ⎥ ⇒ AT = ⎢ ⎥
⎢⎣ 3 5 / 2 3 ⎥⎦ ⎢⎣ z ⎥⎦ ⎣0 2 ⎦ ⎣ −4 2⎦
∴ matrix of the quadratic form is ∴ Symmetric matrix
1 1 ⎡12 −4 ⎤ ⎡ 6 −2⎤
⎡1 2 3 ⎤ C = ( A + AT ) = ⎢ =
2 2 ⎣ −4 4 ⎥⎦ ⎢⎣ −2 2 ⎥⎦
A = ⎢2 2 5 / 2⎥⎥

Example 4.7
⎣⎢ 3 5 / 2 3 ⎦⎥
Find the real symmetric matrix C such that Q =
Example 4.5 XT CX where Q = 2( x1 − x2 )2 = 2 x12 − 4 x1 x2 + 2 x22.
Find a real symmetric matrix C of the quadratic
form Q = x12 + 3x22 + 2 x32 + 2 x1 x2 + 6 x2 x3 . Solution
⎡ 2 −4⎤ ⎡ 2 0⎤
Solution A= ⎢ ⎥ ⇒ AT = ⎢ ⎥
⎣0 2 ⎦ ⎣ −4 2⎦
⎡1 2 0⎤
1 ⎡ 2 −2⎤
The coefficient matrix of Q is A = ⎢⎢0 3 6⎥⎥ Symmetric matrix C = ( A + AT ) = ⎢ ⎥
2 ⎣ −2 2 ⎦
⎣⎢0 0 2⎥⎦
Example 4.8
since Q = x12 + 2 x1 x2 + 0.x1 x3 + 0.x2 x1 + 3x22 + 6 x2 x3 Determine the real symmetric matrix C such that
= + 0.x3 x1 + 0.x3 x2 + 2 x32 Q = XT CX where Q = (x1 + x2 + x3)2.
Quadratic Forms    4-3

Solution We have where P is known as a modal matrix and D is called


a spectral matrix. D is actually a diagonal matrix
Q= x12 + 2 x1 x2 + 2 x1 x3 + 0. x2 x1 + x22 + 2 x2 x3 consisting of the eigenvalues of A as its diagonal
+ 0. x3 x1 + 0. x3 x2 + x32 elements.
The matrix P which is the matrix of transforma-
⎡ 1 2 2⎤ ⎡1 0 0⎤ tion is an orthogonal matrix here, and therefore the
Here A = ⎢0 1 2⎥ and A = ⎢⎢ 2 1 0⎥⎥
⎢ ⎥ T above method of reduction is called the orthogonal
transformation, which is explained in Sections 4.3
⎢⎣0 0 1⎥⎦ ⎢⎣ 2 2 1⎥⎦ and 4.4. There are two more methods: Diagonalisa-
tion by simultaneous row/column transformations
⎡1 1 1⎤
1 and Lagrange’s method, explained in Section 4.4
Symmetric matrix C = ( A + A ) = ⎢⎢1 1 1⎥⎥
T
through examples.
2
⎢⎣1 1 1⎥⎦

Example 4.9 4.3.1 Index and Signature of a Real


Obtain the real symmetric matrix C such that Q = Quadratic Form
XT CX where Q = 4 x1 x3 + 2 x2 x3 + x32. Let r be the rank of a matrix A in the quadratic
form (4.1).
Solution We have
Index s: The index s of the real quadratic form
Q = 0.x12 + 0.x1 x2 + 4 x1 x3 + 0.x2 x1 + 0.x22 + 2 x2 x3 (4.1) is the number of positive square terms in
the canonical form of a real quadratic form, e.g.,
+ 0.x3 x1 + 0.x3 x2 + 1.x32 if the canonical form of a real quadratic form is
2 y12 − y22 − y32 then its index = 1.
⎡0 0 4⎤ ⎡ 0 0 2⎤
⎢ ⎥ ⎢ 0 0 1⎥ Signature: The signature of a real quadratic
Here A = ⎢0 0 2⎥ and AT = ⎢ ⎥ form is the number of the positive terms minus the
⎣⎢0 0 1 ⎥⎦ ⎣⎢ 2 1 1⎥⎦ number of nonpositive terms in the canonical form.
⎡0 0 2⎤ e.g., if the canonical form of a real quadratic form is
1
∴ C = ( A + A ) = ⎢⎢0
T
0 1⎥⎥ 2 y12 − y22 − y32 then its signature = 1 − 2 = −1.
2
⎢⎣ 2 1 1⎥⎦
4.4 NATURE OF REAL QUADRATIC
4.3 CANONICAL FORM (OR) SUM FORMS
OF THE SQUARES FORM A real nonsingular quadratic form Q = XT AX (with
Consider a real quadratic form |A| ≠ 0) is said to be:
n n
Q = X T AX = ∑∑ aij xi x j (4.1) 4.4.1 Positive Denite
i =1 j =1
If the rank and index of Q are equal to the number
where A is the coefficient matrix, X = [x1, x2, …, xn]T of variables, i.e., r = s = n or what is the same thing,
and XT = [x1, x2, …, xn] are, respectively, the column if each of the eigenvalues of A > 0 then Q is positive
and row vectors of n variables. definite.
By the application of an orthogonal transforma-
tion defined by X = PY the quadratic form (4.1) can 4.4.2 Negative Denite
be transformed to the following sum of the squares
If the index of Q equals zero, i.e., r = n and s = 0 or
form or canonical form
what is the same thing if each of the eigenvalues of
Y T DY = λ1 y12 + λ 2 y22 + ! + λ n y x2 A < 0 then Q is negative definite.
4-4    Engineering Mathematics-II

4.4.3 Positive Semi-Denite −1 7 −7 1


1
If rank and index of Q are equal but less than the −1 6 −1
number of variables, i.e., −1 6 −1 0
s = r < n (|A| = 0)
So, the quadratic form is positive definite. Index =
or, what is the same thing, if all of the eigenvalues of
no. of positive eigenvalues = 3.
A are nonnegative, i.e., li ≥ 0 and at least one eigen-
Signature = no. of positive eigenvalues–number
value is 0 then Q is positive semi-definite.
of nonpositive eigenvalues = 3 − 0 = 3.
Example 4.11
4.4.4 Negative Semi-Denite
Find the nature, index and signature of the quadratic
If the index is zero and the rank r < n.
form 2x1x2 + 2x1x3 + 2x2x3.
i.e., s = 0 and r < n (|A| = 0)
or what is the same thing, if all of the eigenvalues of Solution The real symmetric matrix A associated
A are nonpositive, i.e., li ≤ 0 and at least one eigen- with the quadratic form is
value is 0 then Q is positive semi-definite. ⎡0 1 1⎤
A = ⎢⎢1 0 1⎥⎥
4.4.5 Indenite ⎢⎣1 1 0⎥⎦
If Q is not of any type described above or what is the The characteristic equation of A is
same thing, A has some positive and some negative
0−l 1 1
eigenvalues, then Q is indefinite.
P(l ) = 1 0−l 1
Note 1 If Q is negative definite (semi-definite) 1 1 0−l
then −Q is positive definite (semi-definite).
= (0 −l) (l 2 − 1) − 1( − l − 1) + 1(1 + l ) = 0
Example 4.10 ⇒ − l 3 + 3l + 2 = 0 ⇒ l = −1, − 1, 2
Determine the nature, index and signature of the
−1 0 3 2
quadratic form −1
1 −1 −2
2 x12 + 2 x22 + 3x32 + 2 x1 x2 − 4 x1 x3 − 4 x2 x3
−1 1 2 0
−1
Solution The real symmetric matrix A associated 1 −2
with the quadratic form is
−1 2 0
⎡ 2 1 −2⎤ Since one of the eigenvalues is positive, and
A = ⎢⎢ 1 2 −2⎥⎥ the other is negative the quadratic form is indefinite.
⎢⎣ −2 −2 3 ⎥⎦ Index = 1, signature 1 − 2 = − 1
The characteristic equation of matrix A is Example 4.12

2−l 1 −2 Identify the nature, index and signature of the


quadratic form
P(l ) = 1 2 − l −2
−2 −2 3 − l x12 + 4 x22 + x32 − 4 x1 x2 + 2 x3 x1 − 4 x2 x3

= (2 − l )( l 2 − 5l + 2) − 1( − l − 1) − 2(2 − 2l ) = 0 Solution The real symmetric matrix A associated


with the quadratic form is
⇒ − l 3 + 7l 2 − 7l + 1 = 0 ⎡ 1 −2 1 ⎤
⇒ l =1 or l 2 − 6l + 1 = 0 A = ⎢⎢−2 4 −2 ⎥⎥
∴ l = 1, 3 + 2 2, 3 − 2 2, which are all positive. ⎣⎢ 1 −2 1 ⎦⎥
Quadratic Forms    4-5

The characteristic equation of A is −Q = 3x12 + 3x22 + 3x32 + 2 x1 x2 + 2 x1 x3 − 2 x2 x3


1− l −2 1 is positive definite.
P ( l ) = −2 4 − l −2 4.5 REDUCTION OF A QUADRATIC FORM
1 −2 1 − l TOCANONICAL FORM
= (1 − l )( l 2 − 5l ) + 2(2l ) + ( l ) = 0 We know that a homogeneous expression of second
degree in any number of variables is called a qua-
⇒ − l 3 + 6l 2 = 0, l = 0, 0, 6
dratic form. For instance, let
li ≥ 0).

So, the quadratic form is semi-definite (
Index = 3, signature = 3. ⎡a h
g⎤ ⎡x⎤
A = ⎢⎢ h f ⎥⎥ , X = ⎢⎢ y ⎥⎥ , X T = [ x
b y z ] (4.2)
Example 4.13 ⎢⎣ g c ⎥⎦
f ⎢⎣ z ⎥⎦
Classify the quadratic form and find the index and Then, the expression
signature of
XT AX = ax2 + by2 + cz2 + 2fyz + 2gzx + 2hxy (4.3)
−3x12 − 3x22 − 3x32 − 2 x1 x2 − 2 x1 x3 + 2 x2 x3 is a quadratic form.
Let l1, l2, l3 be the eigenvalues of the matrix
Solution The real symmetric matrix A associated A and
with the quadratic form is
⎡ x1 ⎤ ⎡ x2 ⎤ ⎡ x3 ⎤
⎡ −3 −1 −1⎤
X1 = ⎢⎢ y1 ⎥⎥ , X 2 = ⎢⎢ y2 ⎥⎥ , X 3 = ⎢⎢ y3 ⎥⎥
(4.4)
A = ⎢⎢ −1 −3 1 ⎥⎥
⎢⎣ z1 ⎥⎦ ⎢⎣ z2 ⎥⎦ ⎢⎣ z3 ⎥⎦
⎢⎣ −1 1 −3⎥⎦
be the corresponding eigenvectors in the normalized
The characteristic equation of A is form, i.e., in the form in which each component is
divided by the length of the vector.
−3 − l −1 −1 Denote the square matrix
P ( l ) = −1 −3 − l 1
⎡ x1 x2 x3 ⎤
−1 1 −3 − l [ X1 X2 X 3 ] = ⎢⎢ y1 y2 y3 ⎥⎥ by P
= ( −3 − l )( l 2 + 6l + 8) + (4 + l ) ⎢⎣ z1 z2 z3 ⎥⎦
+ (4 + l ) = 0 We have
⇒ 3 2
l + 9 l + 24l + 16 = 0 AP = A[ X 1 X2 X 3 ] = [ AX 1 AX 2 AX 3 ]
= [l1 X1, l2 X 2 , l3 X 3 ]
⇒ l = −1, − 4, − 4
⎡ l1 0 0⎤
1 9 24 16
−1 = [ X1 X2 X 3 ] ⎢⎢ 0 l2 0 ⎥⎥ = PD
−1 −8 −16 ⎢⎣ 0 0 l3 ⎥⎦
18 16 0 (4.5)
−4
−4 −16 Let Q be the quadratic form given by (4.1). The
1 4 0 coefficient matrix A is a real symmetric matrix. So,
Since all the eigenvalues are negative the it has a set of n L.I orthogonal eigenvectors as n
quadratic form is negative definite. columns corresponding to the n eigenvalues which
Index = 0, signature = 0 − 3= −3 may not be distinct.
Let P̂ be a normalized modal matrix of A. Then
Note 1 If we denote the above quadratic form by P̂ is an orthogonal matrix, i.e., a matrix for which
Q then P̂−1 = P̂ T.
4-6    Engineering Mathematics-II

Thus, the transformation X = P̂Y is an orthogonal 2. Orthogonalisation.


transformation. This transforms the quadratic form 3. Lagrange’s method of reduction (completion
into a canonical form as follows. of squares).
We know that P diagonalises A. Thus
We illustrate these methods one by one through
Pˆ −1 APˆ = D or A = PDP
ˆ ˆ −1 = PDP
ˆ ˆT (4.6) solved examples.

since P̂ is an orthogonal matrix satisfying Pˆ −1 = Pˆ T


4.7.1 Diagonalisation (by
Substituting (4.6) and (4.1) we have
Simultaneous Application
ˆ ˆ T X = ( X T Pˆ ) ( D ) ( Pˆ T X )
Q = X T AX = X T PDP of Row and Column
(4.7) Transformations)
Premultiplying X = P̂Y by P̂−1 we get P̂ −1 X = Example 4.14
Pˆ −1 PY
ˆ = Y ⇒ Y = Pˆ −1 X = Pˆ T X (4.8) Reduce the quadratic form −21x12 + 30 x1 x2 − 12 x1 x3 −
Taking transposes of equation (4.8) 11x22 + 8 x2 x3 − 2 x32 to canonical form and find its
nature.
Y T = ( Pˆ T X ) T = X T Pˆ (4.9)
Solution The given quadratic form is
Using (4.8) and (4.9) in (4.7) we have
T T
−21x12 − 11x22 − 2 x32 + 30 x1 x2 − 12 x1 x3 + 8 x2 x3
Q = X AX = Y DY = [ y1 y2  yn ]
⎡ l1 0  0 ⎤ ⎡ y1 ⎤ ⎡ −21 15 −6⎤ ⎡ x1 ⎤
⎢0 l  0 ⎥ ⎢ y2 ⎥
T
= X AX = [ x1 x2 x3 ] ⎢⎢ 15 −11 4 ⎥⎥ ⎢⎢ x2 ⎥⎥
⎢ 2 ⎥ ⎢ ⎥
⎢   ⎥ ⎢  ⎥ ⎢⎣ −6 4 −2⎥⎦ ⎢⎣ x3 ⎥⎦
⎢ ⎥ ⎢ ⎥
⎣0 0  l n ⎦ ⎣ yn ⎦ Let us write A as
or Q = l1 y12 + l 2 y22 +  + ln yn2 ⎡ −21 15 −6⎤ ⎡1 0 0⎤ ⎡1 0 0⎤
⎢ 15 −11 4 ⎥ = ⎢0 1 0⎥ A ⎢0 1 0⎥
(4.10) ⎢ ⎥ ⎢ ⎥ ⎢ ⎥
which is known as the canonical form or sum-of- ⎣⎢ −6 4 −2⎥⎦ ⎢⎣0 0 1⎦⎥ ⎣⎢0 0 1⎦⎥
the squares form or the principal axes form of the
quadratic form. 5 2 5 2
R2 + R1 , R3 − R1 , C2 + C1 , C3 − C1
7 7 7 7
4.6 SYLVESTOR’S (LAW OF INERTIA)
⎡ ⎤ ⎡ ⎤
Theorem 4.1 The signature of a real quadratic ⎢ −21 0 0 ⎥ ⎢ 1 0 0⎥ ⎡ 5

2⎥
form is invariant for all normal reductions (Canonical ⎢ ⎥ ⎢ ⎥ ⎢1 −
⎢ 0 −2 2⎥ 5 ⎢ 7 7⎥
forms). − = ⎢ 1 0⎥ A ⎢
⎢ 7 7⎥ ⎢ 7 ⎥ 0 1 0 ⎥
4.7 METHODS OF REDUCTION OF A ⎢ 2 2⎥ ⎢ 2 ⎥ ⎢ ⎥
⎢ 0 − ⎢ 0 0 1 ⎥
− ⎥ ⎢− 0 1⎥ ⎣ ⎦
QUADRATIC FORM TO A CANONICAL FORM ⎣ 7 7⎦ ⎣ 7 ⎦
A given quadratic form (Q.F) can be reduced to a R3 − R2, C3 − C2
canonical form (C.F) or sum-of-squares form by one
of the following methods. ⎡ −21 0 0 ⎤
⎢ 2 ⎥
1. Diagonalisation (by simultaneous applica- D=⎢ 0 − 0⎥
⎢ 7 ⎥
tion of row and column transformations). ⎢ 0 ⎥
⎣ 0 0 ⎦
1
Refer to footnote on page 1-1.
Quadratic Forms    4-7

⎡ 5 ⎤ Applying R3 − R2, C3 − C2
⎡ 1 0 0 ⎤ ⎢1 −1⎥
⎢5 ⎥ ⎢ 7
⎥ ⎡1 0 0⎤ ⎡ 1 0 0⎤ ⎡1 0 −3⎤
= ⎢ 1 0 ⎥ A ⎢0 1 −1⎥ = P T AP ⎢0 −2 0 ⎥ = ⎢ 0 1 0⎥ A ⎢0 1 −1⎥
⎢7 ⎥ ⎢ ⎢ ⎥ ⎢ ⎥ ⎢ ⎥
⎢ −1 −1 1 ⎥ ⎢0 0 1⎥ ⎢⎣0 0 −4⎥⎦ ⎢⎣ −3 −1 1⎥⎦ ⎢⎣0 0 1 ⎥⎦
⎣ ⎦ ⎥
⎣ ⎦
So, the Linear Transformation is given by X = PY
The Linear Transformation is given by
⎡ x1 ⎤ ⎡1 0 −3⎤ ⎡ y1 ⎤
⎡ 5 ⎤ ⇒ ⎢⎢ x2 ⎥⎥ = ⎢⎢0 1 −1⎥⎥ ⎢⎢ y2 ⎥⎥ and the canonical
1 −1⎥
⎡ x1 ⎤ ⎢ 7 ⎡ y1 ⎤ ⎢⎣ x3 ⎥⎦ ⎢⎣0 0 1 ⎥⎦ ⎢⎣ y3 ⎥⎦
X = PY ⇒ ⎢ x ⎥ = ⎢0 1 ⎥
−1⎥ ⎢y ⎥
⎢ 2⎥ ⎢ ⎢ 2⎥ form is q = y12 − 2 y22 − 4 y32
⎢⎣ x3 ⎥⎦ ⎢0 0 1⎥ ⎢⎣ y3 ⎥⎦
⎢ ⎥ The number of variables n = 3. The rank of
⎣ ⎦
the quadratic form = r = r(A) = 3. The index of the
2 2 quadratic form = number of positive terms = s = 1.
The canonical form is −21y12 − y2 + 0. y32. The
7 The signature = 2s − r = 2.1 − 3 = −1. Since r = n
number of variables = n = 3. The rank of the quadratic it is either positive or negative definite. But s is
form q = r = 2 or r (A) = 2. The index = s = num- neither equal to n nor o, so it is not definite. Hence the
ber of positive terms = 0. The signature = 2s − r = quadratic form is indefinite.
2 × 0 − 2 = − 2. Since s = 0 and r = 2 < n = 3 the nature
of the quadratic form is negative semi-definite. 4.7.2 Orthogonalisation
Example 4.16
Example 4.15
Reduce the quadratic form x12 + 3x22 + 3x32 − 2 x2 x3 by
Reduce to canonical form and find the rank, orthogonal transformation. Find its rank, signature
signature and nature of the quadratic form and nature.
x12 − 2 x22 + 3x32 − 4 x2 x3 + 6 x3 x1 .
Solution The quadratic form is
Solution By comparing the given quadratic form ⎡1 0 0 ⎤ ⎡ x1 ⎤
with X T AX = [x1 x2 x3 ] ⎢⎢0 3 −1⎥⎥ ⎢⎢ x2 ⎥⎥
X T AX = ∑ aij xi x j ⎢⎣0 −1 3 ⎥⎦ ⎢⎣ x3 ⎥⎦

⎡1 0 3 ⎤ ⎡ x1 ⎤ The eigenvalues of the matrix A are l1 = 1,


l2 = 2, l3 = 4 (please refer to Example 2.13, p. 17)
= [ x1 x2 x3 ] ⎢0 −2 −2⎥⎥ ⎢⎢ x2 ⎥⎥

The orthogonal modal matrix is
⎢⎣ 3 −2 3 ⎥⎦ ⎢⎣ x3 ⎥⎦
⎡ ⎤
and writing the matrix A as ⎢1 0 0 ⎥
⎢ ⎥
⎢ 1 1 ⎥
⎡1 0 3⎤ ⎡1 0 0⎤ ⎡1 0 0⎤ P = ⎢0
⎢0 −2 −2⎥ = ⎢0 1 0⎥ A ⎢0 1 0⎥ ⎢ 2 2 ⎥⎥
⎢ ⎥ ⎢ ⎥ ⎢ ⎥ ⎢ 1 1 ⎥
⎢⎣ 3 −2 3 ⎥⎦ ⎢⎣0 0 1⎥⎦ ⎢⎣0 0 1⎥⎦ ⎢0 2

2 ⎥⎦

and applying R3 − 3R1, C3 − 3C1, we have The canonical form of q is y12 + 2 y22 + 4 y32. The
number of variables = n = 3. The rank of the quadratic
⎡1 0 0 ⎤ ⎡ 1 0 0⎤ ⎡1 0 −3⎤
⎢0 −2 −2⎥ = ⎢ 0 1 0⎥ A ⎢0 1 0 ⎥ form = r = r (A) = 3. The index = s = the number of
⎢ ⎥ ⎢ ⎥ ⎢ ⎥ positive terms = 3. The signature = 2s − r = 2.3 − 3 = 3.
⎢⎣0 −2 −6⎥⎦ ⎢⎣ −3 0 1⎥⎦ ⎢⎣0 0 1 ⎥⎦ ∴ The quadratic form is positive definite.
4-8    Engineering Mathematics-II

Example 4.17 ⎛ 1 −1 1 ⎞
Reduce the quadratic form q = x2 + y2 + z2 + 4yz − Normal vector is ⎜ , , ⎟
⎝ 3 3 3⎠
4zx + 4xy to canonical form by orthogonalisation
The orthogonal modal matrix is
and find the rank, index, signature and nature of it.
⎡ 2 1 ⎤
Solution The given quadratic form is ⎢ 0 ⎥
⎢ 3 3 ⎥
⎡ 1 2 −2⎤ ⎡ x ⎤ ⎢ 1 1 1 ⎥
P=⎢ −
X AX = [ x y z ] ⎢⎢ 2 1 2 ⎥⎥ ⎢⎢ y ⎥⎥
T
2 6

3⎥

⎢⎣ −2 2 1 ⎥⎦ ⎢⎣ z ⎥⎦ ⎢ 1 1 1 ⎥
⎢ − ⎥
The characteristic roots of A are obtained from ⎣ 2 6 3 ⎦
1− l 2 −2 1− l 2 −2 ⎡3 0 0⎤
A − lI = 2 1− l 2 = 3−l 3−l 0 P AP = ⎢⎢0 3
T
0 ⎥⎥ = D
−2 2 1− l 0 3−l 3−l ⎢⎣0 0 −3⎥⎦
1 − l 2 −2
The canonical form of the quadratic form is
= (3 − l ) 1
2
1 0 by R3 + R2, R2 + R1
3 y12 + 3 y22 − 3 y32.
0 1 1
The number of variables = n = 3; the rank of the
(3 − l )2 (3 + l ) = 0 ⇒ l = 3, 3, − 3 matrix = r = r(A) = 3.
The eigenvalues are not distinct. So, we have The index = number of positive terms = s = 2;
to see if we can construct two Linearly Independent The signature = 2s − r = 2 × 2 − 3 = 1.
eigenvectors corresponding to l = 3. Since r = n = 3 the quadratic form must be either
To find the eigenvectors we have to solve positive definite or negative definite.
(A − lI) X = 0. But since s ≠ n or 0, it is neither. Hence q is
indefinite.
For l = 3 We have to solve
⎡ −2 2 −2⎤ ⎡ x1 ⎤ ⎡0 ⎤
⎢ 2 −2 2 ⎥ ⎢ x ⎥ = ⎢0 ⎥ 4.7.3 Lagrange’s Method
⎢ ⎥ ⎢ 2⎥ ⎢ ⎥ of Reduction
⎢⎣ −2 2 −2⎥⎦ ⎢⎣ x3 ⎥⎦ ⎢⎣0 ⎥⎦ (Completing Squares)
~ x1 − x2 + x3 = 0 ⇒ x1 = x2 − x3
We may choose two linearly Example 4.18
independent vectors (orthogonal) Reduce the quadratic form x12 + 2 x22 − 7 x32 − 4 x1 x2 +
(0, 1, 1) and (2, 1, 1); normalised vectors 8x1 x3 to canonical form by Lagrange’s method of
⎛ 2 1 1 ⎞ reduction.
(1/ 2, −1/ 2 ),and ⎜ , ,− ⎟
⎝ 6 6 6⎠
Solution The given quadratic form is
For l = -3 We have to solve x12 − 2x1 (2 x2 − 4 x3 ) + 2 x32 − 7 x32
⎡ 4 2 −2⎤ ⎡ x1 ⎤ ⎡0⎤
⎢ 2 4 2 ⎥ ⎢ x ⎥ = ⎢0⎥ = [x1 − 2( x2 − 2 x3 )]2 − 2 x22 − 23x32 + 16 x2 x3
⎢ ⎥ ⎢ 2⎥ ⎢ ⎥
⎢⎣ −2 2 4 ⎥⎦ ⎢⎣ x3 ⎥⎦ ⎢⎣0⎥⎦ = [x1 − 2( x2 − 2 x3 )]2 − 2 ⎡⎣ x22 − 2 x2 ⋅ 4 x3 + 42 x32 ⎤⎦ + 9 x32
x1 + x2 = 0 x1 x2 x3 = ( x1 − 2 x2 + 4 x3 )2 − 2( x2 − 4 x3 )2 + 9 x32
! = =
x2 + x3 = 0 1 −1 1 = y12 − 2 y22 + 9 y32 (say )
Quadratic Forms    4-9

Linear Transformation is X = PY 2. x2 + y2 + z2 − 2xy + 2xz − 2yz


⎡ x1 ⎤ ⎡1 −2 4 ⎤ ⎡ y1 ⎤ ⎡ 1 −1 1 ⎤
⇒ ⎢ ⎥ ⎢ ⎥⎢ ⎥ Ans: ⎢⎢ −1 1 −1⎥⎥
⎢ x2 ⎥ = ⎢0 1 −4⎥ ⎢ y2 ⎥
⎢⎣ x3 ⎥⎦ ⎢⎣0 0 1 ⎥⎦ ⎢⎣ y3 ⎥⎦ ⎢⎣ 1 −1 1 ⎥⎦
3. x2 + y2 + 2xy − 2xz + 2yz
Here, n = number of variables = 3. Rank of the ⎡ 1 1 −1⎤
quadratic form r = r(A) = 3. Number of positive
Ans: ⎢⎢ 1 1 1 ⎥⎥
terms = s = 2. Signature = 2s − r = 2 × 2 − 3 = 1.
⎢⎣ −1 1 0 ⎥⎦
Since r = n = 3 the quadratic form must be positive
or negative definite. But since s ≠ n, s ≠ 0 it is neither 4. −x2 + 4xy + 4y2 + 6xz + 5z2
positive definite nor negative definite. Hence it is ⎡ −1 2 3⎤
indefinite. Ans: ⎢⎢ 2 4 0⎥⎥
Example 4.19 ⎢⎣ 3 0 5⎥⎦
Reduce the quadratic form q = x2 − 4y2 + 6z2 + 2xy − Determine the quadratic form corresponding to the
4xz to canonical form by Lagrange’s method. Find symmetric matrix A = (Qns. 5−8)
its nature. ⎡0 0 1 ⎤
Solution The given quadratic form can be written as 5. ⎢⎢0 1 −1⎥⎥

q = x2 + 2x (y − 2z) − 4y2 + 6z2 ⎣⎢0 −1 0 ⎥⎦


= [x2 + 2x (y − 2z) + (y − 2z)2] Ans: y2 + 2xz − 2yz
− (y2 + 4z2 − 4yz) − 4y2 + 6z2 ⎡ 2 −1 0⎤
= (x + y − 2z)2 − 5y2 + 2z2 + 4yz 6. ⎢⎢ −1 −2 5⎥⎥
= (x + y − 2z)2 + 2 (z2 + 2yz + y2) − 7y2 ⎣⎢ 0 5 1⎥⎦
= (x + y − 2z)2 + 2 (y + z)2 − 7y2
Ans: 2x2 − 2y2 + z2 − 2xy + 10yz
= u2 − 7v2 + 2w2
⎡1 1 1⎤
where u = x + y − 2z, v = y, w = y + z; L.T. is X = PY
7. ⎢⎢1 1 1⎥⎥
⎡1 1 −2⎤
⎢ ⎥ X = [x y z ]T ⎢⎣1 1 1⎥⎦
where P = ⎢0 1 1 ⎥ ,
Y = [u v w ]T Ans: (x + y + z)2
⎢⎣0 0 1 ⎥⎦
⎡ 4 −1 0 ⎤
Here number of variables n = 3, Rank of q = 8. ⎢⎢ −1 1 3 ⎥⎥
r = r(A) = 3; Index = s = 2. Signature = 2s − r = ⎢⎣ 0 3 −3⎥⎦
2 × 2 − 3 = 1. Since r = n, q must either be positive
definite or negative definite. But s ≠ n, s ≠ 0 and so, Ans: 4x2 − 2xy + 6yz + y2 − 3z2
it is neither. Hence q is indefinite. Reduce the following quadratic form to canonical form by
diagonalisation: (Qns. 9−12)
EXERCISE 4.1 9. 6x2 + 3y2 + 3z2 − 4yz − 4zx − 2xy [JNTU 2000]
Write down the matrix of the quadratic form in each case:
Ans: x + y + 7z rank r = 3; index s = 3;
2 2 2
(Qns. 1−4)
Positive definite signature = 3.
1. xy + yz + zx
⎡ 1 1⎤ 10. x12 + 2 x22 + 3x32 + 4 x1x2 − 2 x2 x3 + 6 x1x3
⎢0 2 2⎥ 37 2
⎢ ⎥ Ans: y12 − 2 y22 + y3
1 1⎥ 2
Ans: ⎢ 0
⎢2 2⎥ ⎡1 −2 4 ⎤ rank = r = 3
⎢ ⎥ ⎢ 7⎥
⎢1 1 0⎥ L.T. is X = PY , P = ⎢0 1 − ⎥ index = s = 2
⎢ 2⎥
⎣⎢ 2 2 ⎦⎥ ⎢0 0
⎣ 1 ⎥⎦ Indefinite signature = 1
4-10    Engineering Mathematics-II

⎡ 1 −2 1 ⎤ ⎡ 1 2 ⎤
11. q = X AX , X = [x
T
y z] T
A = ⎢⎢ −2 3 −1⎥⎥ ⎢− 3 0
6 ⎥⎥

⎢⎣ 1 2 5 ⎥⎦ Ans: l1 = 1, l2 = 4, l3 = 4 ⎢ 1 1 1 ⎥
P=⎢
Ans: q = y12 + 4 y22 + 4 y32 ⎢ 3 2 6 ⎥⎥
⎡1 2 7 ⎤ rank = r = 3 ⎢ 1 1 1 ⎥
⎢ 3 − 2 6 ⎥⎦
y12 − y22 + 20 y32 ; P = ⎢⎢0 1 4 ⎥⎥ index = s = 2 ⎣
⎢⎣0 0 1 ⎥⎦ Indefinite signature =1 rank = r = 3
index = s = 3
⎡ 6 −2 2 ⎤
signature = 2s − r = 3; Positive indefinite
12. q = X AX , X = [x
T
y z] T
A = ⎢⎢ −2 3 −1⎥⎥
⎢⎣ 2 −1 3 ⎥⎦ 16. 6 x12 + 3x22 + 3x32 − 4 x1x2 − 2 x2 x3 + 4 x3 x1
⎡ 1 2 2 ⎤
⎡1 1 3 −2 7 ⎤ ⎢ 5 6 ⎥⎥
+ y22 + y32 P = ⎢⎢0 1 1 7 ⎥⎥
7 16 ⎢ 30
Ans: 6 y12 Ans: l1 = 2, l2 = 2, l3 = 8
3 7 ⎢ 5 1 ⎥
⎢⎣0 0 1 ⎥⎦ P = ⎢ 0 −
q = 2 y12 + 2 y22 + 8 y32 ⎢ 30 6 ⎥⎥
rank = r = 3 ⎢ 2 1 1 ⎥
index = s = 2 ⎢− 5 6 ⎥⎦
⎣ 30
Positive definite signature = 3 rank = r = 3
Reduce the following quadratic form to canonical forms. index = s = 3
Find its rank index, signature and nature. signature = 2s − r = 3; Positive indefinite

13. 3x2 + 5y2 + 3z2 − 2y2 + 2zx − 2xy Reduce each of the following quadratic forms to canonical
[JNTU 1999S Dec 2002] forms by Lagrange’s method of reduction. (Qns. 17−20)
17. 2 x12 + 7 x22 + 5 x32 − 8 x1x2 − 10 x2 x3 + 4 x1x3
⎡ 1 1 1 ⎤
⎢− 2 3 6 ⎥⎥ ⎡1 2 −1⎤

Ans: l1 = 2, l2 = 3, l3 = 6 ⎢ 1 2 ⎥ Ans: 2 y12 − y22 + 4 y23 P = ⎢⎢0 1 −1⎥⎥
P = ⎢ 0 −
q = 2 y12 + 3 y22 + 6 y32 ⎢ 3 6 ⎥⎥ ⎣⎢0 0 1 ⎦⎥
⎢ 1 1 1 ⎥ rank = r = 3
⎢ 2 6 ⎥⎦
⎣ 3 index = s = 3
rank = r = 3 signature = 2s − r = 3; Indefinite
index = s = 3
18. x12 + 2 x22 − 7 x32 − 4 x1x2 + 8 x1x3 [JNTU 2003S]
signature = 2s − r = 3; Positive definite
⎡1 2 −12⎤
14. x2 + 3y2 + 4z2 − 2yz [JNTU May/June 2004] Ans: y12 − 2 y22 + 9 y32 ; P = ⎢⎢0 1 −4 ⎥⎥
⎡ ⎤ ⎢⎣0 0 1 ⎥⎦
⎢1 0 0 ⎥
⎢ ⎥ 19. q = X T AX , where X = [x y z ]T ,
Ans: l1 = 2, l = 2, l3 = 4 ⎢ 1 1 ⎥
P = ⎢0
q = y12 + 2 y22 + 4 y32 ⎢ 2 2 ⎥⎥ ⎡1 2 4 ⎤
⎢ 1 1 ⎥ A = ⎢⎢ 2 6 −2⎥⎥ [JNTU 2003S]
⎢0 −
⎣ 2 2 ⎥⎦ ⎢⎣ 4 −2 18 ⎥⎦
rank = r = 3 ⎡1 2 −14⎤
index = s = 3 Ans: y12 + 2 y22 + 48 y32 ; P = ⎢⎢0 1 5 ⎥⎥
signature = (2s − r ) = 3; Positive definite ⎢⎣0 0 1 ⎥⎦

15. 3x12 + 3x22 + 3x32 + 2 x1x2 − 2 x2 x3 + 2 x1x3 20. x1x2 + x22 + 4 x1x3 + x32
17
[JNTU 2003] Ans: y12 + y22 − y32
4


5
   Sin x, cos x, cosec x, sec x are of period 2p and
Fourier series1 is an important tool in solving prob- tan x, cot x are of period p.
lems in conduction of heat, electrical engineering,
 
current and voltage in alternating circuits, electro-


dynamics, acoustics, etc. The basic idea is to repre-
sent periodic functions by a series involving sines 1. A constant function f (x) = c is periodic. Any
and cosines. In this chapter, we find expansions of positive real number p is a period of f: it has
even/odd functions first in an interval of 2p and no primitive period.
later of 2l. Finally, we develop half-range Fourier 2. The period of −f (x) is the same as that of f (x).
series also. 3. If p is a period of a function f then np
First we begin with the notion of periodic func- (a multiple of p) is also a period of f.
tions; and even and odd functions.
4. P ( f ( x )) = p ⇒ P ( f ( ax + b)) = p a
    
      E.g. P (tan ax ) = p a
The graph of a trigonometric function repeats itself in P (sin 3x ) = 2p 3
regular intervals. Such functions are called periodic 5. P ( f1 ( x )) = P ( f 2 ( x )) = p
functions. ⇒ P (c1 f1 ( x ) + c2 f 2 ( x )) = p,
Periodic Function: Definition A function f (x) is where c1, c2 are constants.
called a periodic function if there exists a positive
real number p such that f (x + p) = f (x) ∀x ∈ dom f. ⎛ x x⎞
E.g. P ⎜ 3tan + 4 cot ⎟ = 2p ;
Generally dom f = R (the set of all real numbers). ⎝ 2 2⎠
Examples of periodic functions in engineering: p
P (6 tan 2 x + 5sin 4 x ) =
(i) emf in an AC circuit, (ii) output of half-wave 2
rectifier, (iii) thrust on the piston, (iv) digital signals, etc. 6. A linear combination of two periodic
The smallest value of p for which this holds is functions is a periodic function.
called the fundamental period or the primitive period
Let P( f1(x)) = p1 and P( f2(x)) = p2. Then (c1 f1(x) +
or simply the period of f and is denoted by P ( f ) = p.
c2 f2(x) = k(m, n)) where k is the least positive real
number such that p1 = km, p2 = kn and (m, n) = LCM
1
Named after the French analyst and mathematical physicist, of m and n ∈ N.
Baron de Jean Baptiste Joseph Fourier (1768–1830). While
⎛ x⎞ ⎛p⎞
investigating heat conduction problems, he developed the theory E.g. P ⎜ cos ⎟ = 6p = 18 ⎜ ⎟ = mk ,
of Fourier series and used it first in his memorable work Theorie ⎝ 3 ⎠ ⎝ 3⎠
Analytique de la Chaleur. This initiated a great mathematical
⎛ 3p ⎞ 4p ⎛p⎞
activity and led to the development of a new branch of P ⎜ tan ⎟ = = 4 ⎜ ⎟ = nk
mathematics called Fourier Analysis. ⎝ 4⎠ 3 ⎝ 3⎠
5-2    Engineering Mathematics-II

where k = p 3, m = 18 and n = 4.  Every classifable function defined on a


set S (which is symmetric with respect to x = 0)
⎛ x 3p ⎞ p
P ⎜ 4 cos + 3tan ⎟ = (18, 4) can be expressed as the sum of two functions, one
⎝ 3 4⎠ 3 even and the other odd. Let f (x) be defined on S.
p Then
= × 36 = 12p
3
1
f ( x) = [ f ( x ) + f ( x )]
     2
 1 1
= [ f ( x ) + f ( − x )] + [ f ( x ) − f ( − x )]
A function f defined on a set S is called a classifiable 2""#""$
! 2""#""$
!
even odd
function if −x ∈S whenever x ∈ S.

E.g. x, x + x 2 , sin x, cos x, tan x, e x, e − x, − x +  The graph of an even function f (x) is sym-
cos x, x , log x , log [(1 − x) / (1 + x )], etc., are all metric about the y-axis.
classifiable functions, as they are defined for
positive as well as negative real values of x.   An even function f (x) contains even
powers of x and possibly cos x, sec x, etc., and odd
Nonclassifiable Functions Functions which are not
powers of x multiplied by sin x, cosec x, etc.
classifiable are called nonclassifiable functions.
E.g. log x, x, which are defined for x > 0 only
log(−x), − x which are defined for x < 0 only are y
nonclassifiable functions.
The set of classifiable functions can be divided
into three mutually disjoint2 subsets consisting
of (1). Even functions, (2). Odd functions and
(3). Neither even nor odd functions.
x
Even Function: Definition A function defined on
a set S is an even function if (i) −x ∈ S whenever
x ∈S and (ii) f (−x) = f (x)∀ ∈ S:
1− x
E.g. x 2 , cos x, x sin x, x 4 cos x, x log ,
1+ x a a
Odd Function: Definition A function f defined
  ∫− a f ( x) dx = 2∫0 f ( x ) dx, if f ( x ) is even.
on S is an odd function if (i) −x ∈ S whenever x ∈ S
and (ii) f (−x) = −f (x) ∀x ∈ S. 
 The sum or the product of two or more
even functions is an even function.
E.g. x, sin x, x3cos x, x2 sin x + 2x

  The identity function f (x) = 0 for all   The product of an even number of odd
x ∈ R is the only function which is both even functions is even.
and odd.   The graph of an odd function f (x) is
 There are classifable functions which are symmetric about the origin. (It lies in I and III or II
neither even nor odd. E.g. x2 + x, sin x + cos x, ex and IV quadrants.)

2
With the exception of the identity function f (x) = 0 for all x ∈ R,  An odd function f (x) contains odd powers
which is even as well as odd. of x and possibly sin x, cosec x, etc.
Fourier Series    5-3

y
   $" $! !!%""
! $#
"&$! !%"'#!"%
In order to establish (5.2) we require the following
results:
a + 2p
x a + 2p ⎡ sin nx ⎤
0 1. ∫a cos nx dx = ⎢
⎣ n ⎥⎦a
sin( na + 2np ) − sin na
= =0
n
( n ≠ 0)
a a + 2p
!$ f ( x ) is odd ⇒ ∫− a f ( x) dx = 0 2. ∫a
a + 2p ⎡ − cos nx ⎤
sin nx dx = ⎢
⎣ n ⎥⎦a
!$ The sum of any number of odd functions − cos( na + 2np ) + cos na
is odd. = =0
n
!$  The product of an odd number of odd ( n ≠ 0)
functions is an odd function. a + 2p

!$  The product of an odd and an even func-


3. ∫a cos mx cos nx dx
1 a + 2p
tion is an odd function. =
2 ∫a [cos(m + n) x + cos(m − n) x ] dx
a + 2p

   1 ⎡ sin( m + n) x sin( m − n) x ⎤


= + =0
   ' 2 ⎢⎣ m + n m − n ⎥⎦a
 a a + p
( m2 ≠ n 2 )
The Fourier series corresponding to a function f (x)
in the interval a < x < a + 2p is the trigonometric a + 2p 1 a + 2p
4. ∫a cos2 nx dx = ∫a (1 + cos 2nx ) dx
series on the RHS of (5.1) below and we write 2
a + 2p
1 ∞ ⎛ x sin 2nx ⎞
f ( x ) = a0 + ∑ ( an cos nx + bn sin nx ) (5.1) =⎜ +
⎝2 ⎟ =p
2 4n ⎠ a
n =1
( n ≠ 0)
where the constants a0, an, bn(n = 1, 2, 3, …) are
a + 2p
called Fourier coefficients and are defined by Euler’s
formulae3
5. ∫a sin mx cos nx dx

1 a + 2p 1 a + 2p
= ∫ [sin(m + n) x + sin (m − n) x] dx
p ∫a
( a0 , an , bn ) = (1, cos nx, sin nx ) f ( x ) dx
2 a
(5.2)
a + 2p
1 ⎡ cos( m + n) x cos( m − n) x ⎤
= − ⎢ + =0
3
EULER, Leonhard (1707–1783), was a gifted Swiss mathema- 2⎣ m+n ( m − n) ⎥⎦a
tician, and the most prolific mathematician, who contributed to
almost all branches of mathematics even after becoming totally ( m2 ≠ n 2 )
blind in 1771. He studied under John Bernoulli and became a a + 2p
a + 2p ⎛ sin2 nx ⎞
professor of mathematics at St. Petersburg, Russia. He is the first
Modern Mathematical Universalist.
6. ∫a sin nx cos nx dx = ⎜
⎝ 2n ⎠ a
⎟ =0
5-4    Engineering Mathematics-II

a + 2p 1 a + 2p 1 a + 2p
7. ∫a sin mx sin nx dx =
2 ∫a
[cos (m − n) x  ∴ an =
p ∫a
f ( x ) cos nx dx (5.5)
− cos (m + n) x]dx

1 ⎡ sin( m − n) x sin( m + n) x ⎤
a + 2p a + 2p 1 a + 2p
= − =0 ∫a f ( x ) sin mx dx =
2 ∫a
a0 sin mx dx
2 ⎢⎣ m − n ( m + n) ⎥⎦a ∞
a + 2p ⎛ ⎞
( m2 ≠ n 2 ) +∫
a ⎜ ∑
⎝ n =1
an cos nx ⎟ sin mx dx

a + 2p a + 2p ⎛ ∞ ⎞
a + 2π ⎛ x sin 2nx ⎞ +∫ ⎜ ∑ bn sin nx dx ⎟ sin mx dx
8. ∫a sin2 nx dx = ⎜ − ⎟ =p a ⎝ n =1 ⎠
⎝2 4n ⎠ a
( n ≠ 0) = 0 + 0 + pbn (5.6)
(5.3) by (2), (5), (6), (7), (8);

Let f (x) be represented in the interval


(a, a + 2p ) by the Fourier Series (5.1) By choosing a as 0 and −p, we obtain expansions in
We now find the Fourier coefficients a0, an, bn(n = the intervals (0, 2p) and (−p, p), respectively.
1, 2, …) assuming that the series (5.1) can be integrated
term by term from x = a to x = a + 2p. For this, it is  

  


suffcient that the series is uniformly convergent.   

 

To obtain a0, an and bn1 integrate both sides of  

(5.1) from x = a to x = a + 2p after multiplying it by We have seen above that if a function f (x) can be
1, cos mx and sin mx, respectively as follows: formally expanded in a Fourier series then the coef-
a + 2p 1 a + 2p a + 2p ⎛
∞ ⎞ ficients are given by Euler’s formulae (5.2). The
∫a f ( x ) dx =
2
a0 ∫
a
dx + ∫
a ⎜ ∑
⎝ n =1
an cos mx⎟ dx

theory concerning the conditions under which a
function can be represented by a Fourier series and

a + 2p ⎛ ⎞ its convergence is complicated. However, we state
+∫ ⎜ ∑ bn sin mx⎟ dx here the well-known4 Dirichlet’s conditions, which
a ⎝ n =1 ⎠
1 cover almost all engineering applications.
= a0 (α + 2p − a ) + 0 + 0 = a0p Any function f (x) can be developed as a Fourier
2
by (1) and (2) series (5.1)
1 a + 2p ∞ ∞
1
∴ a0 = ∫ f ( x ) dx a0 + ∑ an cos nx + ∑ bn sin nx
p a 2 n =1 n =1
(5.4)
where a0, an, bn are constants given by Euler’s
a + 2p 1 a + 2p
formulae (5.2) mentioned above provided they
∫a f ( x ) cos mx dx =
2 ∫a
a0 cos mx dx satisfy the following sufficient conditions of Dirichlet.

a + 2p ⎛ ∞ ⎞ (i) f (x) is periodic; f (x) and its integrals are


+∫ ⎜ ∑ an cos nx ⎟ cos mx dx single valued and finite (bounded).
a ⎝ n =1 ⎠

a + 2p ⎛ ∞ ⎞
+ ∫a ⎜ ∑ bn sin nx ⎟ cos mx dx 4
⎝ n =1 ⎠ DIRICHLET, Peter Gustav Lejune (1805–1859), is a German
number theorist, analyst and applied mathematician. He showed
= 0 + p an + 0 that every arithmetical sequence 〈 a, a + b, a + 2b, … 〉 where a
by (1), (3), (4), (5), (6); and b are relatively prime integers, contains an infinite number
of primes.
Fourier Series    5-5

(ii) f (x) has at the most a finite number of At such a point Fourier series gives the value of
discontinuities in any one period. f (x) as the arithmetic mean of the two limits:
(iii) f (x) has at the most a finite number of 1
maxima/minima. At x = c, f ( x ) = [ f (c − 0) + (c + 0)]
2
(5.9)
%$ # & $#  # $%$(
In deriving Euler’s formulae for a0, an and bn above, %$ #&  %"""#
it was assumed that f (x) was continuous. But a
'!# #
function may have a finite number of points of Sin−1x in (−1, 1) is not single valued, tan x in
finite discontinuity, i.e., its graph may consist of a (0, 2p) has infinite discontinuity at x = p/2, 3p/2;
finite number of different curves given by different and sin 1/x in (−p, p) has infinite number of maxima
equations. Such a function is also expressible as a and minima in the nbd of x = 0. So, the set of other
Fourier series. such functions has no Fourier series expansions.
For instance, if f (x) is defined in the interval Dirichlet’s conditions for the existence of Fourier
(a, a + 2p) by series expansions for functions are only suffcient
but not necessary. This means that though a function
f (x) = f (x) a < x < c
does not satisfy the conditions it may have Fourier
= y (x) c < x a + 2p (5.7) series expansion.
i.e., x = c is a point of finite discontinuity for f (x).
  



 


y    
The following properties are useful in deriving
the Fourier series expansions of combinations of
functions when we have derived the Fourier series
expansions for the typical functions.
If the Fourier series expansions for f1(x) and
f2(x) in an interval (a, b) are given by S1 and S2,
respectively, then the Fourier series expansion for
c1 f1(x) + c2 f2(x) in the interval (a, b) is c1 S1 + c2 S2
where c1 and c2 are any constants.
0 a x=c a + 2p x
From this we infer the following:
1. The Fourier series of constant times a func-
Then tion in (a, b) is constant times the Fourier
series of that function in (a, b).
1⎡ c a + 2p
y ( x ) dx ⎤ ,
p ⎣⎢ ∫a
a0 = f ( x ) dx + ∫ 2. The Fourier series of the sum (or difference)
c ⎦⎥
of two functions in (a, b) is the sum (or
1 c + 2
an = ⎡ ∫ f ( x ) cos nx dx + ∫ y ( x ) cos nx dx ⎤ ,
a p
difference) of their Fourier series in (a, b).
p ⎣⎢ a c ⎥⎦
Thus, for example, if we want to find the Fourier
1⎡ c a + 2p
y ( x )sin nx dx ⎤
p ⎢⎣ ∫a
bn = f ( x )sin nx dx + ∫ series of the function (p / 2 − x/ 2) in (0, 2p ), we
c ⎥⎦
may proceed as follows. Suppose we have found
(5.8) Fourier series for f (x) = x in (0, 2p) as
At x = c there is a finite jump discontinuity. ∞
sin nx
Both the left and right limits f (c − 0) and f (c + 0) x = p − 2∑ (5.10)
n =1 n
exist and are different.
5-6    Engineering Mathematics-II

Multiplying both sides of (5.10) by 12 and trans- 2 2 4 2 ∞ (cos nx )


posing, we obtain the Fourier series expansion of
1 − cos x =
p
− ∑
p n =1 (2n − 1)(2n + 1)
(5.17)
p x in (0, 2p) as
2 −2
which is the same for f ( x ) = 1 − cos x in (−p, p)

p x sin nx where it is defined by
− =∑ (5.11)

2 2 n =1 n x
⎢ − 2 sin ( −p , 0)
f ( x ) = 1 − cos x = ⎢ 2 (5.17)
Again, suppose we have found the Fourier series ⎢ x
⎣ + 2 sin (0, p )
expansion for x2 in (0, 2p) as 2
   
4p 2 ∞
cos nx ∞
sin nx
x2 = + 4∑ 2 − 4p ∑   
3 n =1 n n
n =1   
(5.12)
In Section 5.8, it is shown that Fourier series expan-
sion for a function f (x) with period 2l is given by
Using these, the Fourier series of (x + x ), (x − x )2 2

2
p − x⎞ a0 ⎡ ⎛ npx ⎞ ⎛ npx ⎞ ⎤
and ⎛⎜ may be obtained as follows: f ( x) = + ∑ ⎢ an cos ⎜ ⎟⎠ + bn sin ⎜⎝ ⎟
⎝ 2 ⎟⎠ 2 ⎣ ⎝ l l ⎠ ⎥⎦ (5.18)
⎛ 4p 2 ⎞ ∞
cos nx ∞
sin nx

x + x2 = ⎜ p + ⎟ + 4∑ 2 − (2 + 4p )∑ 2
⎝ 3 ⎠ n =1 n n =1 n
npx
Let f (x) be odd. Then f (x) and f ( x ) cos are odd
(5.13) l
functions and their integrals in (−l, l) will be zero.
Hence a0 = an = 0.
⎛ 4p 2 ⎞ ∞
cos nx ∞
sin nx
x − x2 = ⎜ p −
3 ⎠ ⎟ − 4 ∑ 2
+ (4p − 2 ) ∑ 2 npx
f ( x )sin
⎝ n =1 n n =1 n , being the product of two odd
l
(5.14) functions is even.
2 l npx
l ∫0
2 bn = f ( x )sin dx
⎛ p − x⎞ p 2 px x 2 p 2 ∞ cos nx ∴ (5.19)
⎜⎝ ⎟⎠ = − + = +∑ , l
2 4 2 4 12 n =1 n2
The Fourier series (5.18) in this case becomes
after simplication (5.15)

npx
The Fourier series of two different functions f ( x ) = ∑ bn sin (5.20)
n =1 l
in the same interval are evidently different. The
Fourier series of the same function in different inter-

vals may, in certain cases, be the same. npx
Let f (x) be even. Then f ( x )sin being the
Fourier series expansion of f (x) = x in (0, 2p) l
is (5.10) but for the same function in (−p, p) the product of an even and an odd function, is odd and
Fourier series expansion is its integral in (−l, l) will be zero. Hence bn = 0.
∞ n −1 npx
( −1) sin nx f (x) and f ( x ) cos are even. Consequently a0
x = 2∑ (5.16) l
n =1 n and an are given by
But the Fourier series expansion for 2 l ⎛ npx ⎞
f ( x ) = 1 − cos x in (0, 2p ) is ( a0 , an ) =
l ∫0
f ( x ) ⎜1, cos
⎝ l ⎠
⎟ dx (5.21)
Fourier Series    5-7

The Fourier series (5.18) in this case becomes  +#*"'#/%&("-")


a0 ∞
npx %+%*"&%)
f ( x) = + ∑ an cos (5.22) (a) f : [−p, p] → R defined by
2 n =1 l
⎡ 0, in ( −p , 0)

 &+("((")&  f ( x) = ⎢
⎣ cos x , in (0, p )
%,%+%*"&%)
"%*!%*(,#pp (b) g : (−2, 2) → R defined by
)
Let f (x) be odd. Taking l = p in (5.19) we have ⎡ 1, in ( −2, −1)
⎢ 0, in ( −1, 0)
2 p g( x) = ⎢
p ∫0
bn = f ( x )sin nx dx (5.23) ⎢1 − x, in (0,1)

The Fourier series is given by ⎣1 + x, in (1, 2)

∞ are multiply-defined or piecewise-defined functions.


f ( x ) = ∑ bn sin nx (5.24) While evaluating integrals of multiply- or
n =1 piecewise-defined functions we have to split the
while a0 = an = 0 interval of integration into sub-intervals and write
the integral as the sum of integrals over the respec-
)
tive subintervals with the corresponding values of
Let f (x) be even. Taking l = p in (5.21) we have the function taken as the integrands. Thus,
2 p
p ∫0
( a0 , an ) = f ( x )(1, cos nx ) dx (5.25) p 0 p 2
∫−p f ( x) dx = ∫−p 0. dx + ∫0 cos x dx; ∫−2 g ( x) dx
−1 0 1 2
The Fourier series (5.22) in this case becomes = ∫ 1. dx + ∫ 0. dx + ∫ (1 − x ) dx + ∫ (1 + x ) dx
−2 −1 0 1
a0 ∞
f ( x) = + ∑ an cos nx (5.26) and so on.
2 n =1
while bn = 0 .$'# 
Find the Fourier series of the function f (x) = x in
    (0, 2p). [JNTU Nov. 2003]
  &#+*"&% The Fourier series of the function
 
By a function f : S → R we mean a rule or a set of rules f (x) = x in (0, 2p) (1)
assigning to each element in S (domain) a unique is given by
element in R (codomain). If this assignment is by a
a0 ∞ ∞
simple (single) rule or law we call the function a f ( x) = x = + ∑ an cos nx + ∑ bn sin nx (2)
simply-defined function and if this assignment is 2 n =1 n =1
through a multiple rule or law, we call the function
multiply-defined or piecewise-defined function. where
2p
 "$'#/%+%*"&%) 1 2p 1 2p 1 ⎛ x2 ⎞
a0 = ∫ f ( x ) dx = ∫0 x dx = ⎜ ⎟
The functions f, g, h : R → R defined by p 0 p p ⎝ 2 ⎠0

f (x) = x2, g(x) = 1 + ex sin x, h(x) = sin x + cos x, 1 4p 2


= ⋅ = 2p (3)
respectively, are simply-defined functions. p 2
5-8    Engineering Mathematics-II

1 2p 1 2p p −1
Putting a = ,b=
an =
p ∫0
f ( x ) cos nx dx = ∫ x cos nx dx
p 0 2 2
in (7) we have the Fourier

1 ⎡ x sin nx ⎛ cos nx ⎞ ⎤
2p series in (0, 2p) for ⎛⎜ p − x ⎟⎞ :
= ⎢ −1 ⋅ ⎜ − 2 ⎟ ⎥ ⎝ 2 ⎠
p⎣ n ⎝ n ⎠ ⎦0
p − x ∞ sin nx sin 2 x sin 3x
1 ⎡1 =∑ = sin x + + + … (10)
= ⎢ (2p sin 2np − 0 . sin 0) 2 n 2 3
p ⎣n n =1

1 ⎤
+(cos 2np − cos 0)⎥
2 At x = p / 2, a point of continuity we obtain
n ⎦

1 ⎡1 1 ⎤ p p p 1 np
= ⎢ ⋅ 0 + 2 ⋅ 0⎥ = 0 (4) − = = ∑ sin
p ⎣n n ⎦ 2 4 4 n =1 n 2
∞ ∞
1 p ( −1)m
1 2p 1 2p = ∑ 2m + 1 sin (2m + 1) = ∑
2 m = 0 2m + 1
bn = ∫ f ( x )sin nx dx = ∫ x sin nx dx m= 0
p 0 p 0
⎛ np ⎞
1 ⎡ ⎛ cos nx ⎞
2p
⎜⎝! sin = 0 when n = 2m (even)⎟
⎛ sin nx ⎞ ⎤ 2 ⎠
= ⎢x ⎜ − ⎟⎠ − 1⋅ ⎜⎝ − 2 ⎟⎠ ⎥
p⎣ ⎝ n n ⎦0
1 1 1 p
1 ⎡1 ⇒ 1− + − +  =
= ⎢ ( −2p ⋅ cos 2np + 0 ⋅ cos 0) 3 5 7 4
p ⎣n
1 ⎤ 2  
+ (sin 2np − sin 0)⎥ = − (5)
n 2
⎦ n Find the Fourier series of the function f (x) = x2 in
(0, 2p).
Substituting the values of the Fourier coefficients    The Fourier series of the function
a0, an and bn in (2), we have
∞ f (x) = x2 in (0, 2p) (1)
sin nx
x = p − 2∑ in (0, 2p) (6) is given by
n =1 n
a0 ∞ ∞
f ( x) = x2 = + ∑ an cos nx + ∑ bn sin nx (2)

   2 n =1 n =1
Multiplying (6) by b and adding a we obtain the where
Fourier series in (0, 2p) for a + bx: 2p
1 2p 1 2p 1 ⎛ x3 ⎞

sin nx a0 = ∫ f ( x ) dx = ∫ x 2 dx = ⎜ ⎟
a + bx = a + bp − 2b∑ (7) p 0 p 0 p ⎝ 3 ⎠0
n =1 n
1 8p 3 8p 2 (3)
= ⋅ =
Putting a = 1, b = 1 in (7) we have the Fourier series π 3 3
for (1 + x):

sin nx 1 2p 1 2p
f ( x ) cos nx dx = ∫ x 2 cos nx dx
p ∫0
1 + x = 1 + p − 2∑ (8) an =
n p 0
n =1
1⎡ ⎛ sin nx ⎞ ⎛ cos nx ⎞
Putting a = 1, b = −1 in (7) we have the Fourier = ⎢ x2 ⋅ ⎜ ⎟ − 2 x ⋅ ⎝⎜ − 2 ⎠⎟
p⎣ ⎝ n ⎠ n
series for (1 − x):
∞ 2p
sin nx ⎛ sin nx ⎞ ⎤
1 − x = 1 − p + 2∑ (9) +2 ⋅ ⎜− 3 ⎟⎥
n =1 n ⎝ n ⎠ ⎦0
Fourier Series    5-9

1 2 ⋅ 2p ⋅ cos 2np 4 


 To obtain (9) directly take f (x) = ax + bx2
= = 2; (4)
p n 2
n and put a = −p 2, b = 1 4 and add p 2 4 on both
the sides.
t1 = t3 = 0.
 

∵ sin nx = 0 at x = 2p , 0;
Find the Fourier series of the function f (x) = eax in
t2 = 0 at x = 0 and cos 2np = 1 (0, 2p).
1 2p 1 2p
bn = ∫
p 0
f ( x )sin nx dx = ∫ x 2 sin nx dx
p 0
   The Fourier series of the function

1⎡ f (x) = eax in (0, 2p) (1)


⎛ cos nx ⎞ ⎛ sin nx ⎞
= ⎢ x2 ⋅ ⎜ − ⎟ − 2 x ⋅ ⎜⎝ − 2 ⎟⎠ is given by
p⎣ ⎝ n ⎠ n
2p a0 ∞ ∞
⎛ cos nx ⎞ ⎤ f ( x ) = e ax = + ∑ an cos nx + ∑ bn sin nx (2)
+ 2 ⋅ ⎜ 3 ⎟⎥ 2 n =1
⎝ n ⎠ ⎦0 n =1

1 ⎡ −1
= (4p 2 cos 2np − 0 ⋅ cos 0) + 0 1 2p 1 2p

p⎣n where a0 =
p ∫0
f ( x ) dx = ∫ e ax dx
p 0
2 ⎤ 4p 2p
+ (cos 2np − cos 0)⎥ = − (5) 1 ⎛ e ax ⎞ 1 2ap
n 3
⎦ n = ⎜ ⎟ = (e − 1) (3)
p ⎝ a ⎠0 ap
! sin nx = 0 at x = 2p , 0;
t2 = 0 at x = 0, and
1 2p 1 2p
cos 2np = cos 0 = 1 an = ∫
p 0
f ( x ) cos nx dx = ∫ e ax cos nx dx
p 0
Substituting the values of the Fourier coefficients 2p
1 ⎡ e ax ⎤
a0, an and bn in (2), we have = ⎢ 2 ( a cos nx + n sin nx )⎥
p ⎣a + n 2
⎦0
∞ ∞
4p 2 cos nx sin nx
x2 = + 4 ∑ 2 − 4p ∑ (6) 1 1
3 n =1 n n = ⋅ ⎡e 2ap ( a cos 2np + n sin 2np )
n =1
p a 2 + n2 ⎣
x 2 p 2 ∞ cos nx ∞
sin nx
⇒ = + ∑ 2 −p∑ (7) 1
−e 0 ( a ⋅1 + 0)⎤⎦ = ⋅ 2
1
⋅ a ⋅ (e 2ap − 1) (4)
4 3 n =1 n n =1 n p a + n2

The Fourier series expansion for x in (0, 2p) is ! cos 2np = 1, sin 2np = 0, e 0 = 1

sin nx
x = p − 2∑ (8)
n 1 2p 1 2p
n =1
p − x⎞
2 bn = ∫ f ( x )sin nx dx = ∫ e ax sin nx dx
We now have the Fourier series for ⎛⎜
p 0 p 0
⎝ 2 ⎠ ⎟ 2p
in (0, 2p) 1 ⎡ e ax ⎤
= ⎢ 2 ( a sin nx − n cos nx )⎥
2 p ⎣a + n 2
⎦0
⎛ p − x⎞ p 2 p x x2
⎜⎝ ⎟ = − +
2 ⎠ 4 2 4 =
1

1
⎡e 2ap ( a ⋅ 0 − n ⋅1) − e 0 ( a ⋅ 0 − n ⋅1)⎤
p2
p 2
sin nx p ∞ 2
cos nx ∞ p a 2 + n2 ⎣ ⎦
= − +p∑ + +∑ 2 1 ( − n)
4 2 n =1 n 3 n =1 n = ⋅ 2 (e 2ap − 1) (5)

p a + n2
sin nx
−p ∑ n
, by (5) and (6)
Substituting the values of the Fourier coefficients
n =1
a0, an, and bn in (2) we have
p 2 ∞ cos nx
2
⎛ p − x⎞
⇒ ⎜⎝ ⎟⎠ = +∑ (9)
2 12 n =1 n2
5-10    Engineering Mathematics-II

1 f (x) = x sin x in (0, 2p) (1)


e ax = (e 2ap − 1)
2ap is given by

1 1 a0 ∞ ∞
+ (e 2ap − 1)∑ 2 ( a cos nx − n sin nx ) f ( x ) = x sin x = + ∑ an cos nx + ∑ bn sin nx (2)
p n =1 a + n2 2 n =1 n =1
e 2ap − 1 ⎡ 1 ∞
⎛ a cos nx − n sin nx ⎞ ⎤
⎢ + ∑ ⎜⎝
or e ax = where
⎟⎠ ⎥
p ⎣ 2 a n =1 a 2 + n2 ⎦
1 2p 1 2p
p ∫0
(6) a0 = f ( x ) dx = ∫ x sin x dx
p 0

   1 1
= [ x( − cos x ) − 1( − sin x )]20p = [( −2p cos 2p
Replacing a by −a in (6) we have the Fourier series p p
for e−ax: + 0 ⋅ cos 0) + (sin 2p − sin 0)] = −2 (3)
1 − e −2ap
⎡1 ∞
⎛ a cos nx + n sin nx ⎞ ⎤
1. e − ax = ⎢ + ∑ ⎜⎝ ⎟⎠ ⎥ ! cos 2p = cos 0 = 1, sin 2p = sin 0 = 0
p ⎣ 2a n =1 a 2 + n2 ⎦
(7)
1 2p 1 2p
From (6) and (7) the Fourier series for an =
p ∫0
f ( x ) cos nx dx = ∫ x sin x cos nx dx
p 0
e ax + e − ax e ax − e − ax
cosh ax = and sinh ax = are 1 2p
2 2 =
2p ∫0 x[sin( n + 1) x − sin( n − 1) x ] dx

sinh 2ap ⎛ 1 a cos nx ⎞ 2p
2. cosh ax = ⎜ + ∑
⎝ 2a n =1 a2 + n2 ⎠
⎟ =
1 ⎡ ⎛ cos( n + 1) x ⎞
⎢x ⎜ −
⎛ sin( n + 1) x ⎞ ⎤
− 1⋅ ⎜ −
p
⎝ n +1 ⎠
⎟ ⎟⎥
2p ⎣⎢ ⎝ ( n + 1)2 ⎠ ⎦⎥ 0
cosh 2ap − 1 ∞ n sin nx

p
∑ a 2 + n2 (8)
1 ⎡ ⎛ cos( n − 1) x ⎞ ⎛ sin( n − 1) x ⎞ ⎤
2p
n =1 − ⎢x ⎜ − ⎟ −1 ⋅⎜ − ⎟⎥ ;
2p ⎝ n −1 ⎠ ⎝ ( n − 1)2 ⎠ ⎦⎥ 0
⎣⎢

cosh 2ap − 1 ⎛ 1 a cos nx ⎞
3. sinh ax = ⎜ +∑ 2 ⎟ ( n ≠ 1)
p ⎝ 2a n =1 a + n2 ⎠

⎛ All terms vanish except cosines at x = 2p
sinh 2ap n sin nx ⎜

p
∑ a 2 + n2 (9) ⎝ cos(n + 1) 2p = cos(n − 1) 2p = 1
n =1
−1 ⎛ 1 1 ⎞ 2
Also, for a = 1 and a = −1we have = ⋅ 2p ⎜ − ⎟⎠ = 2 (4)
2p ⎝ n +1 n −1 n −1
(e 2p − 1) ⎡ 1 ∞ ⎛ cos nx − n sin nx ⎞ ⎤
4. e x = ⎢ + ∑ ⎜⎝ ⎟⎠ ⎥ (10)
p ⎣ 2 n =1 1 + n2 ⎦
For n = 1
1 2p 1 2p
(1 − e −2p
) ⎡1 ∞
⎛ cos nx + n sin nx ⎞ ⎤
a1 = ∫
p 0
x sin x cos x dx =
2p ∫0
x sin 2 x dx
5. e − x = ⎢ + ∑ ⎜⎝ ⎟⎠ ⎥
p ⎣ 2 n =1 1 + n2 ⎦ 2p
1 2p ⎡ ⎛ cos 2 x ⎞ ⎛ sin 2 x ⎞ ⎤
(11) =
2p ∫0 ⎢ x ⎜⎝ − 2 ⎟⎠ − 1 ⎜⎝ − 2 ⎟⎠ ⎥
⎣ 2 ⎦0
 
Expand f (x) = x sin x as a Fourier series in (0, 2p). 1 ⎛ 1⎞ 1
= ⋅ 2p ⎜ − ⎟ = − (5)
[JNTU, May/June 2004, 1993] 2p ⎝ 2⎠ 2

   The Fourier series of the function ( All terms vanish except cos 2 x = 1 at x = 2p )
Fourier Series    5-11

1 2p f (x) = cosa x, in (0, 2p) (a œ Z) (1)


p ∫0
bn = x sin x sin nx dx
is given by
1 2p
x [cos( n − 1) x − cos( n + 1) x ] dx
2p ∫0
= a0 ∞ ∞
f ( x ) = cos a x = + ∑ an cos nx + ∑ bn sin nx (2)
2 n =1 n =1
1 ⎡ sin( n − 1) x − cos( n − 1) x
= ⎢x ⋅ − 1⋅
2p ⎣ n −1 ( n − 1)2 where
2p
sin( n + 1) x − cos( n + 1) x ⎤
2p 1 2p 1 2p 1 ⎛ sin a x ⎞
−x ⋅ − 1⋅ ⎥⎦ = 0,
a0 = ∫
p 0
f ( x ) dx = ∫ cos a x = ⎜
p 0 p ⎝ a ⎠0

n +1 n +1 0
( n ≠ 1) 1 sin 2a p
(6) = (sin 2a p − sin 0) = (3)
pa pa

For n = 1 1 2p 1 2p
an =
p ∫0
f ( x ) cos nx dx = ∫ cos a x cos nx dx
p 0
1 2p 1 2p

p ∫0 ∫0
b1 = x sin x ⋅ sin x dx = ( x − x cos 2 x ) dx
2p 1 2p
2p
=
2p ∫0 [cos(n + a ) x + cos(n − a ) x ] dx
1 ⎡ x2 ⎛ sin 2 x ⎞ ⎛ cos 2 x ⎞ ⎤
= ⎢ − x ⎜⎝ ⎟⎠ + 1⋅ ⎜⎝ − 2 ⎟⎠ ⎥ 2p
2p ⎣ 2 2 2 ⎦0 1 ⎡ sin( n + a ) x sin( n − a ) x ⎤
= +
1 ⎡ 4p 2 − 0 2p sin 4p − 0 1 ⎤ 2p ⎢⎣ ( n + a ) ( n − a ) ⎥⎦ 0
= ⎢ − − 2 (cos 4p − cos 0)⎥
2p ⎣ 2 2 2 ⎦ 1 ⎡ sin(2np + 2p a ) sin(2np − 2p a ) ⎤
= + (4)
= p , ! cos 4p = cos 0 = 1; sin 4p = 0 (7) 2p ⎢⎣ (n + a ) (n − a ) ⎥

sin 2p a ⎡ 1 1 ⎤ a sin 2p a
Substituting these values in (2) we obtain = ⎢ − ⎥ =−
2p ⎣ n + a n − a ⎦ p ( n2 − a 2 )

1 cos nx ⎡ ! sin(p ± a )2p ⎤
x sin x = −1 − cos x + p ⋅ sin x + 2 ∑ 2 " (8)
2 n= 2 n − 1 ⎢ = sin(2np ± 2p a )⎥
⎢ ⎥
The function is continuous at x = p / 2 and hence ⎣⎢ = ± sin 2p a ⎦⎥
from (8) we obtain
1 2p 1 2p

bn =
p ∫0
f ( x )sin nx dx = ∫ cos a x sin nx dx
p 0
p p 1 p p cos ( np 2)
sin = −1 − cos + π ⋅ sin + 2 ∑
2 2 2 2 2 ( n − 1)( n + 1) 1 2p
n= 2
2 m +1
⎩⎪
=
2p ∫0 [sin( n + a ) x + sin( n − a ) x ] dx
np ⎪⎧ 0, if n is odd; n = 2
p ⎨
cos =⎨ 2p
−1 ⎡ cos( n + a ) x cos( n − a ) x ⎤
⎪⎧
2 ⎪⎩( −1) , if n is even;
m
n = 2m = +
∞ 2p ⎢⎣ ( n + a ) ( n − a ) ⎥⎦ 0
p −2 cos mp 1 1 1
⇒ = −∑ = − + 
4 m =1 (2m − 1)(2m + 1) 1.3 3.5 5.7 −1 ⎡ cos 2p a − 1 cos 2p a − 1⎤
= + (5)
(9) 2p ⎢⎣ ( n + a ) ( n − a ) ⎥⎦
(1 − cos 2p a ) ⎛ n ⎞

 = ⎜⎝ 2 ⎟
p n −a2 ⎠
Obtain the Fourier series of f (x) = cos a x, in (0, 2p)
(a ∉ Z). ⎡! cos(2np ± 2p a )⎤
⎢ ⎥
   The Fourier series of the function ⎣ = cos 2p a ⎦
5-12    Engineering Mathematics-II

Substituting these values the Fourier series is 1 2p x



bn =
p ∫0
2 sin sin nx dx
2
sin 2p a a sin 2p a cos nx
cos a x =
2p a
− ∑ ( n2 − a 2 )
1 2p ⎡ ⎛ 1⎞ ⎛ 1⎞ ⎤
p n =1 =
2p
∫0 ⎢cos ⎜⎝ n − 2 ⎟⎠ x − cos ⎜⎝ n + 2 ⎟⎠

x ⎥ dx

(1 − cos 2p a ) ∞ n sin nx
+
p
∑ ( n2 − a 2 ) (6)
1 ⎡ sin( n − 12 ) x sin( n + 12 ) x ⎤
2p
n =1 = ⎢ − ⎥
2p ⎣⎢ ( n − 12 ) ( n + 12 ) ⎦⎥
0
 

1 ⎡ sin(2n − 1)p − sin 0
Find a Fourier series for f ( x ) = 1 − cos x in (0, 2p) = ⎢
2p ⎢⎣ ( n − 12 ) (5)
and deduce that 1 + 1 + 1 + ! = 1 .
1.3 3.5 5.7 2 sin(2n + 1)p − sin 0 ⎤
− ⎥=0

  The Fourier series for ( n + 12 ) ⎥⎦
f ( x ) = 1 − cos x in (0, 2p ) (1) [∵ sin(2n ± 1)p = sin 0 = 0]
is given by
x
f ( x ) = 1 − cos x = 2 sin Substituting these values in (2), we obtain
2
∞ ∞
a 2 2 4 2 ∞ cos nx
= 0 + ∑ an cos nx + ∑ bn sin nx (2) 1 − cos x = − ∑ (6)
2 n =1 n =1 p p n =1 4n2 − 1
where
f ( x ) = 1 − cos x is continuous at x = 0 and the
2p
1 2p x 2⎛ x⎞ series converges to f (0) = 0
a0 = ∫ 2 sin dx = ⎜ −2 cos ⎟⎠ (3)
p 0 2 p ⎝ 2 0
2 2 4 2 ∞ 1
=
−2 2
(cos p − cos 0) =
4 2 ∴ 0=
p
− ∑
p n =1 (2n − 1)(2n + 1)
(7)
p p
1 1 1 1
1 2p x ⇒ = + + +!
an = ∫ 2 sin cos nx dx 2 1.3 3.5 5.7
p 0 2
1 2p ⎡ ⎛ 1⎞ ⎛ 1⎞ ⎤ p
Also, at x =
=
2p
∫0 ⎢sin ⎜⎝ n + 2 ⎟⎠ x − sin ⎜⎝ n − 2 ⎟⎠ x ⎥ dx
⎣ ⎦ 2
2p
1 ⎡ cos(n + 12 ) x cos(n − 12 ) x ⎤ 2 2 4 2 ∞ cos n2p
= ⎢− + ⎥
2p ⎣⎢ ( n + 12 ) ( n − 12 ) ⎦⎥
0
1=
p
− ∑
p n =1 (2n − 1)(2n + 1)
; (8)

1 ⎡ cos(2n + 1)p − cos 0 np ⎡ 0, if n = 2m + 1, (odd) ⎤


= ⎢− cos =⎢ ⎥
2p ⎢⎣ ( n + 12 ) 2 ⎣( −1)m , if n = 2m (even)⎦
cos(2n − 1)p − cos 0 ⎤
+ ⎥ Multiplying by p/4 2 and transposing,
( n − 12 ) ⎥⎦
1 ⎛ 4 4 ⎞ −4 2 p 1 ∞ −( −1) m
= ⎜⎝ − ⎟⎠ = − =∑
2p 2n + 1 2n − 1 (4n2 − 1)p 4 2 2 m =1 (4 m − 1) (4 m + 1)
⎡" cos(2n + 1)p = cos(2n − 1)p = −1⎤ p 1 1 1 1 1
⇒ − = − + − +
⎢ cos 0 = 1 ⎥ (4) 4 2 2 3.5 7.9 11.13 15.17
⎣ ⎦
(9)
Fourier Series    5-13

  2a
Obtain a Fourier series for the function = ( −1)n +1 ; [sin nx = 0 at x = p , 0; cos np = (−1)n ]
n
f (x) = ax + bx2 in (−p, p).
Substituting these values, we have
   The Fourier series of
bp 2 ∞
( −1)n cos nx
ax + bx 2 = + 4b ∑
f (x) = ax + bx in (−p, p) n2
2
(1) 3 n =1

is given by ∞
( −1)n sin nx
−2a∑ (6)
a ∞
n =1 n
f ( x ) = ax + bx = 0 + ∑ an cos nx
2
(2)
2 n =1
∞ 
  
+ ∑ bn sin nx
n =1 (i) a = 1, b = 1
where p2 ∞
( −1)n cos nx
x + x2 = + 4∑
1 p 1⎛a b ⎞
p 3 n =1 n2
( ax + bx 2 ) dx = ⎜ x 2 + x 3 ⎟
p ∫− p
a0 = (3)
p ⎝2 3 ⎠ −p ∞
( −1)n sin nx
2 −2∑ (7)
1 ⎡a 2 b ⎤ 2bp n
= ⎢ (p − ( −p )2 ) + (p 3 − ( −p )3 ) ⎥ = n =1
p ⎣2 3 ⎦ 3
(ii) a = 1, b = 1
1 p 2b p p2 ∞
( −1)n +1 cos nx
( ax + bx 2 ) cos nx dx = 0 + ∫ x 2 cos nx dx x − x2 = − + 4∑
an =
p ∫ − p p 0 3 n =1 n2
2b ⎡ 2 ⎛ sin nx ⎞ ⎛ cos nx ⎞
= ⎢ x ⎜ ⎟⎠ − 2 x ⋅ ⎜⎝ − 2 ⎟⎠

( −1)n +1 sin nx
p ⎣ ⎝ n n + 2∑ (8)
p n =1 n
⎛ sin nx ⎞ ⎤
+ 2⎜ − 3 ⎟ ⎥
⎝ n ⎠ ⎦0 (iii) f (x) is continuous at x = 0 and the series
⎡ x cos nx is an odd function converges to f (0). So from (8) we obtain
⎢ 2
⎣and x cos nx is an even funtion −p 2 ∞
( −1)n +1
0= + 4∑
=
4b 4b
(p cos np − 0 ⋅ cos 0) = 2 ( −1)n
3 n =1 n2
p n2 n
[" sin nx = 0 at p , 0; cos np = ( −1)n ] p2 1 1 1 1 1 1
(4) ⇒ = 2 − 2 + 2 − 2 + 2 − 2 +! (9)
12 1 2 3 4 5 6
1 p 2 1 p
bn =
p ∫−p (ax + bx )sin nx dx =
p
⋅ 2a∫ x sin nx dx + 0
0 (iv) Putting a = 0, b = −1/4 and adding p 2 /12 to
p both sides of (6) we obtain
2a ⎡ ⎛ cos nx ⎞ ⎛ sin nx ⎞ ⎤
= x⎜− ⎟ − 1 ⋅ ⎜⎝ − 2 ⎟⎠ ⎥
p ⎢⎣ ⎝ n ⎠ n ⎦0 p 2 x2 ∞
( −1)n +1
− =∑ cos nx (10)
⎡ x sin nx is an even function 12 4 n =1 n2
⎢ 2
⎣and x sin nx is an odd funtion
−2a (v) Putting a = −p / 2 , b = 1/4 and adding p 2 /12 to
= (p cos np − 0 ⋅ cos 0) + 0 (5)
pn both sides of (6) we obtain
5-14    Engineering Mathematics-II

2 ⎡1 ∞
⎛ p − x⎞ p 2 ∞ cos nx 2sinh ap ( −1)n
⎜⎝ ⎟⎠ = +∑ (11) e ax = ⎢ + a∑ 2 2
cos nx
2 12 n =1 n2 p ⎣ 2a n =1 a + n
∞ ⎤
n( −1)n
   −∑ 2 2
sin nx ⎥ (6)
ax n =1 a + n ⎦
Find the Fourier series of f (x) = e in (−p, p).
p
or e ax
 
  The Fourier series of f (x) = eax in (−p, p) 2sinh ap

is (1) 1 ( −1)n
= +∑ 2 ( a cos nx − n sin nx ) (7)
a ∞ ∞ 2a n =1 a + n2
f ( x) = e ax
= 0 + ∑ an cos nx + ∑ bn sin nx  (2)
2 n =1 n =1 Taking a = 1 we have
p 1 ∞ ( −1)n
ex = + ∑
p
1 p 1 ⎛ e ax ⎞ (cos nx − n sin nx )
where a0 = ∫ e ax dx = ⎜ (3) 2sinh p 2 n =1 1 + n2
p −p p ⎝ a ⎟⎠ −p
[JNTU 1995] (8)
1 ap 2sinh ap
= (e − e − ap ) = Half of the sum of the series
pa pa
1
f [at x = ± p ] = [ f ( −p + 0) + f (p − 0)]
1 p ax 2
p ∫-p
an = e cos nx dx
1 p p
1 ⎡ e ax ⎤
p = (e −p + ep ) = coth p (9)
= ⎢ 2 ( a cos nx + n sin nx )⎥ 2 2sinh p 2
2
p ⎣a + n ⎦ −p
1 a   
= ⋅ ⋅ ⎡e ap cos np − e − aπ cos( − np )⎤⎦
p a 2 + n2 ⎣ Find the Fourier series for f ( x ) = 1 − cos x in
a (−p, p).
= cos np (e ap − e − ap )
p ( a 2 + n2 )  
  The given function is
n
2a( −1) ⎧ x
= ⋅ sinh ap
⎪⎪ − 2 sin 2 , ( −p , 0)
(4)
p ( a 2 + n2 )
f ( x ) = 1 − cos x = ⎨ (1)
[! sin np = 0, ! cos np = ( −1)n ] ⎪ 2 sin x , (p , 0)
⎪⎩ 2
1 p ax
p ∫− p
bn = e sin nx dx The Fourier series is
p a0 ∞ ∞
1 ⎡ e ax ⎤ f ( x) = + ∑ an cos nx + ∑ bn sin nx (2)
= ⎢ 2 2
( a sin nx − n cos nx )⎥ 2 n =1 n =1
p ⎣a + n ⎦ −p
1 ( − n)( −1)n where
= ⋅ ⋅ (e ap − e − ap ),
p a 2 + n2 (5) 1 p 1 0 ⎛ x⎞
n( −1) n +1
a0 =
p ∫ − p
f ( x ) dx = ∫ ⎜ − 2 sin ⎟ dx
p − p ⎝ 2⎠
= 2sinh ap
p ( a 2 + n2 ) 1 p⎛ x⎞
+
p ∫0
⎜⎝ 2 sin ⎟⎠ dx
2
[! cos np = ( −1)n , sin np = 0]
1 p x 1 p x
= ∫ 2 sin dx + ∫ 2 sin dx (3)
Substituting these values in (2) we obtain p 0 2 p 0 2
Fourier Series    5-15

2 2 p x 2 2⎛ x⎞ 4 2
p  

=
p ∫0
sin dx =
2 p
⎜⎝ −2cos ⎟⎠ =
2 0 p Find the Fourier series for f (x) in (0, 2p) where
⎡ 1, 0< x≤p
1 p ⎢
an = ∫ f ( x ) cos nx dx f ( x) = x
p −p ⎢2 − , p < x ≤ 2p
⎣ p
1 0 x 1 p x
p ∫− p
= − 2 sin cos nx dx + ∫ 2 sin cos nx dx
2 p 0 2    The Fourier series for the function

2 2 p x ⎡ 1, 0< x≤p ⎤
p ∫0
= sin cos nx dx
2 f ( x) = ⎢ x ⎥ (1)
⎢2 − , p < x ≤ 2p ⎥
2 p⎡ ⎛ 1⎞ ⎛ 1⎞ ⎤ ⎣ p ⎦
p ∫0 ⎢⎣ ⎝
= sin ⎜ n + ⎟ x − sin ⎜ n − ⎟ x ⎥ dx
2⎠ ⎝ 2⎠ ⎦ is given by
a0 ∞ ∞

( )
2 ⎡ cos n + 2 x cos n − 2 x ⎤
1 1
( )
p
f ( x) = + ∑ an cos nx + ∑ bn sin nx (2)
= ⎢− + ⎥ 2 n =1 n =1
p ⎢
⎣ ( )
n + 12 n − 12 ( )
⎥⎦ 0
where
2⎛ 1 1 ⎞ 2 −1
= − = ⋅ 1 2p 1 p 1 2p ⎛ 1 ⎞
p ⎝ n + 2 n − 12 ⎟⎠

p ∫0
1
p n2 − 14 a0 = f ( x ) dx = ∫ 1⋅ dx + ∫ ⎜ 2 − x ⎟ dx
p 0 p p ⎝ p ⎠
−4 2 1 1 p 1⎛ 1 2⎞
2p
= (4) = ( x )0 + ⎜ 2 x − x ⎟
p (2n − 1)(2n + 1) p p⎝ 2p ⎠ p
1 p 1 1⎡ 1 ⎤
= ⋅ p + ⎢2(2p − p ) − (4p 2 − p 2 )⎥
p ∫− p
bn = f ( x )sin nx dx
p p⎣ 2p ⎦
1 0 1 p 3 3
x x = 1+ 2 − = (3)
= ∫
p −p
− 2 sin sin nx dx + ∫ 2 sin sin nx dx
2 p 0 2 2 2

1 0 (− x) 1 2p
= ∫ − 2 sin sin n( − x )d ( − x )
p ∫0
p p 2 an = f ( x ) cos nx dx

1 p x 1 p 1 2p ⎛ 1 ⎞
p ∫0
+ 2 sin sin nx dx = ∫ 1⋅ cos nx dx + ∫ ⎜ 2 − x ⎟ cos nx dx
2 p 0 p p ⎝ p ⎠
p
1 p x 1 p x 1 ⎛ sin nx ⎞ 1 ⎡⎛ x ⎞ ⎛ sin nx ⎞
= ∫
p 0
− 2 sin sin nx dx + ∫ 2 sin sin nx dx
2 p 0 2
=
p
⎜⎝
n 0 p
⎟⎠ + ⎢⎜⎝ 2 − p ⎟⎠ ⎜⎝ n ⎟⎠

=0 (5) 1 ⎛ − cos nπ ⎞ ⎤
2p
+ ⎜ ⎟
p ⎝ n2 ⎠ ⎥⎦p
Substituting these values in (2) we obtain
2 2 4 2 ∞ cos nx ⎡ 0, if n is even
−1 ⎡1 − ( −1)n ⎤ ⎢
f ( x ) = 1 − cos x =
p
− ∑
p n =1 (2n − 1)(2n + 1) = 2⎢ ⎥ = −2
p ⎣ n2 ⎦ ⎢ 2 2 , if n is odd (4)
⎢⎣ p p
in ( −p , p ) (6) ! t1 = t2 = 0 at p , 2p , 0
2 1

 The Fourier series expansions for 1 − cos x ⇒ a2m −1 = − 2
in (0, 2p) and in (−p, p) are the same. p (2m − 1)2
5-16    Engineering Mathematics-II

1 2p
1 p2 1 ⎡ 4p 2 − p 2 ⎤
p ∫0
bn = f ( x )sin nx dx
= ⋅ + ⎢2p( 2p − p) − ⎥=p (3)
p 2 p⎣ 2 ⎦
p 1 2p ⎛ 1 ⎞
= ∫ 1⋅ sin nx dx + ∫ ⎜ 2 − x ⎟ sin nx dx
0 p p ⎝ p ⎠
p
1 ⎛ − cos nx ⎞ 1 ⎡⎛ x ⎞ ⎛ − cos nx ⎞ 1 2p
p ∫0
= ⎜⎝ ⎟⎠ + ⎢⎜⎝ 2 − p ⎟⎠ ⎜⎝ ⎟ an = f ( x ) cos nx dx
p n 0 p ⎣ n ⎠
2p 1 p 1 2p
1 ⎛ − sin nx ⎞ ⎤ = ∫ x cos nx dx + ∫ (2p − x ) cos nx dx
+ ⎜ ⎟ p 0 p p
p ⎝ n2 ⎠ ⎥⎦p p
1 ⎡ sin nx − cos nx ⎤
−1 1⎡ cos np 1 ⎤ = x⋅ − 1⋅
= [(cos np − 1)] + ⎢ 0 + 1⋅ − 2 ⋅ 0⎥ p ⎢⎣ n n2 ⎥⎦ 0
pn p⎣ n pn ⎦ 2p
1 − ( −1)n ( −1)n 1 1⎡ sin nx ⎛ − cos nx ⎞ ⎤
+ (2p − x ) ⋅ − ( −1) ⋅ ⎜
= + = (5) ⎢
p⎣ n ⎝ n2 ⎟⎠ ⎥⎦p
pn pn pn
1 ⎡1 1 ⎤
⇒ bm = = ⎢⎣0 + n2 (cos np − cos 0)⎥⎦
pm p
1⎡ 1 ⎤
Substituting these values in (2) we obtain + ⎢0 − 2 (cos 2np − cos np )⎥
p ⎣ n ⎦
3 2 ∞
cos(2m − 1) x 1 ∞ sin mx 2
= 2 ⎡⎣( −1)n − 1⎤⎦
f ( x) = −
4 p2
∑ (2m − 1)2
+ ∑
p m=1 m
(6)
pn
(4)
m =1

 

Find the Fourier series of the function
1 2p
p ∫0
⎡ x, 0≤ x≤p ⎤ bn = f ( x )sin nx dx
f ( x) = ⎢ ⎥ and deduce that
⎣2p − x, p ≤ x ≤ 2p ⎦ 1 p 1 2p
= ∫ x sin nx dx + ∫ (2p − x )sin nx dx
1 1 1 p2 p 0 p p
+ + +! = . [JNTU 2003, 1997 S]
12 32 52 8 1 ⎡ ⎛ cos nx ⎞ ⎛ sin nx ⎞ ⎤
p
= x⎜− ⎟ − 1⋅ ⎜⎝ − 2 ⎟⎠ ⎥
p ⎢⎣ ⎝ n ⎠ n ⎦0

  The Fourier series for the function
2p
⎡ x, 0≤ x≤p ⎤ 1⎡ ⎛ cos nx ⎞ ⎛ sin nx ⎞ ⎤
f ( x) = ⎢ ⎥ (1) + ⎢(2p − x ) ⎜ − ⎟ + 1⎜ − 2 ⎟⎠ ⎥
⎣2p − x, p ≤ x ≤ 2p ⎦ p⎣ ⎝ n ⎠ ⎝ n ⎦p
is given by 1 ⎡ p cos np − 0 ⎤ 1 ⎡⎛ 0 + p cos np ⎞ ⎤
= − + 0⎥ + ⎢⎜ ⎟⎠ + 0⎥
p ⎢⎣ n ⎦ p ⎣ ⎝ n ⎦
a0 ∞ ∞
f ( x) = + ∑ an cos nx + ∑ bn sin nx (2) =0 (5)
2 n =1 n =1

where Substituting these values in (2) we obtain


1 2π 1 p 1 2p p 2 1
a0 = ∫ f ( x ) dx = ∫ x dx + ∫ (2p − x ) dx f ( x) = + ∑ 2 [( −1)n − 1]cos nx
p 0 p 0 p p 2 p n
p 2p
1 ⎛ x2 ⎞ 1⎛ x2 ⎞ p 4 ∞ cos(2m + 1) x
= + 2px − = − ∑
p ⎜⎝ 2 ⎟⎠ 0 p ⎜⎝ 2 ⎟⎠ p
(6)
2 p m= 0 (2m + 1)2
Fourier Series    5-17

 
  1 p
x = p is a point of continuity for f (x) since f (p−) =
= 0+
2p ∫0 [sin(n + 1) x − sin(n − 1) x ] dx
f (p+) = p and the series converges to f (p). Putting
p
x = p in (6) we obtain 1 ⎡ cos( n + 1) x cos( n − 1) x ⎤
= ⎢⎣ − + ( n ≠ 1)
p 4⎛ 1 1 1 1 ⎞ 2p n +1 n − 1 ⎥⎦ 0
p = + ⎜ 2 + 2 + 2 + 2 +" ⎟

2 p 1 3 5 7 ⎠
1 ⎡ cos( n + 1)p − cos 0 cos( n − 1)p − cos 0 ⎤
= ⎢⎣ − + ⎥⎦
p2 1 1 1 2p n +1 n −1
⇒ = + + +" (7)
8 12 32 52
1 ⎡ −( −1)n +1 + 1 ( −1)n −1 − 1⎤
= ⎢ + ⎥
   2p ⎣ n +1 n −1 ⎦
Find the Fourier series for the function
⎡ 0, for − p < x < 0 cos np = ( −1)n ;cos 0 = 1
f ( x) = ⎢
⎣sin x, for 0 < x < p ⎧ 0, if n is odd ( n ≠ 1)

[JNTU 1994S 1996, 2001 S] =⎨ 2 ! ( −1)n +1 = ( −1)n −1
⎪ − p ( n2 − 1) , if n is even
or ⎩
1 sin x 2 ∞ cos 2mx ⎧ +1, ( n, odd ) 2 1
Show that f ( x ) = + − ∑ =⎨ ⇒ a2m = −
p 2 p n =1 4m2 − 1 ⎩ −1, ( n, even = 2m) p (4m2 − 1)
Hence deduce the results (4)
1 1 1 1 n=1
− + − " = (p − 2);
1.3 3.5 5.7 4 1 p
p ∫− p
a1 = f ( x ) cos x dx
1 1 1 1
− + −" =
1.3 3.5 5.7 2 1 0 1 p
=
p ∫ − p
0 ⋅ cos x dx + ∫ sin x cos x dx
p 0
  The given function f (x) is defined by
p
⎡ 0, for − p < x < 0 1 p 1 ⎛ cos 2 x ⎞
f ( x) = ⎢ (1) =
2p ∫0 sin 2 x dx = ⎜−
2p ⎝ 2 ⎠0

⎣sin x, for 0 < x < p
The Fourier series for the function is given by −1
= (cos 2p − cos 0) = 0 (5)
a ∞ ∞ 4p
f ( x ) = 0 + ∑ an cos nx + ∑ bn sin nx (2) (! cos 2p = cos 0 = 1)
2 n =1 n =1

where 1 p
p ∫− p
bn = f ( x )sin nx dx
1 p 1 0 1 p
a0 = ∫ f ( x ) dx = ∫ 0 dx + ∫ sin x dx
p − p p −π p 0 1 0 1 p
1 1 2
= ∫
p −p
0 ⋅ sin nx dx + ∫ sin x sin nx dx
p 0
= 0 + ( − cos x )p0 = ( − cos p + cos 0) = (3)
p p p 1 p
= 0+
2p ∫0 [cos(n − 1) x − cos(n + 1) x] dx
1 p
p ∫− p
an = f ( x ) cos nx dx 1 ⎡ sin( n − 1) x sin( n + 1) x ⎤
p
= − = 0 ( n ≠ 1) (6)
1 0 1 p 2p ⎢⎣ n − 1 n + 1 ⎥⎦ 0
= ∫ 0 cos nx dx + ∫ sin x cos nx dx
p −p p 0 ! sin nx = 0 at x = p , 0
5-18    Engineering Mathematics-II

n=1 
  
Find the Fourier series expansions of the following
1 p
p ∫− p
b1 = f ( x )sin x dx functions in the intervals given against them.

1 0 1 p f (x), Interval
= ∫ 0 ⋅ sin x dx + ∫ sin x sin x dx
p −p p 0 ⎧ 5 (0, p )
1. ⎨
1 p 1 ⎛
p
sin 2 x ⎞ ⎩ −5 (p , 2p )
=
2p ∫0 (1 − cos 2 x) dx = 2p ⎜⎝ x − 2 ⎠0
⎟ 20 sin nx
Ans: ∑
p n odd n
1 ⎡ 1 ⎤
= ⎢⎣(p − 0) − 2 (sin 2p − sin 0)⎥⎦ ⎧ cos x (0, p )
2p 2. ⎨
1 ⎩ − cos x ( −p ,0)
= (" sin 2p = sin 0 = 0) (7) 8 ∞ sin 2mx
2 Ans: ∑
p m =1 (2m − 1)(2m + 1)
Substituting these values in (2) we have 3. cosh ax ( −p , p )

1 2 ∞ cos 2mx 1 2a ⎡ 1 ∞
( −1)n ⎤
f ( x) = − ∑ + sin x (8) Ans: sinh ap ⎢ 2 + ∑ 2 2
cos nx ⎥
p p m=1 4m2 − 1 2 p ⎣ 2 a n =1 a + n ⎦
4. x − x 2 ( −p , p )
x = p /2 is a point of continuity and the series con-
verges to f ( p /2) = sin(p /2) = 1 p2 ∞
cos nx ∞
( −1)n sin nx
Ans: − + 4∑ 2 + 2∑
3 n =1 n n =1 n
Putting x = p /2 in (8) we obtain ⎧− x ( −p , 0)
5. ⎨
⎩ x (0, p )
1 1 2 ∞ cos mp p2 ∞
1= + ×1− ∑ Deduce that =∑
1
p 2 p m=1 (2m − 1)(2m + 1) 8 n =1 (2n − 1)2
1 1 2 ∞ ( −1)m p 4 ∞ cos(2n − 1) x
⇒ − =− ∑ Ans: − ∑
2 p p m=1 (2m − 1)(2m + 1) 2 p n =1 (2n − 1)2

" cos mp = ( −1)m ⎧ ⎛ p p⎞


⎪x ⎜⎝ − , ⎟⎠
⎪ 2 2
6. ⎨
Multiplying both sides by p /2 we obtain ⎪p − x ⎛ p 3p ⎞
,
⎪⎩ ⎝⎜ 2 2 ⎠⎟
1 1
p 1 1 1 1 1 Find the value of 1 + + +!
− = − + − +! (9) 32 52
4 2 1.3 3.5 5.7 7.9
4 ⎛ ∞ ( −1)n sin(2n − 1) x ⎞ p2
Again, x = 0 is a point of continuity and the series Ans: ∑ (2n − 1)2 ⎟⎠
p ⎜⎝ n =1
Sum =
8
converges to f (0) = 0. Putting x = 0 in (8) we obtain
⎧ x + x2 ( −p , p )
∞ ⎪
1 2 1 7. ⎨ 1 p2
0= − ∑ (10) ⎪Show that ∑ 2 =
p p m=1 (2m − 1)(2m + 1) ⎩ n 6
1 1 1 1 p2 ∞
( −1)n ∞
( −1)n +1
⇒ = + + +! Ans: + 4∑ cos nx + 2∑ sin nx
2 1.3 3.5 5.7 3 n =1 n n =1 n
Fourier Series    5-19

8. cos ax (0, 2p ) z
a ∉Z b b + 2p
Deduce that p cosec ap
1 ∞ ⎛ 1 1 ⎞
= + ∑ ( −1)n ⎜ +
a n =1 ⎝ a + n a − n ⎟⎠

sin 2ap α sin 2ap cos nx x
Ans:
2ap
+
p
∑ a 2 − n2
n =1 a a + 2I
(1 − cos2ap ) ∞ n sin nx

p
∑ a 2 − n2 If we change the interval from (b, b + 2p) of length
n =1

⎧ 1 (0, p ) 2p to an interval (a, a + 2l) of length 2l then we


9. ⎨ have to substitute
⎩2 − x p (p , 2p )
variable z x
3 2 ⎡ ∞ cos(2n − 1) x ⎤ 1 ⎡ ∞ sin nx ⎤ = =
Ans: − ⎢∑ ⎥ + ⎢∑ ⎥ length of the interval 2p 2l
4 p 2 ⎣ n =1 (2n − 1)2 ⎦ p ⎣ n =1 n ⎦
px p
⇒ z= , dz = dx
⎧0, ( −p , 0) l l (5.29)

10. ⎨ px
⎪⎩ 4 , (0, p ) Also,
l
p 1⎛ 1 1 ⎞ z=b ⇒ x= b = a (say)
Ans: − ⎜ cos x + 2 cos3x + 2 cos5 x + ! ⎟ p
16 2 ⎝ 3 5 ⎠
1 1 lβ
+
p⎛ ⎞
⎜ sin x − sin 2 x + sin3x − ! ⎠⎟ z = b + 2p ⇒ x l (b + 2p) = + 2l = a + 2l
4⎝ 2 3 p p

Now, the Fourier series of F (z) = F (px l ) =


       
  f (x) (say) is given by
    a a+ a0 ∞ npx ∞ npx
So far we have considered expansion of a function f ( x) = + ∑ an cos + ∑ bn sin (5.30)
2 n =1 l n =1 l
f (x) defined on an interval of length 2p in a Fourier
series of period 2p. In most engineering and physi- where the Fourier coefficients a0, an and bn are given
cal applications, the period is not 2p but is of arbi- by Euler’s formulas
trary length 2l. By a suitable transformation we can
1 a + 2l ⎛ np x np x ⎞
obtain formulas for Fourier series expansion of a
function in an interval (a, a + 2l).
( a0 , an , bn ) =
l ∫a
f ( x ) ⎜1, cos
⎝ l
, sin
l ⎠
⎟ dx
Let F (z) be defined in an interval (b, b + 2p) of (5.31)
length 2p.
Then the Fourier series of F (z) is given by  # Putting x = 0 in (5.31) we have formulas
∞ for f (x) defined in (0, 2l) and putting a = −l in (5.31)
a
F ( z ) = 0 + ∑ ( an cos nz + bn sin nz ) (5.27) we have formulas for f (x) defined in (−l, l).
2 n =1
where the Fourier constants are given by Euler’s
   $!!!"&
formulas
!#!!&#!%
1 b + 2p Let z and x be related by
p ∫b
( a0 , an , bn ) = F ( z )(1, cos nz , sin nz ) dz
z = lx + m (5.32)
(5.28)
5-20    Engineering Mathematics-II

π
We determine x such that z = −p when x = a and 2 p 2⎛ x2 ⎞
p ∫0
z = p when x = b. where a0 = (p − x ) dx = p x − =p
p ⎜⎝ 2 ⎟⎠ 0
la + m = −p ⎫ 2p (2)
∴ ⎬ ∴ l= (5.33)
lb + m = p ⎭ b−a
2p b 2 p
p ∫0
m = p − bl = p − an = (p − x ) cos 2nx dx (3)
b−a
p
b+a 2 ⎡ ⎛ sin 2nx ⎞ ⎛ cos 2nx ⎞ ⎤
m=− p = ⋅ ⎢(p − x ) ⎜ + − ⎟⎥ = 0
⎝ 2n ⎟⎠ ⎜⎝
or
b−a p ⎣ 4 n2 ⎠ ⎦ 0
The Fourier series of F (z′) in (−p, p) is given by
2 p
p ∫0
a ∞ ∞ bn = (p − x )sin 2nx dx (4)
F ( z′ ) = 0 + ∑ an cos nz′ + ∑ bn sin nz′ (5.34)
2 n =1 p
n =1 2⎡ ⎛ cos 2nx ⎞ ⎛ sin 2nx ⎞ ⎤
= ⎢(p − x ) ⎜ − ⎟ +⎜− ⎟⎥
where p⎣ ⎝ 2n ⎠ ⎝ 4 n2 ⎠ ⎦ 0
2 p 1
2p x = =
z′ = p 2n n
b−a
1 p p sin 2 x sin 4 x sin 6 x
( a0 , an , bn ) = ∫ F ( z′ )(1, cos nz ′, sin nz ′ ) dz ′ ∴ p−x= + + + +! (5)
p −p 2 1 2 3
(5.35)
 

⎛ 2p x ⎞ Find the Fourier series expansion of f (x) = eax in
Now let F ( z′ ) = F ⎜ = f ( x ). Then the
⎝ b − a ⎟⎠ (−l, l).
Fourier series of f (x) in (a, b) is given by    The Fourier series expansion of

a0 ∞ 2np x ∞ 2np x f (x) = eax in (−l, l)


f ( x) = + ∑ an cos + ∑ bn sin (1)
2 n =1 b − a n =1 b − a (5.36)
is given by
where
a0 ∞ np x ∞ np x
e ax = + ∑ an cos + ∑ bn sin (2)
2 b ⎛ 2np x 2np x ⎞ 2 n =1 l l
( a0 , an , bn ) =
b−a ∫ a
f ( x ) ⎜1, cos
⎝ b−a
, sin
b−a⎠
⎟ dx n =1
where
(5.37) l
1 l ax 1 ⎛ e ax ⎞
l ∫− l
a0 = e dx =
 
 l ⎜⎝ a ⎟⎠ − l
Find the Fourier series expansion for f (x) = p − x in 1 al 2
0 < x < p. = (e − e − al ) = sinh al (3)
al al
b−a p −0 p
   Here a = 0, b = p, = =
2 2 2 1 l ax np x
The Fourier series is an =
l ∫ − l
e cos
l
dx
a0 ∞ ∞ l
f ( x) = p − x = + ∑ an cos 2nx + ∑ bn sin 2nx (1) 1 ⎡ e ax ⎛ np x np np x ⎞ ⎤
2 n =1 n =1 = ⎢ 2 np 2 ⎜⎝ a cos + sin ⎟⎥
l ⎣⎢ a + ( l ) l l l ⎠ ⎦⎥
−l
Fourier Series    5-21

⎡ al ⎛ (7) + (8)
l np ⎞ : cosh x = 2sinh l
= 2 2 ⎢
e ⎜ a cos np + sin np ⎟ 2
l a +n p ⎣ ⎝
2 2
l ⎠
⎡1 ∞
( −1)n np x ⎤ (9)

−e − al ⎜ a cos np −
np ⎞⎤
sin np ⎟ ⎥ ⎢ + l∑ 2 2 2
cos ⎥
⎝ ⎠⎦ ⎣ 2 l n =1 l + n p l ⎦
l
(7) − (8)
la( −1)n cos np = ( −1)n : sinh x = 2p sinh l
= 2sinh al (4) 2
l 2 a 2 + n2p 2 and sin np = 0
⎡ ∞ ( −1)n −1 np x ⎤
1 l ax np x ⎢∑ 2 2 2
sin ⎥ (10)
+
l ∫− l
bn = e sin dx ⎣ n =1 l n p l ⎦
l
l  
1 ⎡ e ax ⎛ np x np np x ⎞ ⎤
= ⎢ 2 np 2 ⎜ a sin − cos ⎟⎥ Find the Fourier series expansion for
l ⎢⎣ a + ( l ) ⎝ l l l ⎠ ⎥⎦
−l
f (x) = ax + bx2 in (−l, l)
l ⎡ al ⎛ np ⎞
= 2 2 2 2 ⎢e ⎜⎝ a sin np − l cos np ⎟⎠    The Fourier series expansion for
l a +n p ⎣
f (x) = ax + bx2 in (−l, l) (1)
⎛ np ⎞⎤
−e − al ⎜ − a sin np − cos np ⎟ ⎥
⎝ l ⎠⎦ is given by
l ⎛ np ⎞ a0 ∞ np x ∞ np x
= 2 2 ⎜
− ( −1)n ⎟ (e al − e − al ) f ( x ) = ax + bx 2 = + ∑ an cos + ∑ bn sin
2 2
l a +n p ⎝ l ⎠ 2 n =1 l l
n =1
np ( −1)n +1 cos np = ( −1)n (2)
= 2 2 2 2
2sinh al
l a +n p and sin np = 0 (5) where
1 l 2b l 2
( ax + bx 2 ) dx =
l ∫− l l ∫0
The Fourier series expansion is given by a0 = x dx
⎡ 1 ∞
( −1)n np x l
e ax = 2sinh al ⎢ + la∑ 2 2 cos 2b ⎛ x 3 ⎞ 2bl 2
⎣ 2al n =1 l a + n p
2 2
l = ⎜ ⎟ =
l ⎝ 3 ⎠0 3

( −1)n −1 np x ⎤
+p∑ 2 2 2
sin ⎥ (6) a


0, if f is odd
n =1 l + n p l ⎦ ! ∫− a f ( x ) dx = ⎨ a
⎪⎩ 2∫0 f ( x ) dx, if f is even

   (3)
Putting a = 1, a = −1 in turn we obtain
⎡1 ∞ 1 l np x
( −1)n np x an = ∫ ( ax + bx 2 ) cos dx
e x = 2sinh l ⎢ + l ∑ 2 2 2
cos l l − l
⎣ 2l n =1 l + n p l
2b l np x ⎛ np x ⎞

( −1)n −1 np x ⎤ = 0 + ∫ x 2 cos dx ⎜⎝ x cos is odd⎟
+p∑ 2 sin l 0 l l ⎠
2 2 ⎥ (7)
n =1 l + n p l ⎦

⎡1 ∞
( −1)n np x 2b ⎡ 2 l np x l2 ⎛ np x ⎞
e − x = 2sinh l ⎢ + l ∑ 2 cos = ⎢x sin − 2x 2 2 ⎜⎝ − cos ⎟
2l l + n 2 2
p l l ⎣ np l np l ⎠
⎣ n =1
l (4)

( −1)n −1 np x ⎤ l3 ⎛ np x ⎞ ⎤
−p∑ 2 sin ⎥ (8) + 2 3 3 ⎜ − sin ⎟⎥
n =1 l + n p
2 2
l ⎦ np ⎝ l ⎠ ⎦0
5-22    Engineering Mathematics-II

⎧ t1 = t3 = 0 of the wave. Develop a Fourier series for the result-


4bl 2 ⎪⎪ np x ing periodic function.
= ( −1)n ⎨! sin = 0 at x = l , 0
n 2p 2 ⎪ l ⎧ T
⎪⎪ 0, when − < t < 0
⎪⎩ t2 = 0 at x = 0 2
U (t ) = ⎨
⎪ E sin wt , when 0 < t < T where T = 2p
1 l np x ⎪⎩ 2
bn = ∫ ( ax + bx 2 )sin dx w
l −l l
2a l np x ⎛ 2 np x ⎞   The Fourier series for the function
=
l ∫0
x sin
l
dx + 0 ⎜⎝ x sin
l
is odd⎟

⎧ T ⎫
l ⎪⎪ 0, when − < t < 0 ⎪⎪
2a ⎡ l ⎛ np x ⎞ l2 ⎛ np x ⎞ ⎤ U (t ) = ⎨
2
= ⎢x ⋅ ⎜⎝ − cos ⎟⎠ − 1⋅ 2 2 ⎜⎝ − sin ⎟⎥ ⎬
l ⎣ np l np l ⎠ ⎦0 ⎪ E sin wt , when 0 < t < T where T = 2p ⎪
⎩⎪ 2 w ⎭⎪
2a ⎡ l ⎤
= l ⋅ ( − cos np ) − 0 + 0⎥ (1)
l ⎢⎣ np ⎦
2al ⎡ np x ⎤ is given by
= ( −1)n −1 , ⎢⎣sin l = 0 at l , 0⎥ (5)
p ⎦ a0 ∞ 2npt ∞ 2npt
U (t ) = + ∑ an cos + ∑ bn sin (2)
2 n =1 T T
bl 2 4bl 2 ∞
( −1)n −1 np x n =1
∴ ax + bx 2 =
3
− 2
p
∑ n 2
cos
l
n =1 a0 ∞ ∞
∞ n −1 or U (t ) = + ∑ an cos nw t + ∑ bn sin nw t (3)
2al ( −1) np x 2 n =1
+ ∑
p n =1 n
sin
l
(6) n =1

where
 
  2 T /2 2 0 T /2
U (t ) dt = ⎡ ∫ 0 ⋅ dt + ∫ E sin wt dt ⎤
T ∫−T /2
a0 =
a = 1, b = − 1 T ⎣⎢ −T /2 0 ⎥⎦
T /2
− l 3 4l 2 ∞
( −1)n −1 np x 2⎡ ⎛ cos wt ⎞ ⎤
x − x2 =
3
+ 2
p
∑ n2
cos
l
= ⎢
T⎣
0+ E⎜−
⎝ ⎟
w ⎠ ⎥⎦ 0
n =1

2l ∞ ( −1) n −1
np x −2 E 2E
= (cos p − 1) = (! wT = 2p )
+ ∑
p n =1 n
sin
l
(7) wT π
(4)

a = 1, b = 1 2 T /2 2 T /2
l 3
4l 2
( −1) n −1
np x
an =
T ∫ − T /2
U (t ) cos nwt dt = ∫ E sin wt cos nwt dt
T 0
x + x2 = −
3 p2
∑ n 2
cos
l E T /2
= ∫ [sin( n + 1)wt − sin( n − 1)wt ] dt
∞ n −1 T 0
2l ( −1) np x
+ ∑
p n =1 n
sin
l
(8)
E ⎡ − cos( n + 1)wt cos( n − 1)wt ⎤
T /2
= + , ( n ≠ 1)
Choosing l = 1 in the above results (6), (7) and T ⎢⎣ ( n + 1)w ( n − 1)w ⎥⎦ 0
(8) we get Fourier series expansions in the interval E ⎡ − cos(n + 1) p + 1 cos(n − 1) p − 1⎤
(−1, 1) for the functions ax + bx2, x − x2, x + x2, = +
wT ⎢⎣ ( n + 1) ( n − 1) ⎥

respectively.
E ⎡ −( −1)n +1 + 1 ( −1)n +1 − 1⎤
   = ⎢ + ⎥
wT ⎣ n +1 n −1 ⎦
A sinusoidal voltage E sin wt is passed through a
( n ≠ 1)
half-wave rectifier which clips the regative portion
Fourier Series    5-23

⎧ 0, if n is odd 2. Obtain the Fourier series


p−x
expansion of
⎪ f ( x) = in 0 < x < 2.
= ⎨ 2E ⎛ 1 1 ⎞ −2 E if n is even 2
⎪ 2π ⎝⎜ n + 1 − n − 1 ⎠⎟ = π ( n2 − 1) , p − x p − 1 ∞ sin np x
⎩ Ans: = +∑
(5) 2 2 n =1 np
3. Find the Fourier series for f (x) = ex in (−l, l).
n=1 ⎡ n np x ⎤
∞ ( −1) cos
⎢1 l ⎥
2 T /2 E T /2 Ans: e = sinh l ⎢ + 2l ∑ 2
x
2 2 ⎥
+
a1 = ∫
T 0
E sin wt cos wt dt = ∫ sin 2wt dt
T 0


l n =1 l n p ⎥

E E n p x
= ( − cos 2wt )T0 /2 = (cos wT − cos 0) ∞ ( −1)
n −1
n sin
2Tw 2wT + 2p ∑ l
2 2 2
E n =1 l + n p
= (cos 2p − cos 0) = 0 (6)
4p ⎧px, 0 < x < 1
4. Expand f ( x ) = ⎨ into a Fourier series
2 T /2 ⎩ 0, 1 < x < 2
bn = ∫ E sin wt sin nwt dt
T 0 p 2 ∞ cos(2n − 1)p x ∞ ( −1)n −1 sin np x
E T /2 Ans: f ( x ) = − ∑ +∑
= ∫ [cos( n − 1)wt − cos( n + 1)wt ] dt 4 p n =1 (2n − 1)2 n =1 n
T 0
T /2 ⎧ x, − 1 < x < 0
E ⎡ sin( n − 1)wt sin( n + 1)wt ⎤ 5. Find the Fourier series of f ( x ) = ⎨ and
= − = 0, ⎩ x + 2, 0 < x < 1
Tw ⎢⎣ ( n − 1) n + 1 ⎥⎦ 0
1 1 1 p
deduce that 1 − + − + ! = .
( n ≠ 1) (7) 3 5 7 4

2 (1 − ( −1n ))
Ans: f ( x ) = 1 +
p
∑ n
sin np x
n=1 n =1

⎧0 − l < x < 0
2 T /2 E T /2 6. Expand f ( x ) = ⎨ as a Fourier series.
b1 = ∫ E sin wt sin wt dt = ∫ (1 − cos 2wt ) dt ⎩k 0< x<l
T 0 T 0 x
sin(2n − 1)p
T /2 k 2k ∞
=
E ⎛ sin 2wt ⎞
⎜t − ⎟ =
E
(8)
Ans: f ( x ) = + ∑ 2n − 1 l
2 p n =1
T⎝ 2w ⎠ 0 2
(" sin wT = sin 2p = sin 0 = 0) ⎧l − x , 0 < x ≤ l
7. Find the Fourier series of f ( x ) = ⎨
⎩0, l ≤ x < 2l
Substituting the values of the constants in (2) we Ans:
obtain x x
cos(2n − 1)p ∞ sin np
1 ⎛ 2l ⎞ ∞ l +⎛ l⎞
f ( x) = +⎜ ⎟∑ ⎜⎝ ⎟⎠ ∑
l
E E 2E ∞ cos(2m)wt 4 ⎝ p 2 ⎠ n =1 (2n − 1)2 p n =1 n
U (t ) = + sin wt −
p 2

p m=1 (2m − 1)(2m + 1)
(9)
8. Obtain the Fourier series expansion for
⎧0, − 8 < x < 0

f ( x ) = ⎨4, 0 < x < 4

   ⎪0, 4 < x < 8

⎡ d − x, 0 < x < d ( −1)n (2n − 1)p x
1. Find the Fourier series of f ( x ) = ⎢ Ans: f ( x ) = 1 + ∑ cos
⎣ 0, d < x < 2d (2n − 1) 8
np x (2n − 1)p x
sin sin
l 2l ∞ cos(2n − 1)p x l ∞ 4 ∞ 8
Ans: f ( x ) = + 2 ∑ + ∑ l − ∑
4 p n =1 (2n − 1)2 p n =1 n p n =1 (2n − 1)
5-24    Engineering Mathematics-II


   & #
     Find the Fourier series for f (x) = sin x in −p < x < p.
 [JNTU 2002]
We now work out some examples on Fourier series "%$"! The function is defined in S = (−p, p)
expansions of even and odd functions in (−l, l) and by
(−p, p) by applying the properties discussed in
sections 5.3 and 5.6 on pp. 5-2 and 5-5. f (x) = sin x (1)
Here
& #
Find the Fourier series of f (x) = x in (−p, p). (i) x ∈ S ⇒ −x ∈ S and (ii) f (−x) =
sin(−x) = −sin x = −f (x) ∀ x ∈ S.
"%$"! The function is defined in S = (−p, p) by ∴ f is an odd function in S and hence a0 = an = 0
f (x) = x (1) for all n.
The Fourier series of the function is given by
Here (i) x ∈S ⇒ − x ∈S and (ii) f (− x) =
− x = − f (x). ∞

∴ f (x) is an odd function and hence a0 = an = 0 sin x = ∑ bn sin nx (2)


n =1
for all x. where
The Fourier series of the function is given by
2 p
p ∫0
∞ bn = sin x sin nx dx
x = ∑ bn sin nx (2)
n =1 1 p
where = ∫ [cos( n − 1) x − cos( n + 1) x ] dx (3)
p 0
p
2 p 2 ⎡ ⎛ cos nx ⎞ ⎛ sin nx ⎞ ⎤
p ∫0
p
bn = x sin nx dx = ⎢ x ⎜ − ⎟ − 1⎜ − 2 ⎟ ⎥ 1 ⎡ sin( n − 1) x sin( n + 1) x ⎤
p⎣ ⎝ n ⎠ ⎝ n ⎠⎦0 = − = 0, n ≠ 1 (4)
n +1
p ⎢⎣ n − 1 n + 1 ⎥⎦ 0
2 ⎛ −p cos np ⎞ 2( −1)
= ⎜ ⎟= (3) ∵ sin p = 0 for p = ( n + 1)p , ( n − 1)p or 0
p⎝ n ⎠ n
⎡t2 = 0 at x = p , 0 ⎤
⎢t = 0 at x = 0 ⎥ n=1
⎣1 ⎦
2 p 1 p
p ∫0
b1 = sin x sin x dx = ∫ (1 − cos 2 x ) dx
Substituting the value in (2) we obtain p 0

( −1)n +1
p
1⎛ sin 2 x ⎞ 1⎡ 1 ⎤
x = 2∑ sin nx (4) = ⎜x− ⎟ = ⎢(p − 0) − (sin 2p − 0) ⎥ = 1
n =1 n p⎝ 2 ⎠0 p ⎣ 2 ⎦

%$"!
Hence, the required Fourier series for f (x) is
x = p 2 is a point of continuity of f (x) = x and the
f (x) = sin x
series converges to
"$ There is only one term in the Fourier series
⎡0 if n is even
⎛p⎞ p p np ⎢ n −1
in this case.
f ⎜ ⎟ = at x = sin = ( −1) if n is odd
⎝ 2⎠ 2 2 2 ⎢⎢
⎣ and n = 2m − 1 & #

∞ m −1
p ( −1) p 1 1 1 1 ⎧ 2x
∴ = 2∑ ⇒ = − + + +! ⎪⎪1 + p (−p , 0)
2 (2 m − 1) 4 1 3 5 7
m =1 Find the Fourier series for f ( x ) = ⎨
(5) ⎪ 1 − 2x (0, p )
⎪⎩ p
Fourier Series    5-25

  Consider the function defined in S = x = 0 point of continuity for f (x) and the series
(−p, p) by converges to f (0) = 1
Hence, from (6) for x = 0
f (x) = a − b|x| (1)
8 1
where a, b are constants. Here 1= ∑ (8)
(i) x ∈ S ⇒ −x ∈ S and (ii) f (−x) = p 2
(2n − 1)2
a − b |−x| = a − b |x| = f (x) 1 1 1 1 p2
Hence, f (x) is an even function for which the ⇒ ∑ (2n − 1)2 =
12
+
32
+
52
+! =
8
Fourier series is given by
a0 ∞   
f ( x) = a − b x = + ∑ an cos nx (bn = 0 for all n)
2 n =1 Find the Fourier series expansion for
(2) ⎡ −k , − p < x < 0
f ( x) = ⎢ where k > 0;
where ⎣ k, 0 < x < p
2 p 2 p 2 p p ∞
( −1)n −1
=∑
p ∫0
a0 = f ( x ) dx = ∫ ( a − b x ) dx = ∫ ( a − bx ) dx Deduce that
p 0 p 0 4 n =1 2n − 1
p
2⎛ x2 ⎞ 2⎛ p2 ⎞ ⎛ 1 ⎞ [JNTU 1995, 2001, 2002]
= ⎜ ax − b ⎟ = ⎜ ap − b ⎟ = 2 ⎜ a − bp ⎟
p⎝ 2 ⎠0 p ⎝ 2⎠ ⎝ 2 ⎠
⎡ −k , − p < x < 0
2 p 2 p   The function f ( x ) = ⎢
p ∫0
an = f ( x ) cos nx dx = ∫ ( a − bx ) cos nx dx 0< x<p
p 0 ⎣ k,
p (1)
2⎡ ⎛ sin nx ⎞ ⎛ cos nx ⎞ ⎤
= ( a − bx ) ⎜ + b⎜− 2 ⎟⎥ (3) is defined in the set S = (−p, 0) ∪ (0, p).
p ⎢⎣ ⎝ n ⎟⎠ ⎝ n ⎠ ⎦0
∴ We find that (i) x ∈ S ⇒ −x ∈ S and
−2b ⎛ cos np − cos 0 ⎞ (ii) f (x) = k for 0 < x < p.
= ⎜ ⎟
p ⎝ n2 ⎠ − k = − f ( x )⎫
⇒ ⎬ for 0 > − x > −p
2b ⎛ ( −1) − 1 ⎞
n
(4) = f ( − x )⎭
=− ⎜ ⎟
p ⎜⎝ n2 ⎟⎠ ∴ f is an odd function in S.
Also, from Figure 5.1 (p. 5-26) we see that the
⎧0, if n is even
⎪ graph of the function is symmetrical with respect to
=⎨ 4b if n is odd, n = 2m − 1 the origin and hence f is odd in S; consequently, the
⎪ p (2m − 1)2 , (sin nx = 0 at x = p , 0) Fourier series consists of only sines and it is given by

Substituting the values in (2) we have ∞
f ( x ) = ∑ bn sin nx (2)
1 4b ∞ cos(2m − 1) x n =1
a − b x = a − bp +
2

p m=1 (2m − 1)2
(5)
where
p
 
  2 p 2k ⎛ cos nx ⎞

bn =
p ∫0
k sin nx dx =
p ⎝
⎜−
n ⎠0

2 2 8 cos(2n − 1) x
(i) a = 1, b =
p
; 1− x = 2
p p
∑ (2n − 1)2
(6)
=−
2k
(cos np − cos 0)
n =1
n

p 4 cos(2n − 1) x
(ii) a = 1, b = 1; 1 − x = 1 − +
2 p
∑ (2n − 1)2 −2k n
⎧ 0, if n is even

n =1 = (( −1) − 1) = ⎨ 4k (3)
(7) pn ⎪⎩ p , if n is odd
5-26    Engineering Mathematics-II
np x
( −1)n −1 cos
8 16 ∞
4k ∞
sin(2n − 1) x Ans: 4 − x = + 2 ∑
2 l
∴ f ( x) = ∑
p n =1 (2n − 1)
(4) 3 p n =1 n2
5. f (x) = x2 −2 in (−2, 2)
%$ "! 2 16 ⎛ px 1 1 3p x
Ans: x 2 − 2 = − − ⎜ cos − cos p x + 2 cos
3 p2 ⎝ 2 22 3 2
f is continuous at x = p 2. The series (4) converges
1 ⎞
− 2 cos2p x + ! ⎟
to f (p 2 ) = k at x = p 2. 4 ⎠
6. f (x) = cosh x in (−p, p)
4k ⎛ 1 1 1 1 1 ⎞ 2sin hp ⎛ 1 ∞ (( −1)n − 1) ⎞
∴ k= ⎜1 − + + + − + ! ⎟⎠ Ans: cos hx = +∑ cos nx⎟
p ⎝ 3 5 7 9 11 ⎜ 2
p ⎝ 2 n =1 n + 1 ⎠
p 1 1 1 7. f (x) = |sin x| in (−p, p)
or = 1− + − +! (5)
4 3 5 7
2 4 ⎛ cos2 x cos 4 x cos6 x ⎞
Ans: sin x = − ⎜ + + +! ⎟
y p p ⎝ 1.3 3.5 5.7 ⎠

⎧ 2x
y=k ⎪⎪1 + p , − p ≤ x ≤ 0
8. f ( x ) = ⎨
k ⎪ 1 − 2x , 0 ≤ x ≤ p
⎪⎩ p
_p
0 p x 8 ⎛ ∞ cos(2n − 1) x ⎞
k Ans: f ( x ) = ⎜∑ ⎟
y = _k p 2 ⎝ n =1 (2n − 1) 2 ⎠
⎧0, − 2 ≤ x < −1

9. f ( x ) = ⎨k , − 1 < x < 1
⎪0, 1< x < 2
 %# ⎩
k 2k ⎛ px 1 3p x 1 5p x
Ans: f ( x ) = + ⎜⎝ cos − cos + cos
2 p 2 3 2 5 2
1 7p x ⎞
     − cos +! ⎟

7 2
Obtain Fourier series expansions with respect to the follow-
⎧p
ing even/odd functions in the indicated intervals (−l, l). ⎪⎪ 2 + x, − p < x < 0
5.3 f ( x ) = ⎨
1. f (x) = x2 (−l, l) [JNTU 2002] ⎪ p − x, 0 < x < p
np x n −1 ⎪⎩ 2
( −1) cos
l 2 4l 2 ∞
Ans: x = − 2 ∑
2 l 1 1 1 1 p2
3 p n =1 n2 Deduce that 2
+ 2 + 2 + 2 +! = .
1 3 5 7 8
2. f (x) = sin 3x, (−l, l) 4 cos nx
⎛ ( −1)n −1 ⎞

np x
Ans: f ( x ) = ∑
p n odd n2
Ans: sin3x = 2p sin3l ∑ ⎜ 2 2 2⎟
sin
n =1 ⎝ n p − 9l ⎠ l
 

     
3. Show that
2sin pp ∞ n −1
( −1) n sin nx
    
      
sin px =
p
∑ n2 − p 2
in ( −p , p ) ( p ∉ Z ) 

n =1
In most engineering and physical applications we
4. f (x) = 4 −x2 in (−2, 2) may require Fourier series expansion for a func-
[JNTU Dec. 2002(4), 2003 S (4), 2003 S (4), 2005 S(3)] tion defined in the interval (0, l) only. Such a series
Fourier Series    5-27

is known as half-range Fourier series. Since f (x) is 2. g(x) is an even function in the interval (−l, l)
now neither periodic nor classifiable as even or odd and is periodic with period 2l
we have the option of obtaining Fourier sine series
(FSS) only or Fourier cosine series (FCS) only in Such a function g(x) is called an even periodic
the interval (0, l ), as the case may be. To obtain a continuation (or extension) of f (x) (Figure 5.2).
Fourier sine series for f (x) we construct a new func- The FCS for g(x) in (−l, l) is given by
tion g(x) such that a0 ∞ np x
g( x) = + ∑ an cos
1. g(x) = f (x) in the interval (0, l) 2 n =1 l
where
2. g(x) is an odd function in the interval (−l, l)
and is periodic with period 2l 2 l
l ∫0
a0 = f ( x ) dx, and
Such a function g(x) is called an odd periodic
continuation (or extension) of f (x) (Figure 5.2). The 2 l np x
an = ∫ f ( x ) cos dx and bn = 0 for all n
FSS for g(x) in (−l, l) is given by l 0 l

sin np x
g ( x ) = ∑ bn    The above expansions for f (x) are valid
n =1 l
where only in (0, l).
2 l np x 2 l np x
bn =
l ∫0
g ( x )sin
l
dx = ∫ f ( x )sin
l 0 l
dx
y

with a0 = an = 0 for all n.

_I 0 I x

   Even periodic continuation

_I 0 I x   
Find a FSS for f (x) = a + bx in 0 < x < l where a and
b are constants.


 Here the function is defined by

f (x) = a + bx in 0 < x < l (1)

and we are required to find Fourier half-range sine


series for (1), which is given by
   Odd periodic continuation ∞
np x
f ( x ) = a + bx = ∑ bn sin (2)
n =1 l
Similarly to obtain the half-range FCS for f (x)
we define g(x) such that where

1. g(x) = f (x) in (0, l)


5-28    Engineering Mathematics-II

2 l np x 2 l np x sin nx = 0 at x = p , 0
l ∫0
bn = f ( x )sin dx = ∫ ( a + bx )sin dx
l l 0 l t1 = 0 at x = 0
l
2 ⎡ ⎛ l np x ⎞ ⎛ l2 np x ⎞ ⎤ cos np = ( −1)n , cos 0 = 1
= ⋅ ⎢( a + bx ) ⎜ − cos ⎟ − b ⎜ − 2 2 sin ⎥ !
l ⎢⎣ ⎝ np l ⎠ ⎝ np l ⎟⎠ ⎥⎦
0
⎡ −2( a + bp )
t2 = 0 at x = l , 0 ⎢ + 0, if n is even
n
−2 l bn = ⎢ (3)
= ⋅ [( a + bl )(cos np ) − a] + 0 ⎢ 2( a + bp ) − 8b , if n is odd
l np ⎢⎣ n pn3
2
= ⎡ a(1 − ( −1)n ) − bl ( −1)n )⎤ (3)
np ⎣ ⎦ Substituting these values in (2) we obtain

Substituting these values in (2) we obtain ⎛ sin(2m − 1) x sin 2mx ⎞
ax + bx 2 = 2( a + bp ) ∑ ⎜ −
m =1 ⎝ (2m − 1) 2m ⎟⎠
4a ∞ sin[(2m − 1)p x l ]
a + bx = ∑ (2m − 1) 8b ∞ sin(2m − 1) x
p m =1 − ∑
p m=1 (2m − 1)3
(4)
⎛ np x ⎞
( −1) n −1 sin ⎜
2bl ∞ ⎝ l ⎟⎠
+ ∑
p n =1 n
(4)

  
Putting
  (i) a = p, b = −1. We obtain from (4)
Find the half-range sine series for f (x) = x(p −x) in 8 ∞ sin(2m − 1) x
0<x<p x(p − x ) = ∑
p m=1 (2m − 1)3
(5)
1 1 1 1 p2
Deduce that
3
− 3 + 3 − 3 +"
1 3 5 7 32 x = p 2 is a point of continuity for f (x) = x(p − x)
[JNTU Dec. 2002, 2003 (2)] and the series converges to f (p 2) = p 2 4 at x = p 2
   To find the FSS expansion for p
sin(2m − 1)
f (x) = x(p − x) = p x − x2 in (0, p) we consider p2 8 ∞ 2 = 8

( −1)m −1
∴ = ∑ ∑ (6)
f (x) = ax + bx2 in 0 < x < p (1) 4 p m=1 (2m − 1)3 p m=1 (2m − 1)3
The required series is p3 1 1 1 1
∞ ⇒ = − + − +"
f ( x ) = ax + bx 2 = ∑ bn sin nx (2) 32 13 33 53 73
n =1
(ii) a = 1, b = 0. We deduce the sine series for
where f (x) = x in (0, p)
2 p 2 p ∞
bn = ∫ f ( x ) sin nx dx = ∫ ( ax + bx 2 ) sin nx dx ⎛ sin(2m − 1) x sin 2mx ⎞
p 0 p 0 x = 2∑ ⎜ − ⎟ (7)
m =1
⎝ 2m − 1 2m ⎠
2⎡ ⎛ cos nx ⎞ ⎛ sin nx ⎞
= ⎢( ax + bx 2 ) ⎜ − ⎟ − ( a + 2bx ) ⎜ − 2 ⎟ ∞
( −1) n −1sin nx
p⎣ ⎝ n ⎠ ⎝ n ⎠ = 2∑
p n =1 n
⎛ cos nx ⎞ ⎤
+ 2b ⎜ 3 ⎟ ⎥ (iii) a = 0, b = 1. We deduce the sine series for
⎝ n ⎠ ⎦0
f (x) = x2 in (0, p)
2 ⎡ ( ap + bp 2 ) 2b ⎤
= ⎢ ( −1) n +1 + 3 (( −1) n − 1) ⎥ ∞
( −1)n sin nx 1 ∞ sin(2n − 1) x
p⎣ n n ⎦ x 2 = 2p ∑ − ∑ (8)
n =1 n p n =1 (2n − 1)3
Fourier Series    5-29


  ∞
( −1)n −1
x
Obtain the half-range FSS for f (x) = e in 0 < x < 1. x = 2∑ sin nx (4)
n =1 n
[JNTU 2003, 2003(2)]


 
   The half-range FSS of ex is e x =
∞ Find the half-range sine series for f (t) = t − t2 in
∑ bn sin nx ( l = 1) (1) 0 < t < 1. [JNTU May/June 2002]
n =1

where    The half-range FSS for the function


1
bn = 2∫ e x sin np x dx f (t) = t − t2 in 0<t<1 (1)
0
1 is given by
⎡ 2e x ⎤
=⎢ 2 2 (1⋅ sin np x − np cos np x )⎥
⎣n p +1 ⎦0 f (t ) = ∑ bn sin np t (! l = 1 here) (2)
2e 2.1 where
= 2 2 (0 − np cos np ) − 2 2 (0 − np ⋅1)
n p +1 n p +1 2 1 1
2np
= 2 2 (1 − ( −1)n e)
bn =
1 ∫ 0
f (t )sin np t dt = 2∫ (t − t 2 )sin npt dt
0
n p +1 (2) ⎡ ⎛ cos np t ⎞ ⎛ sin np t ⎞
= 2 ⎢ (t − t 2 ) ⎜ − ⎟ − (1 − 2t ) ⎜ − 2 2 ⎟
⎣ ⎝ np ⎠ ⎝ np ⎠
The FSS for f (x) = ex in (0, 1)
1
⎛ cos np t ⎞ ⎤
⎡ 2(1 + e) 2(1 − e) + ( −2) ⎜ 3 3 ⎟ ⎥
ex = p ⎢ 2 sin p x + 2 sin 2p x (3) ⎝ n p ⎠ ⎦0
⎣ p +1 4p + 1
3(1 + e) 4 4
+ 2 sin 3p x + " ⎥
⎤ =− (cos np − 1) = − (( −1)n − 1)
9p + 1 ⎦ np3 3
np 3 3

⎡ 0, if n is even

  =⎢ 8
⎢ , if n is odd
Find the Fourier half-range sine series for ⎢⎣ n3p 3
f (x) = x in (0, p). [JNTU 2003 (4)]
Hence, the FSS is
   The half-range FSS for

8 sin(2n − 1)p t
f (x) = x in (0, p) (1) t − t2 =
p 3 ∑ (2n − 1)3
(3)
∞ n =1

is given by f ( x ) = x = ∑ bn sin nx (2)


n =1 
 
where Expand cos p x in (0, 1) as an FSS (or) show that
2 p 2 p
8 ∞ n
p ∫0
bn = f ( x )sin nx dx = ∫ x sin nx dx
p 0 cos p x = ∑
p n =1 4n2 − 1
sin 2np x in (0, 1)
p
2 ⎡ ⎛ cos x ⎞ ⎛ sin nx ⎞ ⎤ t2 = 0 at x = p , 0 [JNTU 2004 S(3)]
= x⎜− ⎟ − 1⎜ − 2 ⎟⎠ ⎥
p ⎢⎣ ⎝ n ⎠ ⎝ n ⎦0 t1 = 0 at x = 0
2p ( −1)n (3)    The FSS of cos p x is given by
=− cos np = −2 ⋅
pn n ∞
cos p x = ∑ bn sin np x (! l = 1) (1)
Substituting for bn in (2) we obtain n =1
5-30    Engineering Mathematics-II

where 2 p 2 p
1
an = ∫
p 0
f ( x ) cos nx dx = ∫ x cos nx dx
p 0
bn = 2∫ cos p x ⋅ sin np x dx p
0 2 ⎡ ⎛ sin nx ⎞ ⎛ cos np⎞ ⎤
= x⎜ ⎟ − 1 ⎜ − 2 ⎟⎠ ⎥
p ⎢⎣ ⎝ n ⎠ ⎝
1
= ∫ [sin( n + 1)p x + sin( n − 1)p x ] dx n ⎦0
0

−1 ⎡ cos( n + 1)p x cos( n − 1)p x ⎤


1 2⎡ 1 ⎤
= + = ⎢ 0 + 2 (cos nx − cos 0) ⎥
p ⎢⎣ n +1 n −1 ⎥⎦ , ( n ≠ 1) p⎣ n ⎦
0
2
−1 ⎡ ( −1) n +1 − 1 ( −1) n −1 − 1⎤ = 2 ⎡⎣( −1) n − 1⎤⎦ ∵ sin p = sin 0 = 0
= ⎢ + ⎥ pn
p ⎣ n +1 n −1 ⎦
⎡ 0, if n is even
⎡ 0, if n is odd
= ⎢ −4 if n is odd and n = 2m − 1
= ⎢ 4n ⎢ ,
⎢ , if n is even. (2) ⎢⎣ p (2m − 1)2
⎢⎣ p ( n2 − 1)
(4)

n=1 Substituting these values in (2) we obtain


1 1 p 4 ∞ cos(2m − 1) x
b1 = ∫ 2sin p x cos p x dx = (sin2 p x )10 = 0 x= − ∑ (5)
0 p 2 p m=1 (2m − 1)2
∴ The FSS for cos p x is
8 ∞ m sin 2mx
cos p x = ∑
p m=1 4m2 − 1
(3) y

  
Prove that the function f (x) = x can be expanded in
a series of cosines in
p 4 ⎛ cos x cos 3x cos 5x ⎞ –3p –2p –p 0 p 2p 3p x
0 < x < p as x = − ⎜ 2 + 2 + +! ⎟
2 p⎝ 1 3 52 ⎠
[JNTU 2003 S]
2
Hence, deduce that p = 1 + 1 + 1 +!
8 12 32 52
  Here we are required to find Fourier  
 
series in the positive interval (0, p) for f (x) = x in
x = 0 is a point of discontinuity for f (x);
terms of cosines. This means that we have to find
half-range FCS for f (0 −) + f (0+ )
f (0) = =0
2
f (x) = x (1)

The series converges to 0 at x = 0
a
and this is given by f ( x ) = 0 + ∑ an cos nx (2)
2 n =1 p 4⎛ 1 1 1 ⎞
where ∴ 0= − ⎜ + + +! ⎟
2 p ⎝ 12 32 52 ⎠
p
2 p 2 p 2 ⎛ x2 ⎞ 1 1 1 p2
a0 = ∫
p 0
f ( x ) dx = ∫ x dx = ⎜ ⎟ = p (3)
p 0 p ⎝ 2 ⎠0
⇒ + + +! = (6)
12 32 52 8
Fourier Series    5-31


  1 1 1 1 1 1 p2
Find the half-range cosine series for f (x) = x(2 − x) − + − + − +! = (6)
12 22 32 42 52 62 12
in 0 ≤ x < 2 and hence find the sum of the series.
[JNTU 2002, 2003, 2004 (3)]

 
 The half-range FCS for
For the function defined by the graph below, find the
f (x) = x(2 − x) in 0 ≤ x < 2 (1) half-range FCS. [JNTU 2003 S]
is given by
 The graph is composed of two line

a np x ⎛ p⎞
x(2 − x ) = 0 + ∑ an cos (" l = 2) (2) segments OP and PB in the intervals ⎜ 0, ⎟ and
2 n =1 2 ⎝ 2⎠
⎛p ⎞
where ⎜⎝ , p ⎟⎠ , respectively; their slopes being +1 and −1
2
2 respectively.
2 2 2 ⎛ 1 ⎞
a0 = ∫ f ( x ) dx = ∫ (2 x − x 2 ) dx = ⎜ x 2 − x 3 ⎟
2 0 0 ⎝ 3 ⎠0 ⎛ p⎞
Equation of OP : y = x in ⎜ 0, ⎟
⎝ 2⎠
(3)
8 4
= 4− = ⎛p ⎞
3 3 Equation of PB : y = −(x −p) in ⎜ , p ⎟
⎝2 ⎠
2 2 np x 2 np x
an = ∫ f ( x ) cos dx = ∫ (2 x − x 2 ) cos dx y
2 0 2 0 2
⎛ p , p⎞
⎝ 2 2⎠ y=p

2
⎡ ⎛ 2⎞ np x
= ⎢(2 x − x 2 ) ⎜ ⎟ ⋅ sin
P
− (2 − 2 x ) ⋅
⎣ ⎝ np ⎠ 2
2
4 ⎛ np x ⎞ 8 ⎛ np x ⎞ ⎤
2 2 ⎜
− cos ⎟ + ( −2) 3 3 ⎜ − sin ⎟
np ⎝ 2 ⎠ np ⎝ 2 ⎠ ⎥⎦ 0
t1 = t3 = 0 at x = 2,0
8
= 2 2
( − cos np − cos 0) 0 Ap

Bp x
np 2

⎡ 0, if n is odd
8 n ⎢ 16 So, the function f (x) is defined by
=− (( − 1) + 1)
n2 p 2 ⎢− , if n is even (4)
⎢⎣ n2p 2
⎧ x, 0 < x < p / 2⎫
f ( x) = ⎨ ⎬ (1)
Substituting these values in (2) we obtain ⎩p − x, p / 2 < x < p ⎭

2 4 cos mp x
2x − x2 = −
3 p2
∑ m2
(5) The half-range FCS is
m =1
2 4 ⎛ 1 1
= − ⎜⎝ cos px + 2 cos 2p x f ( x) = a0 + ∑ an cos nx (2)
3 p2 2 2 n =1
1 ⎞
+ cos 3p x + ! ⎟ where
3 2 ⎠
   2 p 2 p /2 2 p
Putting x = 1 in (5) we have
a0 =
p ∫0
f ( x ) dx = ∫ x dx + ∫ (p − x ) dx
p 0 p p /2
5-32    Engineering Mathematics-II

p /2
2 ⎛ x2⎞ 2⎛ x2 ⎞
p a2(2k ) = a4 k = 0 ⎫
= + px − ⎟ ⎪
p ⎜⎝ 2 ⎟⎠ 0 ⎜
p⎝ 2 ⎠ p /2 a2(2k −1) = a4 k − 2 = −
2 ⎬ k ∈N (6)
p (2k − 1) 2 ⎪⎭
2 p2 2 ⎡ 1⎛ p2 ⎞⎤ p
= ⋅ + ⎢p (p − p / 2) − ⎜ p 2 − ⎟ ⎥ = 2 2 1 2 1
p 8 p ⎢⎣ 2⎝ 4 ⎠ ⎦⎥ 2 ∴ a2 = − , a6 = − ⋅ 2 , a10 = − . 2 ,!
p p 3 p 5
(3)
k = 1, 2, 3, 4,"

2 p Substituting these values in (2) we obtain


p ∫0
an = f ( x ) cos nx dx
p 2 ⎛ cos 2 x cos 6 x cos10 x ⎞
2 p /2 2 p f ( x) = − ⎜ 2 + + +! ⎟ (7)
= ∫ x cos nx dx + ∫ (p − x )cos nx dx 4 p ⎝ 1 32
52 ⎠
p 0 p p /2
p /2
2 ⎡ ⎛ sin nx ⎞ ⎛ cos nx ⎞ ⎤   
= ⎢ x⎜ ⎟⎠ − 1⋅ ⎜⎝ − ⎟⎥
p⎣ ⎝ n n2 ⎠ ⎦ 0 Find the FCS for f (x) = x sin x in (0, p) and show
p that
2⎡ ⎛ sin nx ⎞ ⎛ cos nx ⎞ ⎤
+ (p − x ) ⎜ − ( −1) ⋅ ⎜ − 2 2 2 2
p ⎢⎣ ⎝ n ⎟⎠ ⎝ ⎟⎥
n2 ⎠ ⎦p /2 1+ − + −
p
+ ! = . [JNTU 2002 (2)]
1.3 3.5 5.7 7.9 2

2 ⎡1 ⎛ p np ⎞ 1 ⎛ np ⎞⎤  
  Here we have to find half-range cosine
= ⎢ ⎜⎝ sin − 0⎟ + 2 ⎜ cos − cos 0⎟ ⎥
p ⎣n 2 2 ⎠ n ⎝ 2 ⎠ ⎦ series for
⎡⎛ p np ⎞ ⎤ f (x) = x sin x in (0, p) (1)
0 − sin ⎟
2 ⎢ ⎜⎝ 2 2⎠ 1 ⎛ np ⎞ ⎥
+ ⎢ − 2 ⎜ cos np − cos ⎟ ⎥ The required Fourier half-range cosine series
p⎢ n n ⎝ 2 ⎠⎥ is
⎢⎣ ⎥⎦
a0 ∞
2 ⎡⎛ p np p np ⎞ ⎤ f ( x ) = x sin x = + ∑ an cos nx (2)
= ⎜ sin − sin ⎟ ⎥ 2 n =1
p n ⎢⎣⎝ 2 2 2 2 ⎠⎦ where
2 ⎡ np ⎤ 2 p 2 p
+ 2 ⎢2 cos − cos 0 − cos np ⎥
p ∫0
a0 = f ( x ) dx = ∫ x sin x dx
pn ⎣ 2 ⎦ p 0
2 ⎡ np ⎤ 2 −2
= 0 + 2 ⎢2 cos − (1 + ( −1) n ) ⎥ = [ x( − cos x ) − 1⋅ ( − sin x )]p0 = p cos p = 2
pn ⎣ 2 ⎦ p p
(3)
⎧ 0, if n is odd ⎧⎧sin x = 0 at x = p , 0
⎪ (# cos p = −1) ⎨
=⎨ 4 ⎩ t1 = 0 at x = 0
⎪ p (2m)2 (cos mp − 1), if n is even and n = 2m
⎩ 2 p 2 p
p ∫0
an = f ( x ) cos nx dx = ∫ x sin x cos nx dx
(4) p 0
1 p
= ∫ x[sin( n + 1) x − sin( n − 1) x ] dx
p 0
1
a1 = a3 = a5 = ! = 0 and a2m = (cos mp − 1) 1 ⎡ ⎧ − cos( n + 1) x cos( n − 1) x⎫
pm 2 = ⎢ x⎨ + ⎬
p⎣ ⎩ n +1 n −1 ⎭
⎧ 0, if m is even
⎪ p
=⎨ −2 (5) ⎧ sin( n + 1) x sin( n − 1) x ⎫⎤
⎪ p (2k − 1)2 , if m is odd, m = 2 k − 1 + 1⎨ 2
− ⎬⎥ , (n ≠ 1)
⎩ ⎩ ( n + 1) ( n − 1)2 ⎭⎦⎥ 0
Fourier Series    5-33

− ( −1)n +1 ( −1)n −1 ⎛ −1 1 ⎞

( −1)n −1 sin nx
= + = ( −1) n −1⎜ + ⎟ Ans: f ( x ) = x = p ∑
n +1 n −1 ⎝ n + 1 n − 1⎠ n =1 n

⎧sin( n + 1) x = sin( n − 1) x = 0 ⎫
⎪ ⎪ ⎡ 2x l
⎨ at x = p , 0 ⎬ ⎢ l , 0≤ x≤
2
⎪ ⎪ 3. f ( x ) = ⎢
⎩ t1 = 0 at x = 0 ⎭ ⎢ 2(l − x ) , l ≤ x ≤ l
⎢⎣ l 2
2( −1)n −1 np
= ( n ≠ 1);
( n2 − 1) 8 ∞ sin 2 npx
Ans: f ( x ) = 2 ∑ sin
−2 2 −2 2 p n =1 n2 l
a2 = , a3 = , a4 = , a5 = .
1.3 2.4 3.5 4.6 ⎡ ⎛ p⎞
2 p 1 p ⎢ sin x, ⎜⎝ 0, ⎟⎠
a1 = ∫ x sin x cos x dx = ∫ x sin 2 x dx 4. f ( x ) = ⎢
4
[JNTU 1999]
p 0 p 0 ⎢ ⎛p p⎞
p ⎢cos x, ⎜⎝ , ⎟⎠
1 ⎡ ⎛ cos 2 x ⎞ ⎛ sin 2 x ⎞ ⎤ 1 ⎣ 4 2
= ⎢ x⎜− ⎟ − 1⋅ ⎜ − 2 ⎟ ⎥ = − (4)
p⎣ ⎝ 2 ⎠ ⎝ 2 ⎠ ⎦0 2
4 2 ∞ ( −1)n +1 sin(4n + 2) x
⎧sin 2 x = 0 at x = p , 0⎫
Ans: f ( x ) = ∑
p n = 0 (4n + 1)(4n + 3)
=⎨ ⎬
⎩ x(⋅) = 0 at x = 0 ⎭
⎡1 1
⎢ 4 − x, 0 < x < 2
Substituting these values in (2) we obtain 5. f ( x ) = ⎢ [JNTU 2003 (1)]
⎢x − 3, 1 < x <1
⎢⎣ 4 2
1 2 2
x sin x = 1 − cos x − cos 2 x + cos 3x 1 ⎡ 3p + 4
2 1.3 2.4 Ans: f ( x ) = (p − 4)sin px + sin3px
p 2 ⎢⎣ 32
2
− cos 4 x +! (5) 5p − 4 ⎤
3.5 + sin5px − ! ⎥
52 ⎦

   (II) Obtain half-range FCS for 6 to 10


x = p / 2 is a point of continuity for f (x) = x sin x and
1 1 1 p2
the series converges to f (p / 2) = p / 2 when x = p / 2 6. f (x) = x in 0 ≤ x ≤ p. Deduce + + + ! = .
12 32 52 8
[JNTU 2003 S]
p 1 2 2 2
= 1 + (0) − ( −1) + (0)+ (1) + ! Ans: f ( x ) = x =
p 4 ∞ cos(2 x − 1) x
− ∑ .
2 2 1.3 2.4 3.5 4 p n =1 (2n − 1)2
2 2 2 2
= 1+ − + − +! (6)
px
1.3 3.5 5.7 7.9 7. f ( x ) = sin , (0 < x < 1). [JNTU 2004 S(4)]
l
⎛ 2px 4px ⎞
  
  px 2 4 ⎜
cos cos

Ans: f ( x ) = sin = − ⎜ l + l +!
(I) Obain half-range FSS for 1 to 5 l p p 1.3 3.5 ⎟
⎜⎝ ⎟⎠
1. f (x) = x in (0, 2).
n −1 npx 8. f (x) = x in (0, 2).
4 ∞ ( −1) sin
2
Ans: f ( x ) = x = ∑ ⎛ 2n − 1 ⎞
p n =1 n cos ⎜ px⎟
8 ∞ ⎝ 2 ⎠
2. f (x) = x in (0, p). Ans: f ( x ) = x = 1 − 2 ∑
p n =1 (2n − 1)2
5-34    Engineering Mathematics-II

⎧ 1  Let the Fourier series for f (x) in (−l, l) be


⎪⎪ kx, 0≤ x≤
2 ∞
9. f ( x ) = ⎨ 1 npx ∞ npx
l
⎪k (l − x ), f ( x) = a0 + ∑ an cos + ∑ bn sin (5.41)
≤x≤l 2 l l
⎪⎩ 2 n =1 n =1

1 1 1 1 Multiplying both sides of (5.41) by f (x) and inte-


Deduce the value of 2 + 2 + 2 + 2 + !
1 3 5 7 grating term by term from −l to l we obtain
[JNTU 2005 S(3)]
x ∞
cos2np l a0 l l npx
kl 2kl ∞
Ans: f ( x ) = − 2 ∑ l ∫− l [ f ( x )]2 dx =
2 ∫ − l
f ( x ) dx + ∑ an ∫ f ( x ) cos
− l l
dx
4 p n odd n2 n =1

l npx
⎧ 3 + ∑ bn ∫ f ( x )sin dx (5.42)
−l
⎪⎪ 1, 0< x< n =1 l
2
5.3 f ( x ) = ⎨
⎪1 − x, 3
< x<3 Also ( a0 , an , bn )
⎪⎩ 2
1 3 ⎡ 2px 1 l ⎛ np x np x ⎞
l ∫− l
px = f ( x ) ⎜1, cos , sin
Ans: f ( x ) = − + 2 ⎢(p + 2)cos − cos ⎝ ⎟ dx
8 p ⎣ 3 2 l l ⎠
2 − 3p 3px ⎤ l ⎛ np x np x ⎞
+
9
cos
3
−! ⎥⎦ . ⇒ ∫−l f ( x) ⎜⎝1, cos l
, sin
l ⎠
⎟ dx = (la0 , lan , lbn )

(5.43)
Substituting (5.42) we have
      


 
l ⎡1 2 ∞ 2 2 ⎤
⎢ a0 + ∑ ( an + bn )⎥
2
In many engineering applications, one has to deal ∫− l [ f ( x )] dx = l (5.44)
with the concept of root mean square (RMS) value ⎣2 n =1 ⎦
or effective value of a function. which is known as Parseval’s formula.
The RMS value of a function y = f (x) over a
given interval (a, b) is defined by !
"
1/2
⎛ 1 b ⎞
y 2 dx ⎟ 1. If f (x) is an even function in (−l, l), (5.43)
⎝ b − a ∫a
y=⎜ (5.38)
⎠ becomes

⎛1 l ⎞
1/2
2 l 1 2 ∞ 2
2
a0 + ∑ an ,
l ∫0
If the interval is ( −l , l ) then y = ⎜ ∫ [ f ( x )]2 dx ⎟ [ f ( x )] dx = bn = 0
⎝ 2l − l ⎠ 2 n =1
1 l (5.45)
y2 = [ f ( x )]2 dx
2l ∫− t (5.39)
2. If f (x) is an odd function in (−l, l), (5.43)
becomes
  #$ 
" ∞
2 l
 5.1 ∫ [ f ( x )]2 dx = ∑ bn2 , a0 = an = 0
l 0 n =1
If the Fourier series for f (x) converges uniformly in (5.46)
(−l, l) then
3. If the half-range cosine series for f (x) in
l ⎡1 2 ∞ 2 2 ⎤ 1 ∞
npx
⎢ a0 + ∑ ( an + bn )⎥ (0, l) is f ( x ) = a0 + ∑ an cos
2
∫− l [ f ( x )] dx = l (5.40) then
⎣2 n =1 ⎦ 2 n =1 l
Fourier Series    5-35

l 1 ⎡1 2 ⎤ By RMS value in series


∫0 [ f ( x )]2 dx = l ⎢ a0 + a12 + a22 + a32 + ! ⎥
2 ⎣2 ⎦
p
⎡ 1 ⎛ 2p 2 ⎞ 2 1 ⎛ 4( −1)n ⎞ 2 ⎤
∫− p [ f ( x ) ] ⎢ + ∑⎜ ⎥
(5.47) 2
dx = 2p ⎜
⎢ 4 ⎝ 3 ⎟⎠ 2 ⎝ n2 ⎟⎠ ⎥
4. If the half-range sine series for f (x) in (0, l) is ⎣ ⎦

np x 4x4 p4
f ( x ) = ∑ bn sin then ⇒
1 1 1 1
= 8∑ 4 ⇒ 4 + 4 + 4 + ! = (5)
n =1 l 45 n 1 2 3 90
l 1 2
∫0 [ f ( x)] dx = l[b1 + b22 + b32 + ! ]
2
 

2
(5.48) Find (a) the half-range cosine series and (b) sine
series for f (x) = x in 0 < x < l.
 
 1 1 1
Also, deduce the value of + + !
Find the Fourier expansion of f (x) = x2 in −p < x < p. 14 4
3 54
Using the series find the RMS value of f (x) in the [JNTU 2001].
1 1 1 p4
interval. Deduce 4 + 4 + 4 + ! = .   
1 2 3 90 (a) The half-range cosine series is given by

   Since f (x) = x2 is even in (−p, p) the 1 npx
f ( x) = x = a0 + ∑ an cos (1)
Fourier series is given by 2 n= 0 l
a0 ∞ where
f ( x) = x2 = + ∑ an cos nx, bn = 0 (1) l
2 n =1 2 l 2 l 2 ⎛ x2 ⎞
a0 = ∫
l 0
f ( x ) dx = ∫ xdx = ⎜ ⎟ = l
l 0 l ⎝ 2 ⎠0
(2)
where
p
2 ⎛ x3 ⎞ 2 l np x 2 l np x
a0 =
2 p 2

2
x dx = ⎜ ⎟ = ⋅ p 2 (2) an = ∫
l 0
f ( x ) cos
l
dx = ∫ x cos
l 0 l
dx
p 0 p ⎝ 3 ⎠0 3
l
2 p 2⎡ ⎛ 1 np x ⎞ ⎛ l2 np x ⎞ ⎤
an = ∫ x 2 cos nx dx = ⎢x ⎜ sin ⎟ − 1⋅ ⎜ − 2 2 cos ⎥
p 0 l ⎢⎣ ⎝ np l ⎠ ⎝ np l ⎟⎠ ⎥⎦
0
p
2 ⎡ 2 ⎛ sin nx ⎞ ⎛ cos nx ⎞ ⎛ sin nx ⎞ ⎤ 2 ⎡ l2 ⎤ 2l
= ⎢ x ⎜ ⎟⎠ − 2 x ⎜⎝ − 2 ⎟⎠ + 2 ⎜⎝ − 2 ⎟⎠ ⎥ = ⎢
n
⎥ (cos np − cos 0) = 2 2 (( −1) − 1)
p⎣ ⎝ n n n ⎦0 l ⎣ n2p 2 ⎦ np
2 cos nx ( −1)n ⎡0 if n is even
= ⋅ 2p = 4 (3)
= ⎢ −41
p n2 n2 (3)
⎢ if n is odd
t1 = t3 = 0 at x = p , 0 t2 = 0 at x = 0 ⎣⎢ n2p 2
np x
RMS value of cos
l 4l
∴ x= − 2
2 p
∑ n2
l (4)
n odd
1/2
⎡ 1 p ⎤ 1 p 4
∫−p [ f ( x)] dx ⎥ 2p ∫−p
2
f ( x) = ⎢ = ⋅ x dx (4) (b) The half-range sine series is given by
⎣ 2p ⎦

np x
∑ bn sin
p
1 ⎛ x5 ⎞ p2 f ( x) = x = (1)
= 2⋅ ⎜ ⎟ = n= 0 l
2p ⎝ 5 ⎠ 0 5 where
5-36    Engineering Mathematics-II

2 l npx 2 l np x 3. Using the Fourier cosine series for f (x) = x(p − x) in


l ∫0
bn = f ( x )sin dx = ∫ x sin dx (2) 0 < x < p show that
l l 0 l
⎡ ⎤
l 1 1 1 p4
+ + + ! =
⎢ 2 ⎥ 14 24 34 90
2 ⎛ l np x ⎞ ⎛ l ⎞⎛ np x ⎞ ⎥
= ⎢ x ⎜ − cos ⎟ − 1⋅ ⎜ 2 2 ⎟ ⎜ − sin ⎟ np x
l ⎢ "⎝# #
nx# $ # l# ⎠# % ⎝ n p ⎠ ⎝ l ⎠⎥ 4. If f ( x ) = ∑ bn sin in 0 < x < l where
⎢ = 0 at x =0
" # # # $ # # #% ⎥ l
⎣ = 0 at x = l ,0 ⎦0 2 l np x
l ∫0
bn = f ( x )sin then show that
2 ⎛ l ⎞ 2l l
= l ⎜ − ⎟ cos np = − ( −1)n
l ⎝ np ⎠ np
l 1 ∞ 2
∞ n −1 ∫0 ( f ( x))
2
dx = l ∑ bn .
2l ( −1) np x 2 n =1
∴ x= ∑
p n =1 n
sin
l
F (3)
 
 
p 1 1 1 ∞
( −1) n −1
 ! " 1. = 1− + − + = ∑ (Gregory series)
4 3 5 7 n =1 2n− 1

⎡ l2 1 p2 1 1 1 ∞
1
l l 16l 2 ⎤ 2. = 1+ 2 + 2 + 2 +! = ∑
(1) ∫0 [ f ( x )] dx = ∫0 x dx = l ⎢ +
2 2
∑ n4 p 4 ⎥ 8 3 5 7 n =1 (2 n − 1)2
⎣⎢ 4 2 n=1,3,5 ⎦⎥ p-2 1 1 1 1
⎡1 1 3. = − + − +!
l3 16 ⎤ p4 1 4 1.3 3.5 5.7 7.9
= l3 ⎢ + ∑ 4 4 ⎥ ⇒ =8∑ 4
3 4 2 n =1,3,5 n p ⎦ 12 ∞
( −1)n −1 ∞
( −1)n −1
⎣⎢ ⎥ n odd n =∑ 2
=∑
(4) n =1 (4 n − 1) n =1 (2n − 1)(2n + 1)

1 1 1 p 4 p2 1 1 1 1 ∞
1
+ + +! = 4. = 2 + 2 + 2 + 2 +! = ∑ 2
6 1 2 3 4
14 34 54 96 n =1 n
p2 1 1 1 1 1 1 ∞
( −1)n −1
2 2 ∞ 5. = 2 − 2 + 2 − 2 + 2 − 2 +! = ∑
l ⎡1 4l ⎤ p 1 12 1 2 3 4 5 6 n2
(2) ∫0 x
2
dx = l ⎢ ∑ 2 2 ⎥ ⇒ =∑ 2. n =1

⎣2 n p ⎦ 6 n =1 n
1 1 1 1 1 ∞
1
(5) 6. = + + + + = ∑
2 1.3 3.5 5.7 7.9 n =1 (2 n − 1)(2 n + 1)
1 ⎡⎛ 1 1 ⎞ ⎛ 1 1 ⎞ ⎛ 1 1 ⎞


 
  Sn = ⎜ − ⎟ + ⎜ − ⎟ + ⎜ − ⎟ +
2 ⎢⎣⎝ 1 3 ⎠ ⎝ 3 5 ⎠ ⎝ 5 7 ⎠

1 np x ⎛ 1 1 ⎞⎤
1. If f ( x ) = a0 + ∑ an cos (0 < x < 1) show that +⎜ −
2 n =1 l ⎝ 2n − 1 2n + 1⎟⎠ ⎥⎦
l 1 ⎛1 2 ⎞ 1⎛ 1 ⎞ 1
2
l ⎜ a0 + a12 + a22 + a32 + ! ⎟ = ⎜1 − ⎟ → as n → ∞
∫0 ( f ( x)) dx =
2 ⎝2 ⎠ 2 ⎝ 2n + 1⎠ 2

1 np x npx (Except the first and the last terms all other terms get
2. If a0 + ∑ an cos
f ( x) = + ∑ bn sin in cancelled)
2 l l
(0 < x < 2l) prove that From (3) and (6) we obtain
2l ⎡1 1 ⎤ 1 1 1 ∞
1
∫0 ( f ( x ))2 dx = 2l ⎢ a02 + ∑ ( an2 + bn2 )⎥ 7.
p
= + + +! = ∑ .
⎣4 2 ⎦ 8 1.3 5.7 9.11 n =1 (4 n − 3)(4 n − 1)
Partial Differential
Equations 6
6.1 INTRODUCTION 6.2 ORDER, LINEARITY
The reader is familiar with ordinary differential AND HOMOGENEITY OF A PARTIAL
equations. These equations involve functions of a DIFFERENTIAL EQUATION
single variable only and their derivatives. In many 6.2.1 Order
problems that arise in geometry, physics, population The order of a partial differential equation is the
dynamics, social sciences, medicine and engineering, order of the highest derivative appearing in it.
one has to deal with equations containing functions
of more than one variable and partial derivatives with 6.2.2 Linearity
respect to these independent variables. Such equations As in the case of an ordinary differential equation,
are called partial differential equations. Thus, a partial we say that a partial differential equation is linear if
differential equation is an equation of the form it is of the first degree in the dependent variable (the
unknown function) and its partial derivatives and are
⎛ ∂z ∂z ∂z ∂2 z ∂2 z ⎞ not multiplied together.
f⎜ , , , ! , , ! , , ! , z , x, y, t⎟ = 0
⎝ ∂t ∂x ∂y ∂x 2 ∂t ∂x ⎠ 6.2.3 Homogeneity
(6.1) A linear partial differential equation is called homo-
geneous if it contains no term free from the unknown
containing independent variables t, x, y, . . . an unknown function and its derivatives; otherwise, it is called a
function z = z(x, y, . . . , t) and partial derivatives nonhomogeneous equation.
The following examples of partial differential
∂z ∂z ∂z ∂2 z ∂2 z
, , , ! , , ! , , ! , with respect equations with their order and nature noted against
∂t ∂x ∂y ∂x 2 ∂t ∂x each of them are meant to illustrate the points
to these variables t, x, y, … explained above.

Linear/nonlinear,
Partial differential equation Order homogeneous/nonhomogeneous
∂z ∂z
1. + =z One Linear, homogeneous
∂x ∂y

∂z ∂z
2. x +y = 2z One Linear, homogeneous
∂x ∂y

∂2 z ∂2 z
3. + = 0 (Laplace’s Equation) Two Linear, homogeneous
∂x 2 ∂y 2

⎛ ∂z ⎞ ⎛ ∂z ⎞
4. ⎜ ⎟ ⎜ ⎟ = 4 xy One Nonlinear, nonhomogeneous
⎝ ∂x ⎠ ⎝ ∂y ⎠

(Continued )
6-2    Engineering Mathematics-II

Linear/nonlinear,
Partial differential equation Order homogeneous/nonhomogeneous

5. ∂z = c 2 ∂ z
2
(Heat conduction equation) Two Linear, homogeneous
∂t ∂x 2

6. ∂ z = c 2 ∂ z
2 2
(Wave equation) Two Linear, homogeneous
∂t 2 ∂x 2
2
⎛ ∂2 z ⎞
7. ⎜ 2 ⎟ = xy Two Nonlinear, nonhomogeneous
⎝ ∂x ⎠
4 3
⎛ ∂2 z ⎞ ⎛ ∂z ⎞
8. ⎜ ⎟ =⎜ ⎟ Two Nonlinear, homogeneous
⎝ ∂x ∂y ⎠ ⎝ ∂x ⎠

6.3 ORIGIN OF PARTIAL DIFFERENTIAL respectively. Eliminating the constants c and a


EQUATION between these equations we again obtain
1. Consider the equation equation (6.5).
3. The spheres and cones are surfaces of revolution
x2 + y2 + (z − c)2 = a2 (6.2) which have the line OZ as their axis of symmetry.
where c and a are arbitrary constants. It repre- Now, consider the equation
sents the set of all spheres with their centres on the
z = f (x2 + y2) (6.9)
z-axis.
Differentiating with respect to x and y we get where f is an arbitrary function. Equation (6.9)
x + p( z − c) = 0, y + q( z − c) = 0; (6.3), (6.4) characterizes surfaces of revolution having the z-axis
as the axis of symmetry.
⎛ ∂z ∂z ⎞ Differentiating equation (6.9) partially with
⎜⎝ p = ∂x , q = ∂y ⎠⎟ respect to x and y we get
Eliminating the arbitrary constant c from (6.3) p = 2xf¢(u), q = 2yf¢(u) (6.10), (6.11)
and (6.4) we obtain the first-order linear partial
differential equation where u = x 2 + y 2 and f ′(u ) = df du. Eliminating f
yp − xq = 0 (6.5) between equations (6.10) and (6.11), we again
obtain the first-order linear partial differential
which characterizes the set of all spheres with equation (6.5).
centres on the z-axis.
Formation of PDE Ordinary differential
2. Consider again the equation
equations are formed by eliminating arbitrary
x2 + y2 = (z − c)2 tan2 a (6.6) constants only, whereas partial differential equations
are formed by eliminating (a) arbitrary constants or
where c and a are arbitrary constants. Equation (6.6)
(b) arbitrary functions.
represents the set of all right circular cones whose
We know that the order of an ordinary differ-
axes coincide with the z-axis.
ential equation is equal to the number of arbitrary
Differentiating (6.6) partially with respect to x
constants to be eliminated from a relation.
and y we get
In the case of partial differential equations, if
p(z − c) tan2 a = x, q(z − c) tan2 a = y the number of arbitrary constants to be eliminated
(6.7), (6.8) is equal to the number of independent variables
Partial Differential Equations    6-3

involved in a relation, one obtains a first-order partial the partial differential equation
differential equation, and if the number of arbitrary z2(p2 + q2 +1) = 1 (2)
constants to be eliminated is more than the number
of independent variables, then one obtains a higher- is obtained, which is nonlinear.
order partial differential equation. Solution Differentiating (1) partially with respect
Further, if one arbitrary function is to be to x and y
eliminated from a relation, then a first-order partial
differential equation is obtained; and if two arbitrary (x − a) + zp = 0, (y − b) + zq = 0 (3), (4)
functions are to be eliminated, then a second-order Eliminating a and b from (1), (3) and (4) we get
partial differential equation is obtained and so on.
z2(p2 + q2 +1) = 1
A first-order partial differential equation is of
the form which is nonlinear.
f (x, y, z, p, q) = 0 (6.12) Examples
Form the partial differential equation by elimi-
where x, y are independent variables, z is a depen-
nating the arbitrary constants a and b from each of
dent variable and p = ∂z ∂x and q = ∂z ∂y .
the relations given below:
6.4 FORMATION OF PARTIAL Example 6.2
DIFFERENTIAL EQUATION BY z = ax + by
ELIMINATION OF TWO ARBITRARY Solution We have
CONSTANTS
Consider a relation of the type z = ax + by (1)
Differentiating (1) partially with respect to x
F (x, y, z, a, b) = 0 (6.13)
and y we have
where a and b are arbitrary constants.
∂z ∂z
Differentiating (6.13) partially with respect to = p = a, =q=b (2), (3)
x and y we have ∂x ∂y
Eliminating a and b from equations (1)–(3) we
∂F ∂F ∂F ∂F
+p = 0 and +q =0 obtain the first-order partial differential equation
∂x ∂z ∂y ∂z
z = px + qy (4)
(6.14), (6.15)
Example 6.3
We can now eliminate the two arbitrary
constants a and b between the equations (6.13)–(6.15) z = (x2 + a2)(y2 + b2)
and obtain a first order partial differential equation Solution We have
of the form
z = (x2 + a2)(y2 + b2) (1)
f (x, y, z, p, q) = 0 (6.16)
Differentiating (1) partially with respect to
We now work out a few examples to show how x and y
partial differential equations are formed by elimi-
nating arbitrary constants. ∂z p
= p = 2 x( y 2 + b2 ) ⇒ y 2 + b2 = (2)
∂x 2x
Example 6.1
Show by eliminating the arbitrary constants a and ∂z q
= q = 2 y( x 2 + a2 ) ⇒ x 2 + a2 = (3)
b from ∂y 2y
(x − a)2 + (y − b)2 + z2 = 1 (1) Eliminating a and b from (1)–(3) we get
6-4    Engineering Mathematics-II

q p Example 6.6
z= ⋅ or pq = 4 xyz (4)
2 y 2x ax2 + by2 + z2 = 1
Example 6.4 Solution We have
x2 y2
z= + ax2 + by2 + z2 = 1 (1)
a2 b2
Solution We have Differentiating (1) partially with respect to
x and y
x 2
y 2
z= + (1)
a2 b2 zp + ax = 0, zq + by = 0 (2), (3)
Differentiating (1) partially with respect to Multiplying (2) by x and (3) by y and adding
x and y
∂z 2x ∂z 2y z ( px + qy ) + ( ax 2 + by 2 ) = 0
= p= 2, =q= 2 (2), (3)
∂x a ∂y b ⇒ z ( px + qy ) = −( ax 2 + by 2 ) = −(1 − z 2 ), by (1)
Eliminating a and b from (1)–(3) we get ⇒ z ( px + qy ) = z 2 − 1 (4)
x y
z= p+ q ⇒ px + qy = 2 z (4)
2 2 EXERCISE 6.1
Example 6.5 Form the partial differential equation in each of the
z = (x + a)(y + b) following cases, by eliminating the arbitrary constants
a and b (Questions 1–10):
Solution We have
1. z = ax2 + by2.
z = (x + a)(y + b) (1)
Ans: 2z = px + qy
Differentiating (1) partially with respect to x
and y 2. (x − a)2 + (y − b)2 = z2 cot2 a .
Ans: p2 + q2 = tan2 a
∂z ∂z
= p = y + b, = q= x+a (2), (3)
∂x ∂y 3. z = ax + by + a2 + b2.
Eliminating a and b from (1)–(3) we have Ans: z = px + qy + p2 + q2

z = pq (4) 4. z = axy + b.

Example 6.6 Ans: px = qy


2z = (ax + y) + b 2

Solution We have
5. z =
1
2
(
x + a + y − a +b . ) [JNTU 2003]

Ans: 16(p2 + q2)(x + y) = 1


2z = (ax + y)2 + b (1)
6. u = a(x + y) + b(x − y) + abz + c.
Differentiating (1) partially with respect to
x and y Ans: ux2 − u 2y = 4uz

2p = 2a(ax + y), 2q = 2(ax + y) (2), (3) 7. z = xy + y x 2 + a 2 + b 2 .


Eliminating a and b from (1)–(3) we get Ans: px + qy = pq
px + qy = q2 (4) y 2 2y
8. z = axe + 12 a e + b .
which is nonlinear.
Ans: q = px + p2
Partial Differential Equations    6-5

⎛ a⎞ ∂z ∂z ∂2 z
9. z = ax + by + ⎜ ⎟ − b. [JNTU 2003] = p = f ′g , = q = fg ′, = s = f ′g ′
⎝ b⎠ ∂x ∂y ∂x ∂y
⎛ p⎞ pq = f ′g ⋅ fg ′ = f ′g ′ ⋅ fg = sz or pq = sz
Ans: z = px + qy + ⎜ ⎟ − q
⎝ q⎠ where
⎡ b( y − 1) ⎤ ∂2 z
10. z = a log ⎢ ⎥. [JNTU 2003] s= (6.21)
⎣ (1 − x ) ⎦ ∂x ∂y
Ans: px + qy = p + q
11. Form the partial differential equation of all spheres of (c) Formation of PDE by elimination of arbitrary
radius a with their centres on the x–y plane. function of the form F (u, v) = 0
Ans: z2(p2 + q2 + 1) = a2 Consider a relation between x, y and z of the type
12. Form the partial differential equation of all planes F (u, v) = 0 (6.22)
through the origin. where u and v are known functions of x, y and z;
Ans: z = px + qy and F is an arbitrary function of u and v. Also, z is a
function of x and y.
Differentiating (6.22) by chain rule, with respect
6.5 FORMATION OF PARTIAL
to x and y we obtain
DIFFERENTIAL EQUATIONS
BY ELIMINATION OF ARBITRARY ∂F ⎛ ∂u ∂u ⎞ ∂F ⎛ ∂v ∂v ⎞
⎜ + p⎟ + ⎜ + p⎟ = 0 (6.23)
FUNCTIONS ∂u ⎝ ∂x ∂z ⎠ ∂v ⎝ ∂x ∂z ⎠
(a) Elimination of one arbitrary function of the ∂F ⎛ ∂u ∂u ⎞ ∂F ⎛ ∂v ∂v ⎞
+ q + + q =0 (6.24)
form z = f (u) where u = u(x, y, z) ∂u ⎝⎜ ∂y ∂z ⎠⎟ ∂v ⎜⎝ ∂y ∂z ⎟⎠
Let
Eliminating F between (6.23) and (6.24) we get
z = f (u) (6.17)
where f (u) is an arbitrary function of u where ux + uz p v x + v z p
=0 (6.25)
u = u(x, y, z) a known function of x, y and z. u y + uz q v y + uz q
Differentiating (6.17) partially with respect to
x and y where ux = ∂u ∂x , u y = ∂u ∂y and so on.
Expanding the determinant in (6.25), collecting
⎛ ∂u ∂u ⎞ ⎛ ∂u ∂u ⎞ the terms and simplifying we have
p = f ′⋅⎜ + p , q = f ′⋅⎜ + q
⎝ ∂x ∂z ⎟⎠ ⎝ ∂y ∂z ⎠⎟ Pp + Qq = R (6.26)
(6.18), (6.19) ∂( u , v ) ∂( u , v ) ∂( u , v )
where P= , Q= , R=
where ′ on f denotes differentiation with respect ∂( y , z ) ∂( z , x ) ∂( x , y )
to the argument u. Eliminating f from equations which are functions of x, y and z and do not contain
(6.18)–(6.19) we obtain a first-order partial differ- p and q. This equation is called Lagrange’s1 linear
ential equation. equation. In fact, it is a quasi-linear equation since
(b) Elimination of two arbitrary functions of the the dependent variable may be present in P, Q and R.
form z = f (x) · g ( y) If P and Q are independent of z and R is linear
in z the equation (6.26) is called linear.
Let
z = f (x) · g ( y) (6.20)
1
LAGRANGE, Joseph-Louis (1736–1813) great French analyst,
where f is a function of x and g is a function of y alone. algebraist, number theorist, probabilist, physicist, and astrono-
Differentiating (6.20) partially with respect to x mer, contributed especially to the calculas of variations, analytic
and y, respectively, we have mechanics and astronomer.
6-6    Engineering Mathematics-II

Equation (6.26) is of the type given by equation Eliminating f between (1)–(3) we have
(6.12), which need not be linear, as Example 6.1
bp – aq =2abz (4)
above will illustrate.
Example 6.11
Examples
⎛ y⎞
Form the partial differential equation by elimi- z = xn f ⎜ ⎟
⎝ x⎠
nating the arbitrary function f from each of the
following relations [Examples 6.8–6.15 except Solution We have
(Example 6.13) in which functions f and g have to ⎛ y⎞
z = xn f ⎜ ⎟ (1)
be eliminated.] ⎝ x⎠
Example 6.8 Differentiating (1) partially with respect to
z = x f (x + y) x and y
Solution We have ⎛ −y⎞ 1
p = nx n −1 f + x n f ′ ⋅ ⎜ 2 ⎟ , q = x n f ′ ⋅ (2), (3)
⎝x ⎠ x
z = xf (x + y) (1)
We have from (2) and (3)
Differentiating (1) partially with respect to x and y
p = 1f + xf ¢ , q = 0 + xf ¢ (2), (3) px + qy = nxn f − yxn−1 f  + yxn − 1 f  = nz,
⇒ px + qy = nz, by (1) (4)
Eliminating f between (1)–(3) we have
z Example 6.12
p = + q ⇒ z = ( p − q) x (4) ⎛ z ⎞.
x yz + zx + xy = f ⎜ [JNTU 2005 S]
which is the required partial differential equation. ⎝ x + y ⎠⎟

Example 6.9 Solution We have


z = x − y + f (xy) ⎛ z ⎞
yz + zx + xy = f ⎜ (1)
Solution We have ⎝ x + y ⎠⎟

z = x − y + f (xy) (1) Differentiating (1) with respect to x and y


( x + y) p − z
Differentiating (1) partially with respect to y + z + ( x + y) p = f ′ ⋅ ,
x and y ( x + y )2
p = 1+ f ¢ · y, q = −1+ f ¢ · x ( x + y )q − z
(2), (3) z + x + ( x + y )q = f ′ ⋅ (2), (3)
( x + y )2
Eliminating f ¢ between (1) and (2) we get the
partial differential equation Eliminating f ¢ between (2) and (3) we have
px − qy = x + y (4) y + z + ( x + y) p ( x + y) p − z y + 2 z ⎛ diff. of nrs.⎞
= = =
Example 6.10 x + z + ( x + y) q ( x + y) q − z x + 2 z ⎜⎝ diff. of drs. ⎟⎠
z = eax−by f (ax + by) ( x + y )[ p ( x + 2 z ) − q ( y + 2 z )] = z [ x + 2 z − y − 2 z ]
Solution We have px − qy + 2z ( p − q ) = z ( x − y ) ( x + y ) (4)

z = eax−by f (ax + by) (1) Example 6.13

Differentiating (1) partially with respect to x and y z = f (x + at) + g(x−at). [Osmania 1999]
p = aeax−by f + eax−by f ¢ · a, Solution We have
ax−by ax−by
q = −be f+e f¢ · b (2), (3) z = f (x + at) + g(x − at) (1)
Partial Differential Equations    6-7

Differentiating (1) partially with respect to ∂( u , v ) 2 z y


x and t twice = = 2 z − y;
∂( z , x ) 1 1
∂z ∂z ∂( u , v ) y x
= f ′ + g ′, = af ′ − ag ′ , (2) = = y−x (3)
∂x ∂t ∂( x , y ) 1 1
∂2 z ∂2 z
= f ′′ + g ′′ , = a2 f ′′ + a2 g ′′ (3) Hence, the required partial differential equation is
∂x 2 ∂t 2
(x − 2z)p − (y − 2z)q = y − x (4)
Eliminating the arbitrary functions from (3)
we have
∂2 z ∂2 z EXERCISE 6.2
2
= a2 2 (4) Form the partial differential equations by eliminating the
∂t ∂x
arbitrary functions from the following relations:
which is a partial differential equation of the second
⎛ xy ⎞
order. 1. z = f ⎜ ⎟ . [Andhra 1991]
⎝ z⎠
Example 6.14
Ans: px = qy
2 ⎛1 ⎞
z = y + 2 f ⎜ + log y ⎟ [Madras 2000] 2. xyz = f (x + y + z). [Osmania 1995, Calicut 1994]
⎝x ⎠
Solution We have Ans: x(y − z)p + y(z − x)q = z(x − y)

⎛1 ⎞ 3. x + y + z = f (x2 + y2 + z2). [Madras 2000, Ranchi 1990]


z = y 2 + 2 f ⎜ + log y ⎟ (1)
⎝x ⎠ Ans: (y − z)p + (z − x)q = x − y
Differentiating (1) with respect to x and y 4. f (x2+ y2, z − xy) = 0. [Madurai 1990]
Ans: py − qx = y − x
2 2
⎛ 1⎞ 1
p = 0 + 2 f ′ ⋅⎜ − 2 ⎟ , q = 2y + 2 f ′ ⋅ (2), (3)
⎝ x ⎠ y 5. z = f (x2 − y2). [Madras 1991]
Ans: py + qx = 0
Eliminating f ¢ between (2) and (3) we get
6. z = yf (x) + xg (y). [Madras 1993, Karnataka 1993,
px2 + qy = 2y2 (4)
Madurai 1998]
2
which is the required partial differential equation. ∂ z ∂z ∂z
Ans: xy = x +y −z
Example 6.15 ∂ x∂ y ∂x ∂y

f (xy + z2, x + y + z) = 0 7. z = f (y) + f (x + y). [JNTU 2005 S (1)]


[Madras 1995, Kerala 1990 S] 2 2
Ans: ∂ z = ∂ z
Solution We have ∂ x∂ y ∂ x 2
f (u,v) = 0 (1) 8. z = xy + f (x2 + y2). [JNTU 2005 S (2)]
where Ans: py − qx = y2 − x2
u = xy + z2, v=x+y+z (2)
We know that the partial differential equation is 6.6 CLASSIFICATION OF FIRST-ORDER
∂( u , v ) ∂( u , v ) ∂( u , v ) PARTIAL DIFFERENTIAL EQUATIONS
p+ q=
∂( y , z ) ∂( z , x ) ∂( x , y ) 6.6.1. Linear Equation
where Let z = z (x, y) be a function of two variables x and y.
∂( u , v ) x 2 z A first-order partial differential equation is said to be
= = x − 2 z; linear if it is linear in p, q and z, i.e. if it is of the form
∂( y , z ) 1 1
6-8    Engineering Mathematics-II

P(x, y)p + Q(x, y)q = R(x, y)z + S(x, y) Compute p = ∂z ∂x and q = ∂z ∂y from
(6.34) and substitute in (6.33). If this reduces (6.33)
where
into an identity in x and y then (6.34) defines a solu-
∂z ∂z
p= and q = (6.27) tion of partial differential equation (4.33).
∂x ∂y Equation (6.34) defines a surface in three-
E.g. yp − xp = xyz + x⎫⎪ dimensional space. It is called an integral surface of
2 ⎬ (6.28) partial differential equation (6.33).
y p − xyq = x( z − 2 y )⎪⎭ There are different types of solutions of
6.6.2. Semi-Linear Equation equation (6.33).
A first-order partial differential equation is said
to be semi-linear if it is linear in p and q and the 6.7.1. Complete Integral
coefficients of p and q are functions of x and y alone, A two-parameter family of solutions
i.e. if it is of the form
z = F (x, y, z, a, b) (6.35)
P(x, y)p + Q(x, y)q = R(x, y, z) (6.29)
is called a complete integral (or complete solution)
E.g. 1. e x p − yxq = xz 2 ⎫⎪ of (6.33) if in the region D the matrix
⎬ (6.30)
2. x p + y q = z ⎭⎪
⎡ ∂F ∂2 F ∂2 F ⎤
6.6.3. Quasi-Linear Equation ⎢ ⎥
∂a ∂a∂x ∂a∂y ⎥
M=⎢ (6.36)
A first-order partial differential equation is said to ⎢ ∂F ∂2 F ∂2 F ⎥
be quasi-linear if it is linear in p and q, i.e. if it is of ⎢ ⎥
the form ⎣⎢ ∂b ∂b∂x ∂b∂y ⎦⎥
is of rank two.
P(x, y, z)p + Q(x, y, z)q = R(x, y, z) (6.31)

E.g. 1. ( x 2 − yz ) p + ( y 2 − zx )q = z 2 − xy ⎪⎫ 6.7.2. General Integral


⎬ (6.32) Let a and b be related and suppose
2. ( x 2 + y 2 ) p − xyq = z 3 x + y 2 ⎪⎭
b = f (a) (6.37)
Note that every semi-linear partial differential
equation is quasi-linear. Substituting this in (6.35) we get

6.6.4. Nonlinear Equation z = F (x, y, a, f (a)) (6.38)


A first-order partial differential equation is said to which is a one-parameter family of solutions of
be nonlinear if it does not fall into any one of the (6.33).
above types. This is a sub-family of the two-parameter family
6.7 CLASSIFICATION OF SOLUTIONS OF given by (6.35). The envelope of (6.38), if it exists, is
obtained by eliminating a between (6.38) and
FIRST-ORDER PARTIAL DIFFERENTIAL
EQUATION ∂F ∂F
+ f ′( a) = 0 (6.39)
Consider a first-order partial differential equation of ∂a ∂b
the form If (6.39) can be solved for a then
f (x, y, z, p, q) = 0 (6.33)
a = a(x, y) (6.40)
Let
z = z(x, y) (6.34) Substituting for a in (6.38) we obtain an
integral surface as
be a continuously differentiable function of x and y
in a region D. z = F(x, y, a(x, y), f (a(x, y))) (6.41)
Partial Differential Equations    6-9

If f is arbitrary then (6.41) is called a general Example 6.16


integral (or general solution) of the partial differ- ∂2 z
ential equation (6.33). Solve: = xy 2.
∂x 2
6.7.3. Particular Integral Solution The given partial differential equation is
When a particular function f is used in (6.41) we ∂2 z
obtain a particular integral or particular solution of = xy 2 (1)
the partial differential equation (6.33). ∂x 2
Integrating once partially with respect to x we get
6.7.4. Singular Integral
∂z x 2 2
In some cases we find another integral which = ⋅ y + f ( y) (2)
∂x 2
satisfies the partial differential equation (6.33) but
is not a particular integral of (6.33). If it exists, it is and integrating (2) partially with respect to x we get
obtained by eliminating a and b from the solution as
∂F ∂F x3 2
z = F ( x, y, a, b), = 0 and =0 z= ⋅ y + xf ( y ) + g ( y ) (3)
∂a ∂b 6
(6.42), (6.43), (6.44)
where f and g are arbitrary functions of y.
and it is the envelope of the two-parameter family
of surfaces Example 6.17
∂2 z
z = F(x, y, a, b) (6.45) Solve: = sin x tan y .
∂x ∂y
and it is called a singular integral (or a singular
solution). Solution The given partial differential equation is
In a sense, a general integral provides a much ∂2 z
broader class of solutions of the partial differential = sin x tan y (1)
equation than does a complete integral. ∂x ∂y
However, it is possible to derive a general Integrating (1) partially with respect to x
integral when a complete integral is known.
∂z
Note that for the partial differential equation = − cos x tan y + f ( y ) (2)
∂y
z = px + qy (6.46)
the relation where f (y) is an arbitrary function of y. Integrating (2)
z = ax + by (6.47) again partially with respect to y
is a complete integral while the relation
z = − cos x · log sec y + g(y) + h(x) (3)
⎛ y⎞
z = yf ⎜ ⎟ (6.48)
⎝ x⎠ where g ( y ) = ∫ f ( y ) dy and h(x) are arbitrary
is a general integral. functions of y and x, respectively.

6.8 EQUATIONS SOLVABLE BY DIRECT


Example 6.18
INTEGRATION
∂2 z
We now consider partial differential equations which Solve: = 2 xe y.
can be solved by direct integration. While carrying ∂y ∂x
out integration with respect to a variable the other Solution The given partial differential equation is
variable is held fixed. So, in place of constant of
integration we have to add an arbitrary function of ∂2 z
= 2 xe y (1)
the variable held fixed. ∂y ∂x
6-10    Engineering Mathematics-II

Integrating (1) partially with respect to y we get Solution The given partial differential equation is
∂z
= 2 xe y + f ( x ) (2) ∂2 z
∂x − a2 z = 0 (1)
∂x 2

where f (x) is an arbitrary function of x. Integrating


(2) again partially with respect to x we get We have to solve (1) under the conditions:

z = x 2 e y + ∫ f ( x ) dx + g ( y ) (3) ∂z ∂z
When x = 0, = a sin y and =0 (2), (3)
∂x ∂y
which is the solution of (1). We can write (3) as
If we treat z as a function of x alone, the solution
z = x2ey + h(x) + g(y) (4) of (1) is
where h(x) and g(y) are arbitrary functions of x and y,
z = A sinh x + B cosh x (4)
respectively.
Example 6.19 where A and B are constants. Since z is a function of
∂ z
3 y also we can take z as
Solve: = cos(2 x + 3 y ).
∂x 2 ∂y z = A(y) sinh x + B(y) cosh x
Solution The given partial differential equation is ∂z
Now = A( y ) cosh x + B( y )sinh x x = 0
∂3 z ∂x x=0
= cos(2 x + 3 y ) (1)
∂x 2 ∂y ⇒ A.1 + B.0 = a sin y
Integrating (1) partially with respect to x ∂z
= A′( y )sin x + B ′( y ) cosh x x = 0
∂ z 2
1 ∂y x=0
= sin(2 x + 3 y ) + f ( y ) (2)
∂x ∂y 2 ⇒ A′.0 + B ′.1 = 0
where f (y) is an arbitrary function of y. Integrating ∴ A( y) = a sin y and B(y) = constant.
(2) partially with respect to x again
Hence z = (a sin y) sinh x + b cosh x,
∂z 1
= − ⋅ cos(2 x + 3 y ) + xf ( y ) + g ( y ) (3) where b is an arbitrary constant.
∂y 4
where g is an arbitrary function of y. Integrating (3)
partially with respect to y, EXERCISE 6.3
1
z = − sin(2 x + 3 y ) + x ∫ f ( y ) dy + ∫ g ( y ) dy + h( x ) 1. Solve ∂ z / ∂x∂y = sin x sin y given that ∂z / ∂y =
2
12 −2sin y when x = 0 and z = 0 when y is an odd multiple
which can be put in the form of p /2. [Madras 1994 S, Mysore 1999 S]
1 Ans: z = (1 + cos x) cos y
z = f1 ( x ) + f 2 ( y ) + xf3 ( y ) − sin(2 x + 3 y ) (4)
12
where f1(x) is an arbitrary function of x and f2(y) and 2. Solve ∂3 z / ∂x 2∂y + 18 xy 2 + sin(2 x − y ) = 0.
f3(y) are arbitrary functions of y. 1
Ans: z = cos(2 x − y ) − x 3 y 3 + x f ( y ) + g ( y ) + h( x )
Example 6.20 4

∂2 z ∂z ∂z
Solve: − a2 z = 0. If = a sin y and =0 3. Solve ∂ 2 z / ∂x 2 + z = 0 given that when x = 0, z = e y
∂x 2
∂x ∂y and ∂z / ∂x = 1. [Mysore 1987 S, Madras 1993,
when x = 0. Karnataka 1994]
Partial Differential Equations    6-11

4. Solve ∂ 2u / ∂x∂t = e − t cos x. [Madurai 1988, satisfy equations (6.54) then


Mysore 1987]
ux dx + uy dy + uz dz = 0, (6.56)
Ans: u = −e−t sin x + f (x) + y (t)
and vx dx + vy dy + vz dz = 0 (6.57)
(
5. Solve ∂ 2 z / ∂x∂y = x + y .) must be compatible with (6.54) so that we must
x y have
Ans: z = e · e + ∫ f (y)dy + f (x)
Pux + Quy + Ruz = 0, Pvx + Qvy + Rvz = 0
6. Solve ∂ 2 z / ∂y 2 = sin( x y ) . (6.58), (6.59)
sin xy
Ans: z = − + yf ( x ) + y ( x ) From (6.58) and (6.59) we have
x2
P Q R
= =
6.9 QUASI-LINEAR EQUATIONS ∂( u , v ) ∂( u , v ) ∂( u , v ) (6.60)
OF FIRST ORDER ∂( y , z ) ∂( z , x ) ∂( x , y )
Quasi-linear partial differential equations of first Now, differentiating (6.53) with respect to x and y
order can be written as
∂F ⎛ ∂u ∂u ⎞ ∂F ⎛ ∂v ∂v ⎞
Pp + Qq = R (6.49) ⎜ + p⎟ + ⎜ + p⎟ = 0 (6.61)
∂u ⎝ ∂x ∂z ⎠ ∂v ⎝ ∂x ∂z ⎠
where P, Q, R are functions of x, y and z. They ∂F ⎛ ∂u ∂u ⎞ ∂F ⎛ ∂v ∂v ⎞
+ q + + q =0 (6.62)
∂u ⎜⎝ ∂y ∂z ⎟⎠ ∂v ⎜⎝ ∂y ∂z ⎟⎠
do not involve p or q. Equation (6.49) is called
Lagrange’s equation. Here, linear means that p and
q appear to the first degree only. This is in contrast Eliminating ∂F ∂u and ∂F ∂v from (6.61)−(6.62) we
to the situation in ordinary differential equations get
where z must also be of first degree.
Note that the partial differential equation ∂u ∂u ∂v ∂v
+ p + p
∂x ∂z ∂x ∂z
∂z ∂z = 0, or
x +y = z2 + x2 (6.50) ∂u ∂u ∂v ∂v
∂x ∂y + q + q
∂y ∂z ∂y ∂z
is linear while the ordinary differential equation
∂( u , v ) ∂( u , v ) ∂( u , v )
p+ q=
x
dz
= z2 + x2 (6.51) ∂( y , z ) ∂( z , x ) ∂( x , y )
dx or Pp + Qq = R (6.63)
is nonlinear.
Theorem 6.1 The general solution of the quasi- ∂( u , v ) ∂( u , v ) ∂( u , v )
where P = , Q= , R= are
linear partial differential equation ∂( y , z ) ∂( z , x ) ∂( x , y )
functions of x, y, z.
Pp + Qq = R (6.52)
is Substituting from (6.60) into (6.63) we see that
F(u, v) = 0 (6.53) (6.53) is a solution of (6.49) if u and v are given by
(6.55).
where F is an arbitrary function and u(x, y, z) = c1
and v(x, y, z) = c2 form a solution of the equations
6.10 SOLUTION OF LINEAR,
dx dy dz SEMI-LINEAR AND QUASI-LINEAR
= = (6.54)
P Q R EQUATIONS
Proof If
If the equation is linear or semi-linear or can be written
u(x, y, z) = c1, v(x, y, z) = c2 (6.55)
in these forms then we can solve them as follows.
6-12    Engineering Mathematics-II

6.10.1. All the Variables are Separable General solution is


Example 6.21 F(lx + my, z l · ey tan(lx+my)) = 0 (6)
Solve: zx p + zy q = 1 − z .
2 2 2
Example 6.23
Solution The equation can be thrown into the Solve: yp − xq = xyz + xy.
form
Solution The given equation is linear. Lagrange’s
2 1 − z2
2 auxiliary equations are
x p+ y q = (1)
z
dx dy dz
which is a semi-linear equation. = = (1)
Lagrange’s auxiliary equations are y − x xyz + xy
dx dy zdz From the first equation we have
= = (2)
x y 2
1− z 2 2 xdx + ydy = 0 (2)
Integrating the first and last equations we have Integrating we get
1 1 1 1 x2 + y2 = c1 (3)
− = c1 , = log(1 − z 2 ) + log c2
x y y 2 From the last equation
(e1 y )
or = c2 , respectively (3), (4) dy dz dz
= ⇒ ydy + =0 (4)
1 − z2 −1 y( z + 1) z +1
General solution is Integrating,
⎛ 1 1 (e1 y ) ⎞ y2 2
F⎜ − , ⎟ =0 (5) + log( z + 1) = log c2 ⇒ ( z + 1)e y 2
= c2 (5)
⎝ x y 1 − z2 ⎠ 2
General solution is
6.10.2. Two Variables are Separable
F ⎛⎜ x 2 + y 2 , ( z + 1)e
y22 ⎞ (6)
Example 6.22 ⎝ ⎟⎠
Solve: mp − lq = z tan(lx + my).
6.10.3. Method of Multipliers
Solution This is a semi-linear equation. In addition to the above methods, we can apply the
Lagrange’s auxiliary equations are following method called method of multipliers.
dx dy dz (1)
= = Example 6.24
m −l z tan(lx + my )
Solve: x2(y − z)p + y2(z − x)q = z2(x − y).
From the first equation we have
Solution Lagrange’s auxiliary equations are
ldx + mdy = 0 ⇒ lx + my = c1 (2)
dx dy dz
Using this in the last equation 2
= 2
= 2
(1)
x ( y − z) y ( z − x) z ( x − y)
dy dz
= (since lx + my = c1 )
−l z tan c1 ⎛ 1 1 1⎞ ⎛ 1 1 1⎞
Using multipliers ⎜ , , ⎟ and ⎜ 2 , 2 , 2 ⎟ in
ldz (3), (4) ⎝ x y z⎠ ⎝x y z ⎠
⇒ tan c1dy + =0 turn we get
z 1
dx + 1y dy + 1z dz
x
Integrating we have each ratio =
x( y − z ) + y( z − x ) + z ( x − y ) = 0
y tan(lx + my) + l log z = log c2
dx dy dz
or zl · e y tan(lx+my) = c2 (5) ⇒ + + =0 (2)
x y z
Partial Differential Equations    6-13

⇒ log x + log y + log z = log c1 Example 6.26


⇒ xyz = c1 (3) Solve yp + xq = (x2 − y2 + z2)x.

1 1 1
Solution Lagrange’s auxiliary equations are
dx + dy + dz
x2 y2 z2 dx dy dz
each ratio = = = 2 (1)
( y − z) + ( z − x) + ( x − y) = 0 y x ( x − y2 + z2 ) x
1 1 1
⇒ dx + dy + dz = 0 (4) From the first two ratios we get
x2 y2 z2
x dx − y dy = 0 ⇒ x2 − y2 = a2
1 1 1 (a2 is an arbitrary constant)
⇒ + + = c2 (5)
x y z Considering the last two ratios we have
dz
General solution is dy = ! x 2 − y 2 = a2 (2)
a + z2
2

⎛ 1 1 1⎞
F ⎜ xyz , + + ⎟ = 0 (6) Integrating,
⎝ x y z⎠
1 −1 z 1 z
y= tan +b = tan −1 +b
2 2
Example 6.25
a a x −y x − y2
2

Solve: zxp + zyq = 1+ z2. (b is an arbitrary constant) (3)

Solution Lagrange’s auxiliary equations are General solution is

dx dy dz ⎛ 1 z ⎞
= = (1) F ⎜ x2 − y2 , y − tan −1 ⎟ = 0 (4)
zx zy 1 + z 2
⎝⎜ x2 − y2 x 2 − y 2 ⎠⎟
dx dy zdz
⇒ = =
x y 1+ z2 Example 6.27
Solve yp + xq =(x + y)z.
Taking the first two ratios,
Solution Lagrange’s auxiliary equations are
dx dy
= ⇒ log x = log y + const., on integration dx dy dz
x y = = (1)
y x ( x + y) z
x
⇒ = c1 (2)
y From the first two ratios
x dx − y dy = 0 ⇒ x2 − y2 = c1 (2)
Taking the last two ratios,
dy zdz Again, each ratio
= ⇒
y 1 + z2 dx + dy dz dz
= = ⇒ d ( x + y) = ,
1 y+x ( x + y) z z
log y = log(1 + z 2 ) + constant (on integration)
2 cancelling ( x + y )
y (3) On integration we have
⇒ = c2
1+ z2 x + y − log z = c2 (3)
⎛x y ⎞ General solution is
General solution is F⎜ , ⎟ =0 (4)
⎝ y 1 + z2 ⎠ F(x2 − y2, x + y − log z) = 0 (4)
6-14    Engineering Mathematics-II

Example 6.28 Example 6.30


Solve xp − yq = xyz. Solve (y − z)p + (z − x)q = (x − y).
Solution Lagrange’s auxiliary equations are Solution Lagrange’s auxiliary equations are

dx dy dz dx dy dz
= = (1) = = (1)
x − y xyz y−z z−x x− y
From the first two ratios
Taking (1, 1, 1) as Lagrange’s multipliers,
dx dy each ratio
= ⇒ x dy + y dx = 0 ⇒ xy = c1 (2)
x −y dx + dy + dz
=
From the last two ratios ( y − z) + ( z − x) + ( x − y) = 0
dy dz ⇒ d ( x + y + z ) = 0 ⇒ x + y + z = c1 (2)
= ! xy = c1
− y c1 z
Taking (x, y, z) as Lagrange’s multipliers,
Integrating each ratio
1 1 x dx + y dy + z dz
log z + log y = constant ⇒ log z + log y = c2 =
c1 xy x( y − z ) + y( z − x ) + z ( x − y ) = 0
(3)
⇒ d( x2 + y2 + z2 ) = 0 ⇒ x 2 + y 2 + z 2 = c2
General solution is
(3)
⎛ 1 ⎞ (4)
F ⎜ xy, log z + log y ⎟ = 0 General solution is
⎝ xy ⎠
F (x + y + z, x2 + y2 + z2) = 0 (4)
Example 6.29
Solve zp + yq = x
Example 6.31
Solution Lagrange’s auxiliary equations are Solve sec x p + (sin x − y sec x tan x)q = (a2 − z2).
dx dy dz
= = (1) Solution Lagrange’s auxiliary equations are
z y x
dx dy dz
From the first and last ratios = = (1)
sec x sin x − y sec x tan x a2 − z 2
dx dz
= ⇒ x dx − z dz = 0
z x From the first and last ratios
Integrating we get dz
cos2 x dx = (2)
x − z = c1
2 2
(2) a − z2
2

dy dx + dy + dz On integration, we get
Each of the ratios (1) = =
y z+ y+x 1 a+ z
Integrating, sin x = log + const.
2a a−z
y
log y = log( x + y + z ) + log c2 ⇒ = c2 (a + z)
( x + y + z) ⇒ 2a sin x = log = c1 (3)
(3) (a − z)
General solution is From the first two ratios
⎛ y ⎞
F ⎜ x2 − z2 , =0 dy
( x + y + z ) ⎠⎟
(4) sec x = sin x − y sec x tan x (4)

dx
Partial Differential Equations    6-15

which is a linear differential equation Solution Lagrange’s auxiliary equations are


dy dx dy dz
sec x + y sec x tan x = sin x = = (1)
dx 2 2 2
2 xy 2 xz
x −y −z
d
(sec x, y) − sin x = 0 dy dz
dx Considering the last two ratios, we have = .
Integrating we have y z
On integration, we get
y sec x + cos x = c2 (5)
y
General solution is log y = log z + constant ⇒ = c1 (2)
z
⎛ a+ z ⎞
F ⎜ 2a sin x − log , y sin x + cos x ⎟ = 0 (6) Using x, y, z as multipliers, we get that each ratio
⎝ a−z ⎠
Example 6.32
x dx + y dy + z dz dz
= 2 2 2
=
Solve x(1 − xy)p − y(1 + xy)q = z(1 − xy). x( x + y + z ) 2 xz

Solution Lagrange’s auxiliary equations are d( x2 + y2 + z2 ) dz


or
2 2 2
= , on multiplication by 2x.
dx dy dz (x + y + z ) z
= = (1)
x(1 − xy ) − y(1 + xy ) z (1 − xy ) This gives, on integration,
dx dz log(x2 + y2 + z2) − log z = log c2
From the first and last ratios = , cancel-
ling (1 − xy). x z
( x2 + y2 + z2 )
Integrating or = c2 (3)
z
x
log x = log z + log c1 ⇒ = c1 (2)
z From (2) and (3), the general solution is
On cross-multiplication the first two ratios yield ⎛ y ( x2 + y2 + z2 ) ⎞
⎟ =0
F⎜ , (4)
− y dx − xy 2 dx = x dy − x 2 y dy ⎝z z ⎠
⇒ x 2 y dy − xy 2 dx = y dx + x dy, on transposition Example 6.34
dy dx d ( xy ) 1 Solve (z2 − 2yz − y2)p + (xy + zx)q = xy − zx.
⇒ − = , multiplying by
y x ( xy )2 ( xy )2 [Bangalore 1990, Madras 1997 S]

On integration we get Solution Lagrange’s auxiliary equations are


1 dx dy dz
log y − log x = − + constant = = (1)
2 2
xy z − 2 yz − y xy + zx xy − zx
1 ⎛ y⎞
+ log ⎜ ⎟ = c2 (3) Using x, y, z as multipliers, each ratio
xy ⎝ x⎠
General solution is x dx + y dy + z dz
=
xz − 2 xyz − xy 2 + xy 2 + xyz + xyz − z 2 x = 0
2
⎛x 1 ⎛ y⎞⎞
F⎜ , + log ⎜ ⎟ ⎟ = 0 (4)
⇒ x 2 + y 2 + z 2 = c1
⎝ z xy ⎝ x⎠⎠ (2)

Example 6.33 Considering the last two ratios, we have


Solve (x − y − z )p + 2xyq = 2xz.
2 2 2
dy dz dy − dz
= = ⇒ 2 z dz = ( y − z ) d ( y − z )
[Bangalore 1990, Gorakhpur 1991, Andhra 1989] y+z y−z 2z
6-16    Engineering Mathematics-II

On integrating and simplifying Example 6.37


Solve (x2 − yz)p +(y2 − zx)q = z2 − xy. [AMIE 1997,
y2 − 2yz − z2 = c2 (3) Madras 1994 S, 1998; Bhopal 1991, Karnataka 1990,
The general solution Ranchi 1996]
F (x2 + y2 + z2, y2 − 2yz − z2) = 0 (4) Solution Langrange’s auxiliary equations are
dx dy dz
Example 6.35
= = (1)
x 2 − yz y 2 − zx z 2 − xy
Solve xnp + ynq = zn.
x dx + y dy + z dz
Solution Lagrange’s auxiliary equations Each ratio =
x 3 + y 3 + z 3 − 3xyz
dx dy dz dx + dy + dz
= = (1) =
xn yn zn x 2 + y 2 + z 2 − xy − yz − zx
On integration we get two solutions x dx + y dy + z dz dx + dy + dz
⇒ = (2)
x+ y+z 1
x−n+1 − y−n+1 = c1, x−n+1 − z−n+1 = c2 (n ≠ 1),
(2), (3)  x3 + y3 + z3 − 3xyz
= (x + y + z)(x2 + y2 + z2 − xy − yz − zx)
log x − log y = log c1, log x − log z = log c2 (n = 1)
⇒ d(x2 + y2 + z2) = 2(x + y + z)d(x + y + z)
x x ⇒ (x + y + z)2 − (x2 + y2 + z2) = constant
⇒ = c1 , = c2 (4), (5)
y z ⇒ xy + yz + zx = c1 (3)
General solution when n ≠ 1 is
dx − dy
F (x−n+1 − y−n+1, x−n+1 − z−n+1) = 0 and (6) Each ratio =
( x − yz) − ( y 2 − zx )
2
⎛ x x⎞ dx − dz
when n ≠ 1 is F⎜ , ⎟ =0 (7) =
⎝ y z⎠ ( x 2 − yz ) − ( z 2 − xy )
d ( x − y) d( x − z)
Example 6.36 = =
( x − y )( x + y + z ) ( x − z )( x + y + z )
Solve x(y − z)p + y(z − x)q = z(x − y). d ( x − y) d ( x − y) x− y
[JNTU 2002, 2003] ⇒ = ⇒ = c2 (4)
x− y x−z x−z
Solution Lagrange’s auxiliary equations are
From (3) and (4) we can write the general
dx dy dz solution as
= = (1)
x( y − z ) y( z − x ) z ( x − y ) ⎛ x − y⎞
F ⎜ xy + yz + zx, ⎟ =0 (5)
⎝ x − z⎠
Now, each ratio
dx dy dz Example 6.38
1 1 +
1 +
dx dy ydz x y z Solve (y − z)p + (x − y)q = z − x. [Punjab 1987 S]
= x
= = = z
⇒ xyz = c1
y−z z−x x− y 0 Solution Lagrange’s auxiliary equations are
(2)
dx + dy + dz dx dy dz
Also each ratio = (3) = = (1)
0 y−z x− y z−x
⇒ x + y + z = c2
dx + dy + dz
General solution is Each ratio =
( y − z) + ( x − y ) + ( z − x ) = 0
F (xyz, x + y + z) = 0 (4) ⇒ d ( x + y + z ) = 0 ⇒ x + y + z = c1 (2)
Partial Differential Equations    6-17

− x dx z dy + y dz or y(sec z + tan z) = c1 (2)


Each ratio = =
− x( y − z ) z ( x − y ) + y( z − x ) dx dy
From the first two ratios =
dx d ( yz ) x2 x + 2 y2 y
⇒ = ⇒ + yz = c2 (3)
y−z − x( y − z ) 2 On cross-multiplication and transposition
From (2) and (3) we write the general solution as y dx − x dy = 2 y 2 dy.
⎛ x2 ⎞ y dx − x dy ⎛ x⎞
F ⎜ x + y + z, + yz ⎟ = 0 (4) = 2 dy ⇒ d ⎜ ⎟ = 2 dy
⎝ 2 ⎠ y2 ⎝ y⎠
Example 6.39 Integrating, we get
Solve xp + (2x − y)q = (1 − z). x
= 2 y + c2 (3)
Solution Lagrange’s auxiliary equations are y
dx dy dz General solution is
= = (1)
x 2x − y 1 − z ⎛ x ⎞
F ⎜ y (sec z + tan z ), − 2 y ⎟ = 0 (4)
dx dz ⎝ y ⎠
From the first and last ratios =
x 1− z
Integrating we get EXERCISE 6.4
1. px + qy = z.
log x = − log(1 − z) + log c1
⎛ x x⎞
⇒ x(1 − z) = c1 (2) Ans: F ⎜ , ⎟ = 0
⎝ y z⎠
dx dy
From the first two ratios = 2. px2 + qy2 = z2.
x 2x − y
⎛ 1 1 1 1⎞
We have, on cross-multiplication, Ans: F ⎜ − , − ⎟ = 0
⎝ x y x z⎠
2x dx − y dx = x dy or 2x dx = x dy + y dx
p q 1
or d(x2) d(xy) = 0 3. + = .
x y z
Integrating Ans: F(x2 − y2, x2 − z2) = 0
x2 − xy = c2 (3)
y2 z
General solution is 4. p + xzq = y 2 or y 2 zp + x 2 zq = xy 2.
x
F(x(1 − z), x2 − xy) = 0 (4) [Madras 1995, 1997]
Ans: F(x − y , x − z ) = 0
3 3 2 2

Example 6.40
5. z(xp − yq) = y2 − x2.
Solve (x +2y2)p + yq = − cos z.
Ans: F(x2 + y2 + z2, xy) = 0
Solution Lagrange’s auxiliary equations are
6. px(z − 2y2) = (z − qy)(z − y2 − 2x3).
dx dy dz
2
= = (1) ⎛y ⎞
x + 2y y − cos z Ans: F ⎜ , y 2 − z ⎟ = 0
⎝z ⎠
7. x(x + y)p − y(x + y)q = −(x − y)(2x +2y + z).
From the last two ratios dy = − sec z dz
y Ans: F(xy, (x + y + z)(x + y)) = 0
Integrating we get 8. x2(y −z)p + y2(z −x)q = z2(x −y). [Madurai 1990]
log y = −log(sec z + tan z) + log c1 −1 −1
Ans: F(xyz, (x + y + z ) = 0 −1
6-18    Engineering Mathematics-II

9. x(y2 − z2)p + y(z2 − x2)q = z(x − y2). so that we have dz = a dx + f (a) dy.
  Ans: F(xyz, (x2 + y2 + z2)) = 0 Integrating, we get the complete integral as
10. (mz − ny)p +(nx − lz)q = ly − mx. z = ax + ∫ f ( a) dy + c or z = ax + f ( a) y + c
[AMIE 1990, Madras 1994 S] (6.66)
Ans: F(lx + my + nx, x2 + y2 + z2) = 0 where a and c are arbitrary constants.
11. (y − zx)p +(x + yz)q = x2 + y2. Note 1 We can put q = a instead of p = a and proceed
Ans: F(x2 − y2 + z2, xy − z) = 0 to obtain the complete integral.
12. (b − c)a−1yzp + (c − a)b−1zxq = (a − b)c−1xy. Example 6.41
Ans: F(ax + by + cz , a x + b y + c z ) = 0
2 2 2 2 2 2 2 2 2 Solve pq = k

13. (y2 + z2 − x2)p − 2xyq = −2zx. Solution The complete integral is


k
⎛y
Ans: F ⎜ ,
(x 2
)
+ y2 + z2 ⎞
⎟ =0
z = ax +
a
y+c (1)
⎜⎝ z y ⎟⎠ where a and c are arbitrary constants
14. p tan x + q tan y = tan z. The general integral is obtained by eliminating
[Andhra 1990, Kerala 1987 S] a between the equations
⎛ sin x sin x ⎞ k
Ans: F ⎜ , =0 z = ax + y + f ( a)1 (2)
⎝ sin y sin z ⎟⎠ a
15. xeyp + (1 − ey)q = zey.
where c has been replaced by f (a) and
k
⎛x ⎞ 0= x− y + f ′( a) (3)
Ans: F ⎜ , ( x + z )(1 − e y )⎟ = 0 a2
⎝z ⎠
obtained by differentiating (2) with respect to a.
6.11 NONLINEAR EQUATIONS The singular integral, if it exists, is determined
OF FIRST ORDER from the equation (1)
The most general partial differential equation of the k
z = ax + y + c
first order in two independent variables is of the form a
f (x, y, z, p, q) = 0 (6.64) k
and 0 = x− 2 , 0 =1 (4), (5)
a
which may not be linear.
We do not propose to study any general method obtained from (1) on differentiating it with respect
of integration of equation (6.64) but consider only to a and c, respectively.
four standard forms which admit integration by The inconsistency of the last equation shows
very short processes. Also, many equations can be that singular integral does not exist in this case.
reduced to one or the other of these four forms. Example 6.42
The general integral, the singular integral and Solve pq = xa yb zg.
the complete integral must be indicated in each case.
Otherwise, the equation is not considered fully solved. Solution The given equation is
∂z ∂z
Standard Form I: pq-equation: Equation of the form = xa y b z g (1)
f (p, q) = 0 where x, y, z do not occur explicitly. ∂x ∂y
To solve such an equation we put p = a and find We can write (1) as
the value of q in terms of a, i.e. q = f (a). Then we
substitute these values in z −g /2 ∂z z −g /2 ∂z
⋅ =1 (2)
dz = p dx + q dy (6.65) x a ∂x y b ∂y
Partial Differential Equations    6-19

Case (i) a ≠ 1, b ≠ −1, g ≠ −2 The general integral is the eliminant of a


−( g /2) +1 between (6) and (7).
z
Put z −g 2 dz = dZ ⇒ Z= , g ≠2
1−
g Case (ii) Writing c = f (a) in the complete
2
Integral at (5)
a +1
x
xa dx = dX ⇒ X = , a ≠ −1, 1 2 1 1
a +1 z = a log x + log x + log y + f ( a) (8)
2 a a
y b +1
y b dy = dY ⇒ Y = , b ≠ −1 Differentiating (8) with respect to a
b +1
1
Equation (2) now becomes, 0 = log x − log y + f ′( a) (9)
a2
∂Z ∂Z (3) Eliminant of a between (8) and (9) is the general
⋅ =1
∂X ∂Y integral.
whose complete integral is
To Find the Singular Integral
Z = aX + bY + c where ab = 1 Case (i) The singular integral, if it exists, is
−( g /2) +1 obtained from the following equations:
z a a +1 1 1 b +1
or
g
= x + ⋅ y +c (4)
1− a +1 a b +1 −( g /2) +1
2 z a a +1 1 1 b +1
g
= x + y +c (10)
Case (ii) a = −1, b = −1, g = −2, we have 1− a +1 a b +1
2

∂z ∂z 1 1 1 z ∂z z ∂z Differentiating partially with respect to a and c,


⋅ = ⋅ ⋅ , ⇒ ⋅ =1
∂x ∂y x y z 2 x −1∂x y −1∂y respectively, we get

1 2 1 a +1 1 1 b +1
Put z dz = dZ ⇒ Z= z 0= x − 2 y 0 =1 (11), (12)
2 a +1 a b +1
x −1dx = dX ⇒ X = log x The inconsistency in the last equation shows
−1
y dy = dY ⇒ Y = log y that the singular integral does not exist in this case.

The CI is Case (ii) We can show that in this case also


1 singular integral does not exist.
Z = aX + bY + c where ab = 1 or b =
a
1 2 1 Example 6.43
z = a log x + log y + c (5) Find a complete integral of
2 a
p + q = pq (1)
To nd the General Integral
a
Case (i) Writing c = f (a) in the complete integral Solution Put p = a, then q = and the equa-
a −1
at (4) we have tion dz = p dx + q dy becomes
−( g /2) +1 a
z a a +1 1 1 b +1 dz = a dx + dy (2)
g
= x + y + f ( a) (6) a −1
1− a +1 a b +1
2 whose solution is
Differentiating (6) with respect to a we get a
z = ax + y+c (3)
1 a +1 1 1 b +1 a −1
0= x − 2 y + f ′( a) (7)
a +1 a b +1 which is the required complete integral.
6-20    Engineering Mathematics-II

Standard Form II: zpq-equation: Equation of on integration, we get


the form f (z, p, q) = 0 where the independent 2/3
⎛ a ⎞ 3 5/3
variables do not appear explicitly. x + ay + c = ⎜ z (4)
In this case put ⎝ 1 + a ⎟⎠ 5
where c is an arbitrary constant.
q = ap (or p = aq) (6.67)
Standard Form III: Separable equation
The given equation becomes f (z, p, a) = 0. f (x, p) = g ( y, q).
Solving for p we have We can take f (x, p) = a and g(y, q) = a (6.71)
p = f (a, z) (6.68) Solving for p and q we get
Substituting in p = f1(a, x), q = g1(a, y) (6.72), (6.73)
we have dz = pdx + qdy = p dx ap dy = pd(x + ay)
(6.69) Substituting in dz = p dx + q dy
= f1(a, x) dx + g1(a, y) dy (6.74)
dz
⇒ d (x + ay) = Integrating we get the CI as
f ( a, z )
z = ∫ f1 ( a, x ) dx + ∫ g1 ( a, y ) dy + c (6.75)
dz
⇒ x + ay + c = ∫ (6.70)
f ( a, z ) Example 6.46
where c is an arbitrary constant. Solve p + q + x + y
Example 6.44 Solution We can write the equation as
Solve p = z (1 − pq).
2 2
(1) p−x=y−q=a (1)
so that
Solution Put q = ap in the given partial differen-
p = x + a, q = y − a (2)
tial equation (1) so that we have
p2 = z 2 (1 − ap2 ) (2) Substituting in
dz = p dx + q dy (3)
1 1 + ap2
⇒ p2 (1 + az 2 ) = z 2 or = (3)
p2 z2 we have
dz = (x + a) dx + (y − a) dy (4)
Now, dz = p dx + q dy
= p dx + ap dy = p d(x + ay) Integrating after multiplying throughout by 2,
the complete integral is
dz ⎛ 1 ⎞
x + ay + c = ∫
p ∫ ⎝ z2
= ⎜ + a⎟ dz 2z = (x + a)2 +(y − a)2 + c (5)

where c is an arbitrary constant.
1
or x + ay + c = − + az (4) Example 6.47
z
Solve zpq = xy.
where c is an arbitrary constant.
Solution The equation can be written as
Example 6.45
⎛ z1/2 ∂z ⎞ ⎛ z1/2 ∂z ⎞
zpq = p1/2 + q1/2.
⎜ ∂x ⎟ ⎜ ∂y ⎟ = xy
Solve (1) (1)
⎝ ⎠⎝ ⎠
Solution Putting q = ap in (1) we get
2 3/2
⎛ ⎞ 1 a Put z1/2 dz = dZ ⇒ Z= z (2)
zap2 = p1/2 ⎜ 1 + a1/2 ⎟ , ⇒ = ⋅ z (2) 3
⎝ ⎠ p 3/2
1+ a
⎛ ∂Z ⎞
Now, dz = p dx + q dy = p d(x + ay) (3) ⎜⎝ ⎟⎠
∂x y
2/3 = =a (3)
dz ⎛ a ⎞ x ⎛ ∂Z ⎞
d ( x + ay ) = = z 2/3 dz
p ⎜⎝ 1 + a ⎟⎠ ⎜⎝ ∂y ⎟⎠
Partial Differential Equations    6-21

∂Z ∂Z y General integral and singular integral (if it


∴ = ax, = (4)
∂x ∂y a exists) are obtained as explained in sec. 6.7.
Substituting in the equation Singular Integral The complete integral is

⎛ ∂Z ⎞ ⎛ ∂Z ⎞ z = ax + by + ab (6.80)
dZ = ⎜ ⎟ dx + ⎜ ⎟ dy, (5)
⎝ ∂x ⎠ ⎝ ∂y ⎠ Differentiating (6.80) partially with respect to
y a and b we get
we have dZ = ax dx + dy , which on integrating
a 0=x+b (6.81)
yields,
0=y+b (6.82)
2 3/2 a 2 y 2
z = x + +c (6)
3 2 2a respectively. Eliminating a and b between (6.81),
(6.82) we have singular solution of (6.76) as
where c is an arbitrary constant.
The general integral and singular integral (if it z + xy = 0
exists) is obtained as explained in sec. 6.7.
Examples
Standard Form IV: Clairaut’s2 Equation Find complete integral in each of the following cases:
z = px + qy + f (p, q).
Example 6.48
A partial differential equation of the type
z = px + qy − 2 pq [JNTU 2003]
z = px + qy + f (p, q) (6.76)
Solution The given partial differential equation
is called a Clairaut’s equation.
z = px + qy − 2 pq (1)
Complete Integral A complete integral of an
equation of the type (6.76) is obtained by replacing is an equation of Clairaut’s type.
p and q by arbitrary constants a and b, respectively. The complete integral of (1) is
Thus, we have
z = ax + by − 2 ab (2)
z = ax + by + f (a, b) (6.77)
where a and b are arbitrary constants.
One can readily verify that (6.77) is a solution Example 6.49
of equation (6.76)
p4 + q4
Also, writing (6.77) as z = px + qy + [JNTU 2003]
pq
z = F(x, y, a, b) = ax + by + f (a, b) (6.78)
Solution The given partial differential equation
we observe that the matrix
p4 + q4
z = px + qy + (1)
⎡ ∂F ∂ F
2
∂ F⎤ ⎡
2
∂f ⎤ pq
⎢ ⎥ x+ 1 0⎥
∂a ∂x ∂a ∂y ∂a ⎥ ⎢ ∂a
M=⎢ =⎢ ⎥ is an equation of Clairaut’s type.
⎢ ∂F ∂ F
2 2 ⎥
∂ F ⎢ ∂f
0 1⎥
The complete integral of this equation is
⎢ ⎥ ⎢y + ⎥⎦
⎢⎣ ∂b ∂x ∂b ∂y∂b ⎥⎦ ⎣ ∂b
a4 + b4
(6.79) z = px + by + (2)
ab
is of rank two. Hence, (6.78) is indeed a complete
Example 6.50
integral of the equation (6.79).
2q(z − px − qy) = 1+ q2 [JNTU 2005 S]
2
CLAIRAUT, Alexis Claude (1713–1765), was a French ana- The given partial differential equation can be
lyst, differential geometer and astronomer. written as
6-22    Engineering Mathematics-II

1 + q2 7. p(1 + q)qz. [Kerala 1990 S, Punjab 1986]


z = px + qy + (1)
2q Ans: log (az − 1 ) = (x + ay + c)
which is an equation of Clairaut’s type.
8. z = p2 + q2. [Calicut 1994, Andhra 1990,
The complete integral of this equation is
Madurai 1990]
1 + b2 Ans: 4z (1 + a2) = (x + ay + c)2
z = ax + by + (2)
2b
9. z(p2 − q2) = x − y. [Madras 1991, Calicut 1991,
Example 6.51 Karnataka 1990]
Find the singular integral in Example 6.48. Ans: z 3/2 = ( x + a)3/2 + ( y + a)3/2 + c
Solution The complete integral is 10. q = xyp2.
1
z = ax + by − 2 ab (1) Ans: z = 2 ax + ay 2 + c
2
Differentiating this with respect to a and b, 11. yp + xq + pq = 0. [Marathwada 1993]
respectively, we get a 2
Ans: 2 z = ax −2
y +c
a +1
b a
0= x− , 0= y− (2), (3) 12. p + q = sin x + sin y. [Madras 1993, Karnataka 1993]
a b
Ans: z = a(x − y) − (cos x + cos y) + c
Eliminating a and b from these equations we get
13. z = px + qy + p2+ q2.
the singular integral as
Ans: z = ax + by + a2 + b2
xy = 1 (4)
14. z = px + qy + l 1 + p2 + q2 .
EXERCISE 6.5 Ans: z = ax + by + l 1 + a2 + b2
1. q2 = e − p /a . 15. z = px + qy + sin(p + q).

Ans: z = ax + e − a 2a y + c Ans: z = ax + by + sin(a + b)

2. p + q = pq. 6.12 EULER’S METHOD OF SEPARATION


Ans: z = ax +
a
y+c OF VARIABLES
a −1
When we model scientific, engineering, biotechno-
3. p2 + q2 = 1. [Osmania 2000] logical and other processes there arise initial and
boundary value problems involving partial different-
Ans: z = ax + 1 − a2 y + c
ial equations. The general solutions of these partial
differential equations are in the form of arbitrary
4. x2p2 + y2q2 = z. [Osmania 2000 s, Madras 1998]
functions which are not suitable for determining
⎡ ⎛ z −1∂z ⎞
2
⎛ z −1∂z ⎞
2 ⎤ the exact solutions satisfying the given initial and
⎢ Hint : ⎜ ⎟ + ⎜ −1 ⎟ = 1.⎥
⎢ ⎝ x −1∂x ⎠ ⎝ y ∂y ⎠ ⎥ boundary conditions.
⎣ ⎦ The method of separation of variables, due to
Ans: log z = a log x + 1 − a2 ⋅ log y + c Euler3 is a simple, yet powerful technique in break-
5. z = pq. 3
Euler, Leonhard (1707–1783), was a gifted Swiss mathema-
Ans: (x + ay + c)2 = 4az tician and the most prolific mathematician, who contibuted to
almost all branches of mathematics even after becoming totally
6. zq = p2. blind in 1771. He studied under John Bernoulli and became a
Ans: z = ceax + a2y professor of mathematics at St. Petersburg, Russia. He is the first
modern mathematical universalist.
Partial Differential Equations    6-23

ing up a partial differential equation into an ordinary log T = lt + log c2 or T = c2 e lt (8)


differential equations, which can be easily solved
using the known methods. Combining (7) and (8) we may write the solu-
For a partial differential equation in the function tion as
u = u(x, y) where x and y are independent variables,
u( x, t ) = XT = c1c2 e(1+ 2l ) x e lt
we assume that the solution is separable, that is,
Now 6e−3x = u(x, 0) = c1c2e(1+2l)x
u(x, y) = X(x)Y(y) (6.83)
⇒ c1c2 = 6 and 1+2l = −3
where X(x) is a function of x alone and Y(y) is a The latter equation gives l = −2
function of y alone. Substituting in the given partial Substituting in the general solution we get
differential equation, separating the variables and
assuming each side must be equal to the same constant u( x, y ) = 6e −3 x e −2t = 6e − (3 x + 2t ) (9)
l (say), we obtain two ordinary differential equations, which is the required solution.
whose solutions give the solution of the problem. The
method is best illustrated through examples. Example 6.53
Solve by the method of separation of variables
Example 6.52 ∂u ∂u
4 + = 3u given that u = 3e−y − 3e−5y when x = 0.
∂u ∂u ∂x ∂y
Solve = 2 + u by the method of separation of
∂x ∂t Solution Let
variables given that u(x, 0) = 6e−3x.
u = X(x)Y (y) (1)
Solution We assume the solution to be
where X(x) is a function of x alone and Y(y) is a
u(x, t) = X(x)T(t) (1) function of y alone. Calculating the derivatives and
substituting in (1) we have
where X(x) is a function of x alone and T(t) is a func-
tion of t alone. We have 4X¢Y + XY¢ − 3XY = 0
∂u ∂u X′ Y′
= X ′T and = XT ′ (2), (3) ⇒ 4−3= − = l (say)
∂x ∂t X Y
Thus, we have two ordinary differential equations
where  denotes differentiation with respect to the
respective argument. 4X¢ − 3X = lX and Y¢ + lY = 0 (2), (3)
Substituting in the given partial differential
Taking a trial solution of the form emx, emy for
equation, we get
the above equations we obtain auxiliary equations
X¢T = 2XT¢ + XT or (X¢ − X)T = 2XT¢ (4) 3+ l
4m − 3 − l = 0 ⇒ m = , and m = − l (4), (5)
Dividing both sides of (4) by 2XT we get 4
respectively.
X′− X T′
= = l (say) The solution for X is of the form
2X T
⇒ X ′ − X − 2lX = 0 X = c1e((3+ l )/4)x (6)
X′ T′ (5), (6) and the solution for Y is of the form
or = 1 + 2l, and =l
X T Y = c2e−ly (7)
Solving (5) we get The solution for u can be taken as
(1+ 2 l ) x
log X = (1 + 2l ) x + log c1 or X = c1e (7) u( x, y ) = c1e(
(3+ l1 )/4)x (3+ l2 )/4)x
⋅ e − l1 y + c2 e( ⋅ e − l2 y +!
and solving (6) we get (8)
6-24    Engineering Mathematics-II

where we have used the principle of superposition u( x, y ) = c1 sinh( 2 + k1 x ) e k1 y


of solutions since the equation is linear.
Putting x = 0 in (8) we get + c2 sinh( 2 + k2 x ) e k2 y +! (8)
(3+ l1 )/4).0
u(0, y ) = 3e − y − 3e −5 y = c1e( ⋅ e − l1 y ∂u
= ( 2 + k1 c1 cosh( 2 + k1 x ) e k1 y
+ c e ( 2 ) 0 e − l2 y
(3+ l )/4 . ∂x
2 x=0

and c3 = c4 = … = 0 + 2 + k2 c2 cosh( 2 + k2 ) e k2 y +! ) x = 0
Equating like terms on both sides (9)
−y −5 y − l1 y − l2 y
3e − 3e = c1e + c2 e = ( 2 + k1 c1 .1.e k1 y + 2 + k2 c2 .1.e k2 y
⇒ c1 = 3, c2 = −3, l1 = 1, l2 = 5
+ c3 ! + c4 ! ) = 1 + e −3 y (10)
and c3 = c4 = º = 0
1
The required solution is ⇒ k1 = 0, c1 = , k2 = −3,
2
u(x, y) = 3ex−y − 3e2x−5y 1
Example 6.54
c2 = , c3 = c4 = ! = 0 (11)
i
∂2u ∂u
Solve = + 2u
subject to conditions. 1
∂x 2
∂y Since 2 + k2 ⋅ c2 = 1 for k2 = −3 ⇒ c2 = .
i
∂u The second solution is
u = 0, = 1 + e −3 y when x = 0.
∂x
1 eix − e − ix −3 y
Solution Let u = X(x)Y(y) be a solution of c2 sinh 2 + k2 xe k2 y = .e sin xe −3 y
i 2
∂2u ∂u
= + 2u. (1) 1
∂x 2
∂y ∴ u( x , y ) = sinh 2 x + sin xe −3 y (12)
2
Substituting in the equation (1)
Example 6.55
X ′′ Y′
X ′′Y = XY ′ + 2 XY ⇒ −2= =k (say) Solve
X Y
∂2u ∂u ∂u
(2)
2
−2 + =0 (1)
∂x ∂x ∂y
We obtain the ordinary differential equations
Solution
X≤ − (2 + k)X = 0, Y≤ − kY = 0, where k is a constant.
Let u = X(x) · Y(y) (2)
Taking the solutions as X = emx, Y = emy we get
(3), (4) where X is a function of x alone and Y is a function
Auxilliary equations m − (2 + k) = 0, m −k = 0
2 of y alone, be a solution of equation (1). Calculating
(5), (6) the derivatives and substituting in (1) we have
X ′′ Y′
2+ k x X ′′Y − 2 X ′Y + XY ′ = 0 ⇒ − 2 = k (say)
∴ The solution is of the form (e , e− 2+ k x
) e ky X Y
If we take the solution as Thus, we obtain two ordinary differential
u = (c1e 2 + k x + c2 e − 2 + k x ) e ky equations
The condition x = 0 ⇒ u = 0 which gives X≤ − 2X ′ − kX = 0 and Y ′ + kY = 0 (3), (4)
c1 + c2 = 0, (eky ≠ 0), or c2 = −c1 (7)
ax −ax where k is a constant. Taking trial solutions, X = emx,
Since e −e = 2 sinh ax, and we have to
Y = emy, the auxiliary equations are
satisfy another condition by the principle of super-
position of solutions, we may take m2 − 2mk = 0, m + k = 0
Partial Differential Equations    6-25

2 ± 4 + 4k 6.13 CLASSIFICATION OF SECOND-ORDER


⇒ m= = 1 ± 1 + k , m = −k PARTIAL DIFFERENTIAL EQUATIONS
2
6.13.1 Introduction
1+ k x
X = (c1e + c2 e − 1+ k x
) e x, Y = c3e − ky Many physical and engineering applications such
1+ k x − 1+ k x x − ky as fluid flow, heat transfer and wave motion involve
u( x, y ) = ( Ae + Be )e
second-order partial differential equations and hence
we take up a study of these equations and their solu-
where A and B are two arbitrary constants, consis-
tion by separation of variables method.
tent with the order of the differential equation.
The general second-order partial differential
equation may be written in the form
EXERCISE 6.6 ∂2u ∂2u ∂2u
L [u ] = A +B +C 2
Solve the following equations by separation of variables: ∂x 2 ∂x ∂y ∂y
∂u(0, t )
1. uxt = e−t cos x with u(x, 0) = 0 and = 0. ⎛ ∂u ∂u ⎞
∂t + H ⎜ x , y , u, , ⎟ = 0 (6.84)
−t
Ans: u(x,t) = sin x − e sin x ⎝ ∂x ∂y ⎠

∂u ∂u
Equation (6.84) is called semilinear if A, B and C are
2. 4 + = 3u with u = e−5y when x = 0. functions of the independent variables x and y only.
∂x ∂y [JNTU 2004 S] On the other hand, if A, B and C are functions of x, y,
Ans: u = e2x−5y u, ∂u / ∂x and ∂u / ∂y then (6.84) is called quasi-
linear. If A, B and C are functions of x and y and H is
⎛ ∂z ∂⎞ a linear function of u, ∂u / ∂x and ∂u / ∂y then equation
3. px 3 + qx 2 = 0 ⎜ p = , q = ⎟ . [JNTU 2004 S]
⎝ ∂x ∂y ⎠ (6.84) is called linear. The general second-order
linear partial differential equation in two independent
Ans: z = ce
(
k x 3 /3− y 4 /4 ) variables x and y may be written as

∂2u ∂2u ∂2u ∂u


∂ 2 u ∂u A( x, y ) 2
+ B( x , y ) + C ( x, y ) 2 + D( x, y )
4. 2 − = 0. ∂x ∂x ∂y ∂y ∂x
∂x ∂y
∂u
+ E ( x, y ) + F ( x , y )u + G ( x , y ) = 0 (6.85)
Ans: u( x, y ) = Ae kx
+ Be − kx ∂y

If G ≠ 0 then equation (6.85) is caled nonhomoge-


∂u ∂u neous and if G = 0 then it is called homogeneous.
5. 3 + 2 = 0, u( x,0) = 4e − x .
∂x ∂y

Ans: u( x, y ) = 4e − (2 x − 3 y )/2 6.13.2 Classication of Equations


We call the quantity Δ = B2 – 4AC the discriminant
and classify equation (6.85) as hyperbolic, parabolic
∂ 2 u ∂u
6. Find a solution of = + 2u in the form or elliptic according as Δ > 0, = 0 or < 0. The follow-
∂x 2 ∂y
ing are well-known examples of these three types.
u = X(x)Y (y). Solve the equation subject to the con-
∂u Hyperbolic Type
ditions u = 0 and = 1 + e −3 y when x = 0 for all
∂x
values of y. [Andhra 2000, Nagpur 1997] 1. One-dimensional wave equation
1 ∂2u ∂2u
Ans: u = sinh 2 x + e −3 y sin x = a2 (6.86)
2 ∂t 2 ∂x 2
6-26    Engineering Mathematics-II

Here A = a2, B = 0, C = –1 and Δ = B2 – 4AC = 4a2 > 0. boundary conditions may be one of the following
three types:
Parabolic Type 1. Dirichlet problem ( First boundary value problem)
2. One-dimensional heat-flow equation The solution is prescribed along the boundary.
2. Neumann problem ( Second boundary value problem)
∂u ∂2u
= a2 2 (6.87) The derivative of the solution is prescribed along
∂t ∂x the boundary.
3. Mixed problem ( Third boundary value problem)
Here A = a2, B = C = 0 and Δ = B2 – 4AC = 0.
The solution and its derivative are prescribed
along the boundary.
Elliptic Type
3. (a) Two-dimensional wave equation Any of the above conditions is called homogeneous
if it is a zero-condition and nonhomogeneous if it is
a nonzero condition.
∂2u ∂2u ⎛ ∂2 ∂2 ⎞
∇2 u = 2
+ 2
= 0 ⎜ ∇2 = 2 + 2 ⎟ (6.88)
∂x ∂y ⎝ ∂x ∂y ⎠ 6.13.4 Solution of One-dimensional
Heat Equation (or diffusion
Here A = 1, B = 0, C = 1 and Δ = B2 – 4AC = – 4 < 0. equation)
(b) Poisson’s equation Consider a long and thin wire, rod or bar OA of
length l and of constant cross-section and homo-
∂2u ∂2u
∇2 u = + = f ( x, y ) (6.89) geneous heat-conducting material. Let the bar be
∂x 2 ∂y 2 placed along the x-axis with one end O coinciding
with the origin and the other end A at a distance l
Equations (6.86) – (6.88) are homogeneous while
from O (Figure 6.1).
equation (6.89) is nonhomogeneous.
y

6.13.3 Initial and Boundary Value


Problems and their Solution
O l A x
The unique solution corresponding to a particular
physical problem is obtained by use of additional Figure 6.1 Heat conduction along a bar
information arising from the physical situation.
If this information is given on the boundary as Suppose that the lateral surface of the bar is perfectly
boundary conditions, the resulting problem is insulated, so that heat flow is along the x-direction
called a boundary value problem (BVP). If this only. Therefore, the temperature u of the bar depends
information is given at one instant as initial on x and t only. The initial boundary value problem
conditions, the resulting problem is called an initial consists of one-dimensional heat equation:
value problem (IVP). ∂u ∂2u
The principle of superposition of solutions = a2 2 (6.90)
is applicable as long as the equation is linear and ∂t ∂x
homogeneous. That is, if un is a solution for each where a2 is the thermal diffusivity. The boundary

n then ∑ n=1 un , which is a linear combination of conditions at the ends O and A are
the solutions {un}is also a solution of the equation. u (0, t) = 0 = u (l, t) (6.91, 6.92)
The hyperbolic and parabolic types of equa-
respectively. The initial temperature distribution in
tions are either initial value problems or initial and
the bar is
boundary value problems, whereas the elliptic-type
equation is always a boundary value problem. The u (x, 0) = f (x) (6.93)
Partial Differential Equations    6-27

where f (x) is a given function of x. The solution by The boundary condtion u (0, t) = 0 implies that
the method of separation of variables reduces the A = 0 so that we have
initial boundary value problem (IBVP) to that of 2 2

two ordinary differential equations. u( x, t ) = BC e − a p t


sin px (6.102)
Assume that the solution u (x, t) is separable
The boundary condition u (l, t) = 0 requires that
i.e., u (x, t) = X(x) T(t) (6.94) sin pl = 0
where X(x) is a function of x only and T (t) is a func- which holds if
tion of t only. np
Differentiating (6.94) with respect to t and x we get pl = n p ⇒ p = pn = ( n = 1, 2,3,...) (6.103)
l
∂u ∂2u The constant pn is an eigenvalue and the function
= XT ′, 2
= X ′′ T (6.95) sin pn x is an eigenfunction. Now, we can write the
∂t ∂x
solution, using the principle of superposition, as
where ′ denotes differentiation with respect to the ∞ ∞ 2 2
corresponding independent variable. Substituting u( x, t ) = ∑ un ( x, t ) = ∑ bn sin pn x e − a pn t
into (6.90) we have n =1 n =1
X ′′ T′ ⎛ nπ ⎞
XT ′ = a2 X ′′ T ⇒ = 2 (6.96) ⎜⎝ pn = , n = 1, 2,3,...⎟
X aT l ⎠ (6.104)

Since the L.H. member is a function of x only and The initial conditon (6.93) will be satisfied at
the R.H. member is a function of t only, both sides t = 0 if

must be equal to the same constant l, say. So, we
u( x, 0) = f ( x ) = ∑ bn sin pn x (6.105)
obtain two ordinary differential equations
n =1

X≤ − l X = 0, T¢ − a2 l T = 0 (6.97, 6.98) that is, if f (x) can be expanded in a convergent half-


range Fourier series in (0, l). The bn are given by
Three cases arise: Case 1. l > 0; Case 2. l = 0;
Case 3. l < 0. 2 l
l ∫0
bn = f ( x ) sin pn x dx (6.106)
Case 1: l > 0. The general solution is
Note 1 Solution of partial differential equations by
lx − lx a2 lt separation of variables method cannot be applied in
u( x, t ) = X ( x ) T (t ) = (C1e + C2 e ) (C3e )
all cases. It is only a certain special set of boundary
(6.99) conditions that allows us to separate the variables.
Equation (6.99) shows that the solution has Example 6.56
unbounded temperature for large t due to exponen-
A long copper rod with insulated lateral surface has
tial growth, which is not physically possible.
its left end maintained at a temperature of 0°C and
Case 2: l = 0. The general solution is its right end at x = 2m maintained at 100°C. Find the
temperature u (x,t) if the initial condition
u (x, t) = X(x) T(t) = (ax + b) (6.100)
⎧100 x 0 < x < 1
which is independent of time. This is also not possible. u( x, 0) = f ( x ) = ⎨
⎩100 1< x < 2
Case 3: l < 0. We write l = −p 2 where p is real.
Solution We have to solve the partial differential
The general solution in this case is
equation for heat conduction
2 2
u( x, t ) = X ( x )T (t ) = ( A cos px + B sin px )(C e − a p t
) ∂u ∂2u
= a2 2 (1)
(6.101) ∂t ∂x
6-28    Engineering Mathematics-II

under the boundary conditions Superimposing the above two solutions we obtain a
u(0, t ) = 0 and u(l , t ) = u(2, t ) = 100 (2), (3) more general solution
2 2
and the initial condition u( x, t ) = 50 x + ( A cos px + B sin px ) Ce − a p t
(11)
⎧100 x 0 < x < 1⎫
u( x, 0) = f ( x ) = ⎨ ⎬ The condition u(0, t) = 0 requires that A= 0 and the
⎩100 1 < x < 2⎭ (4) condition u (l, t) = u (2, t) = 100 demands that
Assuming seperation of variables in the form 2 2
100 = 50 × 2 + BC sin pl e − a p t
⇒ sin pl = 0
u ( x , t ) = X ( x ) T (t ) (5)
This will be satisfied if
we get
X ′′ T′ np np
= 2 =l (6) pl = np ⇒ p = pn = =
X l 2 (12)
aT
n = 1, 2, 3,... (! l = 2)
where l is the separation constant. In this problem
the eigenvalue l = 0 is important. The solution for The solution, by the principle of superposition of
l = 0 is solutions, is
X ( x ) = ax + b, T (t ) = c (7) ∞
u( x, t ) = ∑ un ( x, t ) = 50 x + ∑ bn sin pn x e − a
2 2
pn t
u( x, t ) = c( ax + b) (8) n =1
(13)
The boundary conditions u (0, t) = 0 and u (2, t) = 100
imply that b = 0 and ac =50. Then This must satisfy the initial condition (4) and hence
u( x, t ) = 50 x (9) ∞
np x
f ( x ) = 50 x + ∑ bn sin (14)
Now taking up the case of exponential decay of n =1 2
temperature namely l = −p2 where p is real we have
the general solution as Expanding [f (x) − 50x] in a half-range Fourier sine
2 2
series in [0, 2] we get the solution. The Fourier coef-
u( x, t ) = ( A cos px + B sin px ) Ce − a p t
(10) ficients bn are given by

2 l np x 2 1 np x 2 2 np x
bn =
l ∫0
( f ( x ) − 50 x ) sin
l
dx = ∫ (100 x − 50 x ) sin
2 0 2
dx + ∫ (100 − 50 x ) sin
2 1 2
dx

1 2 2
⎡ 2x np x 4 np x⎤ ⎡ 2 ⎛ np x⎞ ⎛ 2⎞ ⎛ np x⎞ ⎤
= 50 ⎢ − cos + 2 2 sin ⎥ + ⎢(100 − 50 x ) np ⎜⎝ − cos ⎟⎠ + 50 ⎜⎝ ⎟⎠ ⎜⎝ − sin ⎟⎥
⎣ np 2 n p 2 ⎦ 0 ⎣⎢ 2 np 2 ⎠ ⎦⎥
1
= 0 at 0 = 0 at 0 = 0 at 2 = 0 at 2
2
100 np 200 np 2 np ⎛ 2⎞ np 400 np
=− cos + 2 2 sin + 50 × cos + 50 × ⎜ ⎟ sin = 2 2 sin (15)
np 2 np 2 np 2 ⎝ np ⎠ 2 p n 2

Finally, the solution is Example 6.57


400 ∞
1 np np x − a2 n2p 2t/4 An insulated rod of length l has its ends A and B
u( x, t ) = 50 x +
p2
∑n 2
sin
2
⋅ sin
2
e maintained at 0°C and 100°C, respectively, until
n =1 steady-state conditions prevail. If B is suddenly
(16)
Partial Differential Equations    6-29


reduced to 0°C and maintained at 0°C, find the 100
u ( x, 0) = ∑ Bn sin pn x = x, (12)
temperature at a distance x from A at time t. l
n =1
[JNTU 2003 (Set 4)]
Solution Let u(x, t) be the temperature at time t at which is the half-range Fourier series expansion in
a distance x from A. The equation for the conduction (0, l ) for the function (100/l ) x. Therefore, Bn are
of heat is given by
∂u ∂2u
= a2 2 (1) 2 l 100
l ∫0 l
∂t ∂x Bn = x sin pn x dx
where ‘a2’ is the diffusivity of the material of the rod. l
200 ⎡ ⎧ cos pn x ⎫ l cos pn x ⎤
In the steady state when u depends only on x we = 2 ⎢ ⎨ x( − )⎬ − ∫ − dx ⎥
get from (1): l ⎢⎣ ⎩ pn ⎭0 0 pn ⎥⎦
d 2u 200 ⎡ l cos pn l 1 l ⎤
=0 (2) = 2 ⎢0 − + 2 (sin pn x )0 ⎥ ( pn l = np )
dx 2 l ⎣ pn pn ⎦
whose general solution is 200 l ⋅ ( −1) n −1 200 ( −1) n −1 (13)
= ⋅ =
u( x ) = ax + b (3) l2 ⎛ np ⎞ np
⎜⎝ ⎟⎠
Boundary conditions are u(0) = 0 and u(l ) = 100. l
So we obtain b = 0 and a = 100/l. This gives The solution for the problem is
u( x ) = (100/l ) x at time t = 0. Thus, we have the
initial condition ⎛ a 2 n 2p 2 t ⎞
100 200 ∞ ( −1)n −1 np x − ⎜⎝ l2 ⎠

u( x, 0) =
l
x (4) u( x , t ) = ∑
p n =1 n
sin
l
e (14)
Boundary conditions for unsteady flow are:
u(0, t) = 0 and u(l, t) = 0 for all t (5), (6) Example 6.58
Now, we have to solve equation (1) under the condi- A homogeneous rod of conducting material of length
tions (4) – (6). A solution of (1) is of the form 100 cm has its ends kept at zero temperature, and the
2 2
temperature is initially
u( x, t ) = ( A cos px + B sin px ) e − a p t
(7)
− a 2 p 2t ⎧x 0 ≤ x ≤ 50
u(0, t ) = 0 ⇒ Ae =0 ⇒ A=0 (8) u( x, 0) = ⎨
⎩100 − x 50 ≤ x ≤ 100
Equation (7) becomes
2 2 Find the temperature u (x, t) at any time.
u( x, t ) = B sin px e − a p t
(9)
[JNTU 2004s (Set 3)]
2 2 np
u(l , t ) = 0 ⇒ B sin pl ⋅ e − a p t
=0 ⇒ p=
l Solution We have to solve the differential equa-
( B ≠ 0) (10) tion for the conduction of heat
By the principle of superposition of solutions we
may write the solution as ∂u ∂2u
= a2 2 (1)
∞ ∂t ∂x
− a2 pn2t np
u ( x, t ) = ∑ Bn sin pn x e where pn =
n =1 l under the boundary and initial conditions
(11) Boundary conditions:

Imposing the initial condition (4) on the solution 1. u (0, t) = 0 for all t (2)
(11) we have 2. u (100, t) = 0 for all t (3)
6-30    Engineering Mathematics-II

Initial condition: By the principle of superposition of solutions we


⎧x 0 ≤ x ≤ 50 ⎫ may take the solution as
u( x, 0) = ⎨ ⎬ (4)
⎩100 − x 50 ≤ x ≤ 100⎭
u( x, t ) = ∑ Bn sin pn x e − a
2 2
pn t
(8)
A solution of equation (1) may be taken as
2 2 Imposition of the initial condition (4) on (8) yields,
u ( x, t ) = ( A cos px + B sin px ) e − a p t
(5)
u (0, t) = 0 ⇒ A = 0 equation (5) becomes ⎧x 0 ≤ x ≤ 50 ⎫
u ( x, t ) = B sin px e − a2 p2t ∑ Bn sin nx = u( x, 0) = ⎨100 − x ⎬
50 ≤ x ≤ 100⎭
(6) ⎩
u(100, t ) = 0 ⇒ B sin p100 = 0 (9)
⇒ 100 p = np (! B ≠ 0)
We now expand u(x, 0) in a half-range Fourier sine
np series in (0, 100) and determine Bn .
⇒ p = pn = (7)
100 Now, Bn are given by

np x 2 ⎡ 50
x sin pn x dx + ∫ (100 − x )sin pn x dx ⎤
2 l 100
Bn =
l ∫0
u( x )sin
l
dx =
100 ⎣⎢ ∫0 50 ⎥⎦
⎡ 50
⎛ cos pn x ⎞ ⎛ sin pn x ⎞ ⎪⎫ ⎥⎤
100
2 ⎢ ⎪⎧ ⎛ cos pn x ⎞ ⎛ sin pn x ⎞ ⎪⎫ ⎪⎧
= ⎨ x − − 1 − ⎟⎬ + ⎨(100 − x ) − + − ⎟⎬
100 ⎢ ⎩⎪ ⎜⎝ pn ⎟⎠ ⎝⎜ p2n ⎠ ⎭⎪ 0 ⎩⎪ ⎜⎝ pn ⎟⎠ ⎝⎜ p2n ⎠ ⎭⎪50 ⎥
⎣ ⎦
= 0 at 0, 50 = 0 at 0 = 0 at 100, 50 = 0 at 100
⎡ ⎧ 1 at x=0
⎢ np x ⎪
⎢ cos pn x = cos =⎨ 0 at x = 50
⎢ 100 ⎪
n
⎢ ⎩( −1) at x = 100
⎢ ⎧ 0 at x = 0, 100
⎢ np x ⎪
⎢ sin pn x = sin = ⎨ np
⎢ 100 ⎪sin at x = 50
⎣ ⎩ 2
⎛ np np ⎞
sin sin 2
2 ⎜ 2 + 2 ⎟ = 4 ⋅ 100 ⋅ sin np = 400 sin np
=
100 ⎜ p2n p2n ⎟⎟ 100 p 2 n2 2 p 2 n2 2
⎜⎝ ⎠
⎧ 0, if n is even

= ⎨ 400 ( −1)n
⎪ 2 if n is odd
⎩p n2

Finally, the required solution of the problem is


Example 6.59
400 ∞ ( −1n ) (2n − 1) p x
u ( x, t ) = 2 ∑ 2
sin Find the temperature u(x, t) in a homogeneous bar
p n =1 (2n − 1) 100 of heat conducting material of length l cm with its
⎛ 2 ⎞ ends kept at zero temperature and initial tempera-
. exp ⎜ − ⎛⎜ 2n − 1⎞⎟ p 2 a2 t ⎟ (10) ture given by a x (l−x)/l2.
⎝ ⎝ 100 ⎠ ⎠
Partial Differential Equations    6-31

Solution The initial boundary value problem 2a ⎡ 2l 3 ⎤ 8a


consists of the following: = ⎢ 0 + 0 − (( −1)n − 1)⎥ = 3 3 , ( n : odd).
l ⎣
3
np
3 3
⎦ np
1. Partial differential equation for conduction of heat:
∂u ∂2u Hence the temperature distribution in the bar is
= a2 2 (1) given by
∂t ∂x

8a 1 (2n − 1)p x
2. Boundary conditions: u ( x, t ) =
p3
∑ (2n − 1)3 sin l
n =1
u(0, t) = 0, u(l, t) = 0 for all t (2), (3)
⎛ ⎞
. exp − (2n − 1) p 2 a2t
2
3. Initial condition: ⎜ ⎟
⎝ l 2

u(x, 0) = a x (l−x)/l2, 0<x<l (4)
A general solution of (1) is Example 6.60

2 2
Find the temperature in a thin metal rod of length l
u ( x, t ) = ( A cos px + B sin px ) e − a p t
(5) with both ends insulated and with initial tempera-
ture in the rod sin (p x/l ).
Boundary condition (2) is satisfied if we set A = 0
and boundary condition (3) is satisfied if Solution The initial boundary value problem
2 2 consists of the following
u(l , t ) = B sin pl e − a p t = 0 ⇒ pl = np
np 1. Partial differential equation for conduction of heat
⇒ p = pn = (! B ≠ 0) (6)
l ∂u ∂2u
By the principle of superposition of solutions we = a2 2 (1)
∂t ∂x
may write
2. Boundary conditions: (Insulation at both ends)

u( x, t ) = ∑ Bn sin pn x e − a
2 2
p t
( pn = np / l ) (7) ∂u ∂u
= 0; =0 (2), (3)
n =1 ∂x x=0 ∂x x=l

By the imposition of the initial condition (4) on (7) 3. Initial condition:


we get ⎛ px⎞
u( x, 0) = sin ⎜ ⎟ (4)
a x (l − x ) ∞ ⎝ l ⎠
u( x, 0) = = ∑ Bn sin pn x
l2 n =1
The general solution of equation (1) is of the form
A0 2 2
which is the half-range Fourier sine series in (0, l ) for u( x , t ) = + ( A cos px + B sin px ) e − a p t
(5)
2
f (x) = a x(l − x )/l 2. The constants Bn are given by
(Note: we have added the constant A0 / 2 since in
this case we have half-range Fourier cosine series
2 l 2 l a x (l − x )
Bn = ∫
l 0
f ( x )sin pn x dx = ∫
l 0 l2
sin pn x dx expansion)
Differentiating (5) we have
2a l
= 3 ∫ (lx − x 2 )sin pn x dx ∂u 2 2
l 0 = ( − pA sin px + pB cos px ) e − a p t (6)
∂x
2a ⎡ ⎛ − cos pn x ⎞
= 3 ⎢(lx − x 2 ) ⎜ ⎟⎠ Boundary condition (2) is satisfied if we set B = 0.
l ⎣ ⎝ pn
Also, boundary condition (3) is satisfied if
l
⎛ − sin pn x ⎞ cos pn x ⎤
− (l − 2 x ) ⎜ ⎟ + ( −2) ⎥ np
⎝ pn ⎠
2
pn3 ⎥⎦ sin pl = 0 (! A ≠ 0) ⇒ p = pn = (7)
0 l
6-32    Engineering Mathematics-II

Therefore, by the principle of superposition, the Example 6.61


solution may be taken as A homogeneous rod of conduction material of length
l has its ends kept at zero temperature. The tempera-
A0 ∞
+ ∑ An cos pn x ⋅ e − a pn t ture at the centre is T and it falls uniformly to zero at
2 2
u( x , t ) = (8)
2 n =1 the two ends. Find the temperature at any time t.
Here Solution The initial boundary value problem
l consists of solving the partial differential equation
⎛ px ⎞
cos ⎟ for heat conduction:
2 l 2 l px 2⎜ l
l ∫0
A0 = f ( x ) dx = ∫ sin dx = ⎜ − ⎟
l 0 l l ⎜ ⎛p⎞ ⎟ ∂u ∂2u
⎜ ⎟
⎜⎝ ⎝ l ⎠ ⎟⎠ = c2 2
0 ∂t ∂x (1)
4 l 4 (9)
= ⋅ = under the boundary conditions
l p p
2 l px 2 l px px u (0, t ) = 0 = u (l , t ) (2), (3)
l ∫0
A1 = f ( x ) cos dx = ∫ sin cos dx
l l 0 l l
and the initial condition that u (x,0) falls uniformly
l
1⎛ px⎞ to zero at the ends.
= ⎜ sin2 ⎟ = 0 (10)
p⎝ l ⎠0 To find this condition we have to solve the steady-
state equation: d 2 u/dx 2 = 0 whose general solution is
n ≠1
2 l np x 2 l px np x
l ∫0
An = f ( x ) cos dx = ∫ sin cos dx u ( x ) = ax + b (4)
l l 0 l l
1 l ⎡ ( n + 1)p x ( n − 1)p x ⎤ If C = l/2 is the centre of the rod then for the portion
= ∫ ⎢sin − sin ⎥⎦ dx
l 0⎣ l l
AC of the rod we have
1 ⎡ −l ( n + 1)p x
= ⎢ ⋅ cos at A : u (0) = a.0 + b = 0 ⇒ b = 0 ⎫
l ⎣ ( n + 1)p l ⎪
⎛l ⎞ l 2T ⎬ so that
px⎤
l and at C : u ⎜ ⎟ = a = T ⇒ a = ⎪
l ⎝2⎠ 2 l ⎭
+ ⋅ cos( n − 1) ⎥
( n − 1)p l ⎦0
2T l⎫
1⎡ 1 ⎛ ⎞ 1 ⎛ ⎞⎤ u ( x) = x, 0 ≤ x≤ ⎬ (5)
= ⎢ ⎜ 1 − ( −1) n +1 ⎟ − ⎜ 1 − ( −1) n −1 ⎟ ⎥ l 2⎭
p ⎣n +1⎝ ⎠ n −1 ⎝ ⎠⎦
2⎛ 1 1 ⎞ 4 For the portion CB of the rod we have
= ⎜⎝ − ⎟⎠ = − , (n even)
2 at B : u (l ) = al + b = 0 ⇒ b = − al
p n +1 n −1 p ( n − 1) ⎫

4 1 1 2T ⎬
=− (11) and at C : u (l 2) = T = a − al = − al ⇒ a = − ⎪
p (4 n2 − 1) 2 2 l ⎭
2T l
so that u ( x ) = − x + 2T, ≤ x≤l (6)
The temperature distribution in the rod is l 2

2 4 ∞ Consequently the initial condition is


1 ⎛ 2np x ⎞
u( x , t ) = − ∑ cos ⎜
p p n =1 4n2 − 1 ⎝ l ⎟⎠ ⎧ 2T l
⎪⎪ l x, 0≤ x≤
2
⎛ 4 n2 a 2p 2 ⎞ u ( x, 0) = ⎨ (7)
exp ⎜ − t⎟ (12) 2T
⎪ (l − x ), l
⎝ l2 ⎠ ≤ x ≤ l.
⎪⎩ l 2
Partial Differential Equations    6-33

By the method of separation of variables the solution Finally, the solution of the problem is
of (1) may be put in the form
8T ∞
( −1)n nπ x ⎛ n 2π 2 c 2 t ⎞
u (x, t) = (A cos px+ B sin px) e − c 2 p2t (8) u ( x, t ) =
π2
∑ n2
sin
l
exp ⎜ −
⎝ l2 ⎠

n =1
Boundary condition (2) is satisfied if we set (14)
A = 0, and boundary conditon (3) is satisfied if we
set sin pl = 0
EXERCISE 6.7
⇒ p = pn = np / l (9)
∂q ∂ 2q
Thus, by the principle of superposition of solutions, 1. Solve = k 2 such that
we may write the general solution as ∂t ∂x
∞ (a) q is finite as t → ∞.
2 2
u ( x, t ) = ∑ Bn sin pn x. e − c pn t (10)
(b) ∂q /∂x = 0 when x = 0 and q = 0 when x = l for all t.
n =1
Imposing the initial condition (7) we must have (c) q = q0 when t = 0 for all x in 0 < x < l.
⎧ 2T l 4q 0 ⎡ 1
⎪⎪ l x, 0≤x≤ 2 2

2 Ans: q = ⎢ cos p1x , e − p1 kt − cos p3 x , e − p3 kt
u ( x, 0) = ∑ Bn sin pn x = ⎨ p ⎣ 3
2T
⎪ (l − x ), l 1
n =1 ≤ x ≤l 2 ⎤⎛ np ⎞
⎪⎩ l 2 + cos p5 x.e − p5 kt − ! ⎥⎦ ⎜⎝ pn = l ⎟⎠
(11) 5
Equation (11) is a half-range Fourier sine series
expansion in (0, l) for the function u (x, 0) and so the 2. A bar 10 cm long with insulated sides has its ends A
constants Bn are given by and B maintained at temperatures 50°C and 100°C,
respectively, until steady-state conditions prevail. The
2 l 2 l /2 2T
Bn = ∫
l 0
u ( x )sin pn x dx = ∫
l 0 l
x sin pn x dx temperature at A is suddenly raised to 90°C and at the
same time that at B is lowered to 60°C. Find the tem-
2 l 2T nπ ⎞ perature distribution in the bar at time t.

l ∫l /2 l
+ (l − x )sin pn x dx , ⎜ pn = ⎟ [Mysore 1997, Warangal 1996]
⎝ l ⎠
l /2 80 ∞ 1 npx
4T ⎡ ⎛ − cos pn x ⎞ ⎛ − sin pn x ⎞ ⎤ Ans: u = ( x, t ) = 90 − 3x − ∑ sin 5 ⋅ exp( − n2π 2c2t/25)
π n =1 n
= 2 ⎢x ⎜ ⎟⎠ − 1⎜ ⎟⎥
l ⎣⎢ ⎝ pn ⎝ pn2 ⎠ ⎦⎥
0
l 3. Solve ∂u/∂t = ∂ 2u/∂x 2 with boundary conditions u(x ,0) =
4T ⎡ ⎛ cos pn x ⎞ ⎛ sin pn x ⎞ ⎤ 3 sin n π x, u(0, t) = 0, u(l, t) = 0 when 0 < x < 1, t > 0.
+ 2 ⎢(l − x )⎜ − ⎟ + 1⋅ ⎜ − ⎟⎥
l ⎣⎢ ⎝ pn ⎠ ⎝ pn2 ⎠ ⎦⎥ [Osmania 1995, Kerala 1990]
l /2

4T ( −1)n 4T ( −1)n 8T ( −1)n Ans: u ( x, t ) = 3 ∑ sin np x exp( − n2 p 2t )
= 2 ⋅ 2 2 l2 + 2 ⋅ 2 2 ⋅ l2 = 2 ⋅ 2 n =1
l np l np p n
(13)
sin pn l = sin np = sin 0 = 0 4. The ends A and B of a rod 20m long have temperatures
at 30°C and 80°C, respectively, until steady-state condi-
pn l np
sin = sin = ( −1)n tions prevail. The temperatures of the ends are changed
2 2 to 40°C and 60°C, respectively. Find the temperature
cos pn l = cos np = ( −1)n distribution in the rod at time t.
pn l np [Kerala 1995, Madras 1991]
cos = cos =0
2 2 20 ∞ 2( −1)n + 1
cos 0 = 1 Ans: u ( x, t ) = 40 + x − ∑ n ×
p n =1
6-34    Engineering Mathematics-II

6.14 ONE-DIMENSIONAL WAVE EQUATION


sin
np x
20
(
exp −c 2n2 p 2 t 400 )
6. 14.1 Introduction
5. Find the solution of one-dimensional heat equation
Consider the vibrations of an elastic string placed
∂u/∂t = c 2 (∂ 2u/∂x 2 ) under the boundary conditions
along the x-axis, stretched to length l between two
u (0, t) = u (l, t) = 0 and the initial conditions u (x, 0) =
fixed points x = 0 and x = l. First we consider the prob-
x, 0 < x < l, l being the length of the rod.
lem when there is an initial displacement but no initial
velocity (string released from rest). Next we consider
2l ( −1)n −1
Ans: u ( x, t ) =
p
∑ n sin pn x exp( − pn2c2t ) motion of a string with an initial velocity but no initial
displacement (string given an initial blow, but from its
( pn = np /l ) horizontal stretched position). Finally, we consider the
case of both initial velocity and initial displacement.
6. Solve ∂u/∂t = c 2 (∂ 2u/∂x 2 ) subject to the conditions: 6. 14.2 Vibrating String with Zero
(a) u (0, t) = 0 Initial Velocity
∂u Consider an elastic string of length l, fastened at its
(b) (l , t ) = 0 ends on the x-axis at x = 0 and x = l. The string is
∂t
u x displaced, then released from rest to vibrate in the
(c) u ( x,0) = 0 , 0 ≤ x ≤ l. xy-plane. We want to find the displacement function y
l
(x, t), whose graph is a curve in the xy-plane showing
∂u0 ∞ ( −1)n −1 ⎛ 2n − 1 ⎞ the shape of the string at time t. The boundary value
Ans: u ( x, t ) = ∑ sin ⎜
x 2 n =1 (2n − 1) ⎝ 2l
p x⎟
⎠ problem for the displacement function y (x, t) consists
in the solution of the partial differential equation
⎛ ⎛ 2n − 1⎞ 2 2 2 ⎞
exp ⎜ − ⎜ ⎟ p c t⎟ . ∂2 y ∂2 y
⎝ ⎝ 2l ⎠ ⎠ 2
= a2 for 0 < x < l , t > 0 (6.107)
∂t ∂x 2
under the boundary conditions:
7. The ends A and B of a rod 30cm long have their tem-
peratures kept at 20°C and 80°C until steady-state y (0, t ) = y (l , t ) = 0 for t ≥ 0 (6.108, 6.109)
conditions prevail. The temperature of the end B is
suddenly reduced to 60°C and kept so while the end A and the initial conditions:
is raised to 40°C. Find the temperature distribution of
y ( x, 0) = f ( x )⎫ for 0 ≤ x ≤ l (6.110)
the rod at time t. ⎪
∂y ⎬
2 x 40 ∞ 1 npx ( x, 0) = 0 ⎪ (6.111)
Ans: u ( x, t ) = 40 + − ∑ sin ∂t ⎭
3 p n =1 n 15
The graph of f (x) is the position of the string before
exp( − n2p 2c 2t /225) release.
The separation of variables method consists of
attempting a solution of the form y(x,t) = X(t) T(t) where
8. Find the solution of ∂u/∂t = c 2 (∂ 2u/∂x 2 ) under the
X(x) is a function of x only and T(t) is a function of t
conditions
only. Substituting into the wave equation we obtain
(a) u (0, t ) = 0 = u (l , t ) for all t .
XT ′′ = a2 X ′′ T (6.112)
np x
(b) u ( x ,0) = 2sin for all x.
l where ′ denotes differentiation with respect to the

npx respective independent variable. Then
Ans: u ( x , t ) = 2 ∑ sin exp( − n2p 2c 2t /l ).
l X ′′ T ′′
n =1 = 2 (6.113)
X aT
Partial Differential Equations    6-35

The left-hand member is a function of x only and yn ( x, t ) = Cn sin ln x cos ln at


the right-hand member is a function of t only. The
equality is possible only if both the quantities are ⎛ np ⎞
⎜⎝ ln = ; n = 1, 2, 3, ...⎟
⎠ (6.122)
equal to the same constant, which we take as −l2 < 0 l
for convenience. This is called the separation
Each of these functions satisfies the wave equation,
constant. (Taking the separation constant as 0 or
both boundary conditions and the initial condition
positive real number leads to trivial solutions.)
∂y/∂t ( x, 0) = 0 . We need to satisfy the condition
We now have
y (x, 0) = f (x). This is achieved by an infinite super-
X ′′ T ′′ position of solutions in the form
= 2 = −l 2 (6.114)
X aT ∞ ∞
y( x, t ) = ∑ yn ( x, t ) = ∑ Cn sin ln x cos ln at
yielding two ordinary differential equations n =1 n =1
⎛ np ⎞
X ′′ + l 2 X = 0 and T ′′ + a2 l 2T = 0 ⎜⎝ ln = ; n = 1, 2, 3, ...⎟

(6.123)
l
(6.115, 6.116) we must choose the Cn’s to satisfy
The boundary conditions y (0, t) = 0, and y (l, t) = 0 ∞ ∞
yield X(0) = 0 and X(l ) = 0, respectively. The gen- y( x, 0) = ∑ yn ( x, 0) = ∑ Cn sin ln x = f ( x )
eral solutions of equations are n =1 n =1
(6.124)
X ( x ) = A cos l x + B sin l x (6.117) This series is the half-range Fourier sine series of
f (x) in [0, l ]. The Fourier constants are given by
T (t ) = C cos al t + D sin al t (6.118)
2 l np x
The condition X(0) = 0 implies that A = 0 and the
Cn =
l ∫0
f ( x )sin
l
dx (6.125)

condition X(l ) = 0 imples that sin l l = 0 ( B ≠ 0)


Therefore, ll = np or l = ln = np /l, which are Example 6.62
the eigenvalues of the problem. The corresponding A string AB of length l is fastened at both ends A and
eigenfunctions are B. At a distance ‘b’ from the end A, the string is tran-
seversely displaced to a distance ‘d’ and released
X n ( x ) = Bn sin ln x ( n = 1, 2, 3,...) (6.119) from rest when it is in this position. Find the solu-
tion for the initial displacement function and zero
The initial condition (the string is released from rest) initial velocity.
Solution: Let y (x, t) be the displacement of the
∂y
( x, 0) = 0 gives X ( x )T ′(0) = 0 ⇒ T ′(0) = 0 string. The initial displacement is given by APB.
∂t
(6.120) y

Hence
P(b,d )
− aln (C sin aln t − D cos aln t ) =0 ⇒ D=0
t =0

Therefore, we obtain
x
T (t ) = Tn (t ) = Cn cos aln t ( n = 1, 2, 3, ...) (6.121) A(0,0) M (b,0) B(l,0)

Figure 6.2 String with transverse displacement d at a


Now we take the solutions for the problem as
point M (b, 0)
6-36    Engineering Mathematics-II

Equation of AP is y = ( d /b) x (1) and initial conditions


d( x − l)
Equation of PB is y = (2) ⎧d ⎫
b−l ⎪⎪ b x for 0 ≤ x ≤ b⎪

The problem is to solve one-dimensional wave y ( x, 0) = f ( x ) = ⎨ ⎬
⎪ d( x − l) for b ≤ x ≤ l ⎪
equation
⎩⎪ b − l ⎭⎪ (6)
∂2 y ∂2 y
=a 2
(3) The solution is given by
∂t 2 ∂x 2

np x np at
with boundary conditions y( x, t ) = ∑ Cn sin cos
n =1 l l (7)
y (0, t ) = 0 = y (l , t ) (4), (5)
where Cn are given by

2 l np x 2 ⎡ bd np x l d( x − l) np x ⎤
l ∫0
Cn = f ( x )sin dx = ⎢ ∫ x sin dx + ∫ sin dx ⎥
l l ⎣ 0b l b b−l l ⎦
b l
2d ⎡ ⎛ − l ⎞ np x ⎛ −l 2 ⎞ np x⎤ 2d ⎡ ⎛ −l ⎞ np x ⎛ −l 2 ⎞ np x⎤
= ⎢ x ⎜ ⎟ cos − ⎜ 2 2 ⎟ sin ⎥ + ⎢( x − l ) ⎜ ⎟ cos − ⎜ 2 2 ⎟ sin ⎥
bl ⎣⎢ ⎝ np ⎠ l ⎝n p ⎠ l ⎦⎥ l (b − l ) ⎣⎢ ⎝ np ⎠ l ⎝n p ⎠ l ⎦⎥
0 b

−2dbl np b 2dl 2 np b 2d np b 2dl 2 np b


= cos + 2 2 2 sin + cos − sin
bl np l bl n p l np l l (b − l )n p
2 2
l
2dl 2 np b
= sin
b(l − b)n p 2 2
l (8)

Hence, the displacement of the string at any point x ∂2 y ∂2 y


and time t is given by = a2 (1)
∂t ∂x 2
2

2dl 2 1 np b np x np at
y ( x, t ) =
b(l − b)p 2 ∑ n2 sin l
⋅ sin
l
⋅ cos
l
under the boundary conditions
n =1 y(0, l ) = 0 = y(l , t ) (2), (3)
(9)
Example 6.63
and the initial conditions:
A string of length l fastened at both ends A = (0,0) Initial displacement:
and B = (l, 0) undergoes initially a transversal dis- ⎧x for 0 ≤ x ≤ l/ 2⎫
placement given by y( x, 0) = f ( x ) = ⎨ ⎬
⎩ l − x for l/2 ≤ x ≤ l ⎭ (4)
⎧x for 0 ≤ x ≤ l/ 2
f ( x) = ⎨ ∂y
⎩l − x for l/2 ≤ x ≤ l Initial velocity: ( x, 0) = g ( x ) = 0 (5)
∂t
and is released at rest when it is in this position. Find The solution of equation (1) under the boundary
the displacement function y(x, t) for the subsequent conditions (2) and (3) and the zero initial velocity is
motion. ∞
np x np at
Solution The problem consists of solving the y( x, t ) = ∑ Cn sin cos (6)
n =1 l l
wave equation
Partial Differential Equations    6-37

where displacement but with an initial velocity given at x


y
by g (x). The boundary value problem for the dis-
placement function is

l /2 P (l /2, l /2) ∂2 y ∂2 y
= a2 for 0 ≤ x ≤ l , t > 0 (6.126)
∂t 2 ∂x 2

y(0, t ) = 0 = y(l , t ) for t > 0 (6.127, 6.128)


A (0,0) l /2 B (l ,0) x
zero initial
Figure 6.3 String with transverse displacement at displacement: y( x, 0) = 0 ⎫ for 0 ≤ x ≤ l
prescribed ⎪
midpoint ∂y ⎬
initial velocity: ( x , 0) = y ( x ) ⎪⎭ (6.129, 6.130)
2 l np x ∂t
Cn =
l 0∫ f ( x )sin
l
dx

2 l /2 np x 2 l np x
= ∫ x sin dx + ∫ (l − x )sin dx By the method of separation of variables we set
l 0 l l l /2 l y (x, t) = x (t) T (t) and obtain ordinary differential
l /2 equations
2 ⎡ ⎛ −l ⎞ np x ⎛ −l 2 ⎞ np x ⎤
= ⎢ x ⎜ ⎟ cos − ⎜ 2 2 ⎟ sin ⎥
l ⎣⎢ ⎝ np ⎠ l ⎝n p ⎠ l ⎦⎥ X ′′ + l 2 X = 0 and T ′′ + a2 l 2T = 0
0
= 0 at 0 = 0 at 0 (6.131, 6.132)
l The boundary conditions are same as before and
2⎡ ⎛ −l ⎞ np x ⎛ −l 2 ⎞ np x ⎤
+ ⎢(l − x ) ⎜ ⎟ cos − ( −1) ⎜ 2 2 ⎟ sin ⎥ hence we obtain eigenvalues
l ⎢⎣ ⎝ np ⎠ l ⎝n p ⎠ l ⎥⎦
l /2
n 2p 2
= 0 at l = 0 at l ln2 = (6.133)
l2
2⎡l ⎛ l ⎞ np l2 np ⎤
= ⎢ ⎜⎝ − ⎟⎠ cos + 2 2 sin ⎥ and the corresponding eigenfunctions are constant
l ⎣ 2 np 2 np 2 ⎦ multiples of
2⎡l l np l2 np ⎤ np x
+ ⎢ ⋅ cos + 2 2 sin ⎥ X n = sin (6.134)
l ⎣ 2 np 2 np 2 ⎦ l
⎧ 0 if n is even⎫ with the values of l as l = ln = np /l the differen-
4l np ⎪ ⎪
= 2 2 sin = ⎨ 4l ( n −1)/2 ⎬ tial equation for T becomes
np 2 ⎪ 2 2 ( −1) if n is odd ⎪
⎩n p ⎭
(7) n 2p 2
T ′′ + a2 T =0 (6.135)
The solution for the initial displacement given by l2
(4) and zero initial velocity is whose general solution is
∞ m −1
4 ( −1) (2m − 1)p x (2m − 1)p at T (t ) = A cos( npat /l ) + B sin(npat /l )
p m∑
y( x, t ) = sin cos
=1 (2m − 1)
2
l l
(8) The initial condition of zero initial displacement
gives
6.14.3 Vibrating String with Given
Initial Velocity and Zero Initial y (x, 0) = 0 = X (x) T (0) ⇒ T (0) = 0 (6.136)
Displacement Since T (0) = B = 0 solutions for T (t) are constant
Next, we consider the case when the string is multiples of sin npat /l . Thus, for n = 1, 2, 3, ... we
released from its horizontal position with zero initial have functions
6-38    Engineering Mathematics-II

np x np at under the boundary conditions:


yn ( x, t ) = Cn sin sin (6.137)
l l y(0, t ) = 0 = y(2l , t ) (2), (3)
and the initial conditions:
Each of these functions satisfies the wave equation,
the boundary conditions and the zero initial dis- initial displacement: y( x, 0) = f ( x ) = 0 (4)
placement condition. In order to satisfy the initial and initial velocity:
velocity condition (∂y/ ∂t )( x, 0) = g ( x ) we invoke
⎧x /l for 0 ≤ x ≤ l ⎫
the superposition principle and write g ( x, 0) = ⎨ ⎬ (5)
∞ ∞
⎩2l − x / l for l ≤ x ≤ 2l ⎭
np x np at
y( x, t ) = ∑ yn ( x, t ) = ∑ Cn sin sin The solution is
l l
n =1 n =1
2l ∞ bn np x np at
(6.138) y( x, t ) = ∑
p a n =1 n
sin
2l
sin
2l
(6)
We assume that the series admits of term-by-term where
differentiation. So, we get
2 2l np x
2l ∫0
∂y ∞
⎛ np a ⎞ ⎛ np x ⎞ ⎛ np at ⎞ bn = g ( x )sin dx
( x , t ) = ∑ Cn ⎜ sin ⎜ cos ⎜ 2l
⎝ l ⎠ ⎟ ⎝ l ⎠ ⎟ ⎝ l ⎟⎠
∂t n =1 1 lx np x 1 2l 2l − x np x
= ∫ sin dx + ∫ sin dx
(6.139) l l0 2l l l l 2l
l
Now, the initial velocity condition yields 1⎡ x ⎛ 2l ⎞ np x 1 4l 2 np x ⎤
= ⎢ ( −1) ⎜ ⎟ cos − ( −1) 2 2 sin ⎥

l⎣l ⎝ np ⎠ 2l l np 2l ⎦ 0
∂y ⎛ np a ⎞ np x
( x, 0) = ∑ Cn ⎜ ⎟⎠ sin ⋅1 = g ( x ) (6.140)
∂t ⎝ l l 1 ⎡ 2l − x −2l ⎛ np x ⎞
+ ⎢ ⋅ ⋅ cos ⎜
⎝ 2l ⎟⎠
n =1
l⎣ l np
This is the half-range Fourier series expansion of 2l
g(x) on [0, l ]. Here, the entire coefficient of sin np x/l ⎛ −1⎞ − 4l
2
np x ⎤
− ⎜ ⎟ . 2 2 ⋅ sin ⎥
is the Fourier sine coefficient of g(x) on [0, l ] so that ⎝ l ⎠ np 2l ⎦ l
we have
2 npl 4 npl
np a 2 l np x =− cos + 2 2 sin
Cn ⋅ = ∫ g ( x )sin dx np 2l n p 2l
l l 0 l
2 np 4 np
2 l np x + cos + sin
or Cn =
np a ∫0
g ( x )sin
l
dx (6.141) np 2 n 2p 2 2
8 np
= 2 2 sin (7)
Example 6.64 np 2
Find the displacement y(x, t) of a string stretched Hence, the displacement function is given by
between two fixed points at a distance 2l apart when

the string is initially at rest in equilibrium position 2l 8 1 1 np np x np at
and the points of the string are given initial velocity
y( x, t ) = ⋅
pa p 2
∑ n ⋅ n2 sin 2
⋅ sin
2l
sin
2l
n =1
g(x), given by ∞
16l 1 np np x np at
⎧ x/l for 0 ≤ x ≤ l ⇒ y( x, t ) =
p 3
a
∑ n3 sin 2
⋅ sin
2l
sin
2l
g( x) = ⎨ n =1
(8)
⎩(2l − x )/l for l ≤ x ≤ 2l
Example 6.65
Solution We have to solve the wave equation
A tightly stretched string with fixed end points is
∂2 y ∂2 y
=a 2
(1) initially at rest in its equilibrium position, and each
∂t 2 ∂x 2 of its points is given a velocity v, which is given by
Partial Differential Equations    6-39

⎧ l ⎧ l⎫
⎪⎪ Cx for 0 ≤ x ≤
2 ∂y ⎪⎪ Cx for 0 ≤ x ≤ ⎪
2⎪
v ( x) = ⎨ ( x, 0) = v( x ) = ⎨ ⎬ (5)
l ∂t ⎪C (l − x ) for l
⎪C (l − x ) for ≤x≤l ≤ x ≤ l⎪
⎪⎩ 2 ⎩⎪ 2 ⎭⎪
Find the displacement y (x, t). Solving equation (1) by the method of separation of
[JNTU 1994S, 2001, 2002] variables under the conditions is given by
Solution We have to solve the wave equation ∞ ∞
np x np at
y ( x, t ) = ∑ yn ( x, t ) = ∑ Cn sin sin (6)
∂ y
2
∂ y
2
n =1 n =1 l l
=a 2 (1)
∂t 2 ∂x 2 Differentiating (6) partially with respect to ‘t’ we obtain
under the boundary conditions:

∂y np a np x np at
y (0, t ) = 0 = y (l , t ) for t ≥ 0 (2), (3) ( x, t ) = ∑ Cn sin cos (7)
∂t n =1 l l l
and the initial conditions: ∞
∂y ⎛ np a ⎞ np x
zero initial displacement: ⇒ ( x, 0) = ∑ ⎜ Cn ⎟ sin ⋅1 = v ( x ), (8)
∂t n =1
⎝ l ⎠ l
y ( x, 0) = 0 (0 ≤ x ≤ l ) (4)
This is the half-range Fourier sine series expansion
prescribed initial velocity: for v (x) in [0, l ]. Therefore, we have

np a 2 l np x
⋅ Cn = ∫ v ( x )sin dx
l l 0 l
2 l /2 np x 2 l np x
= ∫ Cx sin dx + ∫ C (l − x )sin dx
l 0 l l l /2 l
l /2
2C ⎡ ⎛ l ⎞ np x ⎛ l2 ⎞ np x ⎤
= ⋅ ⎢ x ⎜ − ⎟ cos − 1⋅ ⎜ − 2 2 ⎟ sin ⎥
l ⎣⎢ ⎝ np ⎠ l ⎝ np ⎠ l ⎦⎥
0
= 0 at 0 = 0 at 0
l
2C ⎡ ⎛ l ⎞ np x ⎛ l2 ⎞ np x ⎤
+ ⎢(l − x ) ⎜ − ⎟ cos − ( −1) ⎜ − 2 2 ⎟ sin ⎥
l ⎝ np ⎠ l ⎝ np ⎠ l ⎦⎥
⎣⎢ l /2
= 0 at l = 0 at l
⎡ l
2C np
2
l np ⎤ 2
= ⎢− cos + 2 2 sin ⎥
l
⎣ 2np 2 np 2 ⎦
2C ⎡ l 2 np l2 np ⎤
− ⎢− cos − 2 2 sin ⎥
l ⎣ 2np 2 np 2 ⎦
4C l 2 np 4Cl np
= ⋅ 2 2 sin = 2 2 sin
l np 2 np 2
l 4Cl np 4l 2 C np
⇒ Cn = ⋅ 2 2 sin = 3 3 sin (9)
np a n p 2 an p 2
Hence, the solution of the problem is
6-40    Engineering Mathematics-II

np This is the half-range Fourier series expansion for


∞ sin g (x) in [0, l ]. Therefore, we have
4Cl 2 2 sin np x cos np at
y( x, t ) =
ap 3
∑ n3
l l
(10)
n =1 np a 2 l np x
Cn = ∫ g ( x )sin dx
l l 0 l
Example 6.66 2 l ⎛ np x ⎞ np x
= ∫ x ⎜1 + cos ⎟⎠ sin dx
Find the solution for the above problem when the l 0 ⎝ l l
string is released from its horizontal position with 2 l np x 2 l np x np x
an initial velocity given by g ( x ) = x(1 + cos p x/l ). = ∫ x sin dx + ∫ x sin cos dx
l 0 l l 0 l l
Deduce the result for a = 1 and l = p. (5)

Solution Following the procedure of the above For evaluation of the second intergral, we have to
problem, the solution in the present case is obtained as separate the cases.
Case (1) n = 1 and Case (2) n ≠ 1
∞ ∞
np x np at
y( x, t ) = ∑ yn ( x, t ) = ∑ Cn sin sin (1) Case 1
n =1 n =1 l l
n=1
pa 2 l px 2 l px px
The prescribed initial velocity in this case is C1 = ∫ x sin dx + ∫ x sin cos dx
l l 0 l l 0 l l
∂y ⎛ p x⎞ 2 l px 1 l 2px
( x, 0) = g ( x ) = x ⎜1 + cos ⎟ (2) = ∫ x sin dx + ∫ x sin dx
∂t ⎝ l ⎠ l 0 l l 0 l
l
2⎡ ⎛ l ⎞ px ⎛ l2 ⎞ px ⎤
Differentiating (1) partially with respect to ‘t’ = ⎢ x ⎜ − ⎟ cos − 1⋅ ⎜ − 2 ⎟ sin ⎥
l⎣ ⎝ p ⎠ l ⎝ p ⎠ l ⎦0
we have
= 0 at 0 = 0 at l , 0

∂y ⎛ np a⎞ np x np at
( x, t ) = ∑ ⎜ ⎟ Cn sin cos (3) l
∂t ⎝ l ⎠ l l 1⎡ ⎛ l ⎞ 2px ⎛ l 2 ⎞ 2px ⎤
n =1 + ⎢ x ⎜ − ⎟ cos − ⎜ − 2 ⎟ sin ⎥
l⎣ ⎝ 2p ⎠ l ⎝ 4p ⎠ l ⎦0
Imposing the initial veloctiy condition (2) we obtain = 0 at 0 = 0 at l , 0

∂y ⎛ np a ⎞ np x px 2⎡ l 2 ⎤ 1⎡ l ⎤ 2l
2
l 3l
( x, 0) = ∑ ⎜ Cn ⎟ sin = g ( x ) = x + x cos
∂t ⎝ l ⎠ l l = ⎢ − ( −1)⎥ + ⎢ − ⋅1⎥ = − =
n =1 l⎣ p ⎦ l ⎣ 2p ⎦ p 2p 2p
(4)
3l 2
⇒ C1 = (6)
2ap 2

Case 2
n≠1
npa 2 l npx 2 l npx px
Cn = ∫ x sin dx + ∫ x sin cos dx
l l 0 l l 0 l l
2 l npx 1 ⎡l ( )
n + 1 px ( x − 1)px ⎤
= ∫ x sin dx + ∫ ⎢ x sin dx + x sin dx ⎥
l 0 l l 0⎣ l l ⎦
Partial Differential Equations    6-41

l
2⎡ ⎛ l ⎞ npx ⎛ l2 ⎞ npx ⎤
= ⎢ x ⎜ − ⎟ ⋅ cos − 1⋅ ⎜ − 2 2 ⎟ sin ⎥
l⎣ ⎝ np ⎠ l ⎝ np ⎠ l ⎦0
= 0 at 0 = 0 at l , 0
l
1⎡ ⎛ l ⎞ (n + 1) px ⎛ l2 ⎞ (n + 1) px ⎤
+ ⎢x ⎜ − ⎟ cos − 1⋅ ⎜ − ⎟ sin ⎥
l ⎣⎢ ⎝ (n + 1) p ⎠ l ⎝ (n + 1)2 p 2 ⎠ l ⎦⎥ 0
= 0 at 0 = 0 at l , 0
l
1⎡ ⎛ l ⎞ (n − 1) px ⎛ l2 ⎞ (n − 1) px ⎤
+ ⎢x ⎜ − ⎟ cos − 1 ⎜ − ⎟ sin ⎥
l ⎢⎣ ⎝ (n − 1) p ⎠ l ⎝ (n − 1) p ⎠
2 2 l ⎥⎦ 0
= 0 at 0 = 0 at l , 0
2 ⎡ l2 n⎤ 1⎡ l2 n +1 ⎤ 1 ⎡ l2 n −1 ⎤
= ⎢− ⋅ ( −1) ⎥ + ⎢ − ⋅ ( −1) ⎥ + ⎢ − ⋅ ( −1) ⎥
l ⎣ np ⎦ l ⎣ (n + 1) p ⎦ l ⎣ ( n − 1) p ⎦
n n n
2l ( −1) l ( −1) ⎛ 1 1 ⎞ 2l ( −1) 1
=− + ⎜⎝ + ⎟= ⋅ 2
np p n + 1 n − 1⎠ pn n −1
n
2l 2 ( −1)
⇒ Cn = (n ≠ 1) (7)
ap 2 n2 (n2 − 1)

Therefore, the solution for g (x) = x (1 + cos p x/l ) as and initial velocity g (x). Let y1 (x, t) and y2 (x, t) be
the initial velocity function is the respective solutions of the two problems and let

3l 2 px pat 2l 2 ∞ ( −1)n y( x, t ) = y1 ( x, t ) + y2 ( x, t ) (6.142)


y (x , t ) =
2ap 2
sin
l
sin
l
+ 2
ap
∑ n2 (n2 − 1)
n= 2 Then y satisfies the wave equation and the boundary
npx npat conditions. Further,
sin sin (8)
l l
y( x, 0) = y1 ( x, 0) + y2 ( x, 0) = f ( x ) + 0 = f ( x )
By taking a = 1 and l = p in the above result, we (6.143)
obtain the solution for this particular case as
and
n
3 ∞ ( −1)
y (x, t ) = sin x sin t + 2 ∑ 2 2 sin nx sin nt ∂y ∂y ∂y
2 n = 2 n (n − 1) ( x, 0) = 1 ( x, 0) + 2 ( x, 0) = 0 + g ( x ) = g ( x )
(9) ∂t ∂t ∂t
(6.144)
6.14.4 Vibrating String with Thus, y (x, t) is the solution in the case of nonzero
Initial Displacement and initial displacement and velocity functions.
Initial Velocity
Consider the motion of the string with both initial Example 6.67
displacement given by f (x) and initial velocity given An elastic string of length l, fastened at its ends on
by g (x). We have to now solve two separate prob- the x-axis at x = 0 and x = l, is given initial displace-
lems, one with initial displacement f (x) and zero ment f (x) and initial velocity g (x). Find the displace-
initial velocity and the other with zero displacement ment function y (x, t) by solving the wave equation
6-42    Engineering Mathematics-II

ytt = a2yxx under the conditions: y (0, t) = y (l, t) = 0, 8C ∞ 1 np np x np at


y (x, 0) = f (x) = x for Ans: y( x, t ) = 2∑ 2
sin sin cos .
p n =1 n 2 l l
⎧x for 0 ≤ x ≤ l /2⎫
px
y( x, 0) = ⎨ ⎬ (c) Initial velocity: g ( x ) = b sin3
⎩l − x for l /2 ≤ x ≤ l ⎭ l
and Ans:
bl ⎡ px p at 3p x 3p at ⎤
⎧ ⎛ px⎞ g ( x, t ) = ⎢⎣9sin l sin l − sin l sin l ⎥⎦ .
yt ( x, 0) = g ( x ) = ⎨ x ⎜1 + cos ⎟ . 12ap
⎩ ⎝ l ⎠
3p x 2p x
(d) Initial velocity: g ( x ) = b sin cos
Solution From equations (6.143) and (6.144), we l l
obtain the solution for the present problem as Ans:
bl px p at bl 5p x 5p at
∞ m −1 y( x, t ) = sin sin + sin sin .
4l ( −1) (2m − 1)p x 2ap l l 5ap l l
y( x, t ) = 2 ∑ ⋅ sin
p m=1 (2m − 1)2 l
(2m − 1)p at 3l 2 px p at 6.15 LAPLACE’S EQUATION OR POTENTIAL
⋅ cos + sin sin EQUATION OR TWO-DIMENSIONAL
l 2ap 2 l l
2 ∞ n
STEADY-STATE HEAT FLOW EQUATION
2l ( −1) np x np at
+ 2∑ 2 2 sin sin 6. 15.1 Introduction
ap n = 2 n ( n − 1) l l
The two-dimensional heat conduction equation is
(6.145)
given by

EXERCISE 6.8 ∂u ∂2 ∂2
= a2 — 2 u, ⎜⎝where — 2 = 2 + 2 is the
1. Find the displacement of a string stretched between ∂t ∂x ∂y

two fixed points at a distance 2l apart when the string
is initially at rest in equilibrium position and the points Laplacian operator⎟⎠
of the string are given an initial velocity g (x), which In the case of steady-state heat flow ∂u/∂t = 0 and
are given by ∂y/∂t ( x, 0) = g ( x ) = bx(l − x ). equation reduces to
8bl 3 ∞ 1 (2n − 1)p x ∂2u ∂2u
4∑
Ans: y( x, t ) = sin
ap n =1 (2n − 1) 4
l — 2u = + =0
∂x 2 ∂y 2
(2n − 1)p at
sin .
l The solution u(x, y) of the above equation can be
obtained by the method of separation of variables in
2. A string of length l is stretched and fastened to two fixed a rectangular region both in the Dirichlet problem as
points. Find y (x, t) satisfying the wave equation ytt = well as in Neumann’s problem. A rectangular thin
a2yxx when it is given as: plate with its two faces insulated is considered so that
the heat flow is two-dimensional. The boundary con-
(a) Initial displacement y (x, 0) = f (x) = b sin px/l.
ditions are prescribed on the four edges of the plate.
px pat
Ans: y( x, t ) = b sin cos . Example 6.68
l l
Solve Laplace’s equation
(b) Initial triangular deflection:
⎧ 2Cx ∂2u ∂2u
for 0 < x < l/2 + =0 (1)
⎪⎪ l ∂x 2 ∂y 2
f ( x) = ⎨
⎪ 2C (l − x ) for l /2 < x < l in the rectangle; 0 < x < a, 0 < y < b in the x y-plane,
⎪⎩ l with the boundary conditions
Partial Differential Equations    6-43

u(x, 0) = 0 on OA; u(x, b) = 0 on BC (2), (3) np x


X n ( x ) = D sin
b
u(0, y) = 0 on OC; u(a, y) = f (y) on AB (4), (5)
y Therefore, the solution of equation (1) satisfying
the boundary conditions (2), (3) and (4) is
C(0, b) 0° B(a, b)
np x np y
un ( x, y ) = Cn sinh sin
b b
0° 2u = 0 f(y)
where we have replaced D by Cn.
0 (0, 0) 0° A(a, 0) x
By the principle of superposition we write the
solution as
Figure 6.4 Two-dimensional heat flow in a rectangular
∞ ∞
np x np y
u( x, y ) = ∑ un ( x, y ) = ∑ Cn sinh
plate
sin
n =1 n =1 b b
Solution Let u(x, y) = X(x) Y(y) (6) (16)
Substituting in (1) we get
Lastly we have condition (5) namely u(a, y) = f ( y)
Y″ X″ to be satisfied. This gives
= = −l 2 (7)
Y X ∞
np a np y
u( a, y ) = ∑ Cn sinh sin = f ( y) (17)
where the separation constant is taken as negative to n =1 b b
get non-trivial solutions. The boundary value prob-
lem reduces to solution of the ordinary differential This is a half-range Fourier sine series expansion of
equations f ( y) in (0, b) and the constants Cn are given by
np a 2 b np y
Y ″ + l 2Y = 0, X ″ − l 2 X = 0 (8), (9) Cn = sinh = ∫ f ( y ) sin dy
b b 0 b
under the conditons
2 b np y
0 = u( x, 0) = X ( x )Y (0) ⇒ Y (0) = 0
np a ∫0
(10) or Cn = f ( y ) sin dy (18)
b sinh b
0 = u( x, b) = X ( x )Y (b) ⇒ Y (b) = 0 (11) b
0 = u(0, y ) = X (0)Y ( y ) ⇒ X (0) = 0 (12) Thus, this harmonic function u(x, y) satisfying
Laplace’s equation (1) and the boundary conditions
The general solution of equation (8) is (2) – (5) is given by (16) where the constants Cn are
Y( y) = A cos l y + B sin l y determined by (18) for any specific function f (y).
By (10) we get A = 0 and by (11) we get sin l b = 0 Example 6.69
np A retangular plate is bounded by the lines x = 0,
⇒ ln = ( n = 1, 2, 3," ) (13) y = 0, x = a and y = b and the edge tempera-
b
tures are u (0, y) = u (x, b) = u (a, y) and u (x, 0) =
ln = np /b are the eigenvalues and the correspond- 5sin(5p x/a) + 3sin(3p x/a). Find the steady-state
ing eigenfunctions are temperature at any point of the plate.
np y [JNTU 2002, 2003s]
Yn ( y ) = sin (with B = 1) (14)
b Solution Let u(x, y) be the steady-state tempera-
Now the general solution of (9) is ture at any point p(x, y) of the rectangular plate. We
have to solve Laplace’s equation
np x np x
X n ( x ) = C cosh + D sinh (15)
b b ∂2u ∂2u
+ =0 ! (1)
using (12) we get C = 0 so that ∂x 2 ∂y 2
6-44    Engineering Mathematics-II

under the boundary conditions them. The breadth in p. This end is maintained at a
temperature u0 at all points and the other edges are
1. u (0, y) = 0 for 0<y<b (2)
at zero temperature. Determine the temperature at
2. u (a, y) = 0 for 0<y<b (3) any point of the plate in the steady state.
[JNTU 2002, 2005 (Set 4)]
3. u (x, b) = 0 for 0<x<a (4)
5p x 3p x
4. u ( x, 0) = 5sin + 3sin for 0 < x < a (5) y 8
a a
A suitable solution of equation (1), by the method
of separation of variables, satisfying boundary con-
P(x, y)
ditions (2) – (4) for each n = 1, 2, 3, ... is 0°c 0°c

np x np ( y − b)
un ( x, y ) = Cn sin sinh (6)
a a
By the principle of superposition of solution we 0 u0 x =p x
may write the general solution as
∞ ∞
np x np ( y − b) Figure 6.5 Temperature in an infinitely long plate
u ( x, y ) = ∑ un ( x, y ) = ∑ Cn sin sinh
n =1 n =1 a a
Solution Let u(x, y) be the temperature at any
(7) point p(x, y) of the plate. Then the steady state
Imposing condition (5) we get temperature distribution is given by Laplace’s equation

np x np b 5p x ∂2u ∂2u
u( x, 0) = − ∑ Cn sin sinh = 5sin + =0 ! (1)
n =1 a a a ∂x 2
∂y 2
3p x
+ 3sin with the boundary conditions
a (8)
1. u(0, y) = 0; 2. u(p, y) = 0 for all y
Equating the coefficients of like terms on either side 3. u(x, 0) = u0; 4. u(x, ∞) = 0 for 0 < x < p
we get
The general solution of equation (1) is
5p b 3p b
−C5 sinh = 5 and − C3 sinh = 3 and Cn = 0 u( x, y ) = ( A cos l x + B sin l x)(Ce λ y + De − λ y ) (2)
a a
for all n ≠ 3 or 5 (9) By condition (1): u(0, y) we get A = 0. Now equation (2)
becomes
Hence, the required solution of steady-state tem-
perature is u(x, y) = B sin lx (Cely + De–ly) (3)
3 3p x 3p (b − y )
u( x , y ) = sin sinh By condition (2): u(p, y) = 0 we get
3p b a a
sinh
a sin lπ = 0 ⇒ l = ln = n (4)
5 5p x 5p (b − y )
+ sin sinh (10) so that the solution becomes u(x, y) = (cneny + Dne–ny)
5p b a a
sinh sin nx where BC and BD have been replaced by cn
a and dn, respectively.
By condition (4):
Example 6.70
An infinitely long plane uniform plate is bounded u(x, ∞) = 0 we have cn = 0 for all n (5)
by two parallel edges and an end at right angles to
Partial Differential Equations    6-45

Finally, the general solution satisfying the con- with the boundary conditons:
ditions (1), (2) and (4) is (by the principle of
superposition) 1. u(0, y ) = 0 2. u( a, y ) = 0 for 0 ≤ y ≤ a
∞ ∞ 3. u( x, 0) = 0 4. u( x, a) = u0 for 0 ≤ x ≤ a .
u( x, y ) = ∑ un ( x, y ) = ∑ dn sin nx e − ny (6)
n =1 n =1 The general solution of equation (1) is
Now applying boundary condition (3):
∞ u( x, y ) = (C1 cos px + C2 sin px )(C3e py + C4 e − py )
u( x, 0) = ∑ dn sin nx = u0 (7) (2)
n =1
Equation (7) is a half-range Fourier sine series By condition (1) we get C1 = 0 so that equation (2)
expansion for u0 in (0, p) and hence the Fourier con- becomes
stants dn are given by u( x, y ) = C2 sin px (C3e py + C4 e − py ) (3)
p
2 p −2u0 ⎛ cos nx ⎞
dn = ∫ u0 sin nx dx = ⎜⎝ ⎟ By condition (2) we get u( a, y ) = 0 = C2 sin pa
p 0 p n ⎠0 a(C3e py + C4 e − py ) ⇒ p = ( np /a)( n = 1, 2, 3), so that
⎧0 if n is even we have
−2u0 ⎪
= (cos np − 1) = ⎨ 4u0 (8)
np ⎪⎩ np if n is odd. u( x, y ) = C2 sin
np x
a
(
C3e np y/a + C4 e − np y/a ) (4)

Now (6) reduces to


Now, condition (3) implies that C4 = − C3 (5)
4u0
u( x , y ) = ∑ sin nx e − ny (9) The solution may therefore be written as
n =1,3,5 np
which is the required solution.
np x np y
un ( x, y ) = Cn sin sinh ( n = 1, 2, 3,...) (6)
Example 6.71 a a
The temperature u (x, y) is maintained at 0°C along
three edges of a square plate of side 100 cm and the for each n. By the principle of superposition we may
fourth edge is maintained at a constant temperature write
u0 until steady-state conditons prevail. Find the tem-
perature at any point (x, y) of the plate and also at the ∞ ∞
npx np y
centre of the plate. u( x, y ) = ∑ un ( x, y ) = ∑ Cn sin sinh (7)
n =1 n =1 a a
[JNTU 2003s (Set 4)]
y
u(x,100) =u 0 Imposing the condition (4) we have
C B

np x
u(0,y)=0
P(x,y )
u(100,y) = 0 u( x, a) = u0 = ∑ Cn sinh np, sin (8)
n =1 a
Equation (8) is a half-range Fourier sine series
0 u(x,0) = 0 A x expansion of u0 in (0, a), and the Fourier coefficients
are obtained by
Figure 6.7 Temperature in a square plate.
2 a np x
Solution Let the side of the plate be a = 100 cm. Cn sinh np = ∫ u0 sin dx or
a 0 a
The temperature function u (x, y) satisfies Laplace’s
2u0 a np x
equation Cn = ∫
a sinh np 0
sin
a
dx
∂2u ∂2u
+ =0 (1)
∂x 2 ∂y 2
6-46    Engineering Mathematics-II

a By separating the variables we get the following two


2u0 ⎛ −a ⎞ ⎡ np x ⎤
⇒ Cn = ⋅ ⎜ ⎟ ⎢cos
a ⎥⎦ 0
ordinary differential equations:
a sinh np ⎝ np ⎠ ⎣
2u0 −a X″ + l2 X = 0 and Y ″ − l 2Y = 0 (6), (7)
= ⋅ ⋅ (cos np − 1)
a sinh np np these solutions are
⎧ 0 if n is even
⎪ X ( x ) = A cos l x + B sin l x; Y ( y ) = Ce l y + De − l y
=⎨ 4u0
⎪ np sinh np if n is odd (9) (8), (9)

The above boundary conditions imply that
Substituting for Cn in (7) we have

4u0 np x np y X ′ (0) = (− Al sin l x + Bl cos l x ) x = 0 = 0 ⇒ B = 0
u( x , y ) = ∑ np sinh np
sin
a
sinh
a X ′ (a) = 0 = − Al sin l a ⇒ l = np /a ( n = 0,1, 2)
n =1,3,5,...

where a = 100 cm. (10) np x


∴ X n ( x ) = An cos n = 0,1, 2,!
a
Temperature at the centre = (50, 50) is De − l b

Y ′ (a) = l (Ce l b − De − l b ) = 0 ⇒ C = l b
4u0 np np e
u(50,50) = ∑ sin sinh (11)
n =1,3,5,... np sinh np 2 2
Thus np (b − y )
Y ( y ) = Cn cosh .
Example 6.72 a
Find the steady-state temperature in a rectangular Hence, the required solution is
plate 0 < x < a, 0 < y < b when the sides x = 0, np x np (b − y )
x = a and y = b are insulated while the edge y = 0 is u( x, y ) = ∑ An cos cosh
a a
kept at temperature (c/a) cos p x/a.
where
y
uy = 0 2 a np x
C B An =
a cosh ( np b/a) ∫0
f ( x ) cos
a
dx

2c a px np x
ux =0 ux = 0 = ∫
a cosh ( np b/a) 0
cos
a
cos
a
dx

0 c cos px A x ⎛ p b⎞ a
a a A1 = ⎜ 2c/a cosh ⎟
⎝ a ⎠2
Figure 6.7 Steady state temperature in a rectangular An = 0 for n = 0 and n≥2
plate.
pb px p (b − y )
Solution We have to solve Laplace’s equation ∴ u( x, y ) = c sech cos cosh .
a a a
∂2u ∂2u
+ =0 (1)
∂x 2 ∂y 2 EXERCISE 6.9
Solve the two-dimensional Laplace’s equation.
under the boundary conditions (∂ 2u/∂x 2 ) + (∂ 2u/∂y 2 ) = 0 in the region 0 < x < a,
0 < y < b bounded by a metal plate with the following
ux x=0 = ux x= a = uy y=b =0 (2), (3), (4)
boundary conditions:

px 1. u(0, y ) = u( x,0) = x( x, b) = 0 and u( a, y ) = g ( y )


and u( x, 0) = f ( x ) = c cos . (5) for 0 < y < b
a
Partial Differential Equations    6-47


np x np y temperature and its fourth edge y = p is kept at tem-
Ans: u( x, y ) = ∑ Bn sinh sin perature f (x). Find the steady-state temperature at any
n =1 b b
point of the plate. [JNTU Suppl. 2003]
2 np a h np y
where Bn = b cosech b ∫0 g ( y )sin b dy 2 ∞ sin nx cos h ny p
Ans: u( x, y ) = ∑
p n =1 cosh np ∫0
f ( x )sin nx dx

2. Solve problem 1 completely by taking g (y) = 100.



400 np x np y 5. Find the solution of problem 4 if f (x) = 100.
Ans: u( x, y ) = ∑ sinh sin .
np sinh ( np a/b ) b b 400 ∞ sin nx cosh ny
n =1,3,5
Ans: u( x, y ) = ∑
p n =1,3,5... x cosh np
.
3. u(0, y ) = u( a, y ) = u( x, b) = 0 and u( x ,0)
= f ( x ) = x( a − x ), 0 < x < a 6. Solve (∂ 2u/∂x 2 ) + (∂ 2u/∂y 2 ) = 0 for 0 < x < p , 0 < y < p
8a2 ∞ cosech (mp b/a) mp x given that
Ans: u( x, y ) = ∑
p 3 n =1,3,5... m3
sin
a u(0, y ) = u(p , y ) = u( x, p ) = 0 and u( x ,0) = sin 2 x.
mp (b − y ) 8 ∞ (e ny − e 2np − e − ny )
sinh
a Ans: u( x, y ) = ∑
p n =1,3,5... ( n2 − 22 )(e 2np − 1)
sin nx.
4. A square plate has its faces and the edge y = 0
insulated. Its edges x = 0 and x = p are kept at zero
Fourier Integral Transforms
7

7.1 INTRODUCTION F ( s) = F { f ( x )} = ∫ f ( x )eisx dx (7.3)
−∞
Integral transforms are useful in solving initial and
boundary value problems and in evaluating certain
integrals. Laplace and Fourier transforms are two 7.3 FOURIER INTEGRAL THEOREM
important transforms which are widely used in Let f (x) be a function satisfying the following
engineering and physical applications. They are Dirichlet conditions in every interval (−l, l) (l > 0)
used in the solution of conduction of heat, vibration however large:
of strings, oscillations of elastic beams, transmission 1. f (x) is periodic; f (x) and its integrals are
lines, etc. single-valued and finite (bounded).
Here we define Fourier transform together with
Fourier sine and cosine transforms, their inverses 2. f (x) has at most a finite number of disconti-
and study their properties and consider evaluation nuities in any one period.
of certain integrals. 3. f (x) has at most a finite number of maxima
or minima.
7.2 INTEGRAL TRANSFORMS Then the Fourier series expansion of f (x) in (−l, l)
A linear integral transform or simply an integral is given by
transform of a function f (x) is defined by a0 ∞ npx ∞ npx
b
f ( x) = + ∑ an cos + ∑ bn sin (7.4)
T { f ( x )} = ∫ f ( x ) K ( s, x ) dx (7.1) 2 n =1 l n =1 l
a
where
1 l ⎛ npt npt ⎞
l ∫− l
where K(s, x), called the Kernel of the transforma- ( a0 , an , bn ) = f (t ) ⎜1, cos

, sin ⎟ dt
tion, is a known function. For different choices of l l ⎠
the Kernel K(s, x) and the limits a and b, we obtain (7.5)
different transforms.
Substituting for a0, an, bn from (7.5), equation (7.4)
becomes
7.2.1 Laplace Transform 1 l 1∞ l npt npx
−sx
When K(s, x) = e (s > 0), a = 0 and b = ∞, we obtain f ( x) =
2l ∫ − l
f ( t ) dt + ∑
l n =1 ∫ − l
cos
l
cos
l
f (t ) dt
the Laplace transform of f (x). It is defined by
∞ 1∞ l npt npx
F ( s) = L{ f ( x )} = ∫ f ( x )e − sx dx ( s > 0)
0
(7.2) + ∑
l n =1 ∫ − l
sin
l
sin
l
f (t ) dt

1 l 1∞ l np (t − x )
7.2.2 Fourier Transform =
2l ∫ − l
f ( t ) dt + ∑
l n =1 ∫ − l
cos
l
f (t ) dt
When K(s, x) = eisx, a = −∞ and b = ∞, we obtain the
Fourier transform of f (x). It is defined by (7.6)
7-2    Engineering Mathematics-II

If we further assume that f (x) is absolutely integ- 1 ∞ ⎛ ∞ ⎞


rable for all x =
p ∫0
cos sx ⎜ ∫ cos st f (t ) dt ⎟ ds
⎝ −∞ ⎠

i.e., ∫−∞ f ( x ) dx 1 ∞ ⎛ ∞ ⎞
p ∫0
+ sin sx ⎜ ∫ sin st f (t ) dt ⎟ ds (7.9)
⎝ −∞ ⎠
is convergent, then
When f (t) is an odd function in (−∞, ∞) then
1 l ⎤ the integral in the first brackets in (7.9) is zero
2l ∫− l
f (t ) dt → 0 as l → ∞ ⎥ since the integrand cos st f (t) is an odd function in
⎥ (7.7) (−∞, ∞) and so we obtain
f (t ) dt ≤ ∫ f (t ) dt → 0 as l → ∞ ⎥⎥
1 l 1 l
2l ∫− l 2l − l ⎦ 2 ∞ ⎛ ∞ ⎞
f ( x) =
p ∫ 0
sin sx ⎜ ∫ sin st f (t ) dt ⎟ ds
⎝ 0 ⎠
(7.10)

Now (7.6) becomes


which is called the Fourier Sine Integral (FSI)
1 ∞ l np (t − x )
f ( x ) = ∑ ∫ cos f (t ) dt of f (x).
l n= 0 − l l When f (t) is an even function in (−∞, ∞) then
the integral in the second brackets in (7.9) is zero
p 1 ds since the integrand sin st f (t) is an odd function in
Putting = ds or = we have, on taking limit
l l p (−∞, ∞) and so we obtain
as n → ∞ or ds → 0
2 ∞ ⎛ ∞ ⎞
p ∫0
f ( x) = cos sx ⎜ ∫ cos st f (t ) dt ⎟ ds (7.11)
1 l
∞ ⎝ 0 ⎠
f ( x ) = lim ∑ ds∫ cos[nds(t − x )] f (t ) dt
p n→∞ n = 0 − l
which is called the Fourier Cosine Integral (FCI)
1 ∞⎡ ∞
cos[ s(t − x ) f (t ) dt ]⎤
p ∫0 ⎣⎢ ∫−∞
= (7.8) of f (x).
⎦⎥

as lim f (nds ) = ∫ f ( s) ds 7.4 FOURIER INTEGRAL IN COMPLEX
n→∞
or
0
FORM
ds → 0
cos s(t − x) is an even function of s in (−∞, ∞) and so
we can write (7.8) in the form
which is called the Fourier Integral (FI) of f (x). ∞ ∞
1
Note 1 A rigorous proof of the above theorem is
f ( x) =
2p ∫−∞ ∫−∞ cos[s(t − x)] f (t ) dt ds (7.12)
beyond the scope of this book.
Also, sin s(t − x) is an odd function of s in
Note 2 Fourier integral representation for a func- (−∞, ∞). So,we can write
tion f (x) is useful in solving certain differential and
1 ∞ ∞
integral equations. 0=
2p ∫−∞ ∫−∞ sin[s(t − x)] f (t ) dt ds (7.13)

7.3.1 Fourier Sine and Cosine Now, multiplying (7.13) by ‘i ’ and adding it to
Integrals (FSI’s and FCI’s) (7.12) we obtain
Writing the expansion for cosine in (7.8) we have 1 ∞ ∞
2p ∫−∞ ∫∞
f ( x) = {cos[ s(t − x )]
1 ∞ ∞
p ∫0 ∫−∞
f ( x) = (cos st cos sx + sin st sin sx ) f (t ) dt ds + i sin[ s(t − x )]} f (t ) dt ds
Fourier Integral Transforms    7-3

Using Euler’s formula eiq = cos q + i sin q we can the Fourier transform (FT) of f (x). Then the inverse
write the above integral as Fourier transform (IFT) of F(s) is given by (7.22).
1 ∞ ∞ is ( t − x )
f ( x) =
2p ∫−∞ ∫−∞ f (t )e dt ds (7.14) Fourier Transform
The Fourier transformation of f (x), denoted by
which is called Fourier Integral of f (x) in complex F{ f (x)}, is defined by
form (FICF )

F { f ( x )} = F ( s) = ∫ f ( x )eisx dx (7.23)
−∞
7.4.1 Fourier Integral
Representation of a Function
Inverse Fourier Transform
By (7.9) a function f (x) may be represented by a FI
as The inverse FT of F(s), denoted by F −1 {F (s)} is
defined by
1 ∞
p ∫0
f ( x) = [ A( s) cos sx + B( s)sin sx ] ds (7.15)
1 ∞
F −1{F ( s)} = f ( x ) = ∫−∞ f ( s)e
− isx
ds (7.24)
2p
where ∞
A( s) = ∫ f (t ) cos st dt (7.16) Thus, the function F (s) defined by (7.23) is called
−∞
the Fourier Transform of f (x) and is denoted by
and F{ f (x)} = F (s) (7.25)

B( s) = ∫ f (t )sin st dt (7.17)
−∞
and the function f (x) given by (7.24) is called the
If f (x) is an odd function in (−∞, ∞) then Inverse Fourier transform of F (s) and is denoted by
1 ∞ F −1{F (s)} = f (x) (7.26)
p ∫0
f ( x) = B( s)sin sx ds (7.18)
The process of obtaining the FT F{f (x)} = F (s)
where from a given function f (x) is called the FT method

B( s) = 2∫ f (t )sin st dt (7.19) or simply Fourier Transform.
0

and if f (x) is an even function in (−∞, ∞) then Existence of Fourier Transform


1 ∞
f ( x ) = ∫ A( s) cos sx ds (7.20) The following conditions are sufficient for the exis-
p 0 tence of the FT of a function f (x):
where ∞ 1. f (x) is piecewise continuous on every finite
A( s) = 2∫ f (t ) cos st dt (7.21) interval.
0
2. f (x) is absolutely integrable for all x.
7.5 FOURIER TRANSFORM OF f (x)
Writing the exponential function eix (t − x) as a product
of two exponential functions eix (t − x) = eist · e−isx the
7.5.1 Fourier Sine Transform
FI (7.14) may be put in the following form (FST) and Fourier Cosine
Transform (FCT)
1 ∞ − isx ⎛ ∞ ⎞
∫−∞ e ⎜⎝ ∫−∞ f (t )e dt ⎟⎠ ds
ist
f ( x) = (7.22) The FSI in (7.10) can be written by replacing t by
2p x as
The expression inside the brackets is a function ∞ ⎛ ∞ ⎞
f ( x ) = ∫ sin sx ⎜ ∫0 f ( x )sin sx dx ⎟ ds (7.27)
of s. Denoting it by F (s) and replacing t by x, we have 0 ⎝ ⎠
7-4    Engineering Mathematics-II

The expression in the brackets in (7.27) is a Similarly the finite Fourier cosine transform of f (x)
function of s denoted by Fs(s). Writing in 0 < x < l is defined by
∞ l npx
Fs { f ( x )} = Fs ( s) = ∫ f ( x )sin sx dx (7.28) Fc ( n) = ∫ f ( x ) cos dx (7.35)
0 0 l

equation (7.20) becomes where n is an integer.


The function f (x) is then called the inverse finite
2 ∞ Fourier cosine transform of Fc(n) and is given by
Fs−1{Fs ( s)} = f ( x ) =
p ∫0
Fs ( s)sin sx ds (7.29)
1 2 ∞ npx
f ( x ) = Fc (0) + ∑ Fc ( n) cos (7.36)
The function defined by (7.28) is called the l l n =1 l
Fourier sine transform (FST) of f (x) and that defined
by (7.29) is called the inverse Fourier sine transform Note 1 The finite Fourier sine transform is useful
of Fs(s). for solving problems with boundary condition of
The Fourier cosine integral (7.11) can be written heat distribution (say) on two parallel boundaries
by replacing t by x as while the finite Fourier cosine transform is useful
for solving problems in which the velocity distri-
∞ ⎛ ∞ ⎞ bution (say) normal to two parallel boundaries are
f ( x ) = ∫ cos sx ⎜ ∫ f ( x ) cos sx dx ⎟ ds (7.30)
0 ⎝ 0 ⎠ prescribed.

The expression in the brackets (7.30) is a function of 7.6.1 FT, FST and FCT Alternative
s denoted by Fc(s). Writing denitions
∞ The Fourier transform, FST and FCT are alterna-
Fc { f ( x )} = Fc ( s) = ∫ f ( x ) cos sx dx (7.31)
tively defined as follows:
0

Denition 1
2 ∞
Fc −1{Fc ( s)} = f ( x ) =
p ∫0
Fc ( s) cos sx dx (7.32)
1 ∞
∫−∞ f ( x)e
isx
FT F ( s) = dx,
The functions Fc(s) and Fc−1 {Fc(s)} = f (x) are, 2p
respectively, called the Fourier cosine transform and ∞
inverse Fourier cosine transform, respectively. IFT f ( x) = ∫ F ( s)e − isx ds (7.37)
−∞

7.6 FINITE FOURIER SINE TRANSFORM 2 ∞


p ∫0
AND FINITE FOURIER COSINE FST Fs ( s) = f ( x )sin sx dx,
TRANSFORM (FFCT) ∞
IFST f ( x) = ∫ Fs ( s)sin sx ds (7.38)
The finite Fourier sine transform of f (x)in 0 < x < l 0
is defined by
l npx
Fs ( n) = ∫ f ( x )sin dx (7.33) 2 ∞
p ∫0
0 l FCT Fc ( s) = f ( x ) cos sx dx,

where n is an integer. IFCT f ( x ) = ∫ Fc ( s) cos sx ds (7.39)
The function f (x) is then called the inverse finite 0

Fourier sine transform of Fs(n) and is given by


Denition 2
2 ∞ npx
f ( x ) = ∑ Fs ( n)sin 1 ∞
(7.34)
∫ f ( x )e
isx
l n =1 l FT F ( s) = dx,
2p −∞
Fourier Integral Transforms    7-5

1 ∞ − isx understand which notation is used and then answer,


IFT f ( x) = ∫ F ( s )e
2p −∞
ds (7.40)
clearly stating the notation he or she is using.

2 ∞ 7.7 CONVOLUTION THEOREM


p ∫0
FST Fs ( s) = f ( x )sin sx dx FOR FOURIER TRANSFORMS

2 ∞ 7.7.1 Convolution
p ∫0
IFST f ( x) = Fs ( s)sin sx ds (7.41)
Denition: The convolution of two functions
f (x) and g (x) over the interval (−∞, ∞) denoted by
2 ∞ f * g is defined by
p ∫0
FCT Fc ( s) = f ( x ) cos sx dx ∞
h( x ) = f ∗ g = ∫ f (u ) g ( x − u ) dx (7.43)
−∞
2 ∞
p ∫0
IFCT f ( x) = Fc ( s) cos sx ds (7.42)
7.7.2 Convolution Theorem
Theorem 7.1 If F (s) = F{ f (x)} and G(s) =
Note 1 Each of these notations has its advantages F{g (x)} are the Fourier transforms of f (x) and g (x)
and disadvantages. So, one can adopt any one of then the Fourier transform of the convolution of f (x)
these definitions. If both transformation and inverse and g (x) is the product of their transforms
transformation are involved completing one cycle
there will not be any difference in the results. If only F{ f (x) * g (x)} = F{ f (x)} · F{g (x)} (7.44)
one of these is involved the results will be differ- Proof By the definition of Fourier transforms, we
ent. The student, while answering a question, should have

⎧ ∞ ⎫ ∞ ∞
F{ f ( x ) * g ( x )} = F ⎨ ∫ f (u ) g ( x − u ) du ⎬ = ∫ ⎡ ∫ f (u ) g ( x − u )du ⎤ eisx dx
⎩ −∞ ⎭ −∞ ⎢
⎣ −∞ ⎦⎥
∞ ∞
= ∫ f (u ) ⎡ ∫ g ( x − u )eisx dx ⎤ du, Changing the order of integration
−∞ ⎢⎣ −∞ ⎥⎦
∞ ∞ ! dx = d ( x − u )
= ∫ f (u ) ⎡ ∫ eis( x − u ) g ( x − u )d ( x − u )⎤ eisu du
−∞ ⎣⎢ −∞ ⎦⎥ eisx = eis( x − u ) ⋅ eisu
∞ ∞
=∫ f (u )eisu ⎡ ∫ g (t )eist dt ⎤ du, where t = x − u, dt = d ( x − u )
−∞ ⎢⎣ −∞ ⎥⎦

=∫ f (u ) ⋅ eisu du ⋅ G( s) = F ( s) ⋅ G( s) = F { f ( x )} ⋅ F {g ( x )}
−∞

7.7.3 Relation between Laplace and Proof


Fourier Transforms

⎡ g (t )e − xt, t>0 F { f (t )} = ∫ f (t ) ⋅ eist dt
If f (t ) = ⎢ −∞
⎣ 0, t<0 (7.45) 0 ∞
=∫ 0 ⋅ dt + ∫ g (t ) ⋅ e − xt ⋅ eist dt
then F { f (t )} = L{g (t )} −∞ 0
7-6    Engineering Mathematics-II

∞ p = x − is and 1 ax = u, adx = du
=∫ g (t )e − pt dt = F ( s a)
−∞ Re p = Re ( x − is) a The limits are same
= L{g (t )}
Corollary If Fs(s) and Fc(s) are the Fourier sine
Thus, the Fourier transform of a function f (t) transform and Fourier cosine transform of f (x),
defined by (7.45) is the Laplace transformation respectively, then Fs ( f ( ax )) = (1 a) Fs ( s a) and
of g (t). Fc ( f ( ax )) = (1 a) Fc ( s a).

7.8 PROPERTIES OF FOURIER TRANSFORM 7.8.3. Shifting Property


7.8.1. Linearity Property Theorem 7.4 If F (s) is the complex Fourier
Theorem 7.2. If F (s) and G (s) are the Fourier transform of f (x) then
transforms of f (x) and g (x), respectively, then
F{ f (x − a)} = eisa F{ f (x)} = eisa F (s) (7.48)
F{af (x) + bg(x)} = aF (s) + bG(s)

= aF{f (x)} + aF{g (x)}
∫−∞ f ( x)e
isx
Proof It is given that F ( s) = dx
where a and b are any constants. By definition

Proof By definition we have F { f ( x − a)} = ∫ f ( x − a)eisx dx
−∞

F (s ) = ∫ f ( x )eisx dx and = eisa ∫

f (u )eisu du = eisa F ( s),
−∞
∞ −∞
G ( s) = ∫ g ( x )eisx dx where u = x − a and du = dx.
−∞

F {af ( x ) + bg ( x )} = ∫ {af ( x ) + bg ( x )}e dx isx
−∞
∞ ∞ 7.8.4. Modulation Theorem
= a∫ f ( x )eisx dx + b∫ g ( x )eisx dx
−∞ −∞ Theorem 7.5 If F (s) is the complex Fourier
= aF ( s) + bG( s)
transform of f (x) then
= aF { f ( x )} + bF {g ( x )} (7.46)
1
(a) F { f ( x ) cos ax} = [ F ( s + a) + F ( s − a)] and
7.8.2. Change of Scale Property or 2
Damping Rule 1
(b) F { f ( x ) sin ax} = [ F ( s + a) − F ( s − a)].
2
Theorem 7.3 If F (s) is the complex Fourier
(7.49)
transform of f (x) then

∫−∞ f ( x)e
isx
1 Proof It is given that F ( s) = dx
F { f ( ax )} = F ( s a) ( a ≠ 0) (7.47)
a (a) By definition

Proof Since F (s) is the complex Fourier F { f ( x ) cos ax} = ∫ f ( x ) cos axeisx dx
−∞
transform of f (x) we have
∞ ⎛ eiax + e − iax ⎞ isx
∞ =∫ f ( x) ⎜ ⎟ e dx
F { f ( x )} = F ( s) = ∫ isx
f ( x )e dx −∞ ⎝ 2 ⎠
−∞
1⎡ ∞ ∞
f ( x )eix ( s + a ) dx + ∫ f ( x )eix ( s − a ) dx ⎤
2 ⎣⎢ ∫−∞
=
So, the complex Fourier transform of f (ax) is −∞ ⎦⎥
given by 1
= [ F ( s + a) + F ( s − a)]
2
∞ 1 ∞
F { f ( ax )} = ∫ f (u )e ( ) du
a ∫−∞
i sa u
f ( ax )eisx dx =
−∞
(b) can be proved similarly.
Fourier Integral Transforms    7-7

Corollary If Fs(s) and Fc(s) are the Fourier sine Example 7.3
transform and the Fourier cosine transform of f (x),
⎡ 0, − ∞ < x < −1
respectively, then
Express f ( x ) = ⎢⎢ 1, − 1 < x < 1 as a Fourier
1
(a) Fs { f ( x ) cos ax} = [ Fs ( s + a) + Fs ( s − a)] ⎢⎣ 0, 1 < x < ∞
2
1 ∞ sin s cos sx
(b) Fc { f ( x )sin ax} = [ Fs ( s + a) − Fs ( s − a)] integral and hence evaluate ∫ ds
2 0 s
1
(c) Fs { f ( x )sin ax} = [ Fc ( s − a) − Fc ( s + a)] Solution The Fourier integral of f (x) is given by
2
1 ∞ ∞
p ∫0 ∫−∞
(7.50) f ( x) = f (t ) cos s(t − x ) dt ds (1)
Example 7.1
Using Fourier integral show that The funtion f (t) is defined by

2a ∞ cos lx ⎡0, in ( −∞,1)


e − ax =
p ∫−∞ l 2 + a2
a > 0, x > 0
f (t ) = ⎢⎢1,
d l,
in ( −1,1) (2)
⎢⎣0, in (1, ∞)
Solution The Fourier cosine integral is given by
∴ We obtain
2 ∞ ∞

p ∫0
f ( x) = cos l x ∫ f (t ) cos lt dt d l (1)
0 1 ∞ 1
p ∫0 ∫−1
f ( x) = [cos s(t − x ) dt ] ds
Let f (t) = eat so that f (x) = eax. Substituting in the 1
1 ∞ ⎡ sin s(t − x ) ⎤
p ∫0 ⎢⎣
above integral, we have =
s ⎥⎦ ds
−1
2 ∞ ∞ ⎛ ∞ ⎞
e − ax = ∫ cos lx ∫ cos lx ⎜ ∫ e − at cos lt dt ⎟ d l 1 ∞1
p ∫0 s
p 0 0 ⎝ 0 ⎠ = [sin s(1 − x ) − sin s( −1 − x )] ds

2 ∞ ⎡ e − at ⎤ 2 ∞ sin s cos sx
= ∫ cos lx ⎢ 2 ( − a cos lt + b sin lt )⎥ d l = ∫ ds (3)
⎣a + l
2
p 0
⎦0 p 0 s
2 ∞ ⎡ e − a0 ⎤
=
p ∫0
cos lx ⎢ 0 −
a +l
2 2
( − a ⋅1 + b ⋅ 0)⎥ d l which is the Fourier integral of f (x). From (3) we
⎣ ⎦ obtain
2a ∞ cos lx
p ∫0 a2 + l 2
= dl ⎡p
∞ sin s cos sx p , x ≤1
cos lx pe − ax ∫0 ds = f ( x ) = ⎢ 2 by (2)

s 2 ⎢
⇒ ∫0 a2 + l 2
d l =
2a
(2) ⎣0, x ≥1

Example 7.2 At x = ±1, f (x) is discontinuous and the value of the


integral is
−x
Prove that pe = ∞ cos lx d l .
2 ∫0 l 2 + 1 1 1⎛p ⎞ p
[ f (1 − 0) + (1 + 0)] = ⎜ + 0⎟ = (4)
2 2⎝ 2 ⎠ 4
Solution Putting a = 1 in Example 7.1, we
obtain Example 7.4
−x
∞ cos lx pe Prove that ∞ sin x p
∫0 a +l
2 2
dl =
2 ∫0 x
=
2
7-8    Engineering Mathematics-II

Solution From Example 7.3 ⎡0, x<0


∞ cos sx + s sin sx

p ⎢p ,
∫0
∞ sin s cos sx
ds = , for x < 1 ∫0 1+ s 2
ds =
⎢2
x=0
s 2 ⎢ −s
⎣pe , x > 0.
Putting x = 0 we have
Solution Let f (x) be defined by
∞ sin x p ⎡0, ( x < 0)
∫0 x
dx =
2 ⎢
1
f ( x) = ⎢ , ( x = 0) (1)
⎢2
where we have replaced the dummy variable s by x. ⎢ −x
⎣e , ( x > 0)
Example 7.5 We now find the Fourier integral representation
Using the Fourier integral representation, show that of f (x) in the exponential form, which is given by

∞ ⎡ ∞
dt ⎤ e − isx ds
1
∫−∞ ⎢⎣∫−∞ f (t )e
ist
f ( x) = (2)
2p ⎥⎦

∞ 0 ∞ ⎡ e − t (1− is ) ⎤ 0 − 1 1 + is
Let I =∫ 0 ⋅ eist dt + ∫ e − t ⋅ eist dt = ⎢
f (t )eist dt = ∫ ⎥ = =
−∞ −∞ 0
⎣ is − 1 ⎦0 is − 1 1 + s2
1 ∞ 1 + is isx 1 ∞ (1 + is)(cos sx − i sin sx )
∴ f ( x) = ∫
2p −∞ 1 + s 2
e ds =
2p ∫−∞ 1 + s2
ds

1 ∞ cos sx + s sin sx ⎧ The second integral = 0


=
2p ∫ −∞ 1+ s 2
ds + 0, ⎨
⎩ since the integrand is odd in(−∞, ∞ )
1 ∞ cos sx + s sin sx
p ∫0
= ds, since the integrand is even in( −∞, ∞ )
1 + s2
= e− x , for x > 0 by definition of f ( x ) (3)

The function f (x) is discontinuous at x = 0 and its Example 7.6


value is
⎡1, for x ≤ a
Express f ( x ) = ⎢ as a Fourier integral.
1 ∞ 1 1 ⎣0, for x > a
f (0) = ∫ [ f (0 + 0) + f (0 − 0)] = (1 + 0) = Evaluate
2 −∞ 2 2
1 ∞ cos sx + s sin sx ∞ sin x ∞
sin as cos sx
1
(a ) ∫0 (b) ∫−∞
p ∫0
∴ ds = dx ds
1 + s2 2 x s
∞ cos sx + s sin sx p [JNTU 2003(2), 2004(3)]
⇒ ∫ ds = , for x = 0
0 1+ s 2
2
Solution The Fourier integral of f (x)is
= pe − x , for x > 0.
1 ∞ ∞
p ∫0 ∫−∞
f ( x) = f (t ) cos s(t − x ) dt ds
(4)
Fourier Integral Transforms    7-9

a
⎧p
1 ∞ a 1 ∞ sin s(t − x ) sin sx (1 − cos sp ) ⎪ , 0≤ x<p
= ∫ ⎡ ∫ cos s(t − x )dt ⎤ ds = ∫
∞ p
⎢ ⎥
ds ⇒ ∫0 ds = f (x ) = ⎨ 2
p 0 ⎣ − a ⎦ p 0 s s 2 ⎪⎩ 0, x > p
−a

1 ∞ sin s( a − x ) + sin s( a + x ) At x = p which is a point of discontinuity for f (x)


p ∫0
= ds the value of the integral is
s
2 ∞ sin sa cos sx
= ∫ ds p ⎛ f (p − 0) + f (p + 0) ⎞ p ⎛ 1 + 0 ⎞ p
p 0 s ⋅⎜ ⎟⎠ = ⎜⎝ ⎟=
2 ⎝ 2 2 2 ⎠ 4
⎡p
∞ sin sa cos sx p .1, x ≤a
⇒ ∫0 ds = f ( x) = ⎢ 2 Example 7.8
s 2 ⎢
⎣0, x >a Find the Fourier transform of

For |x| = a ⇔ x = ± a, which are points of ⎧⎪ 1 − x 2 , x ≤1


f ( x) = ⎨
discontinuity the value of the integral is ⎪⎩ 0, x >1

p f ( a + 0) + f ( a − 0) p ⎛ 1 + 0 ⎞ p ∞ x cos x − sin x x
= ⋅⎜ ⎟=
2 2 2 ⎝ 2 ⎠ 4 Hence, evaluate ∫0 x3
cos dx
2
[JNTU 2002, 2004S]
p∞ sin x
Choosing x = 0 and a = 1 we obtain
x
dx = ∫0
2 Solution The Fourier transform of f (x) is given by
where we have replaced the dummy variable s by x. ∞
F { f ( x )} = F ( s) = ∫ f ( x )eisx dx
−∞
Example 7.7 −1 1 ∞
= ∫ 0 ⋅ eisx dx +∫ (1 − x 2 )eisx dx +∫ 0 ⋅ eisx dx
⎧1, for 0 ≤ x ≤ p −∞ −1 1
Express the function f ( x ) = ⎨ as a 1
⎩0, for x > p ⎡ eisx eisx eisx ⎤
Fourier sine integral. = ⎢(1 − x 2 ) + 2x ⋅ 2
−2 ⎥
⎣ is (is) (is)3 ⎦ −1
∞ 1 − cos p
Hence, evaluate ∫0 s
sin sx ds
= 0 + 2⋅
1⋅ eis − ( −1)e − is
+ 2⋅
eis − e − is
− s2 is3
Solution The Fourier sine integral of f (x) is 4 4 4
= − 2 cos s + 3 sin s = − 3 ( s cos s − sin s)
given by s s s
(1)
2 ∞ ⎛ ∞ ⎞
f ( x ) = ∫ sin sx ⎜ ∫ f (t )sin st dt ⎟ ds By the inverse Fourier transform, we have
p 0 ⎝ 0 ⎠
1 ∞
F −1{F ( s)} = f ( x ) = F ( s)e − isx ds
2p ∫−∞
2 ∞ ⎛ ∞ ⎞
= ∫ sin sx ⎜ ∫ 1⋅ sin st dt ⎟ ds
p 0 ⎝ 0 ⎠
1 ∞ 4
− ( s cos s − sin s)e − isx ds
2p ∫−∞ s3
p =
2 ∞ ⎛ − cos st ⎞
p ∫0
= sin sx ⎜ ds
⎝ s ⎟⎠ 0
⎪⎧ 1 − x , x ≤1
2
=⎨
2 ∞ ⎛ − cos p + cos 0 ⎞ ⎪⎩ 0, x >1
=
p ∫0
sin sx ⎜
⎝ s
⎟⎠ ds
(2)

2 ∞ sin sx (1 − cos sp ) Putting x = 1/ 2 we obtain from the above


p ∫0
= ds
s equation (2)
7-10    Engineering Mathematics-II

2 ∞ ( s cos s − sin s) 2
− is 2 Putting x = 0 we get
⎛ 1⎞ 3
p ∫−∞
− e ds = 1 − ⎜ ⎟ =
s3 ⎝ 2⎠ 4 1 ∞ sin s ∞ sin s
p ∫−∞ s ∫−∞
ds = 1 or ds = p (4)
∞ s cos s − sin s ⎛ s s⎞ 3p s
⇒ ∫ 3 ⎜⎝ cos − i sin ⎟⎠ ds = −
−∞ s 2 2 8 Since the integrand is an even function in
(3) (−∞, ∞) we have
∞ sin x p
Equating the real parts on either side
∫0 x dx = 2 (5)
∞ s cos s − sin s ⎛ s⎞ 3p
∫−∞ s3
⎜⎝ cos ⎟⎠ ds = −
2 8
where we have replaced the dummy variables s by x.
∞ x cos x − sin x x 3p Example 7.10
⇒ ∫0 x 3
cos dx = − ,
2 16
(4)
Find the Fourier (a) cosine and (b) sine transform of
f (x) = eax (x > 0, a > 0).
since the integrand is even in (−∞, ∞) and we have Deduce the values of
replaced the dummy variable s by x.
∞ cos sx ∞ s sin sx
Example 7.9 ∫0 a +s
2 2
ds and ∫0 a2 + s2
ds
Find the Fourier transform of
Solution
⎪⎧1, for x < 1 (a) The Fourier cosine transform of f (x) is given by
f ( x) = ⎨
⎪⎩0, for x > 1 ∞
Fc { f ( x )} = Fc ( s) = ∫ f ( x ) cos sx dx (1)
0
∞ sin x
Hence evaluate ∫0 x
dx


Fc {e − ax } = Fc ( s) = ∫ e − ax cos sx dx
0
− ax ∞
Solution The Fourier transform of f (x) is given by ⎡ e ⎤
= ⎢ 2 2 ( − a cos sx + s sin sx )⎥
F { f ( x )} = F ( s) = ∫
∞ 1
f ( x )eisx dx = ∫ 1⋅ eisx dx ⎣a + s ⎦0
−∞ −1
e0 a
1 = 0− ( − a.1 + 0) =
⎛ eisx ⎞ eis − e − is 2 ⎛ eis − e − is ⎞ a +s
2 2
a + s2
2
=⎜ ⎟ = = ⎜
⎝ is ⎠ −1 is s⎝ 2i ⎟⎠ (2)
2
= sin s ( s ≠ 0) (1) The inverse Fourier cosine transform of Fc(s) is
s
2 ∞
Fc −1{Fc ( s)} = f ( x ) = e − ax =
p ∫0
Fc ( s) cos sx ds
sin s
We have lim = 1 and hence F{ f (x)} =
ss→ 0 2a ∞ cos sx ds
p ∫0 a2 + s2
=
F (s) = 2 when s = 0 (2)
By the inverse Fourier transform we have ∞ cos sx pe − ax
1 ∞
⇒ ∫0 a2 + s2
ds =
2a
(3)
−1
F ( s)e − isx ds
2p ∫−∞
F {F ( s)} =
(b) The Fourier sine transform of f (x) is given by
1 ∞ sin s − isx
= ∫ e ds
p −∞ s ∞
Fs { f ( x )} = Fs ( s) = ∫ f ( x )sin sx dx (4)
⎧⎪ 1, for x < 1 0
= f ( x) = ⎨ ∞ − ax
⎪⎩0, for x > 1 (3) Fs {e − ax } = Fs ( s) = ∫ e sin sx dx (5)
0
Fourier Integral Transforms    7-11

∞ 2 ∞
⎡ e − ax ⎤ F −1{Fs ( s)} = f ( x ) =
p ∫0
= ⎢ 2 2 ( − a sin sx − s cos sx )⎥ Fs ( s)sin sx ds
⎣a + s ⎦0
2 ∞ s
1 ⇒ e− x = ∫ sin sx ds (2)
= 0− 2 ( − a.0 − s.1) p 0 1 + s2
a + b2
s If we replace ‘x’ by ‘a’ we obtain
= 2 2
a +s (6) 2 s 2 x sin ax
e−a =
p ∫0 1 + s2
sin sa ds = ∫ dx (3)
The inverse Fourier sine transform of Fs(s) is p 0 1 + x2

2 ∞ where we have replaced the dummy variable s by x.


Fs −1{Fs ( s)} = f ( x ) = e − ax =
p ∫0
Fs ( s) sin sx ds (7)
∞ x sin ax pe − a
Hence ∫0 1+ x 2
dx =
2
(4)
2 ∞ s
= ∫ 2 2 sin sx dx
p 0 a +s Example 7.12
∞ s p
⇒ ∫ 2 2 sin sx dx = e − ax (8) Find the Fourier sine transform of e − ax/ x
0 a +s 2

Corollary Putting a = 0 in the above result we Solution By definition, the Fourier sine trans-
obtain form of f (x) is
∞ sin sx p ∞
∫0 s
ds =
2
(9) Fs { f ( x )} = {Fs ( s)} = ∫ f ( x )sin sx dx
0
(1)

e − ax ∞ e − ax
Example 7.11 f ( x) = ⇒ Fs ( s) = ∫ sin sx dx (2)
|x|
x 0 x
Find the Fourier sine transform of e . Hence,
show that Differentiating both sides with respect to ‘s’
∞ x sin ax pe − a − ax

∫0 1+ x 2
dx =
2
( a > 0) d
[ Fs ( s)] = ∫
∞e ∞
(sin sx )dx = ∫ e − ax cos sx dx
ds 0 x ∂s 0

Solution The variable x is positive in (0, ∞) so ⎡ e − ax ⎤
that |x| = x for (0, ∞). = ⎢ 2 2 ( − a cos sx + s sin sx )⎥
⎣a + s ⎦0
The Fourier sine transform of f (x) = e|x| = ex
is given by e0 a
= 0− ( − a.1 + 0) = (3)
a +s
2 2
a + s2
2
Fs { f ( x )} = Fs {e − x } = Fs ( s)
∞ ∞ Integrating now with respect to ‘s’ we obtain
= ∫ f ( x )sin sx dx = ∫ e − x sin sx dx
0 0
1 s
⎡ e− x ⎤
∞ Fs ( s) = a∫ ds = tan −1 + c
=⎢ ( − sin sx − s cos sx )⎥ a2 + s2 a
⎣1 + s
2
⎦0 When s = 0, Fs (0) = 0 ⇒ 0 = tan −1 0 + c ⇒ c = 0
e0 s Hence, Fs ( s) = tan −1 (s/a). (4)
= 0+ (0 + s cos 0) = (1)
1+ s 2
1 + s2

By the inverse Fourier sine transform of Fs(s) Example 7.13


we have Find the Fourier cosine transform of
7-12    Engineering Mathematics-II

⎧ x, for 0 < x < 1


∞ ⎡(1 + x ) − 1⎤
2
⎪ = −∫ ⎣ ⎦ sin sx dx
f ( x ) = ⎨2 − x, for 1 < x < 2
⎪0,
0 x(1 + x 2 )
⎩ for x > 2
∞1 ∞ sin sx
[JNTU2002, 2003S, 2004S] = −∫ sin sxdx + ∫ dx
0 x 0 x (1 + x 2 )

Solution The Fourier cosine transform of f (x) is p ∞ sin sx ∞ sin sx p


=− +∫ dx !∫ dx = (2)
given by 2 0 x(1 + x 2 ) 0 s 2

Fc { f ( x )} = Fc ( s) = ∫ f ( x ) cos sx dx
0
1 2 ∞
Differentiating both sides of (2) with respect to s
= ∫ x cos sx dx + ∫ (2 − x ) cos sx dx + ∫ 0.dx again
0 1 2

⎡ sin sx ⎛ − cos sx ⎞ ⎤
!
d2 ∞ 1 ∂
= ⎢x −⎜ Fc ( s) = ∫ (sin sx ) dx
⎣ s ⎝ s2 ⎟⎠ ⎥⎦ 0 ds 2 0 x(1 + x ) ∂s 2

2 ∞ cos sx
⎡ sin sx ⎛ − cos sx ⎞ ⎤ =∫ dx = Fc ( s), by (1)
+ ⎢(2 − x ) − ( −1) ⎜
⎣ s ⎝ s2 ⎟⎠ ⎥⎦1 0 1 + x2

1⋅ sin s − 0 1 ⇒ ( D 2 − 1) Fc ( s) = 0 (3)
= + 2 (cos s − cos 0)
s s
The general solution of (3) is
(0 − sin s) (cos 2s − cos s)
+ −
s s2 Fc ( s) = c1e s + c2 e − s (4)
2cos s − cos 2s − 1 d
= ( Fc ( s)) = c1e s − c2 e − s
s2 (5)
ds
Example 7.14
When s = 0, (1) and (4) give
Find the Fourier cosine transform of f ( x ) = 1/ (1 + x 2 ).
Hence, derive the Fourier sine transform of ∞ dx p
Fc (0) = c1 + c2 = ∫ = (6)
x
0 1+ x 2
2
f ( x) =
1 + x2 p
Also Fc (0) = c1 − c2 = − , by(5) (7)
2
Solution By definition, the Fourier cosine
transform of f (x) is given by Solving (6) and (7) we get
∞ c1 = 0, c2 = p / 2 (8)
I = Fc { f ( x )} = Fc ( s) = ∫ f ( x ) cos sx dx
0
∞ cos sx From (1) and (4) we have
=∫ dx (1)
0 1 + x2
p −s
Fc { f ( x )} = I = e (9)
Differentiating both sides of (1) with respect to ‘s’ 2
Now
d ∞ 1 ∂ ∞ x sin sx
Fc ( s) = ∫ (cos sx )dx = − ∫ dx
ds 0 1 + x ∂s
2 0 1 + x2 ⎧ x ⎫ ∞ x
2 Fs {f ( x )} = Fs ⎨ 2⎬
=∫ sin sx dx
∞ x sin sx ⎩1 + x ⎭ 0 1 + x2
= −∫ dx, inserting x in the num-
x(1 + x 2 )
0 d p
= Fc ( s) = − e − s (10)
erator and denominator ds 2
Fourier Integral Transforms    7-13

Example 7.15 Solution By definition, the Fourier cosine trans-


Find the Fourier sine and cosine transform of form of f (x) is
e−ax(a > 0) and deduce the inverse formula. ∞
Fc { f ( x )} = Fc ( s) = ∫ f ( x ) cos sx ds
[JNTU 2002, 2004] 0

⎧1 − s, 0 ≤ s ≤ 1
Solution The Fourier sine transform of f (x) = =⎨ (1)
e−ax is ⎩0, s >1

Fs {e − ax } = Fs ( s) = ∫ e − ax sin sx dx By the inversion formula we have
0
∞ 2 p
Fc−1{Fc ( s)} = f ( x ) =
p ∫0
⎡ e − ax ⎤ Fc ( s) cos sx ds
= ⎢ 2 2 ( − a sin sx − s cos sx )⎥
⎣a + s ⎦0 2 1
p ∫0
= (1 − s) cos sx ds
s s
= 0+ 2 2 = 2 2 (1)
a +s a +s 2⎡ sin sx − cos sx ⎤
1
= ⎢ (1 − s) − ( −1)
The inverse Fourier sine transform for Fs ( s) = p⎣ x x 2 ⎥⎦ 0
s 2 ⎡⎛ 0 − 0 ⎞ cos x − 1⎤ 2 1 − cos x
is = ⎜ ⎟− ⎥=
a + b2 p ⎢⎣⎝ x ⎠
2
x2 ⎦ p x2
2 ∞ s
Fs −1{Fs ( s)} = f ( x ) = e − ax = Now
p ∫0 a2 + s2
sin sx ds
∞ 2 ∞ 1 − cos x
Fc ( s) = ∫ f ( x ) cos sx dx =
p ∫0
(2) cos sx dx
0 x2
(2)
The Fourier cosine transform of f (x) = e−ax is
∞ From (1) and (2) we obtain
Fc { f ( x )} = Fc {e − ax } = Fc ( s) = ∫ e − ax cos sx dx
0
2 ∞ 1 − cos x ⎧1 − s, 0 ≤ s ≤ 1
p ∫0
− ax ∞ cos sx dx = ⎨ (3)
⎡ e ⎤ 2 s >1
= ⎢ 2 2 ( − a cos sx + s sin sx )⎥ x ⎩0,
⎣a + s ⎦0
Taking the limit as s → 0 we obtain
1 a
= 0 + 2 2 ( a.1 + 0) = 2 2 (3)
a +s a +s x
2 ∞ 1 − cos x 4 ∞ sin2 x 2 Put =t
p ∫0 p ∫0
dx = dx = 1 2
The inversion formula for x2 x2 dx = 2 dt
a
Fc ( s) =
a + s22 4 ∞ sin2 t 2 ∞ sin2 t
p ∫0 4t 2 p ∫0 t 2
⇒ 2 dt = dt = 1
2a ∞ cos sx
is Fc −1{Fc ( s)} = f ( x ) = ds = e − ax (4)
p ∫0 a2 + s2
2
∞ sin x p
⇒ ∫0 x 2
dx =
2
(4)

Example 7.16
where we have replaced the dummy variable t by x.
Solve the intergral equation
Example 7.17
∞ ⎧1 − s, 0 ≤ s ≤1
∫0
2
f ( x ) cos sx dx = ⎨ Show that the Fourier transform of e − x /2 is reciprocal.
⎩0, s >1
[JNTU 2002, 2004S]
2
∞ sin x
Hence evaluate ∫ dx Solution Here we have to use definition 2 for the
0 x2 Fourier transform, viz.
7-14    Engineering Mathematics-II

1 ∞ Here the function f (x) and the Fourier transform


∫ f ( x )e
isx
F { f ( x )} = F ( s) = dx 2
2p −∞ F (s) are the same. Hence, e − x /2 is self-reciprocal.
2 1 ∞ − x 2 /2 isx Note 1 If we use the other definitions there will be
F {e − x /2
} = F ( s) = ∫ e e dx
2p −∞ difference of a constant factor.
1 2
∞ − ( x − 2isx )
Example 7.18
1
=
2p
∫ −∞
e 2 dx Find the Fourier sine transform of (1/ x ) ( x 2 + a2 )

x 2 − 2isx + (is)2 − (is)2 − (is)2 = ( x − is)2 + s2 Hence, deduce the Fourier cosine transform of
1
⎛ x − is ⎞
2 . [JNTU 2001, 2002, 2003]
1 ∞ − ⎜⎝ ⎟ x( x + a2 )
2
2 ⎠
2
= e − s /2 ∫ e dx
2p −∞
Solution The Fourier sine transform of f (x) is
given by
x − is 1
Put =u ⇒ dx = du; ∞
2 2 Fs { f ( x )} = Fs ( s) = ∫ f ( x )sin sx dx
0

⎧ x = −∞ ⇒ u = −∞ ⎧ 1 ⎫ ∞ sin sx
limits are ⎨ Fs ⎨ 2 ⎬
= Fs ( s) = ∫ dx
⎩x = ∞ ⇒ u=∞ ⎩ x( x + a ) ⎭
2 0 x( x 2 + a2 )
∞ sin sx
I ( s) = ∫
2
− x2 2 e−s /2
∞ 2 Now, let dx (1)
Hence F {e }= ⋅ 2∫ e−u /2
du 0 x( x 2 + a2 )
2p −∞
2 Differentiating both sides of (1) with respect to ‘s’
e−s /2 2
= , 2p = e − s /2
2p dI ( s) ∞ 1 ∂ ∞ cos sx
=∫ (sin sx )dx = ∫ 2 dx
∞ 2
− u /2 ds 0 x ( x 2 + a 2 ) ∂s 0 x + a2
! ∫−∞ e du = p
(2)

Again differentiating both sides of (2) with respect to ‘s’


d 2 I ( s) ∞ 1∂ ∞ x sin sx
=∫ (cos sx ) dx = − ∫ 2 dx
ds 2 0 x + a ∂s
2 2 0 x + a2
∞ x 2 sin sx
= −∫ dx, (inserting x in the numerator and denominator)
0 x( x 2 + a2 )

∞ ⎡( x + a ) − a ⎤
2 2 2
= −∫ ⎣ ⎦ sin sx dx, (adding and subtracting a2 )
0 x( x + a )
2 2

⎡ ∞ sin sx p
= −∫
∞ sin sx
dx + a2 ∫
∞ sin sx
dx = −
p
+ a2 I ⎢∵ ∫0 x dx = 2
0 x 0 x( x 2 + a2 ) 2 ⎢
⎣corollary of Example (7.10)
p
⇒ ( D 2 − a2 ) I = −
, which is a second-order linear differential equation (3)
2
The general solution of (3) comprises two parts: the complementary function (CF) and the particular
intergal (PI).
Fourier Integral Transforms    7-15

1
⎛ p⎞ p 1 dI ( s) p
PI = ⎜⎝ − ⎟⎠ = 2 1 = = 2 (0 + ae − as )
2 2
D −a 2 2a ⎛ D 2 ⎞ ds 2a
⎜1 − a2 ⎟ p − as
⎝ ⎠ = e
−1 2a
p ⎛ D2 ⎞ p ⎛ D2 ⎞ p
= 2 ⎜
1 − 2 ⎟ 1 = 2 ⎜1 + 2 + ! ⎟1= 2 ∴ The Fourier cosine transform of g ( x ) = 1/( x 2 + a2 )
2a ⎝ a ⎠ 2a ⎝ a ⎠ 2a
is Gc ( s) = (p / 2a) e − as . (10)
CF = Ae as + Be − as
∴ The general solution is Examples of Finite Fourier Sine/Cosine
p transforms
I ( s) = Ae as + Be − as + (4)
2a 2 Example 7.19
dI ( s) Find the finite Fourier (a) sine and (b) cosine trans-
= aAe as − aBe − as (5)
ds form of f(x) = x in 0 < x < p.
Putting s = 0 in (1) and (4) we obtain Solution (a) The finite Fourier sine transform of
p p f(x) in 0 < x < l is
I (0) = A + B + 2 = 0 ⇒ A + B = − 2 (6)
2a 2a
l nπ x p
and similarly from (2) and (5) we obtain Fs (n) = ∫ f ( x) sin dx = ∫ x sin nx d x
0 l 0

dI ∞ ⎡1dx x⎤ (∴f(x) = x, l = p)
= aA − aB = ∫ 2 = ⎢ tan −1 ⎥
ds 0 x + a2 ⎣a a ⎦0 π
s=0 ⎡ ⎛ cos nx ⎞ ⎛ sin nx ⎞ ⎤ cos nπ ( −1) n −1
p p = ⎢x ⎜ − ⎟ − 1⎜ − 2 ⎟ ⎥ = − =
= ⇒ A− B = 2 (7) ⎣ ⎝ n ⎠ ⎝ x ⎠ ⎦0 n n
2a 2a
= 0 at 0 = 0 at π, 0
Solving (6) and (7) we obtain
p (b) The finite Fourier cosine transform of f(x) in 0
A = 0, B = − 2
(8) < x < l is
2a
l np x
Substituting these values in (4) we have Fc (n) = ∫ f ( x) cos dx; Here l = p and f(x) = x.
p
0 l
I ( s) = 2 (1 − e − as ), (9) So,
2a p
p ⎡ sin nx ⎛ cos nx ⎞ ⎤
Fc (n) = ∫ x cos nx dx = ⎢ x − 1. ⎜ − ⎟
n 2 ⎠ ⎥⎦ 0
which is the required Fourier sine transform of ⎝
0
⎣ n
1
f ( x) =
x( x + a2 )
2
⎧ 0 if n is even;
cos nπ ( −1) n −1 ⎪
Now we deduce the Fourier cosine transform of = = = ⎨ −2
1 n2 n2 ⎪⎩ n 2 if n is odd.
g( x) = 2
x + a2
1 Example 7.20
The Fourier cosine transform of g ( x ) = 2
is given by x + a2 Find the finite Fourier (a) sine and (b) cosine trans-
form of f(x) = x (p − x) in 0 < x < p.

Fc {g ( x )} = Gc ( s) = ∫0 g ( x ) cos sx dx
Solution (a) The finite Fourier sine transform of
∞ f(x) in 0 < x < l is
Fc {g ( x )} = Gc ( s) = ∫0 g ( x ) cos sx dx
l np x
Fs (n) = ∫ f ( x) sin dx, Here l = p, f(x) = x
⎧ 1 ⎫ ∞ 1 0 l
⇒ Fc ⎨ 2 ⎬ = Gc ( s) =
⎩ x + a2 ⎭
∫0 x 2 + a2
cos sx dx
(p − x) = px − x2
7-16    Engineering Mathematics-II

⎛ nπ ⎞ ⎛ nπ ⎞
π
∴Fs (n) = ∫ (p x − x 2 ) sin nx dx, e al 1
= 2 2 ⎜
− ⎟ cos nπ − 2 2 ⎜
− .1
0
n π ⎝ l ⎠ n π ⎝ l ⎟⎠
⎡ ⎛ cos nx ⎞ ⎛ sin nx ⎞ a2 + 2 a2 + 2
= ⎢(p x − x 2 ) − ⎟ − (p − 2 x ) ⎝⎜ − 2 ⎟⎠ l l
⎣ ⎝⎜ n ⎠ n nπ
p
⎛ cos nπ ⎞ ⎤
+ ( −2) ⎜ = l
(1 − e al cos nπ )
⎝ n3 ⎟⎠ ⎥⎦ 0 a + 2
2 n2π 2
l
cos nπ − 1 2 nπ
= −2 = 3 ⎡⎣1 − ( −1) n ⎤⎦
n3
⎧ 0
n
if n is even
= l
n2π 2
( )
1 − ( −1) n e al
⎪ a + 2
2
=⎨ 4 l
⎪⎩ − n3 if n is odd
(b) The finite Fourier cosine transform of f(x) in 0
< x < l is
(b) The finite Fourier cosine transform of f(x) in
0 < x < l is l nπ x l nπ x
l nπ x Fc (n) = ∫ f ( x) cos dx = ∫ e ax cos dx
Fc (n) = ∫ f ( x) cos dx. Here l = p, f(x) = x 0 l 0 l
0 l l
(p − x) = px − x 2 ⎡ ⎤
π ⎢ e ax ⎛ nπ x nπ nπ x ⎞ ⎥
∴Fc (n) = ∫ (π x − x 2 ) cos nx dx =⎢ ⎜⎝ a cos + .sin ⎟⎥
0
⎢ a2 + n π
2 2
l l l ⎠⎥
⎡ ⎛ sin nx ⎞ ⎢⎣ l2 ⎥⎦ 0
(
= ⎢ π x − x2 ⎜ −

) n ⎠

⎣ ae al a a
= 0 at π , 0 = cos nπ − =
n π
2 2
n2 2
π n2π 2
π a2 + 2 a2 + 2 a2 + 2
⎛ cos nx ⎞ ⎛ sin nπ ⎞ ⎤ l l l
− (π − 2 x ) ⎜ − ⎟ + ( −2) ⎜ − ⎟
⎝ x ⎠
2
⎝ n3 ⎠ ⎥⎦ 0 ( )
( −1) n e al − 1
= 0 at π = 0 at π , 0
Example 7.22
⎛ cos nπ − 1⎞ 1 − ( −1) n
= −π ⎜ Find the finite Fourier (a) sine and (b) cosine trans-
⎝ 2 ⎟⎠ = π
n n2 x
2
form of f(x)= ⎛⎜1 − ⎞⎟ in 0 < x < p.
⎧ 0 if n is even; ⎝ π⎠

= ⎨ 2π π⎛ x⎞
− if n is odd. Solution (a) Fs (n) = ∫ ⎜ 1 − 2 ⎟ sin nx dx
⎩⎪ n 2 0 ⎝ π ⎠
Example 7.21
⎡⎛ 2
x ⎞ ⎛ cos nx ⎞ ⎛ −2 ⎞ ⎛ x⎞
= ⎢⎜1 − ⎟ ⎜ − ⎟⎠ − ⎜⎝ ⎟⎠ ⎜⎝1 − ⎟⎠
Find the finite Fourier (a) sine and (b) cosine transform ⎢⎣ ⎝ π ⎠ ⎝ n π π
of eax in (0,1).
π
⎛ sinn nx ⎞ 2 cos nx ⎤
⎜⎝ − 2 ⎟⎠ + 2 . 3 ⎥
Solution (a) The finite Fourier sine transform of x π n ⎥⎦
0
f(x) in 0 < x < l is [JNTU 2009S; Set 2] 1 2
= − + 3 (cos nπ − 1)
l nπ x l nπ x n πx
Fs (n) = ∫ f ( x) sin dx = ∫ e ax sin dx
0 l 0 l ⎧ 0 if n is even

⎡ ⎤
l
=⎨ 1 4
⎢ e ax ⎛ nπ x nπ nπ x ⎞ ⎥ ⎪⎩ − n − nπ 3 if n is odd.
=⎢ 2 2 ⎜
a sin − .cos ⎟⎥
⎢ a2 + n π ⎝ l ⎠⎥ 2
l l π⎛ x⎞
⎢⎣ ⎥⎦ 0 (b) Fc (n) = ∫ ⎜1 − ⎟ cos nx dx
l2 0 ⎝ π⎠
Fourier Integral Transforms    7-17

2 Example 7.24
⎡⎛ x ⎞ sin nx ⎛ −2 ⎞ ⎛ x ⎞ ⎛ cos nx ⎞
= ⎢⎜1 − ⎟ − ⎜ ⎟ ⎜1 − ⎟ ⎜ − ⎟ Find the finite Fourier (a) sine and (b) cosine trans-
⎢⎣ ⎝ p ⎠ n ⎝ p ⎠ ⎝ p⎠⎝ n2 ⎠ form of
= 0 at p , 0 = 0 at p
⎧ x if 0 ≤ x p 2
p f(x) = ⎨
2 ⎛ sin nx ⎞ ⎤ ⎩p − x if 0 p 2 ≤ x p
+ 2 .⎜ − 3 ⎟ ⎥
p ⎝ n ⎠ ⎦0
Solution
= 0 at p , 0
p p 2
2 ⎛ −1⎞ −2 Fs (n) = ∫ f ( x) sin nx dx = ∫ x sin nx dx
= ⎜ ⎟=
p ⎝ n 2 ⎠ pn 2
0 0
(a) p 2
+∫ (p − x) sin nx dx
0
Example 7.23
Find the finite Fourier (a) sine and (b) cosine trans- p 2
⎡ ⎛ cos nx ⎞ ⎛ sin nx ⎞ ⎤
forms of f(x) = cos ax in 0 < x < p. = ⎢x ⎜ − ⎟ − 1 ⎜ − 2 ⎟⎠ ⎥
⎣ ⎝ n ⎠ ⎝ n ⎦0
Solution = 0 at 0 = 0 at 0
p p
(a) Fs (n) = ∫ cos ax sin nx dx cos (n ± a) p ⎡ ⎛ cos nx ⎞ ⎛ sin nx ⎞ ⎤
0
cos np cos ap ± + ⎢ (p − x) ⎜ − ⎟ + 1⎜ ⎟
⎣ ⎝ n ⎠ ⎝ n 2 ⎠ ⎥⎦ p
1 p sinp sinap = 0 2
=
2 ∫0
[sin(n + a) x + sin(n − a) x ]dx = 0 at p = 0 at p

p p np 1 np p np
1 ⎡ cos(n + a ) x cos(n − a ) x ⎤ =− cos + sin + cos
=− ⎢ + (−n)2 cos ap 2n n n2 2 2n 2
2⎣ x+a n − a ⎥⎦ 0
1 np
+ 2 sin
1 p ⎡ cos(n + a )p − 1 cos(n − a )p − 1⎤ n 2
2 ∫0 ⎢⎣
=− + ⎥⎦
x+a n−a 2 np
= sin .
n2 2
1 − ( −1) n ⎡ 1 1 ⎤ 1 − ( −1) n −2a
2 ⎢⎣ n + a + n − a ⎥⎦ = 2
. 2
n − a2 p p 2

0 if n is even; (b) Fc (n) = ∫ f ( x) cos nx dx = ∫ x cos nx dx


⎧ 0 0

=⎨ 4a +∫
p 2
(p − x) cos nx dx
⎪⎩ if n is odd 0
2
a − n2
p 2
p ⎡ sin nx ⎛ cos nx ⎞ ⎤
(b) Fc (n) = ∫ cos ax cos nx dx sin(n ± a)p = ⎢ x. − 1⎜ − 2 ⎟ ⎥
0
= sin np cos ap ⎣ n ⎝ n ⎠ ⎦0
1 p
= ∫ [ cos(n + a ) x + cos(n − a ) x ] dx ± cos np sin ap = 0
p
⎡ ⎛ sin nx ⎞ ⎛ cos nx ⎞ ⎤
2 0 + ⎢(p − x) ⎜ − ⎟⎠ − ( −1) ⎜⎝ − 2 ⎟⎠ ⎥
⎣ ⎝ n n ⎦p 2
1 p ⎡ sin(n + a ) x sin(n − a) x ⎤
= ∫ ⎢ + = ±(−1)n sin np
2 0 ⎣ x+a n − a ⎥⎦
p np 1 ⎛ np ⎞ p np
= 0 at 0 = 0 at 0 = sin + ⎜⎝ cos − 1⎟ + sin
2n n n2 2 ⎠ 2n 2
( −1)n sin ap ⎡ 1 1 ⎤ ( −1)n −1 sin ap 1 ⎛ np ⎞
= ⎢⎣ n + a + n − a ⎥⎦ = + 2 ⎜
cos − ( −1)n ⎟
2 n2 − a 2 n ⎝ 2 ⎠
7-18    Engineering Mathematics-II

p np 1 ⎛ np ⎞ ∞
cos ( 2np 3)
= + ⎜ 2 cos − (1 + ( −1) n )⎟
n
sin
n n2 ⎝ 2 ⎠ = 1+ 2 ∑
n =1 (2n + 1) 2
cos npx.
⎧2
( )
⎪⎪ n 2 ( −1) − 1 , if n is even
n2

=⎨
⎪ p ( −1)( n −1) 2 , if n is odd EXERCISE 7.1
⎪⎩ 2 1. Find the finite Fourier sine transform of

Example 7.25 ⎧ x if 0 < x p 2


f ( x) = ⎨
Find f(x) if the finite Fourier cosine transform of f(x) is ⎩p − x if p 2 < x < p
(a) ⎧ 1 np ⎫ 2 np
⎪ sin , for n = 1, 2, 3, . . .⎪ Ans: Fs (n) = 2
sin .
⎪ 2n 2 ⎪ n 2
Fc ( x) = ⎨ ⎬
⎪ p ⎪
, for n = 0 2. Find the finite Fourier cosine transform of

⎩ 4 ⎪

in 0 < x < 2p x2 p
f ( x) = − in 0 ≤ x ≤ p
2p 6
(b) ⎧ 2np ⎫ ( −1)n
⎪ cos , for n = 1, 2, 3, . . .⎪
Ans: Fc (n) = 0 + .
Fc ( x) = ⎨ 3 ⎬
⎪⎩ ⎪⎭ n2
1, for n = 0
3. Find the finite Fourier cosine transform of f(x) = x in
in 0 < x < 1. 0≤x<4
1 − cos np
Ans: Fc (x) =
Solution (a) The inverse finite Fourier cosine n 2p 2
transform of Fc (n) in 0 < x < l is
4. Find the inverse finite sine transform f(x) of
1 2 ∞ npx
f ( x) = Fc (0) + ∑ Fc (n) cos 1 − cos np
l l n =1 l Fs (n) = when 0 < x < p
p n 2p 2
Here Fc (0) = and l = 2p
4 4 ∞
sin nx
1 π 2 ∞
1 np npx
Ans: f ( x) =
p3

n =1,3,5... n
2
∴ f ( x) = , +
2p 4 2p

n =1 2 n
sin
2
cos
2p 5. Find the Fourier transform of e − x
2
/2
, (∞ < x < ∞).

p − s2 /4
1 1 1 np nx Ans: e
= +
8 2p
∑ n sin
n =1 2
cos
2
2
6. Find the Fourier transform of
(b) The inverse finite Fourier cosine transform of ⎧⎪1, x <a ⎧⎪ x 2 , x <a
Fc (n) is (a) f ( x ) = ⎨ (b) f ( x ) = ⎨
⎪⎩0, x >a ⎩⎪0, x >a
1 2 ∞ npx 2sin as
f ( x) = Fc (0) + ∑ Fc (n) cos Ans: (a) (b) [( a2 s2 − 2)sin ax + 2as cos as]
l l n =1 l s

Here Fc (0) = 1 and l = 1 ⎧⎪1, x ≤ 1


7. Express f ( x ) = ⎨ as a Fourier integral
⎪⎩0, x > 1
1 2 ∞ 1 2np npx
∴ f ( x) = + ∑ cos cos Hence, evaluate ∫
∞ sin s cos sx
1 1 n =1 (2n + 1)2 3 1 0 s
ds
Fourier Integral Transforms    7-19

⎧p 14. Find the Finite fourier sine and cosine transforms of


⎪ 4 , x ×1 f (x) = 2x, 0 < x < 4

2 ∞ sin s cos sx ⎪p −32( −1)n ( −1)n − 1
Ans: ∫ ds, I = ⎨ , x = 1 Ans: Fs ( n) = ; Fc ( n) = 32 2 2
p 0 s ⎪4 np np
⎪ 0, x > 1
⎪ ∞
f ( x )cos sx dx = e − s
⎩ 15. Solve ∫0
8. Using Fourier integral show that
2 1
⎧p Ans: f ( x ) = ⋅
∞ sin lp sin lx ⎪ sin x, (0 ≤ x ≤ p ) p 1 + x2
∫0 1 − l2
= ⎨2
⎪⎩0, (x > p )
⎧1, 0 ≤ s < 1
∞ ⎪
9. Using Fourier integral show that 16. Solve ∫0 f ( x )sin sx dx = ⎨2, 1 ≤ s ≤ 2
⎪0, s > 2
∞ l sin lx p ⎩
∫0 d l = e − ax , ( x > 0, a > 0)
a2 + l 2 2 Ans: f (x) = (2 + 2 cos x − 4 cos 2x)/px
10. Using Fourier intergral representation show that
17. Find the Fourier cosine transform of
∞s sin sx p
(a) ∫ ds = e − x ( x > 0) (a) eaxcos ax (b) eaxsin ax
0 1 + s2 2
∞ cos sx p −x ⎛ 2a 2 + b 2 ⎞ a(2a2 − s2 )
(b) ∫ ds = e ( x ≥ 0) Ans: (a) 2a ⎜ 4 ⎟ (b)
0 1 + s2 2 ⎝ s + 4a 4 ⎠ s2 + 4a 4
∞ sin s cos sx p
(c) ∫ ds = (0 < x < 1) 18. Find the Fourier cosine transform of
0 s 2
⎧ x, 0 < x < 1

f ( x ) = ⎨2, 1 < x < 2
11. Prove that ⎪0, x > 2
∞ l sin lx p ⎛ e − ax − e − bx ⎞ ⎩
∫0 (l 2 + a2 )(l 2 + b2 ) d l = 2 ⎜⎝ b2 − a2 ⎟⎠ , 2(1 − cos s)cos s
Ans:
a > 0, b > 0. s2
[Hint: Use FSI with f (t) = eat  ebt.]

12. Find the Fourier sine transform of 7.9 PARSEVAL’S IDENTITY FOR FOURIER
⎧sin x, 0< x<a TRANSFORMS
(a) f ( x ) = ⎨
⎩0, x>a Theorem 7.6 If F (s) and G(s) are the Fourier
⎧ x, 0 < x <1 transforms of f (x) and g(x), respectively, then

(b) f ( x ) = ⎨2 − x, 1< x < 2
⎪0, 1 ∞ ∞
⎩ x>2 (a)
2p ∫−∞ F ( s)G( s)ds = ∫−∞ f ( x) g ( x) dx (7.51)
1 ⎛ sin a(1 − s) sin a(1 + s) ⎞
Ans: (a) ⎜ − ⎟ 1 ∞ ∞
2 ⎝ 1− s 1+ s ⎠
∫−∞ [ F ( s)] ds = ∫
2
(b) 2
f ( x ) dx (7.52)
2p −∞
13. Find the Fourier sine transform of
1 Where g denotes the complex conjugate of g.
f ( x) =
x( x 2 + a2 )
p Proof Let g(x) be the inverse Fourier transform of
Ans: (1 − e − as )
2a 2 G(s) so that
7-20    Engineering Mathematics-II

1 ∞ − isx Using Parseval’s identity prove that


g( x) =
2p ∫−∞ G( s)e dx
∞ ⎛ sin t ⎞
2
p [JNTU 2003 S (1)]
∫0 ⎜⎝ t
⎟⎠ dt = .
2
(a) Taking conjugates on both sides
1 ∞ isx Solution The Fourier transform of f (x) is given by
g ( x) =
2p ∫−∞ G( s)e ds
1
∞ isx ⎛ eisx ⎞
1 isx
∞ ∞ ⎛ 1 ∞ isx ⎞ F { f ( s)} = F ( s) = ∫ f ( x )e dx = ∫ e dx = ⎜ ⎟
∴ ∫−∞ f ( x) g ( x) dx = ∫−∞ f ( x) ⎜⎝ 2p ∫−∞ G( s)e ds⎟ dx

−∞ −1 ⎝ is ⎠ −1
eis − e − is 2
1 ∞ ⎛ ∞ isx ⎞ = = sin s (1)
=
2p ∫−∞ G( s) ⎜⎝ ∫−∞ f ( x )e dx ⎟ ds

is s
Parsevel’s identity for Fourier transform is
1 ∞
=
2p ∫−∞ G ( s) f ( s) ds,
∞ 1 ∞
∫−∞ ∫−∞ F ( s)
2 2
f ( x ) dx = ds
(Changing the order of integration) 2p
2 2
4 ∞ sin s 4 ∞ sin s
by the definition of Fourier transform ( x )1−1 = 2 =
2p ∫−∞ s
ds =
2p
⋅ 2∫
0 s
ds
(b) Setting g(x) = f (x) for all x we obtain 2 2
∞ sin s p ∞ ⎛ sin t ⎞ p
1 ∞ ∞ ⇒ ∫0 ds = ⇒ ∫ ⎜
0 ⎝ t ⎟
dt = ,
2p ∫−∞ F ( s) F ( s) ds = ∫−∞ f ( x) f ( x) dx s 2 ⎠ 2
replacing the dummy variable s by t
(2)

1 ∞ ∞
∫−∞ F ( s) ds = ∫
2 2
⇒ f ( x ) dx Example 7.20
2p −∞
Using Parseval’s identities prove that
7.10 PARSEVAL’S IDENTITIES FOR FOURIER ∞ ⎛ 1 − cos x ⎞
2
p
SINE AND COSINE TRANSFORMS ∫0 ⎜⎝ x
⎟⎠ dx =
2
Theorem 7.7 Similarly, we can obtain the follow-
ing results. Solution The Fourier sine transform of
2 ∞ ∞ ⎧1, 0 < x < 1
p ∫0
1. Fs ( s)Gs ( s) ds = ∫ f ( x ) g ( x ) dx (7.53) f ( x) = ⎨ is given by
0
⎩0, x > 1
2 ∞ ∞ ∞
Fs { f ( x )} = Fs ( s) = ∫ f ( x )sin sx dx
2. ∫
p 0
Fc ( s)Gc ( s) ds = ∫ f ( x ) g ( x ) dx
0
(7.54) 0
1 ∞
2 ∞ ∞ = ∫ 1sin sx dx + ∫ 0sin sx dx
∫ Fs ( s) ds = ∫ f ( x ) dx
2 2 0 1
3. (7.55)
p 0 0 1
⎛ cos sx ⎞ ⎛ 1 − cos s ⎞
= ⎜− ⎟ + 0 = ⎜⎝ ⎟ (1)
2 ∞ ∞ ⎝ s ⎠0 s ⎠
p ∫0
Fs ( s) ds = ∫ f ( x ) dx
2 2
4. (7.56)
0
By Parseval’s identity for Fourier sine transform
Example 7.19 we have
⎧⎪1, x < 1 2 ∞ ∞

p ∫0
Fs ( s) ds = ∫ f ( x ) dx
2 2
If f ( x ) = ⎨ find the Fourier transform.
⎪⎩0, x > 1 0
Fourier Integral Transforms    7-21

a
2 ∞ 1 Fc { f ( x )} = Fc {e − ax } = Fc ( s) = (1)
∫0 Fs ( s) ds = ∫ 12 dx = 1
2
⇒ s + a2
2
p 0
2 Parseval’s identity is
∞ ⎛ 1 − cos s ⎞ p
⇒ ∫0 ⎜⎝
s ⎠
⎟ ds = 2 ∞ ∞

p ∫0
Fc ( s) ds = ∫ Fc ( x ) dx
2 2
2
0
2
∞ ⎛ 1 − cos x ⎞ p
⇒ ∫0 ⎜⎝ x
⎟⎠ dx = (2) 2 ∞⎛ a ⎞ ∞ 2
2

ds = ∫ e − ax dx = ∫ e −2ax dx
2 ⇒
p ∫0 ⎜⎝ 2
s +a 2⎟
⎠ 0 0
where we have replaced the dummy variable s by x. (2)
2 2 ∞ ds ∞ 1
Example 7.21 a ∫ 2 = ∫ e −2ax dx = − (e −2ax )0∞
p 0 s +a 2 0 2a
∞ dx
Evaluate ∫0 ( x 2 + a2 )( x 2 + b2 )
using Parseval’s 1 1
identity =− (0 − 1) = (3)
2a 2a
Solution The
∞ ds p
−ax
Fourier cosine transform of ⇒∫ = (4)
f (x) = e is given by 0 s + a2
2
4a3
a
Fc { f ( x )} = Fc {e − ax } = Fc ( s) = (1)
s + a2
2
EXERCISE 7.2
−bx
and the Fourier cosine transform of g(x) = e is Using Parseval’s identities prove the following:
given by ∞ ( x cos x − sin x )2 p
1. ∫0 x 6
dx =
15
b
Fc {g ( x )} = Fc {e − bx } = Fc ( s) = (2)
s2 + b2 ∞ dx p
Parseval’s identity for Fourier cosine transforms is
2. ∫0 =
( x 2 + 9)( x 2 + 4) 60

∞ x2 p
2 ∞ ∞
∫0 dx =
p ∫0
Fc ( s)Gc ( s) ds = ∫ f ( x ) g ( x ) dx 3.
0 ( x + 1)
2 2
4
2 ∞ a b ∞ ⎧⎪1 − x , x <1
ds = ∫ e − ax e − bx dx
p ∫0 s2 + a2 s2 + b2
⋅ 4. Find the Fourier transform of f ( x ) = ⎨
0
⎪⎩0, x >1

∞ −( a+ b) x ⎡ e −( a+ b) x ⎤ [JNTU 2002S]
=∫ e dx =⎢ ⎥
⎣ −( a + b) ⎦ 0
4
0 ∞ ⎛ sin x ⎞ p
0 −1 1
Hence, deduce that ∫0 ⎜⎝ ⎟ dx =
x ⎠ 3
=
=
−( a + b) a + b
⎧⎪a − x , x <a
∞ ds p 5. Give that f ( x ) = ⎨
⇒ ∫0 ( x 2 + a2 )( x 2 + b2 ) = 2ab(a + b) (3)
⎪⎩0, x >a

∞ sin
4
x
Example 7.22
6. Evaluate ∫0 x 2
dx

∞ dx ∞ ⎛ sin x ⎞ p
2
Evaluate ∫ using Parseval’s identity.
0 (a + x )
2 2 2 Prove that ∫0 ⎜⎝ ⎟ dx =
x ⎠ 2
Solution The Fourier cosine transform of p
Ans:
f (x) = e−ax is given by 2
Z-Transforms and Solution
of Difference Equations 8
8.1 INTRODUCTION A Z B

Let A and B be the domain and range sets of specified


functions. A transform is a mapping T : A → B
from A into B. Recall that we have earlier studied ∞
Laplace and Fourier transforms. These are integral
〈un 〉 –
Σ un z – n
u(z)=
n =0

transforms by which we mean that these mappings


are defined through integrals. Further, these trans-
forms belong to a class of transforms dealing with Figure 8.1 Z-transform of sequence 〈un〉
functions of continuous variables. Now we consider
a transformation called the Z-transformation which for all z in a domain D. Then these values define a
deals with functions of discrete variables. Indeed the complex-valued function u ( z ) on the domain D in
Z-transform is the discrete analogue of the Laplace B, which is called the Z-transform of 〈 u(n) 〉. Then,
transform. Consequently, for every operational the Z-transform of 〈 u(n) 〉 or 〈 un 〉 is
rule and application of Laplace transform we have ∞
an operational rule and application of Z-transform.
The Z-transforms find application in the areas of
Z (〈 un 〉) = ∑ u(n) ⋅ z − n = u ( z ) (8.1)
n= 0
communication systems, control systems, analysis
of signals and linear time-invariant (LTI) systems. Note 1 The transform defined by (8.1) is called a
Further, we have seen that Laplace transforms one-sided (right-sided) or unilateral Z-transform.
are applied to solve initial value problems involv- Note 2 The Z-transform defined for all integers
ing linear differential equations. Now, we will see n = 0, ±1, ±2, ±3, … is called a two-sided or bilate-
how Z-transforms are applied to solve difference ral Z-transform.
equations.
Note 3 One has to show the sequence 〈 un 〉 as the
8.2 Z-TRANSFORM: DEFINITION argument of Z as in (8.1) and not the nth term un.
Let 〈 u(n) 〉, or 〈 u(1), u(2), … u(n) … 〉 or 〈 un 〉 = 〈 u0, In the following we consider only one-sided or
u1, … un, … 〉 be a sequence of real numbers, which unilateral Z-transforms.
is a special function defined on the set of natural
numbers ! = {0, 1, 2, …, n, …} Inverse Z-transform
Suppose that Let the infinite series on the RHS of (8.1) converge
1 1 1 to u ( z ) in a domain D depending on the sequence
u0 + u1 ⋅ + u2 ⋅ 2 + " + un ⋅ n + " 〈 un 〉. Then the mapping is invertible and the inverse
z z z
Z-transform denoted by Z −1 [u ( z )] is defined by
where z is a complex number, a series generated
by the sequence 〈 un 〉. Let the series converge Z −1 [u ( z )] = 〈 u(n) 〉 or 〈 un 〉 (8.2)
8-2    Engineering Mathematics-II

Convergence of the series. An infinite series of the


form

∑ an ( z − a)n = a0 + a1 ( z − a) 1
d (n)
n= 0

+ a2 ( z − a)2 + ! (8.3)
–2 –1 0 1 2 n
where a, a0, a1, a2, … are complex numbers is called Figure 8.2 Graph of unit impulse sequence 〈δn〉
a power series. With each series of the form (8.3) we
can associate a real number r (0 < r < ∞) defined by Unit Step Sequence
1 1/ n
The unit step sequence 〈 H (n) 〉 or 〈 Hn 〉, n = 0, ±1,
= lim sup an , (0 ≤ r < ∞) (8.4) ±2, … is defined by (Fig. 8.3)
r
⎧1 for n ≥ 0
It is called the radius of convergence of the H ( n) or H n = ⎨ (8.8)
series and satisfies the conditions: ⎩0 for n < 0
(i) The series converges absolutely in the open
disc D (a, r) = {z ∈ " | 0 < | z − a | < r}.
1 1 1 1
(ii) the series diverges outside the closed disc H (n)
D (a, r) = { z ∈ " | 0 < | z − a | ≤ r}.
Consequently, the region of convergence (ROC) –2 –1 0 1 2 3 n
of one-sided Z-transform
Figure 8.3 Graph of unit step sequence 〈Hn〉

The relation between the unit impulse sequence and
u ( z) = ∑ un ⋅ z − n (8.5)
the unit step sequence
n= 0
We have
is |z| > a > 0, i.e. the exterior of a circle with centre
at the origin and of radius ‘a’ (‘a’ is a positive real ⎧1 for n = k
d ( n − k ) or d n − k = ⎨ (8.9)
number). ⎩0 for n ≠ k
Similarly, the ROC of a left-sided Z-transform
⎧1 for n ≥ k
0 and H ( n − k ) or H n− k =⎨ (8.10)
⎩0 for n < k
u ( z) = ∑ un ⋅ z − n (8.6)

n = −∞

is the region |z| < b and the ROC of a two-sided (or


∴ H ( n) or H n = ∑ d (k )
k = −∞
bilateral) Z-transform u ( z ) = ∑
0
u z − n is the
n = −∞ n and d ( n) = H ( n) − H ( n − 1)
annular region a < |z| < b where a and b are some
positive real numbers. Also,

f ( n) = ∑ f ( k )d ( n − k ) (8.11)
8.2.1 Two Special Sequences k = −∞

Unit Impulse Sequence


The unit impulse sequence 〈 d (n) 〉 or 〈 dn 〉, n = 0, ±1, 8.2.2 Z-Transforms of Unit Step
±2 … is defined by (Fig. 8.2) and Unit Impulse Sequences
Example 8.1
⎧1 for n = 0
d ( n) or d n = ⎨ (8.7) Find the Z-transform of discrete unit step sequence
⎩0 for n ≠ 0 〈 Hn 〉 where
Z-Transforms and Solution of Difference Equations    8-3

⎧1 for n ≥ 0 Example 8.4


H n=⎨ (8.12) n
⎩0 for n < 0 Find the Z-transform of un = 1/ 2 , − 3 ≥ n ≥ 3 .

Solution We have
Solution By definition

Z (〈 un 〉) =

∑ un z −n
=

∑ Hn z −n
=

∑ 1⋅ z −n Z (〈 un 〉) = ∑ un ⋅ z − n
n = −∞
n= 0 n= 0 n= 0
1 1 1 = u−3 ⋅ z − ( −3) + u−2 ⋅ z − ( −2)
= 1+ + 2 +" + n +"
z z z + u−1 ⋅ z − ( −1) + u0 ⋅ z 0 + u1 ⋅ z −1
(an infinite geometric series with common ratio = 1/ z + u2 z −2 + u3 ⋅ z −3 ,
which converges absolutely for |z| > 1) since n takes values − 3, − 2, − 1, 0, 1, 2, 3 only
1 z 1 1
= = (8.13) = 8z 3 + 4 z 2 + 2 z + 1 + ⋅
1− z z −1
1
2 z
1 1 1 1
+ ⋅ 2+ ⋅ 3
Example 8.2 4 z 8 z
Find the Z-transform of the unit impulse sequence
〈 d (n) 〉 where

⎧1 for n = 0 Example 8.5


d ( n) or d n = ⎨ (8.14) Find the Z-transform of un = (2 / 3)n , n ∈! .
⎩0 for n ≠ 0
Solution We have
Solution By definition ∞
∞ Z (〈 un 〉) = ∑ un ⋅ z − n
Z (〈 un 〉) = ∑ un z − n ⇒ Z (〈d n 〉) n= 0
n= 0 1 1

= u0 ⋅1 + u1 ⋅ + u2 ⋅ 2
z z
= ∑ d n ⋅ z −n = 1+ 0 + 0 + " =1 (8.15)
1 1
n= 0 + u3 ⋅ 3 + " + un ⋅ n + "
z z
2
Example 8.3 2 1 ⎛ 2⎞ 1
= 1⋅ 1 + ⋅ + ⎜ ⎟ 2
Find the Z-transform of 〈 un | 4n = 0 〉 where u0 = −3, 3 z ⎝ 3⎠ z
u1 = 2, u2 = 5, u3 = 0 and u4 = 12. 3 n
⎛ 2⎞ 1 ⎛ 2⎞ 1
+ ⎜ ⎟ 3 +" + ⎜ ⎟ n +"
⎝ 3⎠ z ⎝ 3⎠ z
Solution We have
2 3
∞ ⎛ 2⎞ ⎛ 2⎞ ⎛ 2⎞
= 1+ ⎜ ⎟ + ⎜ ⎟ + ⎜ ⎟
Z (〈 un 〉) = ∑ un z − n = u0 ⋅1 + u1 ⋅ z −1 + u2 ⋅ z −2 ⎝ 3z ⎠ ⎝ 3z ⎠ ⎝ 3z ⎠
n= 0 n
⎛ 2⎞
+ u3 ⋅ z −3 + u4 ⋅ z −4 +" + ⎜ ⎟ + "
⎝ 3z ⎠
Since n takes values n = 0,1, 2,3, 4 only
[which is a GS with c.r. = 2/3z and converges
1 1 1 1
= ( −3)1 + 2 ⋅ + 5 ⋅ 2 + 0 ⋅ 3 + 12 ⋅ 4 absolutely for 2/3z < 1, i.e. z > 2/3]
z z z z
1 1 1 1 3z
= −3 + 2 ⋅ + 5 ⋅ 2 + 12 ⋅ 4 = =
z z z 1− ( )2
3z
3z − 2
8-4    Engineering Mathematics-II

8.3 Z-TRANSFORMS OF SOME 1 z


STANDARD FUNCTIONS = a
=
1− z z − a (8.21)
(SPECIAL SEQUENCES)
8.3.1. Unit Constant Sequence Corollary Putting a = 1 and −1 in turn we get
results 1 and 2, respsectively.
un = 1 " n ∈ ! ;
〈 un 〉 = 〈1, 1, 1, 1, 1," 〉 (8.16)
8.3.4. Natural Number Sequence
∞ ∞
1 un = n ∀ n ∈ ! ;
Z (〈 un 〉) = Z (〈1〉) = ∑ un ⋅ z −n
= ∑ 1⋅
zn 〈 un 〉 = 〈 n 〉 = 〈 0, 1, 2, … 〉 (8.22)
n= 0 n= 0
1 1 1 ∞ ∞
1
= 1+ + 2 +" + n +"
z z z
Z (〈 un 〉) = Z (〈 n〉) = ∑ un ⋅ z − n = ∑ n ⋅ zn
n= 0 n= 0
⎡infinite GS which converges⎤
⎢ absolutely for z >1 ⎥ 1 1 1
⎣ ⎦ = 0 + 1⋅ + 2 ⋅ 2 + " + n ⋅ n + "
z z z
1 z
= = (8.17) 1⎛ 2 3 n ⎞
1− z z −1
1 = ⎜1 + + + " + n −1 ⎟⎠
z ⎝ z z2 z
8.3.2. Alternating Unit Constant Sequence (1 − x ) −2 = 1 + 2 x + 3x 2
un = (−1)n ∀ n ∈ ! ; + " + nx n −1 + "
〈 un 〉 = 〈 (−1)n 〉 = 〈 1, −1, 1, −1, … 〉 (8.18)
( Binomial Theorem )
Z (〈 un 〉) = Z (〈( −1)n 〉) −2
1 ⎛ 1⎞ 1 z2 z
∞ ∞
1 = ⎜1 − ⎟ = ⋅ = ,
= ∑ un ⋅ z − n = ∑ ( −1)n z n z ⎝ z⎠ z ( z − 1) 2
( z − 1)2
n= 0 n= 0 converges absolutely for z > 1
1 1 1 1
= 1 − + 2 − 3 + " + ( −1)n n + " , (8.23)
z z z z
⎡infinite GS which converges ⎤ 8.3.5. Reciprocal Factorial Sequence
⎢ ⎥
⎣absolutely for z > 1 ⎦ 1
1 z un = n ∈! ;
= = (8.19) n!
1 − ( − 1z ) z + 1 1 1 1 1
〈 un 〉 = 〈 〉 = 〈1, , ," , ," 〉 (8.24)
n! 1! 2! n!
8.3.3. Geometric Sequence
un = an ∀ n ∈ ! ; a ∈ $ %or & ; ⎛ 1 ⎞

〈 un 〉 = 〈 an 〉 = 〈 1, a, a2, … an, … 〉 (8.20) Z (〈 un 〉) = z ⎜ 〈 〉⎟ = ∑ un z − n


⎝ n! ⎠ n= 0
∞ ∞
1 ∞
Z (〈 un 〉) = Z (〈 a n 〉) = ∑ un ⋅ z − n = ∑ an ⋅ z n =
1
∑ n! ⋅ z n
1
n= 0 n= 0
n= 0
2 n
a ⎛ a⎞ ⎛ a⎞ 1 1 1 1 1 1
= 1+ + ⎜ ⎟ + " + ⎜ ⎟ + " = 1+ ⋅ + ⋅ 2 +" + ⋅ n +"
z ⎝ z⎠ ⎝ z⎠ 1! z 2! z n! z
⎡infinite GS which converges ⎤ ⎛ t t2 tn ⎞
⎢ ⎥ = e(1/ z ) ⎜# et = 1 + + + " + + " ⎟
⎢absolutely for a < 1(i.e.) z > a ⎥ ⎝ 1! 2! n! ⎠
⎢⎣ z ⎥⎦ (8.25)
Z-Transforms and Solution of Difference Equations    8-5

8.3.6. Power-Cum-Reciprocal a − a a2 − a
Factorial Sequence = 1⋅ e − a + e + e +!
1! 2!
an an − a
un = n ∈" ; + e + ! = e − a e(a / z ) = e a(1/ z −1) (8.31)
n! n!
an a a2 an
〈un 〉 = = 1, , ,! , ,! (8.26) 8.4 RECURRENCE FORMULA
n! 1! 2! n!
FOR THE SEQUENCE OF A POWER
⎛ an ⎞ ∞ OF NATURAL NUMBERS
⎟ = ∑ un z
−n
Z (〈 un 〉) = z ⎜
⎝ n ! ⎠ n= 0 Theorem 8.1 Let un = n p; n, p ∈ " , 〈 un 〉 = 〈 n p 〉 =
n 〈 0. 1 p, 2 p, 3 p … n p, … 〉
an 1
∞ ∞
1 ⎛ a⎞
=∑ =∑ ⎜ ⎟ If Z (〈 un 〉) = Z (〈 n p 〉)
n= 0 n ! z n
n ⎝ ⎠
n= 0 ! z
∞ ∞
∑ un z − n = ∑ n p ⋅ z −n
2 3
1 ⎛ a⎞ 1 ⎛ a⎞ 1 ⎛ a⎞ = (8.32)
= 1+ ⎜ ⎟ + ⎜ ⎟ + ⎜ ⎟

1! z ⎠ ⎝
2! z ⎠ 3! ⎝ z ⎠ n= 0 n= 0

1 ⎛ a⎞
n d
+! + (a / z ) then Z (〈 n p 〉) = z [ Z (〈 n p −1 〉)] or
⎜⎝ ⎟⎠ + ! = e (8.27)
dz
n! z
− z ⋅ Z ′(〈 n p −1 〉) (8.33)
8.3.7. Binomial Coefcient Sequence ∞
un = nCk 0 ≤ k ≤ n
p
Proof Differentiating Z (〈 n 〉) = ∑ n= 0 n p z − n with
respect to ‘z’ we have
〈 un 〉 = 〈 nCk 〉 = 〈 nC0, nC1, nC2, …, nCn 〉
(8.28) d d ⎛ ∞ ⎞
Z ′(〈 n p 〉) = [ Z (〈 n p 〉)] = ⎜ ∑ n p ⋅ z − n ⎟
n n dz dz ⎝ n = 0 ⎠
Z (〈 uk 〉) = Z (〈 nCk 〉) = ∑ uk z − k = ∑ nCk z − k ∞ ∞
k =0 k =0 = ∑ n p ⋅ ( −n) z − n−1 = − z −1 ∑ n p+1 z − n
1 1 n= 0 n= 0
= nC0 ⋅1 + nC1 ⋅ + nC2 ⋅ 2 (8.34)
z z
n
1 ⎛ 1⎞ Replacing p by (p − 1) in (8.7) we get
+ ! + nCn ⋅ = ⎜1 + ⎟
zn ⎝ z⎠ ∞
d
( Binomial Theorem ) (8.29) Z ′(〈 n p −1 〉) = [ Z (〈 n p −1 〉)] = − z −1 ∑ n p ⋅ z − n
dz n= 0

8.3.8. Power-Cum-Reciprocal Factorial = − z −1 ⋅ Z (〈 n p 〉) by (13.5)


Sequence with a Multiple ⇒ Z (〈 n p 〉) = − z ⋅ Z ′(〈 n p −1 〉)
an − a an d
un = e ; 〈un 〉 = 〈 e − a 〉 = − z ( Z 〈 n p −1 〉)! (8.35)
n! n! dz
a a an
= 〈e − a , e − a , e − a ,! , e − a ! 〉 (8.30)
1! 2! n! Corollary 1 Putting p = 1 in (8.6) we get
⎛ an −e ⎞
Z (〈un 〉) = Z ⎜ 〈 e 〉⎟ d d ⎛ z ⎞
⎝ n! ⎠ Z ( 〈 n〉) = − z ⋅ ( Z 〈1〉) = − z ⋅ ⎜ ⎟
dz dz ⎝ z − 1⎠
∞ ∞
an − a ( z − 1) ⋅1 − z ⋅1 z
= ∑ un z − n = ∑ e = −z ⋅ =
n= 0 n= 0 n! ( z − 1)2 ( z − 1)2
8-6    Engineering Mathematics-II

Corollary 2 Putting p = 2 in (8.6) we get Proof By definition,



d d ⎡ z ⎤
Z (〈 n 〉) = − z ⋅ ( Z 〈 n〉) = − z ⋅ ⎢
2

Z ( k 〈 un 〉) = Z (〈 kun 〉) = ∑ (kun )z − n
dz dz ⎣ ( z − 1)2 ⎦ n= 0

( z − 1)2 ⋅1 − z ⋅ 2( z − 1) z2 + z = k ∑ un z − n = k Z (〈un 〉)
= (− z) ⋅ = n= 0
( z − 1) 4
( z − 1) 3

(8.36) Combining the above two results, the linearity


property can be stated as
For p = 3 and 4 we get
Z (k 〈 un 〉 + l 〈 vn 〉) = Z (〈 kun + lvn 〉)
z3 + 4 z2 + z
Corollary 3 Z ( 〈 n3 〉 ) = ; (8.37) = kZ (〈 un 〉) + lZ (〈 vn 〉)
( z − 1)4
and
8.5.2. Change of Scale or Damping Rule
z 4 + 11z 3 + 11z 2 + z
Corollary 4 Z ( 〈 n 〉) =
4
Theorem 8.3 If Z (〈 un 〉) = u (z) then
( z − 1)5
(8.38) Z (〈 a–n un 〉) = u (az) (8.41)

Proof By definition,
8.5 PROPERTIES OF Z-TRANSFORMS
∞ ∞
8.5.1. Linearity Z (〈 a − n un 〉) = ∑ a − n un z − n = ∑ un (az )− n
Theorem 8.2 n= 0 n= 0

(a) Z distributes over sum ⎛ z⎞


Corollary Z (〈 a n un 〉) = u ⎜ ⎟ (8.42)
Z (〈 un + vn 〉) = Z (〈 un 〉 + 〈 vn 〉) ⎝ a⎠
= Z (〈 un 〉) + Z (〈 vn 〉) (8.39) Results Obtained by the Application of Damping Rule
az
(b) Scalar multiplication (a) Z (〈 na n 〉) = (8.43)
( z − a )2
Z (k 〈 un 〉) = k Z (〈 un 〉) (8.40) z
In Z (〈 n 〉) = , replacing z by z / a
(Transform of sum = sum of the transforms) ( z − 1)2
Proof By definition, ( z a) az
we get Z (〈 na n 〉) = =
[( z a) − 1] 2
( z − a )2
(a) Z (〈 un 〉 + 〈 vn 〉) = Z (〈 un + vn 〉)

az 2 + a 2 z
= ∑ (un + vn )z − n (b) Z (〈 n 2 a n 〉 ) =
n= 0 ( z − a )3
∞ ∞
(8.44)
= ∑ un z − n + ∑ vn z − n
n= 0 n= 0 z z +z 2
Replacing z by in Z (〈 n2 〉) =
= Z (〈un 〉) + Z (〈 vn 〉) a ( z − 1)3
Scalar multiplication ( z a )2 + ( z a )
we get Z (〈 n 2 a n 〉) =
(b)  Z (k 〈 un 〉) = k Z (〈 un 〉) [( z a ) − 1]3

(Transform of scalar times a sequence = Scalar a( z 2 + az )


=
times the transform of the sequence) ( z − a )3
Z-Transforms and Solution of Difference Equations    8-7

Note 1 In the following (8.48) and (8.49) involving (d) By the application of the damping rule we
cos and sin functions z is a real variable. obtain, on replacing z by z/a
z ( z − cos t )
(c) (i) Z (〈 cos nt 〉) = 2 z ( z − a cos t )
z − 2 z cos t + 1 (i) Z (〈 a n cos nt 〉) = ;
z 2 − 2az cos t + a2
[JNTU 2001, 2002, S] (8.45)
az sin t
(ii) Z (〈 a n sin nt 〉) =
z sin t z 2 − 2az cos t + a2
(ii) Z (〈sin nt 〉) = (8.46)
z − 2 z cos t + 1
2

(e) By the application of the damping rule to


z ⎛ an ⎞
We have Z (〈1〉) = (8.13) ⎛ 1 ⎞ a /z
z −1 Z⎜ ⎟ = e1/ z we obtain Z ⎜ ⎟ =e .
⎝ n! ⎠ ⎝ n ! ⎠
By the damping rule
( z a) z
Z (〈 a n ⋅1〉) = = (8.47)
( z a) − 1 z − a 8.5.3. Shifting Property
Theorem 8.4
Now Z (〈 e − int 〉) = Z (〈(e − it )n ⋅1〉) (a) First Shifting Theorem (Shifting un to the
z z − eit right)
= ⋅
z − e − it z − eit If Z (〈 un 〉) = u (z) then Z (〈 un−k 〉) = z−k u (z) for
k > 0.
z[ z − (cos t + i sin t )]
=
z 2 − 2 z cos t + 1 (8.48) Proof By definition,
it ∞
 e = cos t + i sin t (Euler’s Theorem) and
it
e +e it
Z (〈 un − k 〉) = ∑ un− k z − n
n= 0
= cos t.
2 = u0 z − k + u1 z − ( k +1) + u2 z − ( k + 2) + !
By Linearity Property (" un = 0 for n < 0, i.e. u− k ,

Z (〈 e −int
〉) = Z (〈 cos nt − i sin nt 〉) ; u1− k , u2 − k , u−1 are zero)

e−int = cos nt − i sin nt (De Moivre’s Theorem) = z − k ∑ un z − n = z − k u ( z )
= Z (〈 cos nt 〉) − iZ (〈 sin nt 〉) n= 0
(8.49)
On equating the real and imaginary parts we Theorem 8.5
obtain from (8.9) and (8.10) (b) Second Shifting Theorem (Shifting un to the
left)
z ( z − cos t )
(i) Z (〈cos nt 〉) = ; If Z (〈 un 〉) = u(z) then Z (〈 un + k 〉) = z k ⎡u ( z ) −
z 2 − 2 z cos t + 1 ⎣
u u
z sin t u0 − z1 − ! − kk −−11 ⎤
z ⎦
(ii) Z ( 〈sin nt 〉) =
z 2 − 2 z cos t + 1
Proof By definition,
Corollary Replacing t by ‘it’ we get ∞ ∞

z ( z − cosh t )
Z (〈 un + k 〉) = ∑ un+ k z − n = z k ∑ un+ k z −( n+ k )
Z (〈cosh nt 〉) = ; n= 0 n= 0
z − 2 z cosh t + 1
2
k −k
= z ⎡⎣uk z + u1+ k z − (1+ k )
z sinh t
Z (〈sinh nt 〉) = + u2+ k z − (2+ k ) + ! ⎤⎦
z 2 − 2 z cosh t + 1
8-8    Engineering Mathematics-II

⎡adding and subtracting ⎤ 8.5.5. Division by n


⎢ −1 −2 − ( k −1) ⎥ Theorem 8.7 If Z (〈 un 〉) = u ( z ) then
⎣u0 + u1 z + u2 z + ! + uk −1 z ⎦
⎡ ∞ k − 1 ⎤ ⎛ u ⎞ z
= z k ⎢ ∑ un z − n − ∑ un z − n ⎥ Z ⎜ n ⎟ = − ∫ t −1u (t ) dt (8.54)
⎝ n ⎠ 0
⎣ n= 0 n= 0 ⎦
= z ⎣u ( z ) − u0 − u1 z − u2 z −2
k ⎡ −1
Proof By definition,
− ! − uk −1 z − ( k +1) ⎤⎦ ⎛ u ⎞ u ⎛ z −n ⎞
Z ⎜ n ⎟ = ∑ n z − n = − ∑ un ⎜ ⎟,
Particular results: For k = 1, 2, 3 we get ⎝ n ⎠ n ⎝ −n ⎠
Z (〈 un+1 〉) = z[u(z) − u0] inserting −1 both in the numerator and denominator
−1
Z (〈 un+2 〉) = z [u(z) − u0 − u1z ]
2
(8.50) z z − n −1
−1 −2 = − ∑ un ∫ t − n −1dt since ∫0 t dt
Z (〈 un+2 〉) = z [u(z) − u0 − u1z − u2z ]
3
0

t −n z z −n
− n ∫0 − n
8.5.4. Multiplication by n = = for n ≠ 0
Theorem 8.6 If Z (〈 un 〉) = u(z) then z
= −∫
0
∑ un ⋅ t − n −1dt , interchanging ∑ and ∫
du ( z )
(∑ u t ) dt = −∫
Z (〈 nun 〉) = − z (8.51) z −1 −n z −1
dz = −∫ t n t u (t ) dt (8.55)
0 0
Proof By definition,
∞ ∞ Now, we deal with two theorems — the initial
Z (〈 nun 〉) = ∑ n ⋅ un z − n = ( − z ) ∑ un ( − n) z − n−1 , value theorem (IVT) and the final value theorem
n= 0 n= 0 (FVT) — which determine the values of un for n = 0
multiplying and dividing by z and for n → ∞, respectively, without the complete
∞ knowledge of un.
⎛d⎞
= ( − z ) ∑ un ⎜ ⎟ ( z − n )
n= 0
⎝ dz ⎠
8.5.6. Initial Value Theorem
d −n
" ( z ) = − nz − n −1 Theorem 8.8 If Z (〈 un 〉) = u ( z ) then
dz

⎛d⎞ u0 = lim u ( z ) (8.56)
= ( − z ) ∑ ⎜ ⎟ (un z − n ) z →∞
⎝ ⎠
n = 0 dz
" un does not contain z Proof By definition,
d ⎛ ∞ ⎞ d u ( z ) = Z (〈 un 〉)
= (− z) ⎜ ∑
dz ⎝ n = 0
un z − n ⎟ = ( − z ) u ( z )
⎠ dz 1 1 1
= u0 + u1 ⋅ + u2 ⋅ 2 + ! + un ⋅ n + !
(8.52) z z z
assuming the uniform convergence of the series, and Taking the limit as z → ∞ we get
that term-wise differentiation is valid.
By the principle of mathematical induction we (a) lim u ( z ) = u0
z →∞
can prove the general result: (b) lim z[u ( z ) − u0 ]
d pu (− z) z →∞
Z (〈 n p un 〉) = ( − z ) p , p = 0, 1, 2, 3,! ⎡ 1 1 ⎤
dz p = lim ⎢u1 + u 2 ⋅ + u 3 ⋅ 2 + ! ⎥ = u1
(8.53) z →∞ ⎣ z z ⎦
Z-Transforms and Solution of Difference Equations    8-9

⎡ u ⎤ Proof By definition,
(c) lim z 2 ⎢u ( z ) − u0 − 1 ⎥
z →∞ ⎣ z⎦ ⎛ ∞ ⎞⎛ ∞ ⎞
u ( z ) ⋅ v ( z ) = ⎜ ∑ un z − n ⎟ ⎜ ∑ vn z − n ⎟
⎡ u u ⎤ ⎝ n= 0 ⎠ ⎝ n= 0 ⎠
= lim ⎢u2 + 3 + 42 + ! ⎥ = u2 (8.57)
z →∞ ⎣ z z ⎦ = u0 v0 + (u0 v1 + u1v0 ) z −1 + (u0 v2 + u1v1
8.5.7. Final Value Theorem + u2 v0 ) z −2 + ! + (u0 vn + u1vn −1 + u2 vn − 2
Theorem 8.9 If Z (〈 un 〉) = u ( z ) then + ! + um vn − m + ! + un v0 ) z − n + !
lim un = lim( z − 1)u ( z ) ∞
n→∞ z →1 = ∑ (u0 vn + u1vn−1 + u2 vn−2
[JNTU 2002 S] (8.58) n= 0

Proof By linearity property + ! + um vn − m + ! + un v0 ) z − n


= Z (〈u0 vn + u1vn −1 + u2 vn − 2 + !
Z (〈 un+1 − un 〉) = Z (〈 un+1 〉) − Z (〈 un 〉)
+ um vn − m + ! + un v0 〉), by definition
= Z [u (z) − u0] − u (z),
⎛ m ⎞
by shifting property =Z⎜

∑ um vn− m ⎟

= ( z − 1)u ( z ) − u0 z m= 0

⇒ ( z − 1)u ( z ) − u0 z = (〈 un +1 − un 〉) Taking inverse Z-transforms of both sides


∞ n
= ∑ (un+1 − un ) z − n , Z −1 (u ( z ) ⋅ v ( z )) = ∑ um vn− m (8.60)
n= 0 m= 0
by definition Example 8.6
Taking the limits as z → 1we have ⎛ 1 ⎞
Evaluate Z⎜ . [JNTU 2001]
∞ ⎝ ( n + 1)! ⎟⎠
lim [( z − 1)u ( z )] − u0 1 = ∑ (un+1 − un ) ⋅1
z →1
n= 0 Solution By definition of Z-transforms
n ∞
= lim
n→∞
∑ (uk +1 − uk ) Z (〈 un 〉) = ∑ un z − n
k =0 n= 0

= lim[(u1 − u0 ) + (u2 − u1 ) 1
n→∞ Here 〈 un 〉 =
( n + 1)!
+ ! + (un +1 − un )]
= lim (un +1 − u0 ) 1 1 1
n→∞
= , ,! ,! (1)
1! 2! ( n + 1)!
= lim un +1 − u0
n→∞ ⎛ ⎞ ∞
1 1 1 1
Hence ∴ Z⎜ ⎟ = ∑
⎝ ( n + 1)! ⎠ n = 0 ( n + 1)!
z − n = + z −1
1! 2!
lim un = lim[( z − 1)u ( z )] (8.59)
n→∞ z →1 1 −2 1
+ z +! + z −n + !
8.5.8. Convolution Theorem 3! ( n + 1)!
⎡1 1 1
Theorem 8.10 = z ⎢ z −1 + z −2 + z −3 + !
⎣1! 2! 3!
Z −1 (u ( z )) = un and Z −1 ( v ( z )) = vn then
1 ⎤
Z −1 (u ( z ).v ( z )) = un * vn = convolution of un and vn + z − ( n +1) + ! ⎥ ,
( n + 1)! ⎦
n
(multiplying and dividing by z)
= ∑ um vn− m (8.60)
m= 0 = z (e1/ z − 1) (2)
8-10    Engineering Mathematics-II

Example 8.7 1 −1 1 −2 1 −n
= 1+ z + z +" + z +"
⎛ 1 ⎞ 2 3 n +1
Find Z ⎜ .
⎝ n + 1 ⎟⎠ ⎛ 1 1 1 − n −1 ⎞
= z ⎜ z −1 + z −2 + z −3 + " + z +" ⎟⎠
Solution By definition of Z-transforms ⎝ 2 3 n +1

(multiplying and dividing by z)
Z (〈 un 〉) = ∑ un z − n (2)
n= 0 ⎡ ⎛ 1⎞ ⎤
= z ⎢ − log ⎜1 − ⎟ ⎥
1 1 1 1 ⎣ ⎝ z⎠ ⎦
Here 〈 un 〉 = = 1, , ," , ," (1)
n +1 2 3 n +1 t2 tn
∞ # − log (1 − t ) = t + +" + +"
⎛ 1 ⎞ 1 −n 2 n
Z⎜ ⎟ =∑ z
⎝ n + 1 ⎠ n= 0 n + 1

Table 8.1 Z-Transforms of some Standard Functions (Special Sequences)


S.No Sequence 〈 un 〉 Z-transform: u(z)
z
1. 〈1〉
z −1
z
2. 〈n〉 2
( z − 1)

3. 〈 n2 〉 (z2 + z) / (z − 1)2
d p −1
4. 〈 np 〉 (p ∈ ! ) −z [ Z (〈 n 〉)]
dz

1 1
5. e z
n!
6. 〈 an 〉 z/(z− a)
d
7. 〈 nun 〉 −z ( u ( z ))
dz
8. 〈 anun 〉 u (z/a)
z
9. 〈 Hn 〉 (Unit step sequence)
z −1
10. 〈 dn 〉 (Unit impulse sequence) 1
11. 〈 un+1 〉 Z ( u (z) − u0)
12. 〈 un+2 〉 z2( u (z) − u0 − u1z−1)
13. 〈 un+3 〉 z3( u (z) − u0 − u1z−1 − u2z−2)
14. 〈 un−k 〉 z-k u (z)

15. u0 lim ( z − 1)u ( z )


z →∞

16. lim un lim ( z − 1)u ( z )


n→∞ z →1
z ( z − cos t )
17. 〈 cos nt 〉 2
z − 2 z cos t + 1
z sin t
18. 〈 sin nt 〉 2
z − 2 z cos t + 1
Z-Transforms and Solution of Difference Equations    8-11

EXERCISE 8.1 9. Evaluate z (〈 nan 〉) using the definition of Z-transform.


az
Denition Ans:
( z − a )2
1. Find the Z-transform of 〈 un 〉, (n = 0, 1, 2, 3, 4) where
u0 = 4, u1 = −5, u2 = 0, u3 = 0 and u4 = 2. Recurrence Formula
5 2 10. Using the recurrence formula Z (〈 n p 〉) = z ( d /dz )
Ans: 4 − +
z z4 [ Z (〈 n p −1 〉)] , find Z (〈 n3 〉) and Z (〈 n4 〉) given that

Z (〈 n2 〉) = z + 2 .
2

Linearity
( z − 1)3
2. Find the Z-transform of 〈 un 〉 where un = 6n + z3 + 4 z2 + z
np Ans: Z (〈 n3 〉) = ;
3sin − 5a2 n ∈ ! . ( z − 1)4
4
z 4 + 11z 3 + 11z 2 + z
z2 + z Z ( 〈 n 4 〉) =
3. Using Z (〈 n 〉) =
2
show that Z (〈 (n + 1)2 〉) = ( z − 1)5
( z − 1)3
z3 + z2 . 8.6 INVERSE Z-TRANSFORM
( z − 1)3 We have already defined the inverse Z-transforms of
n u (z), a function of a complex variable z, by
4. Evaluate Z (〈 (cos q + i sin q) 〉). Hence prove that
z ( z − cosq ) Z−1 ( u (z)) = 〈 un 〉 (8.61)
Z (〈 cos nq 〉) = 2 and
z − 2 z cos q + 1 ∞

Z (〈sin nq 〉) = 2
z sin q
.
which exists provided the series, ∑ n= 0 un z − n con-
z − 2 z cos q + 1 verges and 〈 un 〉 is the sequence generating the series.
[JNTU 2001, 2002S] We will now consider methods for finding the
inverse Z-transforms when a function u(z) is given.
Second Shifting Theorem
8.6.1 Methods for Evaluation
z 2 sin q
5. Show that Z (〈sin( n + 1)q 〉) = . of Inverse Z-Transforms
z − 2 z cos q + 1
2
Method 1: Use of the Table
[JNTU 2002S] of Transforms
The Z-transforms of some special sequences are
Initial Value Theorem and Second
tabulated. If the given function u(z) is one of those
Shifting Theorem
tabulated functions or is a linear combination of
z z them we can readily find the inverse transform.
6. Find Z (〈 un+2 〉) if Z (〈 un 〉) = + .
z + 1 z2 + 1
z ( z 2 − z + 2) Example 8.8
Ans:
( z − 1)( z 2 + 1) −1 ⎛ z ⎞
Find Z ⎜
⎝ z − 1⎟⎠
.
z
Damping Rule Solution Since Z (〈 1 〉) = , the inverse
z −1
7. Evaluate Z (〈e−an sin nq 〉). ⎛ z ⎞
Z-transform of ⎜ is the sequence 〈 1 〉 =
ze a sin q ⎝ z − 1⎟⎠
Ans: 〈 1, 1, 1, 1, 1, … 〉.
2 2a
z e − 2 ze a cos sq + 1
Example 8.9
Multiplication by n Find Z −1 (u(z)) where
n
8. Evaluate Z (〈 na 〉). 3z ⎛ 1 ⎞ 1 z ( z − cos q ) .
u ( z) = + 5e1/ z + 2 z log ⎜ z − ⎟ +
az ( z − 1)2 ⎝ z ⎠ 4 z 2 − 2 z cos q + 1
Ans:
( z − a )2 (1)
8-12    Engineering Mathematics-II

Solution
z+2 ⎫
−1 −1 ⎛ z ⎞ A= 2 1⎪
Z (u ( z )) = 3Z ⎜ 2⎟
+ 5Z −1 (e1 z ) ( z − 2)( z − 3) = = ⎪
z = 0 ( −2)( −3) 3
⎝ ( z − 1) ⎠ ⎪⎪
z+2
⎡ ⎛ 1⎞ ⎤ B= 2+2 ⎬ (3)
+ 2Z −1 ⎢ z log ⎜ z − ⎟ ⎥ z ( z − 3) = = −2 ⎪
⎝ z⎠ ⎦ z = 2 2(2 − 3)
⎣ ⎪
z+2
1 ⎛ z ( z − cos q ) ⎞ C= 3+ 2 5 ⎪
+ Z −1 ⎜ 2 z ( z − 2) = = ⎪
⎝ z − 2 z cos q + 1⎟⎠
,
4 z =3 3(3 − 2) 3 ⎭
by linearity Multiplying both sides of (2) by z and
1 substituting for A, B, C from (3) we have
= 3 ⋅ 〈 n〉 + 5 ⋅ 〈 〉
n! 1 z 5 z
1 1 u ( z) = + ( −2) ⋅ + ⋅ (4)
−2〈 〉 + 〈 cos nq 〉 3 z −2 3 z −3
n +1 4
Now, we take the inverse Z-transforms of both
5 2 1 sides of (4) and obtain
∴ 〈 un 〉 = 〈3n + − + cos nq 〉, (2)
n! n + 1 4 1 5
〈 un 〉 = 〈d n 〉 − 2〈 2n 〉 + 〈3n 〉
is the inverse Z-transform of the given function. 3 3
(5)
1
= d n − 2n −1 + 5 ⋅ 3n −1
Method 2: Method of Partial 3
Fractions by linearity property, where dn is the unit impulse
function.
If u (z) is a rational function of z then put u ( z ) / z
into partial fractions by the usual procedure. Example 8.11
Multiply both sides by z. Write the inverse 3z 2 + z
[JNTU, 2001]
transform of each fraction. By the linearity property, (5 z − 1)(5 z + 2)
their sum is the inverse transform of u (z).
3z 2 + z
Examples
Solution Let u ( z) = (1)
(5 z − 1)(5 z + 2)
Find the inverse Z-transforms of each of the follow-
ing functions (8.10–8.12). write u = 3z + 1
=
A
+
B (2)
z (5 z − 1)(5 z + 2) 5 z − 1 5 z + 2
Example 8.10
z+2 where A and B are constants to be determined.
.
z 2 − 5z + 6 3( 15 ) + 1 ⎫ 8
3z + 1
Solution A= 1= ⎪=
5z + 2 z= 5 ⋅ 15
+2 ⎪ (3)15
z+2 5

Let u ( z) = 3z + 1 3( − 5 ) + 1 1 ⎪
2
z − 5z + 6
2
B= = =
z+2 5 z − 2 z = − 2 5( − 2 ) − 1 15 ⎪
= (1) 5 5 ⎭
( z − 2)( z − 3) Multiplying both sides of (2) by z and
substituting for A and B from (3) we have
Write u ( z) z+2
=
z z ( z − 2)( z − 3) 8 z 1 z
u ( z) = ⋅ + ⋅
A B C 15 5 z − 1 15 5 z + 2
= + + (2)
z ( z − 2) ( z − 3) 8 z 1 z (4)
= ⋅ + ,
where A, B, C are constants to be determined. 75 z − ( 5 ) 75 z − ( − 25 )
1
Z-Transforms and Solution of Difference Equations    8-13


putting the fractions in the form z ( z − a) for division we can write u ( z ) = ∑ n = 0 un z − n so that
inversion. Now we apply the inverse Z-transforms z−1( u(z)) = 〈 un 〉.
on both sides and by linearity property we have
This method is simple and straightforward.
8 −1 ⎛ z ⎞ Example 8.13
Z −1 (u ( z )) = 〈 un 〉 = Z ⎜ 1⎟
75 ⎝ z − 5⎠
−1 ⎛ 10 z ⎞
Find Z ⎜ 2
⎝ z − 32 + 2 ⎟⎠
.
1 ⎛ z ⎞
+ Z −1 ⎜ ⎟
75 ⎝z− −5 ⎠
2
( ) Let u ( z) =
10 z
=
10 z −1
(1)
n n z 2 − 3z + 2 1 − 3z −1 + 2 z −2
8 ⎛ 1⎞ 1 ⎛ 2⎞
= ⎜ ⎟ + ⎜− ⎟ (multiplying the numerator and
75 ⎝ 5 ⎠ 75 ⎝ 5 ⎠
denominator by z−2)
1
〈 un 〉 = 〈(8 − 2n )5− n − 2 〉 (5) By long division
3
10 z −1 + 30 z −2 + 70 z −3 + 150 z −4 + !
Example 8.12 10 z −1
2z
10 z −1 − 30 z −2 + 20 z −3
( z − 1)( z 2 + 1)
30 z −2 − 20 z −3
2z 30 z −2 − 90 z −3 + 60 z −4
Solution Let u ( z) = (1) 1−3z −1 + 2 z −2
( z − 1)( z 2 + 1)
70 z −3 − 60 z −4
u ( z) 2 A Bz + C
write = = + 2 (2) 70 z −3 − 210 z −4 + 140 z −5
2 ( z − 1)( z + 1) z − 1 z + 1
2
150 z −4 − 140 z −5
where A, B and C are constants to be determined.

A( z 2 + 1) + ( Bz + C )( z − 1) = 2⎤ ∴ u (z) = 10z−1 + 30z−2 + 70z−3



Put z = 1, 0 we get A = 1,⎥ + 150z−4 + … (2)
and A + C ( −1) = 2 ⇒ C = −1⎥ −1 −2
= 10 [z + 3z + 7z + 15z + …] −3 −4

Equating the coefficient of z 2⎥ = 10 [(1+ z−1 + 3 . z−2 + 7z−3

A + B = 0 ⇒ B = −1⎦ (3) + 15z−4 + …) −1], adding and
subtracting 1
z z2 + z z z2 z
u ( z) = − 2 = − 2 − (4) = 10 [(20 −1) + (21 −1)z−1 + (22 −1)z−2
z − 1 z + 1 z − 1 z + 1 z2 + 1
+ (23 −1)z−3 + (24 −1)z−4+ …]
Now, we apply the inverse Z-transforms to both

sides of (4) and by linearity we have = 10 ∑ (2n − 1) z − n (3)
np np n= 0
〈 un 〉 = 〈1〉 − cos − sin
2 2 By definition

np np
= 1 − cos
2
− sin
2
(5) u ( z ) = Z (〈 un 〉) = ∑ un z − n (4)
n= 0

Method 3: Long Division Method Comparing (3) and (4) we have the inverse
Z-transform as
If u (z) is a rational function of z and the
denominator cannot be factored, then by long 〈 un 〉 = 10 〈 2n −1 〉 n = 0, 1, 2, … (5)
8-14    Engineering Mathematics-II

Method 4: Inverse Integral Method 〈 un 〉 = Sum of residues


[Contour Integral Method or Method ( −2)n ( −5)n
of Residues] =〈 〉+〈 〉 〈 un 〉
3 −3
Using the theory of complex variables, it can be
( −1)n n
shown that the inverse Z-transform of a function =〈 (2 − 5n )〉
u (z) is given by 3
1
u ( z ) z n −1dz
2pi ∫c
〈 un 〉 = (8.62) Example 8.15

where C is a closed contour in the ROC of u(z) ⎛ z2 + z ⎞


Evaluate Z −1 ⎜ ⎟ using the method of
enclosing the origin and all the n (say) isolated sin- residues. ⎝ ( z − 1)( z + 1) ⎠
2

gularities of the integrand [u(z)zn−1]. But by the


Cauchy’s Residue Theorem the integral in (8.62) is z2 + z
Solution Let u ( z ) .
⎛ n ⎞ ( z − 1)( z 2 + 1)
equal to 2pi ⎜ ∑ k =1 rk ⎟ , where rk is the residue at the
⎝ ⎠ 1
z n −1u ( z ) dz =
2pi ∫c
The inverse transform is 〈 un 〉 =
kth isolated singularity of u(z). z = a is a singularity
of u (z) if u (z) is not analytic (not differentiable or ∑ rk (= sum of the residues)
has no power series expansion) at z = a.
1 z2 + z
= ∫ z n −1 dz
Example 8.14 2pi c ( z − 1)( z 2 + 1)
Evaluate Z −1 ⎛⎜ z ⎞ by contour integral 1 z +1
⎝ ⎟⎠ = ∫ zn dz
method. z 2
+ 7 z + 10 2pi c ( z − 1)( z 2 + 1)
z z
Solution Let u ( z ) = 2 = .
where C is a simple closed curve enclosing the
z + 7 z + 10 ( z + 2)( z + 5)
singularities at z = 1, ±i (which are the zeros of the
The inverse transform is 〈 un 〉 = 1 z n −1u ( z ) dz =
denominator).
2pi ∫c Singularities are at z = 1, ±i which are simple
∑ rk (= sum of the residues) poles (poles of order one)
1 z
= ∫ z n −1 dz r1 = Res (u ( z ) ⋅ z n −1 )
2pi c ( z + 2)( z + 5) z =1
1 zn z +1
= ∫ dz = lim( z − 1) ⋅ z n ⋅ =1
2pi c ( z + 2)( z + 5) z →1 ( z − 1)( z 2 + 1)
where C is a simple closed curve enclosing the r2 = Res (u ( z ) ⋅ z n −1 )
z =i
singularities at z = −2 and −5 (which are the zeros of
z +1
the denominator). = lim( z − i ) ⋅ z n
Singularities are at z = −2, −5 which are simple z →i ( z − 1)( z − i )( z + i )
poles (poles of order one) i n (1 + i ) i n (1 + i ) 1
= = = − in
r1 = Res (u ( z ) z n −1 ) (i − 1)(2i ) −2(1 + i ) 2
z = −2

zn ( −2)n r3 = Res(u ( z ) z n −1 )
= lim ( z + 2) = z = −i
z →−2 ( z + 2)( z + 5) 3 1
n −1
= − ( −i )n ( replacing i by − i in r2 )
r2 = Res (u ( z ) z ) 2
z = −5

zn ( −5)n The required inverse transform


= lim ( z + 5) =
z →−5 ( z + 2)( z + 5) −3 〈 un 〉 = ∑ rk = Sum of the residues
Z-Transforms and Solution of Difference Equations    8-15

1 1 ⎛ z2 ⎞
= 〈1〉 + − i n + − ( −i )n ∴ z −1 ⎜
2 2 ⎟ = 〈 un 〉
⎝ ( z − 1)2 ⎠
1
= 〈1〉 − 〈i n + ( −i )n 〉 = 〈 n + 1〉 n = 0, 1, 2,!
2
1 np np Example 8.18
= 〈1〉 − 2 cos = 1 − cos
2 2 2
−1 ⎛ z
2 ⎞
since Find z ⎜ 2 ⎟ .
np np np np ⎝ z + 1⎠
i n = cos + i sin ; ( −i )n = cos − i sin ;
2 2 2 2
Solution Let
n n np
i + ( −i ) = 2cos z2 1 ⎛ 1⎞
−1
2 u ( z) = = = ⎜1 + 2 ⎟
z2 + 1 1+ 21 ⎝ z ⎠
Method 5: Power Series Method z
1 1 1
= 1− + − +! +!
We expand u(z) in powers of 1 z using a known z 2
z 4
z6

formula and then identify the sequence 〈 un 〉, which
is the inverse Z-transform of u (z).
= ∑ ( −1)n 2 z − n (n is even)
n= 0

Example 8.16 Here the coefficient sequence is 〈 u0, u1, u2, u3, … 〉
⎛ ⎛ z ⎞⎞
Find Z −1 ⎜ log ⎜ np
⎝ z + 1⎟⎠ ⎟⎠
by power series method.
⎝ = 〈1, 0, 1, 0, 1, 0, 1, ! 〉 = cos
2
Solution Let np
⇒ u ( z ) = 〈 un 〉 = cos n = 0, 1, 2, 3, !
⎛ z ⎞ ⎛ z + 1⎞ ⎛ 1⎞ 2
u ( z ) = log ⎜ ⎟ = − log ⎜ ⎟ = − log ⎜1 + ⎟
⎝ z + 1⎠ ⎝ z ⎠ ⎝ z⎠
1 1 1 1 1 ( −1)n − n Method 6: Convolution Method
=− + − ⋅ + ! + z +!
z z z2 3 z3 n When the product of two transforms is given we can
apply the following convolution theorem and find
⎧⎪ 0 for n = 0 the inverse transform.
∴ 〈 un 〉 = Z −1 (u ( z )) = ⎨ ( −1)n
⎪⎩ n for n > 0
Convolution Theorem
Example 8.17
If z −1 (u ( z )) = 〈 un 〉 and
⎛ z ⎞
2
Find Z −1 ⎜ by power series method. z −1 ( v ( z )) = 〈 vn 〉 then
2⎟
⎝ ( z − 1) ⎠ n
Solution Let z −1 (u ( z ) ⋅ v ( z )) = ∑ um vn− m
−2 −2 m= 0
z2 ⎛ z − 1⎞ ⎛ 1⎞
u ( z) = =⎜ ⎟⎠ = ⎜⎝1 − ⎟⎠ = 〈 un 〉 * 〈 vn 〉 (8.63)
( z − 1) 2 ⎝ z z
1 1 1 Example 8.19
= 1 + 2 ⋅ + 3 ⋅ 2 + ! + ( n + 1) n + !
z z z
⎛ z2 ⎞
1 1 1 Find Z −1 ⎜ ⎟ using the convolution
= u0 + u1 ⋅ + u2 2 + ! + un n + !
z z z theorem. ⎝ ( z − a)( z − b) ⎠
8-16    Engineering Mathematics-II

Solution Let EXERCISE 8.2


z Partial Fractions Method
u ( z) = ⇒ 〈 un 〉 = Z −1 (u ( z ))
z−a ⎛ 8z 2 ⎞
1. Find Z −1 ⎜ ⎟.
⎛ z ⎞ ⎝ (2 z − 1)(4 z − 1) ⎠
= Z −1 ⎜ = 〈an 〉
⎝ z − a ⎟⎠ 1 1
Ans: 〈 un 〉 = −
z 2n −1 4n
v ( z) = ⇒ 〈 vn 〉 = Z −1 ( v ( z ))
z−b
⎛ z 2 − 20 z ⎞
⎛ z ⎞ 2. Find Z −1 ⎜ ⎟. [JNTU 2002]
= Z −1 ⎜ = 〈bn 〉 ⎝ ( z − 2)3 ( z − 4) ⎠
⎝ z − b ⎟⎠
Ans: 〈un〉 = 〈2n−1 + 2n . n2 − 22n−1 〉
Then
⎛ z z ⎞ ⎛ 2 z 2 + 3z ⎞
Z −1 ⎜ ⋅ = 〈 un 〉 ∗ 〈 vn 〉 3. Find Z −1 ⎜ ⎟.
⎝ z − a z − b ⎟⎠ ⎝ ( z + 2)( z − 4) ⎠
= 〈 an 〉 ∗ 〈bn 〉 Ans: 〈 un 〉 =
1 11
( −2)n + 4n
∞ n 6 6
= ∑ a m b n − m = b n ∑ ( a b) m , Long Division Method
m= 0 m= 0
a finite GS with c.r. = a b ⎛ 2( z 2 − z ) ⎞
4. Find Z −1 ⎜ 2 ⎟.
⎝ ( z + 1)2 ⎠
( a b)n +1 − 1 a n +1 − b n +1
= bn = np
( a b) − 1 a−b Ans: 〈 un 〉 = 2n sin
2

Example 8.20 ⎛ z ⎞
5. Find Z −1 ⎜ , by division method.
⎛ z3 ⎞ ⎝ ( z + 1)2 ⎟⎠
Find Z −1 ⎜ ⎟ using the convolution
theorem. ⎝ ( z − 3)( z + 1) ⎠
2
〈 un 〉 = 〈 (−1)n−1n 〉 n = 0, 1, 2, …
Solution Let Ans: 〈 un 〉 = 〈 (−1)n−1n〉
z Inverse Integral Method
u ( z) = ⇒ 〈 un 〉 = Z −1 (u ( z ))
z −3 6. Find
⎛ z ⎞
= Z −1 ⎜ = 〈3n 〉 ⎛ z ⎞ np
⎝ z − 3⎟⎠ Z −1 ⎜ 2 〈 u 〉 = 2n 2 sin
⎝ z − 2 z + 2 ⎟⎠ n 4
n = 0, 1, 2,!

z2 np
v ( z) = ⇒ 〈 vn 〉 = Z −1 ( v ( z )) Ans: 〈 un 〉 = 2n 2 sin
z2 + 1 4
⎛ z2 ⎞ np ⎛ z ( z + 1) ⎞
= Z −1 ⎜ 2 ⎟ = cos 7. Find Z −1 ⎜ 〈 un 〉 = 〈 n2 〉 n = 0, 1, 2, …
⎝ z + 1⎠ 2 ⎝ ( z − 1)3 ⎟⎠
⎛ z z2 ⎞ Ans: 〈 un 〉 = 〈 n2 〉
Z −1 ⎜ ⋅ 2 ⎟ = 〈 un 〉 ∗ 〈 vn 〉
⎝ z − 3 z + 1⎠
Power Series Method
n np
= 〈3 〉 ∗ cos z ⎞
8. Find Z −1 ⎛⎜ by the power series method
2 ⎝ z + 1⎟⎠
n
( n − m)p ⎧⎪ 0 for n = 0
= ∑ 3m ⋅ cos 2 〈 un 〉 = ⎨ n2
m= 0 ⎪⎩ ( −1) , n >0
Z-Transforms and Solution of Difference Equations    8-17

⎧ 0 for n = 0 ⇒ yn+2 − yn+1 − 2yn = 3 ∴ Δ yn+1 = yn+2 − yn+1




Ans: nu 〉 = ⎨ ( −1)n
⎪ for n > 0 ⇒ (E2 − E − 2)yn = 3, (E = 1 + Δ) (8.65)
⎩ n

Convolution Theorem 8.7.3 Order of a difference equation


9. Find the inverse Z-transform of z2 by the The order of a difference equation is the difference
convolution theorem method. ( z − 2)( z − 3) of the largest and smallest arguments divided by the
Ans: 〈 un 〉 = 〈 3n+1 − 2n+1 〉 length of the interval.
The order of the equation in Example 1 above is 2.
10. Using the Convolution Theorem evaluate
⎛ z2 ⎞
Z −1 ⎜ ⎟.
8.7.4 General Solution (Complete
⎝ ( z − 1)( z − 3) ⎠ Solution)
1
Ans: 〈 un 〉 = 〈3n +1 − 1〉 The expression for yn satisfying the given difference
2 equation is called its solution. The solution in which
8.7 APPLICATION OF Z-TRANSFORMS: the number of arbitrary constants is equal to the
order of the difference equation is called its General
SOLUTION OF A DIFFERENCE
(Complete) Solution or Integral.
EQUATION BY Z-TRANSFORM
8.7.1 Introduction 8.7.5 Particular Solution
We have earlier seen that the Laplace transform (Particular Integral)
method is quite useful in solving ordinary linear Any solution obtained from the general solution by
differential equations. Now we will see how giving particular values to the arbitrary constants is
Z-transform can be applied in solving linear called a Particular Solution or Particular Integral.
difference equations.
The following are the steps to be followed in
solving difference equations.
8.7.6 Linear Difference Equation
A difference equation in which yn+1, yn+2, … are
1. Apply Z-transform on both sides of the of first degree only and are not multiplied together
difference equation. is called a linear difference equation. If the coeffi-
2. Solve for u(z), using the given initial cients in the equation are constants then the equation
condition. is called a linear difference equation with constant
3. Split the rational expression into partial coefficients. The equation
fractions.
yn+m + a1yn+m−1 + a2yn+m−2 + … + amyn = f (n)
4. Apply inverse Z-transform which gives the
(8.66)
solution sequence 〈 un 〉.
Note 1 The reader may refer to the topic “Finite where a1, a2, …, an are constants is a linear differ-
Differences” in Chapter 6 of the text to know the ence equation with constant coefficients.
meaning of the terms and the symbols used below. If f (n) ≠ 0, equation (8.66) is called a non-
homogeneous equation. Otherwise, it is called a
8.7.2 Difference Equation homogeneous equation or reduced equation.
An equation involving differences of an unknown
function at one or more values of the argument is 8.7.7 Complementary Function
called a difference equation. and Particular Integral
Consider the reduced linear difference equation
E.g.
Δ yn+1 − 2yn = 3 (8.64) yn+m + a1yn+m−1 + a2yn+m−2 + … + amyn = 0 (8.67)
8-18    Engineering Mathematics-II

The Complete (general) Solution of (8.67) is holds. Equation (8.75) is called the auxiliary equa-
tion (AE) or subsidiary equation of the difference
un = c1u1(n) + c2u2(n) + … + cmum(n) (8.68) equation. Since the degree of the equation (8.75) is
where c1, c2, …, cm are constants and u1(n), u2(n), …, m it has m roots l1, l2, …, lm (say). Solving the AE
um(n) are m linearly independent solutions of (8.68). (8.75) we can write the Complementary Function
The Complete Solution of the nonhomogeneous (CF) of the difference equation (8.70) as shown in
equation (8.66) is the table below.
yn = un + vn (CS = CF+ PI) (8.69)
8.8.1(b) Particular Integral
where un is the Complete Solution of the reduced equa-
tion (8.67) and vn is the Particular Integral of (8.66). Operating equation (8.70) by the inverse operator
1 f ( E ) we get the particular integral of equation
8.8 METHOD FOR SOLVING A LINEAR (8.58) as
1
DIFFERENCE EQUATION WITH yn = f (n) (8.77)
CONSTANT COEFFICIENTS f(E )
8.8.1(a) Complementary Function 8.8.2 Short Methods for Finding
Writing yn+k = Ekyn k = 0, 1, 2, …, n equations the Particular Integral
(8.66) and (8.67) can be put in operator notation as
Rule 1 f (n) = an and f (a) ≠ 0 (8.78)
f (E)yn ≡ (E m + a1E m−1 + a2Em−2 + … am1) yn = f (n)(NH)
(8.70) 1 1
m m−1 m−2 …
(1) PI = an = an (replace E by ‘a’)
f (E)yn = (E + a1E + a2E + am1) yn = 0 (H) f(E ) f ( a )
(8.71)
(2) If f (E) ≡ (E − a) y (E) and y (a) ≠ 0 then
Assuming a solution of (8.71) as
yn = ln (8.72) PI =
1 1
an =
1
( na n −1 ) (8.79)
y ( E ) ( E − a) y ( a)
We have Ekyn = ln+k (K = 0, 1, 2, …, m) (8.73)
and we obtain (3) If f (E) = (E − a)2 y (E ) and y (a) ≠ 0 then
(lm + a1lm−1 + a2lm−2 + … + am1)ln = 0 (8.74) 1 1 1
PI = an = n( n − 1)a n − 2
Equation (8.74) will be satisfied if the algebraic y ( E ) ( E − a) 2
y ( a) (8.80)
equation
and so on.
(lm + a1lm−1 + a2lm−2 + … + am1) = 0 (8.75)

S.No. Roots of AE Nature of Roots Complementary Function (CF)


1. l1, l2, ..., lm Real distinct C1λ1n + C2 λ 2n + ! + Cm λ nm
li ∈ R
2. l1, l2, l3, …, lm Two roots equal, others (C1 + C2 n)λ1n + C3λ 3n + ! + Cm λ m
n

distinct

li ∈ R (C1 + C2 n + C3n2 +  + Cs n s −1 )λ1n


3. l1, l1, …, l1 (s roots) ls+1, …, lm
s roots equal, others distinct + Cs +1λns +1 +  + Cn λ m
n

r n (C1 cos nq + C2 sin nq )


where (8.76)
4. a ± ib A pair of complex roots
b
r = a2 + b 2 , q = tan −1
a
Z-Transforms and Solution of Difference Equations    8-19

Rule 2 f (n) = sin an or cos an Example 8.23


eia n − e − ia n 1 ia n Solve yn+2 − 2yn+1 + yn = 0.
Since sin a n = = [(e ) − (e − ia ) n ]
2i 2i Solution In operator notation the equation is
ia n − ia n
e −e 1
and sin a n = = [(eia ) n − (e − ia ) n ] (E2 − 2E +1) ≡ (E − 1)2 yn = 0
2i 2i
apply Rule 1, using eia and e−ia in place of ‘a’ A.E. is (a −1)2 = 0 ⇒ a = 1, 1

Rule 3 f (n) = np ∴ The Required Solution is yn = (C1 + C2n)(1)n


1 1 or yn = C1 + C2n
P .I . = np = n p = f −1 (1 + Δ )n p
f(E ) f (1 + Δ ) Example 8.24
(8.81) The sequence 〈 Fn 〉 = 〈 0, 1, 1, 2, 3, 5, 8, 13,
Expand f−1(1 + Δ) in increasing powers of Δ up 21, … 〉 is called the Fibonacci sequence. Form the
to Δ and express np in the factorial form and apply
p difference equation for the sequence and solve it.
on each term of the expansion. Solution In the given sequence each term beyond
Example 8.21 the second is the sum of the two terms immediately
preceding it. If yn denotes the nth term then
Form a difference equation from
yn = yn−1 + yn−2 (n > 2) (1)
yn = c13n + c2(−1)n (1)
or yn+2 = yn+1 − yn = 0 (n > 0) (2)
Solution Applying the shift operator E twice
We have to solve this under the conditions
Eyn = yn+1 = 3 · C13n − C2(−1)n (2) y1 = 0, y2 = 1
E2yn = yn+2 = 9 · C1 · 3n + C2(−1)n (3) Equation (2) ⇒ (E2 − E −1)yn = 0
The Auxiliary Equation is
Eliminating C1 and C2 from (1)–(3) we get
1± 5
yn 1 1 x2 − x −1 = 0 ⇒ x = (3)
⇒ 12 yn + 8 yn +1 − 4 yn + 2 = 0 2
yn +1 3 −1 = 0 ∴ The Complete Solution yn is
⇒ yn + 2 − 2 yn +1 − 3 yn = 0
yn + 2 9 1 n n
⎛ 1+ 5 ⎞ ⎛1− 5 ⎞
(4, 5) yn = C1 ⎜ ⎟ + C2 ⎜ ⎟ for n > 0 (4)
⎝ 2 ⎠ ⎝ 2 ⎠
Example 8.22
where C1 and C2 are arbitrary constants.
Form a difference equation from
When n = 1, y1 = 0
yn = a·2n + b·3n (1)
⎛ 1+ 5 ⎞ ⎛1− 5 ⎞
Solution Applying the shift operator E twice ∴ C1 ⎜ ⎟ + C2 ⎜ ⎟ =0 (5)
⎝ 2 ⎠ ⎝ 2 ⎠
Eyn = yn+1 = 2a · 2n +3b · 3n (2) When n =2, y2 = 1
n n
E yn = yn+2 = 4a · 2 + 9b · 3
2
(3)
⎛ 1+ 5 ⎞ ⎛1− 5 ⎞
Eliminating a and b from (1)–(3) we get ∴ C1 ⎜ ⎟ + C2 ⎜ ⎟ =1 (6)
⎝ 2 ⎠ ⎝ 2 ⎠
yn 1 1 Solving (5) and (6) we get
⇒ 6 yn − 5 yn +1 + yn + 2 = 0
yn +1 2 3 =0 (4) 5− 5 5+ 5
⇒ yn + 2 − 5 yn +1 + 6 yn = 0 C1 = , C2 = (7)
yn + 2 4 9 10 10
8-20    Engineering Mathematics-II

Hence, the solution under the stated conditions Solution Applying Z-transform we get
n n Z (un+2) − 4Z (un+1) + 3Z (un) = Z (5n),
5 − 5 ⎛ 1+ 5 ⎞ 5 − 5 ⎛1− 5 ⎞
yn = ⎜ ⎟ +
10 ⎝ 2 ⎠ 10 ⎜⎝ 2 ⎟⎠ (8) by linearity (1)
⎛ 1⎞
Example 8.25 ⇒ z 2 ⎜ u ( z ) − u0 − u1 ⎟ − 4 z (u ( z ) − u0 )
⎝ z⎠
Solve yn+2 + 2yn+1 + yn = n with y0 = y1 = 0.
z
Solution Taking Z-transform of both sides, + 3u ( z ) =
z −5
Z (yn+2) + 2Z (yn+1) + Z (yn) = Z (n) by linearity.
⇒ u ( z )( z 2 − 4 z + 3) − u0 ( z 2 − 4 z )
2 −1
⇒ z ( y ( z ) − y0 − y1 z ) + 2 z z
− u1 z =
z z −5
y ( z ) − u0 + y ( z ) = (1)
( z − 1)2 u ( z) 1 u ( z − 4) + u1
⇒ = + 0
Putting y0 = y1 = 0 and solving for y (z) z ( z − 1)( z − 3)( z − 5) ( z − 1)( z − 3)
z y ( z) 1 A B C D E
y ( z) = or = = + + + + (2)
( z − 1)2 ( z + 1)2 z ( z − 1)2 ( z + 1)2 z −1 z − 3 z − 5 z −1 z − 3

1 1
We have to put the rational function into partial Where A= z =1 = (3)
fractions. ( z − 3)( z − 5) 8
1 1 2 1 1
We see that − = (2) B= z =3 = −
z − 1 z + 1 ( z − 1)( z + 1) ( z − 1)( z − 5) 4
Squaring both sides of (1) 1 1
C= z =5 =
( z − 1)( z − 3) 8
1 1 2 4
2
+ 2
− = u0 ( z − 4) + u1 −3u0 + u1
( z − 1) ( z + 1) ( z − 1)( z + 1) ( z − 1) ( z + 1)2
2
D= z =1 =
z −3 −2
1 1 1 1 1 u0 ( z − 4) + u1 −u0 + u1
or = + E= z =3 =
( z − 1)2 ( z + 1)2 4 ( z − 1)2 4 ( z + 1)2 z −1 2
1 1 1 1
− + , by (2) Taking the inverse Z-transforms and substituting
4 z −1 4 z +1 for A, B, C, D, E,
1 z 1 z
∴ y ( z) = ⋅ + ⋅ 1 1 1 1
4 ( z − 1)2 4 ( z + 1)2 un = ⋅1 − ⋅ 3n + 3n + 5n
1 2 1 z 8 4 8 8
− + ⋅ − u1 − 3u0 u1 − u0 n
4 z −1 4 z +1 (3) − ⋅1 + ⋅3
2 2
Taking inverse Z-transform of both sides
⎛ 1 3u − u ⎞ ⎛ u − u 1⎞ 1
1 = ⎜ + 0 1 ⎟ + ⎜ 1 0 − ⎟ 3n + ⋅ 5n (4)
yn = [n − n( −1)n − 1n + ( −1)n ] ⎝8 2 ⎠ ⎝ 2 4 ⎠ 8
4
1 Example 8.27
= ⋅ ( n − 1)[1 − ( −1)n ] (4)
4 Solve un+2 − 4un+1 + 4un = 2n.
Example 8.26
Solution Writing the equation in operator form
Using Z-transform, solve the difference equation
un+2 − 4un+1 + 3un = 5n. (E2 − 4E + 4)un ≡ (E − 2)2 un = 2n (1)
Z-Transforms and Solution of Difference Equations    8-21

The auxiliary equation x2 − 4x + 4 = 0 has two EXERCISE 8.3


equal roots x = 2, 2 1. Solve yn+2 − 7yn+1 + 12yn = 0 given that y0 = 1, y1 = 2.
CF = (c1 + c2n)2n (2) Ans: yn = 2 .3n − 4n

1 n( n − 1) n − 2
PI = 2
2n = 2 = n( n − 1)2n − 3 (3) 2. Solve yn+2 − 5yn+1 − 6yn = 2n.
( E − 2) 2! 1
Ans: yn = c16n + c2 ( −1)n − 2n − 2
3
∴ CS is un = (c1 + c2n)2n + n(n − 1)2n−3 (4)
3. Solve yn+2 − 5yn+1 + 9yn = 3n given that y0 = 0, y1 = 1.
Example 8.28
1
Solve Ans: yn = (5n ⋅ 3n + n2 ⋅ 3n )
18
yn+2 − 2yn+1 + yn = n2 · 2n (1)
4. Solve un+2 − 3un+1 + 2un = 4n given that u0 = 0, u1 = 1.
Solution Writing the equation in operator nota-
tion (E2 − 2 E + 1)yn = n2 · 2n. The auxiliary equation 2 1
Ans: yn = − + 2n −1 + 2n −1
x2 − 2x + 1 = 0 has equal roots x = 1, 1 3 3

CF = c1 + c2n (2) 5. Solve yn+3 − 7yn+2 + 14yn+1 − 8yn = 0.


1 Ans: yn = c1 + c22n + c322n
PI = 2n ⋅ n2
( E − 2)2
6. Solve yn+2 − 5yn+1 + 6yn = n + 2n.
1 1
= 2n n2 = 2n n2. Ans: yn = c1 ⋅ 2n + c2 ⋅ 3n + ⎛⎜
n + 1⎞
(2 E − 1) 2
(1 + 2Δ )2 − n ⋅ 2n −1
⎝ 2 ⎟⎠

= 2n (1 + 2Δ)−2 [n(n − 1) + n]
7. Solve yn+2 − 2yn+1 + yn = 3n + 5 with y0 = 1, y1 = 3.
= 2n(1 − 4Δ + 12Δ2 …) [(n)2 + (n)1]
Ans: yn = (2n + 1) + 1 n( n − 1)( n + 3)
n
= 2 [(n)2 + (n)1 − 4 2(n)1 + 1 + 12 × 2] (3) 2

= 2n [(n)2 − 7(n)1 + 20] = 2n(n2 − 8n + 20) 8. Solve yn+2 − 4yn+1 + 4yn = n2 · 2n with y0 = 2, y1 = 3.

Hence, the complete solution is ⎡ n( n − 1)2 ( n − 2) ⎤ n


Ans: yn = ⎢( n + 2) + ⎥2
yn = c1 + c2n + 2n (n2 − 8n + 20) (4) ⎣ 48 ⎦
Wavelets
9
9.1 INTRODUCTION 
 !)5!%*0!.!/0! %*/%#*(0$0%/*+0
$!0$!+.5+"32!(!0/%/*!3 %0%+*0+0$!)0$! ,!.%+ %!*!.((53!)5$2!/%#*(
)0%(0++(+4"+.!*#%*!!./!!*0(532!(!0/$2! 0$0%/ !*! +2!.0$!!*0%.!.!(0%)!3%0$
"+1* 1/!%*)! %(%)#%*#0!400+/,!!$/5/0!)/ *+,!.%+ %%053%0$%0/!*!.#5!%*#*%0!
#!+(+#%( .!/!.$ .0%%( %*0!((%#!*! *  +)   +.  #%2!* f 3! )5 $2! 0+ $++/! 
,10!.*%)0%+*0$!.,,(%0%+*/+"32!(!0/%*(1 ! 2!.5(.#!0+)+ !(ft5,.0%(/1)+"
/%#*(*(5/%/ 0+),.!//%+*(0!.%*#* !(!0.+ +1.%!./!.%!/
)#*!0%/2!(!0*(5/%/%/-1%'(5&+%*%*#+1.%!.   " 3! .! %*0!.!/0!  %* "+1//%*# +* 0$!
*(5/%//*%),+.0*0)0$!)0%(%*/0.1)!*0 !$2%+1.+"ft%*/+)!*%0!0%)!%*0!.2(
$%*' +"  "1*0%+* !*!  +* 0$! .!( 0%)! +. *!. /+)! ,.0%1(. 0%)! 3! **+0
//%#*("0$!/%#*(+*0%*/+*!"1* )!*0( %/+(0!0$+/!0!.)/%*0$!!4,*/%+*0$0
".!-1!*5 w  0$!* f %/  ,!.%+ % "1*0%+* 3%0$ !/.%!0$%/!$2%+1.*/0! 3!$2!0+
,!.%+  w $!+1.%!./!.%!/+"ft%/+*!0++("+. 0'!0$!!*0%.!/!.%!/+.%0/,.0%(/1)%"3!
*(56%*#0$!/%#*(7/".!-1!*5+*0!*0$!),(%01 ! .!)+ !((%*#0$!/%#*(
/,!0.1)+"f+*/%/0/+",(+0+",+%*0/nw cn 
3$!.!cnan bn 
 an* bn!%*#0$!+1.%!. 1%0!+"0!*3!)5$2!/%#*(+2!.0$!!*0%.!.!(
+!"%!*0/+"f* !.!.0%*+* %0%+*/+*f0$%/ 0%)!3%0$+10*5,!.%+ %%05* 3!.!-1%.!+*(50$0
!
!*(!/1/0+.!,.!/!*00$!/%#*(/0.%#+*+)!0.% %0/!*!.#5!*%0!$%/)!*/0$0 % [ f (t )]2 dt
/!.%!/ %/,(5%*#0$!*01.(".!-1!*%!/
$!
%/*%0!"ft%/+),(!42(1! "1*0%+*0$!*0$!
!
1 !
+* %0%+*%/0$0 % [ f (t )]2 dt !*%0!$%/%*0!#.(
f (t ) ! a0 " " [an cos( nw0 t ) " bn sin( nw0 t )] $!
2 n! 1
%/ 0$! !*!.#5 +*0!*0 +" 0$! /%#*( 1*0%+*/
 

$2%*#*%0!!*!.#5* /0%/"5%*#0$!+* %0%+*0$0
"0!* 3! )+ !( 0$! /%#*( 5 0'%*#  ,.0%( !
/1)+"0$!+1.%!./!.%!/ % [ f (t )]2 dt %/*%0!.!((! square integrable
$!

1 N "1*0%+*/*#!*!.(+1.%!./!.%!/!4,*/%+*/.!
f (t ) # a0 " " [a n cos( nw0 t ) " bn sin( nw0 t )] *+0/1%0(!"+.0$!*(5/%/+"0$!/!"1*0%+*/
2 n! 1
  
9.1.2 Haar Wavelets
9.1.1 Disadvantages with the
* +. !. 0+ +2!.+)! 0$! /$+.0+)%*#/ "+1*  %*
Fourier Series +1.%!. /!.%!/ !4,*/%+*/ 32!(!0 *(5/%/ 3/
$+1#$0$%/,.+!//%/1/!"1(%*)*5/!/0$!+1.%!. %*0.+ 1!  $! 0$!+.5 +10 32!(!0/ 3/ %*0.+
/!.%!/.!,.!/!*00%+*%/*+0(35/0$!!/0 !2%!"+. 1! %*0$!5!.

50$!1*#.%*)0$!)
*(56%*#/%#*(/"+.0$!"+((+3%*#.!/+*/ 0%%*(".!  . $! "1*0%+*/ .! ((!  .
9-2    Engineering Mathematics-II

6!5%+%32!-$2%15%!2!,.$%+.-%!//1.!#(3.3(%   $$)3)5% )$%-3)38  %7)232 )- L


 24#( 3(!3
$%5%+./,%-3.&.3(%138/%2.&6!5%+%3243"%&.1% ff
6%3!*%4/!$)2#422).-.&6!5%+%32+%342)-31.$4#%   $$)3)5% -5%12% ;f %7)232 )- L
 24#( 3(!3
!&%6#.-#%/321%04)1%$)-3()2#.-3%73 f;f
9.2 CHARACTERISTIC FUNCTION OF AN   f∈L

INTERVAL I   ff


(% #(!1!#3%1)23)# &4-#3).- .& !- )-3%15!+ I .1 .&   )231)"43)5%+!6fff
!-8 2%3 .& -4,"%1 .- 1%!+ 3),% $%-.3%$ "8 χ I (t ) 
fgfg
)2$%-%$!2&.++.62
  <ff
⎧1 if t ∈I
 χ I (t ) = ⎨     7!,/+%2.&5%#3.12/!#%2!1%
⎩0 if t ∉ I
  V 9   :
-/!13)#4+!13(%#(!1!#3%1)23)#&4-#3).-.&3(%(!+&
./%-4-)3)-3%15!+)2 
 9 #.2x#.2
x#.2nx:
n
⎧1 for 0 ≤ t < 1  9pnx∑ ar x :
r

χ[0,1) =⎨    r =0
 ⎩0 for t < 0 or t ≥ 1
  9fnxRR:
'1!/(.& )22(.6-"%+.6
x(0.1)
9.3.1 Basis of a Vector Space
- ),/.13!-3 /1./%138 .& ! 5%#3.1 2/!#% )2 3(!3 )3
(!2!basis"!2)2)2!2%3.&5%#3.12)-3(%5%#3.1
1
2/!#%3(!3!1%+)-%!1+8)-$%/%-$%-3!-$3(!32/!-3(%
5%#3.12/!#%()2)$%!)2%04)5!+%-33.3(%&.++.6)-'
0 x
3(%.1%,23!3%$6)3(.43/1..&
Theorem 9.1 %3v"%!5%#3.12/!#%6)3("!2)2
B = [b1 , b 2 ,..., b x ] (%-3(%1%%7)234-)04%2#!+!12
Figure 9.1 Graph of x(0,1)  
k24#(3(!3

9.3 VECTOR SPACE OF FUNCTIONS WITH υ = α1 b1 + α 2 b 2 +! + α k b k  



FINITE ENERGY
&4-#3).-ft$%-%$.-1%!+3),%)22!)$3.(!5% 6(%1% u)2!-85%#3.1)-V-.3(%16.1$23(%1%)2
-)3%%-%1'8.-3(%)-3%15!+Iab )&3(%&.++.6 .-%!-$.-+8.-%6!83.61)3% u!2!+)-%!1#.,")
)-')-%04!+)38)22!3)2%$ -!3).-.&3(%5%#3.12)-B
b .3%3(!33(%2%3B,!8"%)--)3%
 ∫ a
[ f (t )]2 dt < ∞    
(%.41)%12%1)%2%7/!-2).-.&!&4-#3).-ft
(% 2%3 .& &4-#3).-2 6)3( -)3% %-%1'8 .- I  a )-;ttT)2')5%-"8
b  )2 424!++8 $%-.3%$ "8 L
I .1 2),/+8 "8 L
 )& 1 ∞
⎛ nπ t nπ t ⎞
3(%)-3%15!+)2*-.6-()22%3.&&4-#3).-2&.1,2! f (t ) = a0 + ∑ ⎜ an cos + bn sin ⎟  
2 ⎝ T T ⎠
5%#3.1 2!3)2&8)-' 3(% &.++.6)-' /1./%13)%2 %3 f g n =1

!-$h"%&4-#3).-2)-L
!-$!-$2#!+!12(%- 6(%1% 3(% .41)%1 #.%&#)%-32 a an !-$ bn !1%
  +.241%+!6f g∈L
$%-%$"8

 .,,43!3)5%+!6f  g  g  f 1 T ⎛ nπ t nπ t ⎞ 
(a0 , an , bn ) = ∫ f (t ) ⎜1, cos , sin ⎟ dt
  22.#)!3)5%+!6fghfgh  T −T ⎝ T T ⎠ 
Wavelets    9-3

2$%$ %. ) !3(+'! *" !,0/%*)  !-! /$! 9.5.1 Examples
%)/!-1' %. 8T T $! 1!/*- .+! %. ( ! 0+
*" *)/%)0*0. "0)/%*) *) /$! %)/!-1' $! .%.
 
 $!.!/*"-!')0(!-.R0) !-/$!0.0' %/%*)
!'!(!)/. -! *. nptT )  .%) nptT  )  /$! ) (0'/%+'%/%*)9*+!-/%*).
0)%,0!.'-.-!/$!*0-%!-*!"%!)/.#%1!)4  
 $! .!/ *" -/%*)' )0(!-. Q 0) !- /$! 0.0'
!,0/%*)  %/%*)) (0'/%+'%/%*)9*+!-/%*).

9.3.2 Orthogonal/Orthonormal
Vectors 9.6 n-VECTOR SPACE
! ) 1%!2 *)/%)0*0. -!'1'0!  "0)/%*). )*- !-! .!/*"n!'!(!)/.*"F)(!'4a
a 
!)!  *) ) %)/!-1' I  a b . !'!(!)/. *"  anai∈F%.''! )n/0+'!/%.1!/*-''! 
1!/*-.+!) /$!%-*-/$*#*)'%/4*-/$*)*-('%/4 n1!/*-$!.!/*"''n1!/*-.*"!' F%.''! 
(4! !)! ."*''*2. )n1!/*-.+!*1!-/$!!' F !)*/! 4VnF
2*-!'1'0! "0)/%*).ft) gt !)! 
*)Iab-!.% /*!*-/$*#*)'*)I%" 9.6.1 Subspace and Span
!/v
v vr!.!/*"r3! 1!/*-.*"Vn$!)
b ⎧0 if f ≠ g /$!.!/S*"''n1!/*-.*"/$!"*-(
∫ a
f (t ) g (t ) dt = ⎨
⎩k (constant) if f = g

 a v  a v  a v


r r  

) *-/$*)*-('%" 2$!-!ai-!)4.'-.%.1!/*-.0.+!*"Vn
$0.1!/*-.+!%..% /*!.+))! 4/$!
b ⎧0 if f ≠ g
∫ a
f (t ) g (t ) dt = ⎨
⎩1 if f = g
 
 1!/*-.vii
 r$0.1!/*-.+!-%.%)#
"-*('%)!-*(%)/%*)*")4#%1!).!/*"1!/*-.

%..% /*!.+))! 4/$!#%1!).!/*"1!/*-.
!)!.%'4$!&/$/ft.%)t) gt*.t-!
*-/$*#*)')  f (t ) = sin t p ) g(t ) = cos t p 
9.7 SCALING AND TRANSLATION FUNCTIONS
-!*-/$*)*-('*)/$!%)/!-1'I  p
9.7.1 Translation of ()
$!#-+$*"ft8k%./$!#-+$*"ft/-).'/! 
9.4 NORM OF A VECTOR /$-*0#$ %./)!6k60)%/./*/$!-%#$/*-/*/$!'!"/
!/u = (v1 , v2 ,... vn )!1!/*-%)V$!)/$!)*-( *- %)#.k%.+*.%/%1!*-)!#/%1!*).% !-"*-
!3(+'!/$!"*''*2%)#"0)/%*)
*"u !)*/! 466u66%. !)! 4

⎛ n ⎞
1/ 2
⎧t sin t for 0 ≤ t ≤ 15
u = ⎜ ∑ vi2 ⎟ = v12 + v22 + ... + vn2  

 f (t ) = ⎨  

⎝ i =1 ⎠ ⎩0 for t < 0 and t > 15

 #-+$ *" /$! *1! "0)/%*) %. .$*2) %) %#0-!


9.5 FIELD  #%)*).% !-/$!"0)/%*)ft 8 2$*.!
 .!/ F /*#!/$!- 2%/$  %/%*)  )  (0'/%+'% #-+$%..$*2)%)%#0-! '!-'4/$!#-+$*"
/%*)9*+!-/%*). !)! *)%/.!'!(!)/.%.''!  ft 8 %./$!.(!./$/*"ft0/.$%"/! /$-*0#$
!' %"  %./)!*" 0)%/./*/$!-%#$/0-/$!-*).% !-
/$!"0)/%*)ft 2$*.!#-+$%..$*2)%)%#0-!
 
 〈F〉%.)!'%)#-*0+)  !*.!-1!/$//$!#-+$*"ft %./$!
   〈F85 79〉%.#-*0+%)2$%$/$! %./-%0 .(!./$/*"ft0/.$%"/! /$-*0#$ %./)!*"
/%1!'2$*' . 0)%/./*/$!'!"/
9-4    Engineering Mathematics-II

f (t) f (t)

f (t )
15 15

10 10 10

5 5 5

–5 0 10 15 t –5 0 5 10 15 20 t –5 0 t
–5 –5

–10 –10

(a) (b) (c)

⎧ t sin t for 0 ≤ t ≤ 15
Figure 9.2 +*$)" f (t ) = ⎨
⎩0 for t < 0, t > 15
+*$)"ft+*$)"ft−+*$)"ft

9.7.2 Scaling of () − / 


 (  %#.+!   ,$)0,  #+*$ )" ft/3
$!#+*$)"fkt0$!+!k%,,)'!*),%-%/!)(,-(- 0$%$,-+!-$!,-$!#+*$)"f t")+− t )(-)
%,&&! ,&%(#)"-$!#+*$)"ft"k&%!,%(-$! -$!%(-!+/&3
%(-!+/& k
-$!(fkt,-+!-$!,-$!#+*$)"ft")+
at b )(-)-$!%(-!+/&akbk)+!1'*&!)(,% !+ 9.8 HAAR SCALING FUNCTION f(t)
-$!".(-%)( ft !(! %(-$!%(-!+/&,")&&)0, +,&%(#".(-%)(ft%, !(! ,")&&)0,
⎧t sin p t for − 2 ≤ t ≤ 3 ⎧1 for 0 ≤ t < 1
f (t ) = ⎨  
  f (t ) = ⎨  

⎩0 for t < −2 and for t > 0 ⎩0 elsewhere

%#.+!   ,$)0,  #+*$ )" ft %( −   t   #+*$)"ft%,,$)0(%(%#.+! $! )'%()"
%#.+!   ,$)0,  #+*$ )" f t 0$%$ )' ft%,-$!%(-!+/&0,1( -$!".(-%)(ft%,)"(%-!
1
*+!,,!,-$!#+*$)"ft")+3 t )(-)-$!%(-!+/& !(!+#2)( 
i.e.∫ | f (t ) |2 dt!1%,-,( %,(%-!
0

f (t) f (3t) f (t)

10 2 –2

5 –1

t t
–4 –2 0 2 4 1 –10 –5 0 5 10

t
–4 –2 0 2 4

–1

(a) (c)
(b)
Figure 9.3 &%(#)"ft(a) +*$)"ft(b)+*$)"f t(c) +*$)"ft/3
Wavelets    9-5

f (t) 9.10 HAAR WAVELET FUNCTIONS


%2ftx t%$%,%!!0!4%*%2&3,#2)-,
1 yt!1&-**-51

0 1 t ⎧ 1
⎪1, 0≤t<
2
 ⎪
⎪ 1
Figure 9.4 0!.(-&!!01#!*),'&3,#2)-,ft y (t ) = f (2t ) − f (2t − 1) = ⎨ −1, ≤ t <1 
 ⎪ 2

⎪0, for t < 0 and t > 1
⎩  
(%$%,)2)-,-&2(%&3,#2)-,#!,"%13)2!"*7+-$)%$
2-%62%,$)22-#-4%02(%%,2)0%0%!*2)+%),5()#( '0!.(-&yt)11(-5,),)'30% 

#!1%5%(!4%∫ | f (t ) |2 dt%6)121!,$)1,)2%
−∞

9.9 SCALING AND TRANSLATION OF f(t)


Y (t)
9.9.1 Translation of f(t)
3,#2)-,1-&2(%27.%ft 9kk 
 !0% 1
#!**%$2(%20!,1*!2%1-&ft5()#(%6)121&-0!**t),
 (31
0 1 t
⎧1 for k ≤ t ≤ k + 1
 f (t − k ) = ⎨    –1
⎩0 elsewhere

(%0%&-0% ft : k (!1 ,-,8%0- 4!*3% &-0 t ), 2(% Figure 9.5 0!.(-&!!05!4%*%2&3,#2)-,yt
),2%04!*kk 

9.9.2 Scaling of f(t) %62#-,1)$%02(%20!,1*!2% t:k5(%0%k)1!,7


),2%'%0-5
-5#-,1)$%0fkt5(%0%ft)1$%,%$"7%/3!
2)-, !,$k)11-+%,-,8%0-0%!*,3+"%0(%
fkt120%2#(%12(%'0!.(-&ft&-0t -,2-2(% y (t − k ) = f (2(t − k )) − f (2(t − k ) − 1)
),2%04!* /k )&k  !,$#-+.0%11%1)2-,2- = f (2t − 2k ) − f (2t − 2k − 1)
2(%),2%04!*k )&k  ⎧ 1
, .!02)#3*!0 2(% &3,#2)-, f
t (!1 ,-,8%0- ⎪1 for k ≤ t < k +
2
1
4!*3%5(%,t)1),2(%),2%04!* 2!,$8%0-4!*3% ⎪
⎪ 1   
5(%, t )1 -321)$% 2(% ),2%04!* 5()*% f
t :  (!1 = ⎨ −1 for k + ≤ t < k + 1
2
(
,-,8%0-4!*3%5(%,t)1),2(%),2%04!* 3 , 4 ,
5 5 ) ⎪
⎪0 for t < k and t > k + 1
'%,%0!*fat :k(!1!,-,8%0-4!*3%5(%,t*)%1), ⎪

2(%),2%04!*k/ak+ 1/a!≠(31fat :k)1
!1#!*%$!,$20!,1*!2%$4%01)-,-&ft '0!.(-&yt:k)11(-5,),)'30% 
9-6    Engineering Mathematics-II

Y (t-k) %,( (+'1#'-!+,m'n



y
t3myt3n
1
Proof -mn"'-"#'-+/%,mm ('
0"#"yt3m"/'('2+(/%.,'-"#'-+
0 t /%,nn ('0"#"yt3n"/'('/%.,+
K k+ 12 K+1
#,$(#'-(yt3m yt3n (+%%t'
–1

y (t − m) ⋅ y (t − n) = ∫ y (t − m)y (t − n)dt = 0
Figure 9.6 +)"( y t−k*   −∞

!m (m ! 1) "
#&#%+%1 (+m  n-"#'-+/%, # , $'
#2
% 2 & $
!n (n ! 1) "
(0 0 (',#+ -+',%-#(' ' ,%#'! -(!-"+ #, $ (' 0"#" y
t 3 m ' y
t 3 n +
-k'1#'-!+''y
t3k1 %2
# 2 $ &
'('2+(+#,$(#'-',(y
t3m4 y
t3n
y (2t − k ) = f(2(2t − k ) − f (2(2t − k ) − 1) ( )+(/ 
 %- m ' n  '1 #'-!+,  
-"#'-+/%,('0"#"yt3m' y
t3n"/
= f (4t − 2k ) − f (4t − 2k − 1)
'('2+(/%.,+#,$(#'--"'-", .'-#(',+
⎧ k k 1 (+-"(!('%
⎪1 for ∈t < +
2 2 4 "++-0(,,#'0"#"-",#'-+/%,+

⎪ k 1 k +1 '(-#,$(#'-
= ⎨ −1 for + ∈t <    
⎪ 2 4 2 n
⎪ k k +1 Case 1 m " . In this case
2
⎪0 for t < 2 and t ≥ 2

⎧ 1
!+)"( y
t3k#,,"(0'#'#!.+   ⎪1, m ≤ t < m+
4

⎪ 1 1
y (t − m)y (2t − n) = ⎨ −1, m + ≤ t < m +
⎪ 4 2
Y (2t–k)
⎪ 1
⎪0, t < m, t ≥ m + 2
1 ⎩
,(-"-
0 t m +1/4 m +1/2
k k + 1 k+1


2 2 4 2
–1
y (t − m) ⋅ y (2t − n) =
m
1. dt + ∫
m +1/4
( −1)dt = 0

Figure 9.7 +)"( -+',%-#('',%#'! .'-#('


y
t3k Case 2 m ! 1 " n . In this case
2 2
⎧ 1 3
0#%%'(0)+(/-"--"-+',%-#('',%#'! ⎪ −1, m + 2 ≤ t < m + 4

.'-#(',+(+-"(!('%#'L
R ⎪ 3
y (t − m)y (2t − n) = ⎨1, m + ≤ t < m +1
Theorem -mn∈Z'mn"' ⎪ 4
yt3m4yt3n' ⎪ 1 3
 y
t3m4y
t3n ⎪0, t < m + 2 , t ≥ m + 4

Wavelets    9-7

)%**  +(  )%-)*(&)%snts nts


nt
$s nt%$*)#)*%.)%(*)!%
m +3/4
m +1 %#&( )%$*#/%)(,%-(%+*%$*
y (t − m)y (2t − n) =∫ ( −1)dt + ∫ 1 dt = 0 t. )*(&%+() "m%$*(%")*) 0
m +3/4
m +1/2
%* $*(,"%,(- *+$* %$ )$%$0(%
 )%#&"*)*&(%%%**%(# * $*(," ))%(*( m )&%) * ,$ $() $
+$* %$)yt1m$y
t1n%$) ( $"%$( m )$* ,$1m ) $() $
%,(%(*%%$" $L
R+**/%$%*%(# $ +(  n )*!$)&%) * , $*(*$
%#&"*)*)m$n,(/%,( $*() ) ")%  *!$ )  $* , $*( $ -  )
)*$.*$/+) $)" $*%()-  *(&)- ""*%*"*%*,(* ". )
)%$) ("%- *ym,nt
m
m,nt
* )
9.11 SCALING FACTORS OF THE FORM 2m
y m, n (t ) = 2m / 2 ⎣⎡f (2m +1 t − 2n) − f (2m +1 t − 2n − 1) ⎦⎤
$%-%$) (.*$) %$%*)*%+$* %$/
+) $)" $*%()%*%(#
m-(m )$ ⎧ m/2 n n 1
$*( $%((*%%* $+$* %$)***!%$$%$ ⎪2 , for 2m ≤ t < 2m + 2m +1

0(%%$)*$*,"+)%$ $*(,")**$# ⎪ n 1 n 1
)%(*(/*! $m&%) * ,%("%$(/*! $ = ⎨ −2m / 2 , for m + m +1 ≤ t < m + m
⎪ 2 2 2 2
m$* , ⎪ n n 1
⎪0, for t < m t and t ≥ m + m
⎩ 2 2 2
*s m, n (t ) = y (2m t − n) %( $*(m$

$*(n$  +$* %$) ym,nt %(# $ %(*%$%(#" )* %$
L
R)+$* %$)(""(-,"*)$
* +$* %$ f ) "" * )" $ +$* %$")%
s m, n (t ) = f (2m +1 t − 2n) − f (2m +1 t − 2n − 1) * +$* %$ yt  f
t 1 f
t 1  ) "" *
⎧ n n 1 #%*(-,"*
⎪1, ≤t< + m +1
2m 2 m
2 Note +$* %$)ym,nt
n
y
mt 1n%(#

⎪ n 1 n 1 $%(*%$%(#") )%(L
R
= ⎨ −1, + m +1 ≤ t < m + m
⎪ 2m 2 2 2
$%*-,"*)
⎪ n n 1
⎪0, t < 2m
,t≥ m + m 1 ⎛ t − n⎞
⎩ 2 2 y m, n (t ) = y⎜ ⎟
m ⎝ m ⎠
-(m ))" $&(#*($n )"%* %$
sm,n (t)
&(#*(

s3,0 (t) s2,n (t)


9.12 A Wavelet Expansion
1
* ft   )'+( $*(" +$* %$  #/
n n n n n+1
t .&$f $)( )%(-,"*)- %(#
–1
8 4 2
s1,n (t) n2 + 12
%#&"* %(*%$%(#" )* $ L
R $ * %""%- $
n + 1 n + 1 n+ 1 s (t)
8 16 4 8 2 0,n
%(#
∞ ∞

Figure 9.8 m,nt%(m 



f (t ) = ∑ ∑c mn y m, n (t )
m = −∞ x = −∞
9-8    Engineering Mathematics-II

  (,#!+0 !% +  &- (,+!&% !* +&  &) /$'# +  *,*' & ## &%+!%,
,%)*+&& +& $% + + +  *)!* &% +   &,* *(,) !%+)# ,%+!&%* !* %&+ #&*  
&%-)*+&+ ,%+!&%ft !*!$'#!*+ + )*&%&)+ !*!*+ +#!$!+!%+ *%*&$%
2 &%-)% & &%+!%,&,* ,%+!&%* % %&+ 
m ∞
∞ ⎛ ⎞ &%+!%,&,*
lim
m →∞ ∫−∞ ⎜⎝ f (t ) − ∑ ∑ cm, ny m, n (t )⎟⎠ dt = 0 *,*'S!*%&+#&*.%&)$+ 
m = −∞ n = −∞
3*$##*+4*,*'&L
R&%+!%!%##+ ,%
 &!%+*cm,n)&,%!%+ ,*,#.00 +!&%*!%S+&+ ).!+ ##+ #!$!+*&&%-)%+
,*!%+ &)+ &%&)$#!+0&+ ).-#+* *(,%* & ,%+!&%* !% S !* *,*' . ! 
∞ ∞ $0##&L
R!*##+ closure &S+!*
f ⋅ y m0 , n0 = ∑ ∑c mn y m , n ⋅y m0 , n0 = cm0 n0 %&+0 S  S !*#&*,*0!+*&)$+!&%
m = −∞ n = −∞
!+ *##+ #!$!+*&&%-)%+*(,%*&,%
+!&%*+ +)!%+ !**'
9.13 MULTIRESOLUTION ANALYSIS WITH
HAAR WAVELETS 9.16 GENERATION OF A SEQUENCE OF
  ' )* multiresolution analysis ))* +&  CLOSED SUBSPACES OF L2(R) BY
*(,%&#&**,*'*&L
R **, HAAR WAVELETS
*'*))#++&+ *#!%,*!%%!%
) .-#+* %)+  *(,% & #&* *,
*+&.-#+*%+ &%+/+&+ ).-#+*
*'*&L
R *'*)%)+0!)
+ !*$%*+ &##&.!%
%+*#!%*&+ *#!%,%+!&%f .!+ !)%+
"%&.+ +L
R *+ *+),+,)&-+&)
)*&)*&#,+!&%&+ *!%#
*' % +  &##&.!% + ) &%!+!&%* &#
+S&%*!*+&###!%)&$!%+!&%*&+ 
!%L
R
+)%*#+*#!%,%+!&%
  f i∈L
R+ %+ !)#!%)&$!%+!&%*
N

!
N
c f

S0 = ∑ c j f (t − n j )
i! 1 i i )#*&!%L R j =1

. )N!*'&*!+!-!%+)cj ))#%,$)*%

  1)&,%+!&%%0qt &)## nj)!%+)*&)j  
N
t!*!%L
R+*)-**+ 1)&-+&)& V!*+ #&*,)&S+ % V0 ! S 0 
L
R  + !* &) %0 ,%+!&% f !% L
R
+Sm#!%)&$!%+!&%&+ ,%+!&%*
. -f! f
f
mt2nj +!*
   f !* !% L
R + % 2f +  %+!- & f
%02ft 2ft!*#*&!%L
R N
Sm = ∑ c j f (2m t − n j )
j=1
2
9.14 SUBSPACES OF L (R) . )m!*/!%+)%!%+ m+ ,%+!&%
*+S&*(,)!%+)#,%+!&%*!**!+& Sm
*,*'&L
R!S *+#*+&%,%+!&%!%!+ )&$+ *#!%')&')+0ft f
tf 
t2 
%. %-)f%g)!%L
R+ %f – g!*!% .&*)-+ +ft!*!%Vm. %-)f
t!*!%
S&)/$'#+ *+&##&%*+%+$,#+!'#*& Vm % Vm!*&%+!%.!+ !%Vm + 1
X &)$**,*'&L
R  ,*+ #&**,*'*Vm&)$%*%
!% !%
9.15 CLOSED SUBSPACE S ÃV2
ÃV2 ÃVÃV ÃV
Ã
*,*' S!*#&*!&%-)%+*(,%*&  !*  !% * +  &##&.!% +.& !$'&)+%+ !
,%+!&%*!%S -+ !)#!$!+,%+!&%*!%S +!&%#')&')+!*
Wavelets  „  9-9

1. There is a trivial function contained in V0 is a subspace of V1 that contains functions


every Vm (i.e.) the intersection of all the orthogonal to every function in V0. The subspace
closed subspaces Vm consist of only the V1 is called the orthogonal complement of V0. Now,
zero function. consider a scaling function f to produce a mother
2. The ascending chain ends in L2(R). This wavelet y.
implies that every function in L2(R) has Let f(t) be a function satisfying the following
a series expansion in terms of the Haar conditions:
functions.

The spaces Vm are said to form a multiresolu- 1. ∫−∞
f (t ) dt = 1 (1)
tion analysis of L2(R). This multiresolution analysis
is generated by the scaling function f. 2 ∞ 2
Assuming that (Vm | m ∈ Z) is a multiresolution
2. f (t ) = ∫
−∞
f (t ) dt (2)
analysis in L2(R) with scaling function f, the func- 3. The set consisting of f(t) and its translates
tion system ym,n = 2m/2 y(2mt – n) is an orthonormal are orthonormal:
wavelet basis of L2(R). f(t) · f(t – n) = Sn (3)
Since the set {f(t – k): k an integer} consists of
9.17 GENERAL CONSTRUCTION OF linearly independent vectors, it is clear that it consti-
WAVELETS AND MULTIRESOLUTION tutes a basis for a linear space V0.
ANALYSIS Also, {f(23t – n): n ∈ Z} is an orthonormal set
Multiresolution analysis: def for a given integer. Let Vj be a linear space gener-
A sequence of closed subspaces {Vj : j ∈ Z} of L2(R) ated by this set. Then the subspaces ... V–1, V0, V1,
together with a function f ∈ V0 is called a multireso- V2, ... satisfy f(t) ∈ Vj ⇔ f(2t) ∈ Vj+1.
lution analysis (MRA) if it satis¿es the following Let V0 ⊂ V1. This condition implies that every
conditions: vector in V0 belongs to V1. Also, f(t) which is in V0
must be in V1. Therefore, f(t) can be expressed as a
1. Nesting property: ... Ã V2 Ã V1 Ã V0 Ã V1 linear combination of the basis for V1: {f(2t – n): n
à V2 à ... ∈ Z}. Thus
2. Density of union in L2(R): * V j = L2 (R) ∞
j ‰Z f (t ) = ∑ cn f (2t − n) (4)
−∞
3. Separation property:  V j = {0}
j ‰Z This equation is considered as a dilation
4. Scaling property: f(t) ∈ Vj ⇔ f(2t) ∈ Vj+1 equation or a two-scale difference equation since
∀j∈Z f(t) is expressed in terms of its own dilation and
translation.
5. Orthonormal basis:
Again, replacing t by 2jt in equation (4) we get
f(t) ∈ V0 ⇒ f(t − n) ∈ V0 ∀ n ∈ Z

This implies that any Vj, for j ∈ Z, is f (2 j t ) = ∑ c f (2 n
j +1
t − n)
n = −∞
given by translates of normalized dilation:
which implies that the basis for Vj is contained in
2 j / 2 f (2 j t − n) ( j , n ∈ Z) of f. Vj+1. Therefore, the linear space forms a nested
It is possible to construct a multiresolution analysis sequence {... Ã V–2 Ã V–1 Ã V0 Ã V1 Ã V2 ... }.
by an appropriate scaling function f0 and obtain V0 The second condition of the multiresolution
by taking the linear span of integer translates of f. analysis de¿nition implies that the union of all nested
The other subspaces Vj can be generated as scaled linear vector spaces yields a space that is not the same
versions of f0. as the space of signals with ¿nite energy, L2(R), but
9-10  „  Engineering Mathematics-II

is dense in it. Therefore, the union of the subspaces ⎛ 1⎞ sin 2p t − cos p t


y (t ) = f ⎜ t − ⎟ − 2f (2t − 1) =
is dense in L2(R). It can be seen that the initial set of ⎝ 2⎠ ⎛ 1⎞
properties of f(t) along with the dilation equation is p ⎜p − ⎟
⎝ 2⎠
suf¿cient to construct a multiresolution analysis.
Therefore, the Shannon mother wavelet (Figure 9.8)
9.18 SHANNON WAVELETS is given by y(t), which can also be written as
In the case of the Haar wavelets, we have started
with time-limited scaling function f(t) = x[0,1](t). ⎛ 1⎞ ⎛ 1⎞
sin 2p t ⎜ t − ⎟ − cos p ⎜ t − ⎟
Now we start with the frequency-limited Fourier ⎝ 2⎠ ⎝ 2⎠
transform given by y (t ) =
⎛ 1 ⎞
p ⎜t − ⎟
f (w ) = χ (w ) ⎝ 2⎠
[1,0]

whose inverse Fourier transform is given by


1 p sin p t f (t )
f (t ) =
2p ∫ −p
eiwt dt =
pt
Now, a signal f is said to be band-limited if its 0.8

Fourier transform f has compact support, That is, 0.6


for some l,
f (w ) = 0 if ⏐w⏐> l ,where l is to be the
smallest number which is then called the bandwidth
of the signal. The total frequency content of the sig-
t
nal f lies in the band (l, l). –6 –4 –2 0 2 4 6
By the sampling theorem, if f is a continuous and
a band-limited function in L2(R) such that
f (w ) = 0 for
w > lfor some positive constant l then f is determined –0.6
by sampling its values at a discrete set of points.
–0.8
⎛ np ⎞ sin(lt − np )
f (t ) = ∑ f⎜ ⎟ –1.0
n ∈Z
⎝ l ⎠ (lt − np )
Choosing l = ʌ we have Figure 9.9 Shannon mother wavelet


sin p (t − n) ∞ sin 2p t − cos p t
f (t ) = ∑ f ( n)
p (t − n)
= ∑ f (n) f (t − n) y (t ) =
⎛ 1⎞
n = −∞ n = −∞ p ⎜t − ⎟
⎝ 2⎠
Let V0 be the space of band-limited functions

{ }
A graph of the amplitude spectrum of the Shannon
V0 = f ∈ L2 (R ) :

f (w ) = f for w > p mother wavelet is show in Figure 9.9. The frequency


content of this function can be obtained from its
Therefore, {f (t – n) : n ∈Z} is an orthonormal basis Fourier transform
for V0. Let V1 be the space of functions of bandwidth

(w ) = −e iw /2 ⎡ x
y ⎣ [ −2p ,2p ] + x[p ,2p ] ⎤⎦
not exceeding 2p and so on, forming a multiresolu-
tion analysis with scaling function sin p t/p t.
Hence, the Shannon wavelets are the functions
V1 = {f ∈ L2(R):
f (w ) = 0 for w > 2p} scaling
their function by 2t, we get y m , n (t ) = 2m / 2 y (2m t − n)
Wavelets  „  9-11

f (t ) cases of signal processing, data compression, image


processing and in the solution of integral equations.
1.0
0.8 EXERCISE 9.1
0.6 1. Show that sm,n(t) · smƍ,nƍ (t) = 0 if (m, n)  (mƍ, nƍ)

0.4 ∞

0.2
2. Show that f (w ) = ∑ y (2w )
n =1
2.

t 2
–8 –6 –4 –2 0 2 4 6 8 ⎡ Hint: First show that f (w ) 2 = f (2w ) 2 + f (2w ) .⎤
⎣ ⎦
3. Prove (that) any scaling function is an orthogonal
Figure 9.10 Graph of amplitude spectrum of a wavelet system satisfying the relation
Shannon mother wavelet
∑ f (t − n) = 1
n
or
m/2
2 4. Plot a graph for the function y(t – 3).
ψ m , n (t ) = [sin 2t ( 2m t − n) − cos π ( 2m t − n)]
π (t −1/2 ) 5. Plot a graph for the function y(2t – b).
The other types of wavelets are Meyer wave- 6. Suppose f(t) is a scaling function that gives rise to
lets, Daubechs wavelets and Stomberg wavelets. an orthonormal MRA. Determine whether f(–t) also
Different types of wavelets are de¿ned for different gives rise to an orthonormal MRA. How about f(t – m)
speci¿c purposes. These wavelets are applicable in where m is an integer?
QUESTION BANK
MULTIPLE CHOICE QUESTIONS

CHAPTER 1 MATRICES AND LINEAR (c) They are of the same size and of the same rank
EQUATIONS (d) Their ranks are same. Ans: (c)

⎡ 0 1 −6⎤ ⎡0 c −a⎤ 5. A square matrix A = [aij] is upper triangular if


⎢ ⎥ ⎢
1. If A = ⎢ −1 0 5 ⎥ and B = ⎢ −c 0 b ⎥⎥ (a) aij = 0 for i > j (b) aij = 0 for i = j
⎢⎣ 6 −5 7 ⎥⎦ ⎢⎣ a −b 7 ⎥⎦ (c) aij = 0 for i < j (d) aij > 0 for all i, j
and A = B then (a, b, c) = Ans: (a)

(a) (6, 5, 1) (b) (5, 6, 1) ⎡0 0 0⎤


6. The rank of ⎢ ⎥ is
(c) (6, 1, 5) (d) (0, 1, −1) Ans: (a) ⎣0 0 0⎦
(a) 0 (b) 2 (c) 1 (d) 3 Ans: (a)
2. Which one of the following is a scalar matrix?
⎡0 −1 0⎤ ⎡ −3 0 0 ⎤ ⎡1 1 1⎤
(a) ⎢⎢0 1 0⎥⎥ (b) ⎢⎢ 0 −3 0 ⎥⎥ 7. The rank of ⎢1 1 1⎥ is
⎢ ⎥
⎢⎣0 2 0⎥⎦ ⎢⎣ 0 0 −3⎥⎦ ⎢⎣1 1 1⎥⎦
⎡0 4 5⎤ ⎡0 0 1⎤ (a) 3 (b) 2 (c) 1 (d) 0 Ans: (c)
(c) ⎢⎢0 −4 0 ⎥⎥ (d) ⎢⎢0 1 0⎥⎥ Ans: (b)
⎡ 1 2 3⎤
⎢⎣0 0 4⎥⎦ ⎢⎣1 0 0⎥⎦
8. If A = ⎢⎢ 3 4 5⎥⎥ then r (A) =
⎢⎣ 4 5 6⎥⎦
⎡1 0 0⎤
3. The matrix ⎢0 0 0⎥ is a __________ (a) 1 (b) 2 (c) 3 (d) 0 Ans: (b)
⎢ ⎥
matrix. ⎢⎣0 0 0⎥⎦
⎡0 1⎤
9. If A = ⎢ ⎥ then r(A ) =
2
(a) Unit (b) Diagonal ⎣ 1 0 ⎦
(c) Zero (d) Rectangular Ans: (b)
(a) 0 (b) 1 (c) 2 (d) 4 Ans: (c)
4. Two matrices are said to be equivalent if
⎡ 0 2⎤ ⎡ 0 3⎤
(a) They are of the same size and have the 10. If A = ⎢ ⎥ and B = ⎢ ⎥ then r (AB) =
same elements ⎣0 0⎦ ⎣0 0⎦
(b) One is a submatrix of the other (a) 0 (b) 1 (c) 2 (d) 3 Ans: (a)
A-2  !   Engineering Mathematics-II

11. The solution matrix of the equation AX = B where ⎡1 −2⎤


17. If we express A = ⎢ ⎥ as the sum of a
⎡1 1⎤ ⎡ x⎤ ⎡ 2⎤ ⎣3 4 ⎦
A= ⎢ ⎥ X = ⎢ y ⎥ and B = ⎢ 4⎥ is X =
⎣ 2 2 ⎦ ⎣ ⎦ ⎣ ⎦ symmetric matrix P and a skew-symmetric
matrix Q then P =
⎡1 + k ⎤ ⎡ k ⎤ ⎡k ⎤
(a) ⎢ ⎥ (b) ⎢ ⎥ (c) ⎢ ⎥ ⎡ 1⎤
⎣ k ⎦ ⎣1 − k ⎦ ⎣ −k ⎦ ⎢1 − ⎥
⎡1 0⎤ 2
(a) ⎢ ⎥ (b) ⎢ ⎥
⎡ k ⎤ ⎣0 4⎦
(d) ⎢ ⎥ Ans: (d) ⎢1 4 ⎥
⎣2 − k ⎦ ⎢⎣ 2 ⎥⎦
12. The equation AX = 0 has a nontrivial solution if ⎡ 1⎤ ⎡ 1⎤
and only if A is a/an ______ matrix ⎢ −1 2 ⎥ ⎢1 2⎥
(c) ⎢ ⎥ (d) ⎢ ⎥ Ans: (d)
(a) Singular (b) Nonsingular ⎢ 1 −4⎥ ⎢1 4⎥
(c) Nonidentity (d) Identity Ans: (a) ⎣⎢ 2 ⎥⎦ ⎣⎢ 2 ⎦⎥

13. The Echelon matrix among the following is 18. In Question 17 the matrix Q =

⎡ 1 0 7⎤ ⎡1 2 0⎤ ⎡ −5 ⎤
⎢0 2 ⎥ ⎡ 0 5⎤
(a) ⎢0 0 0⎥ (b) ⎢⎢0 0 3⎥⎥ (a) ⎢ ⎥ (b) ⎢ −5 ⎥
⎢ ⎥ ⎢ 0⎥
⎢⎣0 0 2⎥⎦ ⎢⎣0 6 0⎥⎦ ⎢5 0 ⎥ ⎣2 ⎦
⎢⎣ 2 ⎥⎦
⎡ 1 6 3⎤ ⎡0 6 0⎤ ⎡ −5 ⎤ ⎡ 5⎤
(c) ⎢0 7 0⎥ (d) ⎢⎢ 2 0 0⎥⎥ Ans: (c) ⎢0 2⎥ ⎢0 2⎥
⎢ ⎥ (c) ⎢ ⎥ (d) ⎢ ⎥ Ans: (a)
⎢⎣0 0 0⎥⎦ ⎢⎣0 0 0⎥⎦ ⎢ −5 0 ⎥ ⎢5 0⎥
⎢⎣ 2 ⎥⎦ ⎢⎣ 2 ⎥⎦
14. The triangular matrix among the following is
⎡ 1 0 1⎤ ⎡1 6 −1⎤ ⎡1 1 3⎤
⎢ 3 −3⎥⎥ is
(a) ⎢⎢0 0 3⎥⎥ (b) ⎢ 3 −2 0 ⎥⎥ 19. The cofactor of ‘−2’ in A = ⎢ 1

⎣⎢0 0 2⎥⎦ ⎢⎣ 4 0 0 ⎥⎦ ⎣⎢ −2 −4 −4⎥⎦
⎡ 0 0 2⎤ ⎡0 0 0⎤ (a) −10 (b) 10 (c) 12 (d) −12 Ans: (d)
(c) ⎢⎢0 −1 0⎥⎥ (d) ⎢⎢ 0 8 0⎥⎥ Ans: (a)
20. The minor of a32 element in matrix A of
⎢⎣8 4 5⎥⎦ ⎢⎣ −1 4 2⎥⎦ Question 19 is
15. The Elementary matrix among the following is (a) 6 (b) −6 (c) 12 (d) −12 Ans: (b)
⎡ −1 0 0⎤ ⎡1 0 0⎤ 21. (Adj A)−1 =
(a) ⎢ 0 0 1⎥ (b) ⎢0 0 1⎥⎥
⎢ ⎥ ⎢ 1 A
⎣⎢ 0 −1 0⎥⎦ ⎣⎢0 1 0⎦⎥ (a) (b)
A
(c) |A|A (d) |A| Ans: (b)
A
⎡1 0 0 ⎤ ⎡1 0 0⎤
(c) ⎢⎢0 0 −1⎥⎥ (d) ⎢⎢0 0 −1⎥⎥ Ans: (b) 22. (Adj I) =
⎢⎣0 1 0 ⎥⎦ ⎢⎣0 −1 0 ⎥⎦ (a) 1 (b) 0 (c) I (d) Not defined Ans: (c)

16. If r (A) = r1 and r (B) = r2 then r (AB) 23. Adj (kA), if A is an nth order square matrix, is
(a) < min(r1, r2) (b) = max(r1, r2) (a) kn−1(Adj A) (b) kn+1(Adj A)
(c) ≥ max(r1, r2) (d) ≤ min(r1, r2) Ans: (d) (c) kn−2(Adj A) (d) None of these Ans: (a)
Multiple Choice Questions  !   A-3

⎡1 −1 3 ⎤ ⎡ 2 1⎤
30. If A = ⎢ ⎥ then the characteristic equation
24. A 2 × 2 submatrix pair of matrix A = ⎢ 0 2 4 ⎥⎥

⎣ 3 2⎦
is ⎢⎣ 5 −3 −2⎦⎥ satisfied by A−1 is
(a) 2l2 − 3l + 2 = 0 (b) l2 + 4l + 1 = 0
⎡1 −1⎤ ⎡1 −1⎤ ⎡0 2 ⎤ ⎡ 2 4 ⎤
(a) ⎢ , (b) ⎢ ⎥,⎢ ⎥ (c) l2 − 4l + 1 = 0 (d) l2 − 9l + 1=0
⎣0 2 ⎥⎦ ⎢⎣5 2 ⎥⎦ ⎣ 5 −3⎦ ⎣ −3 −2⎦ Ans: (c)
⎡ −1 3⎤ ⎡ 2 4 ⎤ ⎡0 4 ⎤ ⎡1 −3⎤
(c) ⎢ , (d) ⎢ ⎥,⎢ ⎡ 3 −1⎤ ⎡ x ⎤ ⎡ 4 ⎤
⎣2 4⎥⎦ ⎢⎣ −1 3⎥⎦ ⎣ 5 −2⎦ ⎣5 −3⎦

31. The solution of the system ⎢ ⎥⎢ ⎥ = ⎢ ⎥
is (x, y)T = ⎣2 5 ⎦ ⎣ y ⎦ ⎣ −3⎦
Ans: (b)
(a) (1, −1) (b) (−1, 1)
25. If the system AX = B is consistent then for (c) (1, 1) (d) (−1, −1) Ans: (a)
unique solution A must be a _______ matrix
⎡ a −1 0 ⎤
(a) Singular (b) Nonsingular
32. If A = ⎢ 0 a −1⎥ and r (A) = 2 then a =
(c) Upper triangular (d) Lower triangular ⎢ ⎥
Ans: (b) ⎢⎣ −1 0 a ⎥⎦

⎡ −1 0 4⎤ (a) 0 (b) 1 (c) 2 (d) 3 Ans: (b)


26. The matrix ⎢⎢ 5 2 3⎥⎥ is a/an _______ 33. Among the following sets of vectors the one
matrix ⎢⎣ 0 7 2⎥⎦ which is linearly independent is
(a) Nonsingular (b) Singular (a) (1, 2, 1) (0, 1, 2) (3, 7, 5)
(c) Involutory (d) Idempotent Ans: (a) (b) (0, 1, 1) (0, −1, −1) (0, 2, 2)
(c) (2, 1, 1) (−4, −2, −1) (0, 0, −3)
⎡ b −1 + c −1 a −1 1⎤ (d) (1, 2, 4) (2, −1, 3) (0, 1, 2) Ans: (d)
⎢ ⎥
27. If A = ⎢ c −1 + a −1 b −1 1⎥ then det A =
⎢ −1 ⎥ 34. Among the following sets of vectors the one
−1
⎢⎣ a + b c −1 1⎥⎦ which is linearly dependent is
(a) 1 (b) 0 (c) −1 (d) abc Ans: (b) (a) (1, 1, 1) (2, 3, 3) (1, 2, 2)
(b) (1, 1, 1) (2, 1, 1) (0, 1, 2)
⎡0 0 1⎤ (c) (1, 0, 0) (2, 1, 1) (3, 2, −2)
28. The matrix A = ⎢⎢0 0 0⎥⎥ is a/an ______ (d) (0, 0, −1) (1, 1, 0) (2, 0, 0) Ans: (a)
matrix of ______ ⎢0 1 0⎥
⎣ ⎦ 35. If the augmented matrix of a system of NH
(a) Nilpotent, index 2 (b) Nilpotent, index 3 ⎡1 1 1 6 ⎤

equations is [ A B] ∼ ⎢0 0 ⎥
(c) Orthogonal, order 3 (d) Periodic, period 3 2 4 ⎥
Ans: (b) ⎢⎣0 0 a − 3 b − 10 ⎥⎦
⎡ −4 −7 −5⎤ then the system has no solution if
29. The matrix A = ⎢⎢ 2 3 3 ⎥⎥ is a/an _____ (a) a = 3, b ≠ 10 (b) a ≠ 3, b arbitrary
⎢⎣ 1 2 1 ⎥⎦ (c) a = 3, b = 10 (d) a, b arbitrary Ans: (a)
matrix of ______
36. The system in Question 35 has a unique solu-
(a) Nilpotent, index 3 (b) Nilpotent, index 2 tion if _______ Ans: (b)
(c) Orthogonal, order 3 (d) Periodic, period 3
Ans: (a) 37. The system in Question 35 has an infinite num-
[Hint: Characteristic equation l = 0.]
3 ber of solutions if _______ Ans: (c)
A-4  !   Engineering Mathematics-II

38. If the augmented matrix of a system of NH 5. If the characteristic equation of matrix A of


⎡1 1 1 1 ⎤ order 2 is ax2 + bx + c = 0 then the characteris-
equations is [ A B] ∼ ⎢⎢0 1 3 l −1


tic equation of A−1 is
⎢⎣0 0 0 ( l − 1)( l − 2) ⎥⎦ (a) ax2 + bx + c = 0 (b) bx2 + cx + a = 0
then the system is consistent if l = (c) cx2 + bx + a = 0 (d) cx2 + ax + b = 0
Ans: (c)
(a) 1 (b) 2 (c) 1 or 2 (d) None of these
Ans: (c)
6. If the eigenvalues of A are 3, −1, 7 then the
39. In Question 38 for l = 1 the solution set is eigenvalues of (A + I) are
(x, y, z)T = (a) 4, 0, 8 (b) 2, −2, 6
(a) (1 + 2k, −3k, k) (b) (2k, k, −k) (c) 0, 1, 3 (d) 3, −1, 7 Ans: (a)
(c) (1 + 2k, 4k, k) (d) (k, k, k) Ans: (a)
7. The eigenvalues a, b, c of A are in A.P and those
40. In Question 38 for l = 2 the solution set is of A2 are in A.P too. Then b =
(x, y, z)T = 2 2 2 2
(a) a + 2c (b) 2a + c
(a) (2k, 1 − k, k) (b) (3k, 1 + 2k, k)
a + 2c 2a + c
(c) (2k, 1 − 3k, k) (d) (k, −k, k) Ans: (c)
a+c a 2
+ c2
(c) (d) Ans: (d)
a2 + b2 a+c
CHAPTER 2 EIGENVALUES AND
EIGENVECTORS
8. If the product of the eigenvalues of A is zero
1. The sum of the characteristic roots of then A is a _______ matrix.
⎡0 1⎤
A= ⎢ ⎥ is (a) Nonsingular (b) Singular
⎣1 0⎦ (c) Involutory (d) Symmetric Ans: (b)
(a) 1 (b) 2 (c) 0 (d) −1 Ans: (c)
9. If the product of the eigenvalues of A is positive
2. If the sum and the product of the eigenvalues of then A is a ______ matrix.
⎡ a + 1 −1 ⎤
⎥ are s1 and s2 respectively then s1 =
2
⎢ 1 a − 1 (a) Nonsingular (b) Singular
⎣ ⎦ (c) Symmetric (d) Skew-symmetric
(a) 4s2 (b) 2s2 (c) s2 (d) s2 /4 Ans: (a) Ans: (a)
⎡ 2 −2 −4⎤
10. If 2 and 1/2 are the eigenvalues of a 2-square
3. By diagonalisation of A = ⎢⎢ −1 3 4 ⎥⎥ we
matrix A then those of A−1 are
⎡0 0 0⎤ ⎣⎢ 1 −2 −3⎦⎥
obtain D = ⎢⎢0 1 0⎥⎥. From this we conclude (a) 2, 4 (b) 2, 1
(c) 1, 1 (d) 1, −1
2
that A is ⎢⎣0 0 1⎥⎦ Ans: (b)

(a) Symmetric (b) Skew-symmetric 11. If the characteristic equation of a 2-square


(c) Singular (d) Involutory Ans: (c) matrix A is ax2 + bx + c = 0 then that of 3A is

4. A is an n-square matrix with tr. A = 0 then the (a) ax3 + 3bx + 9c = 0


coefficient of ln−1 in its characteristic equation is (b) 3ax2 + bx + c = 0
(c) 3ax2 + 9bx + c = 0
(a) 1 (b) 0 (c) −1 (d) n Ans: (b) (d) ax2 + 3bx + 6c = 0 Ans: (a)
Multiple Choice Questions  !   A-5

⎡ 3 −9 −12⎤ 18. The algebraic multiplicity of the eigenvalue


12. If 2 is an eigenvalue of ⎢⎢1 a 4 ⎥⎥ then a = ⎡1 0 2⎤
l = 2 of the matrix ⎢⎢ −1 2 2⎥⎥ is
⎣⎢0 0 1 ⎦⎥
⎢⎣ −1 0 4⎥⎦
(a) 0 (b) 4 (c) −6 (d) −7 Ans: (d)
(a) 0 (b) 1 (c) 2 (d) 3 Ans: (c)
⎡ 3 − 2 −9
⎢ Hint : A − 2 I = 0 ⇒ ( −1) =0 [Hint: Expanding by c2
⎢⎣ 1 a−2 1− l 2
⎤ A − lI = (2 − l )
⇒ a = −7.⎥ −1 4 − l
⎦ = −( l − 2)2 ( l − 3).]
⎡1 1 −1⎤ 19. If x3 − s1x2 + s2x − s3 is the characteristic poly-
13. The latent roots of ⎢⎢0 2 −1⎥⎥ are ⎡1 2 1⎤
⎢⎣0 0 −2⎥⎦ nomial of ⎢0 3 5⎥ then (s1, s3) =
⎢ ⎥
(a) 1, 0, 0 (b) 1, 2, −2 ⎢⎣0 0 4⎥⎦
(c) 0, 0, −2 (d) 1, 1, −1 Ans: (b)
(a) (8, −12) (b) (−8, −12)
14. A is similar to D = diag (1, −1, 3) then the proper (c) (−8, 12) (d) (8, 12) Ans: (d)
values of A2 are
⎡ 4 2⎤
20. If the product of the eigenvalues of A = ⎢ ⎥
(a) 1, −1, 3 (b) 2, −2, 6
⎣ 6 5⎦
(c) 1, 1, 9 (d) 12 , −12 , 32 Ans: (c) is divisible by the product of the eigenvalues of
⎡8 −3⎤
⎡1 2 2 ⎤ B=⎢ ⎥ then a =
⎣0 a ⎦
15. If ‘0’ is a latent root of ⎢0 a 1 ⎥ then a =
⎢ ⎥ (a) ±2 (b) 2, 1 (c) ±1 (d) 0, 1 Ans: (c)
⎢⎣0 1 −1⎥⎦

(a) 2 (b) 0 (c) 1 (d) −1 Ans: (d)


⎡ −1 0 ⎤
⎡ a 1 ⎤ 21. If A = ⎢ ⎥ then the eigenvalues of B =
⎢ Hint : 0 = ⇒ a = −1.⎥ ⎣ 1 −4⎦
⎣ 1 −1 ⎦ AT + I are

⎡ 2 1⎤ (a) −1, −4 (b) ±1 (c) ±3 (d) 0, −3


16. The eigenvalues of ⎢ ⎥ are the roots of the Ans: (d)
equation ⎣1 5⎦ [Hint: λ1 = −1, λ2 = −4 are e.v.s of A
(a) x2 + 7x − 9 = 0 (b) x2 − 7x + 9 = 0 ⇒ − 1 + 1 = 0, − 4 + 1 = −3 are those of B
(c) x2 + 7x + 9 = 0 (d) x2 − 7x + 10 = 0
Ans: (b) 22. If l1, l2 are distinct eigenvalues of A then the
⎡ 2 1 ⎤ corresponding eigenvectors X1, X2 of A are
⎢ Hint : x − (2 + 5) x + = 0.⎥
2

⎣ 1 5 ⎦ (a) Linearly independent (b) Linearly dependent


(c) Orthogonal (d) Orthonormal Ans: (a)
⎡ 0 2⎤
17. If X1 and X2 are the eigenvectors of A = ⎢ ⎥
then X1 and X2 are ⎣ 2 0⎦ ⎡ 1 ⎤ ⎡1⎤
23. If ⎢ ⎥ ⎢ ⎥ are the eigenvectors of A then those
(a) Collinear (b) Orthogonal ⎣ −1⎦ ⎣1⎦
(c) Identical (d) Linearly dependent Ans: (b) of A2 are
A-6  !   Engineering Mathematics-II

1
2
1
2 ⎡12 ⎤ ⎡ 12 ⎤ 31. If A is a nonsingular matrix with li as its eigen-
(a) ⎡ ⎤ , ⎡ ⎤ (b) ⎢ ⎥ , ⎢ ⎥ values then the eigenvalues of Adj A are
⎢ −1⎥ ⎢1⎥ 2 2
⎣ ⎦ ⎣⎦ ⎣⎢1 ⎦⎥ ⎣⎢( −1) ⎦⎥
⎡0⎤ ⎡0⎤ 1 A
⎡ 1 ⎤ ⎡1⎤ (a) (b) (c) l2i (d) l−2i Ans: (b)
(c) ⎢ ⎥ , ⎢ ⎥ (d) ⎢ ⎥ , ⎢1⎥ Ans: (c) λi λi
⎣ −1⎦ ⎣1⎦ ⎣0⎦ ⎣ ⎦

24. −1, −2 are two eigenvalues of the third-order ⎡1 2 ⎤


32. If A = ⎢ ⎥ then A = xI where x =
4
square matrix A with det A = 6. The eigenvalues ⎣2 −1⎦
of −A are
(a) 1 (b) 5 (c) 25 (d) 625 Ans: (c)
(a) 1, −2, −3 (b) −1, −2, 3
(c) −1, 2, −3 (d) 1, 2, −3 Ans: (d)
CHAPTER 3 REAL AND COMPLEX MATRICES
25. The characteristic equation of second-order Symmetric/Skew-Symmetric
matrix A is ax2 + bx + c = 0 then that of −A is 1. If A and B are nth order symmetric matrices
then BT AB is a/an _______ matrix.
(a) a + bx + cx2 = 0 (b) ax2 − bx + c = 0
(c) ax2 + bx − c = 0 (d) −ax2 + bx + c = 0 (a) Singular (b) Skew-symmetric
Ans: (b) (c) Symmetric (d) Orthogonal Ans: (c)

26. The characteristic equation of a second-order 2. If A is a nonsingular symmetric matrix then


singular matrix A is ax2 + bx + c = 0. Then A−1 is a/an ______ matrix.
(a) a = 0 (b) b = 0 (a) Skew-symmetric (b) Symmetric
(c) c = 0 (d) b = c = 0 Ans: (c) (c) Orthogonal (d) Unit Ans: (b)

⎡ 5 1⎤ 3. If A and B are symmetric and are commuta-


27. If matrix A = ⎢ ⎥ then the latent roots of B,
⎣0 5⎦ tive w.r.t multiplication then AB is a/an ______
which is similar to A, are matrix.

(a) 0, 5 (b) 4, 6 (c) 5, −5 (d) 5, 5 (a) Orthogonal (b) Idempotent


Ans: (d) (c) Skew-symmetric (d) Symmetric Ans: (d)

28. If the characteristic equation of a third-order 4. The eigenvalues of a real symmetric matrix are
matrix A is al3 + bl + c = 0 then tr. A = (a) Real (b) Zero (c) Imaginary
−c
(a) −b (b) (c) 0 (d) 1 Ans: (c) (d) Real or complex Ans: (a)
a a
5. The eigenvalues of a real skew-symmetric
29. If the characteristic equation of a second-order matrix are
matrix A is al2 + bl + c = 0 then the product of
the eigenvalues of A is (a) Purely imaginary (b) Real
−c c (c) Purely imaginary or zero
(a) c (b) (c) −c (d) Ans: (d) (d) Real or complex Ans: (c)
a a
30. If a, b, c are the elements of a nonsingular diag- 6. If A is a square matrix then AAT is a/an ________
onal matrix D then the eigenvalues of D−1 are matrix.
(a) (1, 1, 1) (b) (−a, −b, −c) (a) Symmetric (b) Skew-symmetric
(c) (a−1, b−1, c−1) (d) (a, b, c) Ans: (c) (c) Orthogonal (d) Involutory Ans: (a)
Multiple Choice Questions  !   A-7

7. If A is a square matrix then (A + AT ) is a/an ⎡a b 4⎤


________ matrix. ⎢ ⎥
15. If ⎢ −7 0 −5⎥ is skew-symmetric then (a, b) =
⎣⎢ −4 5 0 ⎥⎦
(a) Skew-symmetric (b) Symmetric
(c) Orthogonal (d) Involutory Ans: (b)
(a) (0, 10) (b) (0, −7) (c) (0, 7)
8. If A is a square matrix then (A − AT ) is a/an (d) (10, 7) Ans: (c)
______ matrix.
16. A square matrix A is idempotent if
(a) Skew-symmetric (b) Symmetric
(c) Orthogonal (d) Involutory Ans: (a) (a) A3 = A (b) A2 = A (c) A2 = I
m+1
(d) A = 0 Ans: (b)
2
9. If A is symmetric then A is a/an _______
matrix. 17. Unit matrix is an example of _____ matrix

(a) Idempotent (b) Nilpotent (a) Symmetric (b) Idempotent


(c) Nilpotent (d) Symmetric Ans: (d) (c) Symmetric, idempotent (d) Orthogonal
Ans: (c)
10. If A is skew-symmetric then A2 is
18. The one matrix which is idempotent among the
(a) Idempotent (b) Nilpotent following is
(c) Periodic (d) Symmetric Ans: (d)
⎡ 1 1⎤
1 T ⎢ 2 − 2⎥ ⎡1 1 ⎤
11. If A = P + Q where P = (A+ A ) and Q = (a) ⎢ ⎥ (b) ⎢
2 ⎥
1 ⎢− 1 1 ⎥ ⎣1 −1⎦
(A− AT ) then P and Q are ______, ______
2 ⎣⎢ 2 2 ⎦⎥
matrices respectively.
⎡ 0 2⎤ ⎡0 −1⎤
(c) ⎢ ⎥ (d) ⎢ ⎥ Ans: (a)
(a) P: symmetric, Q: skew-symmetric ⎣0 0⎦ ⎣1 0 ⎦
(b) P is skew-symmetric and Q is
skew-symmetric
(c) P and Q are symmetric ⎡ −1 1 −1⎤
(d) P and Q is skew-symmetric Ans: (a) 19. The matrix ⎢⎢ 3 −3 3 ⎥⎥ is a /an ______
matrix. ⎢⎣ 5 −5 5 ⎥⎦
12. A square matrix A = [aij]m×m is called symmetric
if (a) Periodic (b) Idempotent
(c) Orthogonal (d) Nilpotent Ans: (b)
(a) A = AT (b) AT = A−1
(c) A−1 = A (d) A−T = A Ans: (a) 20. A square matrix A is periodic if

13. If A is a skew-symmetric matrix then tr. A = (a) Am+1 = 0 (b) AT = A−1


(c) Am = mA (d) Am+1 = A Ans: (d)
(a) 0 (b) 1 (c) positive (d) negative
Ans: (a) 21. Unit matrix is not an example of one of the fol-
⎡ 3 a b⎤ lowing; which one ?
14. If ⎢⎢ −2 2 4⎥⎥ is symmetric then (a, b) = (a) Symmetric (b) Idempotent
⎢⎣ 7 4 5⎥⎦ (c) Periodic (d) Orthogonal Ans: (d)

(a) (2, 7) (b) (2, −7) 22. Among the following which one is a periodic
(c) (−2, 7) (d) (7, 2) Ans: (a) matrix with period 4?
A-8  !   Engineering Mathematics-II

⎡0 −1⎤ ⎡1 1⎤ ⎡1 2 3⎤
(a) ⎢ ⎥ (b) ⎢
⎣1 0 ⎦ ⎣1 1⎥⎦ 29. The matrix ⎢ 1 2 3 ⎥ is _______ matrix
⎢ ⎥
⎡ 1 −1⎤ ⎡1 0⎤ ⎣⎢ −1 −2 −3⎦⎥
(c) ⎢ ⎥ (d) ⎢ Ans: (a)
⎣ −1 1 ⎦ ⎣1 1⎥⎦ with _______
(a) Nilpotent, index 2
23. If A is idempotent then its transpose is (b) Periodic, period 3
(a) Involutory (b) Idempotent (c) Orthogonal, order 3
(c) Orthogonal (d) Symmetric Ans: (b) (d) Idempotent, index 2 Ans: (a)
1− l 2 3
⎡1 0 0⎤ [Hint: n A − lI = 1 2−l 3 if
24. The matrix ⎢⎢0 1 0⎥⎥ is a/an _______ matrix. −1 −2 −3 − l
⎢⎣1 0 0⎥⎦ we put l = 0, three rows become L.D. so that
(a) Idempotent (b) Involutory l 2 = 0 ⇒ A2 = 0.]
(c) Orthogonal (d) Symmetric Ans: (a)
⎡ ab b2 ⎤
⎡ 2 −2 −4⎤ 30. The matrix ⎢
2
⎥ is _______ matrix
25. The matrix ⎢⎢ −1 3 4 ⎥⎥ is a/an ______ ⎢⎣ − a − ab ⎥⎦
with ______
matrix. ⎢⎣ 1 −2 −3⎥⎦
(a) Orthogonal, order 2
(a) Involutory (b) Orthogonal (b) Idempotent, index 2
(c) Idempotent (d) Symmetric Ans: (c) (c) Nilpotent, index 2
(d) Periodic, period 2 Ans: (c)
⎡1 b⎤
26. The matrix ⎢ ⎥ is a/an _______ matrix for ⎡ a − b −( a + b) ⎤
all real b ⎣0 0⎦ 31. The matrix ⎢ − a b
⎢ ( a + b) ⎥⎥ is ______
(a) Idempotent (b) Involutory ⎢⎣ a −b −( a + b)⎥⎦
(c) Orthogonal (d) Symmetric Ans: (a) matrix with _______
⎡ −1 3 5 ⎤ (a) Orthogonal, order 3 (b) Nilpotent, index 2
27. The matrix ⎢ 1 −3 −5⎥ is a/an _______ (c) Nilpotent, index 3 (d) Periodic, period 2
⎢ ⎥ Ans: (b)
matrix. ⎢⎣ −1 3 5 ⎥⎦
[Hint: Please see hint under Question 29.]
(a) Involutory (b) Idempotent
(c) Orthogonal (d) Skew-symmetric Ans: (b) 32. A square matrix A is Involutory if
(a) A2 = A (b) A3 = A (c) A2 = I
28. A square matrix A such that An = 0 is called a
(d) A2 = 0 Ans: (c)
nilpotent matrix, and the least n for which this
holds is called the index of nilpotent matrix. 33. The one matrix which is not Involutory among
⎡1 1 3⎤ the following.
Find the index for A = ⎢⎢ 5 2 6 ⎥⎥ .
⎡ 0 1 −1⎤
⎢⎣ −2 −1 −3⎥⎦ ⎡6 5⎤
(a) ⎢ ⎥ (b) ⎢ 4 −3 4 ⎥
⎣ −7 −6⎦ ⎢ ⎥
(a) 1 (b) 2 (c) 3 (d) 4 Ans: (c) ⎢⎣ 3 −3 4 ⎥⎦
Multiple Choice Questions  !   A-9

⎡ −5 −8 0 ⎤ ⎡1 2 2 ⎤ 38. If A is an idempotent matrix then An = I pro-


vided n =
(c) ⎢⎢ 3 5 0 ⎥⎥ (d) ⎢2 1 −2⎥ Ans: (d)
⎢ ⎥
Ans: (b)
⎣⎢ 1 2 −1⎦⎥ ⎢⎣2 −2 1 ⎥⎦ (a) 1, 0 (b) 2, 0 (c) ±1 (d) 3, 0

39. If A is an idempotent matrix then An = A pro-


34. A square matrix A is orthogonal if vided n =

(a) AAT = I (b) A2 = I (c) AAT = 0 (a) ±1 (b) 3, 2 (c) 2, 1 (d) 1, 3 Ans: (c)
(d) AT = A Ans: (a)
⎡ −1 0 ⎤
40. The matrix ⎢ ⎥ ______ is a/an _____
⎡1 0 0 ⎤ matrix ⎣ 0 −1⎦

35. The matrix A = ⎢0 sin q cos q ⎥⎥ is a/an
(a) Nilpotent (b) Idempotent
⎣⎢0 − cos q sin q ⎦⎥ (c) Orthogonal (d) Involutory Ans: (d)
______ matrix.
⎡0 −i ⎤
(a) Orthogonal (b) Idempotent 41. The matrix A = ⎢ is a/an _______
(c) Involutory (d) Periodic Ans: (a) matrix. ⎣ −i 0 ⎥⎦

(a) Hermitian (b) Skew-Hermitian


36. The matrix which is not orthogonal among the (c) Nilpotent (d) Idempotent Ans: (a)
following is
⎡ a 0 b⎤
⎡ 3 −4 4 ⎤ ⎡1 2 2 ⎤
1 ⎢ 42. The matrix ⎢ 0 1 0 ⎥ is orthogonal if
(a) ⎢ 0 −1 0 ⎥ (b) ⎢ 2 1 −2⎥⎥ ⎢ ⎥
⎢ ⎥ 3 ⎢⎣ −b 0 a ⎥⎦
⎢⎣ −2 2 −3⎥⎦ ⎢⎣2 −2 1 ⎥⎦

⎡1 0 0 ⎤ (a) a2 + b2 = 1 (b) a2 − b2 = 1
⎢ (c) a + b2 = 1 (d) a − b =1
cos q ⎥⎥
Ans: (a)
(c) ⎢0 sin q
⎢⎣0 − cos q sin q ⎥⎦ ⎡1 0 0⎤
⎡ −1 1 1 1 ⎤ 43. The matrix ⎢⎢0 b a ⎥⎥ is orthogonal if
⎢ ⎥
1 1 −1 1 1 ⎥
(d) ⎢ Ans: (a) ⎣⎢0 − a b ⎥⎦
2 ⎢ 1 1 −1 1 ⎥ (a) b = 1 − a (b) b = 1 − a2
⎢ ⎥
⎣ 1 1 1 −1⎦ (c) b2 = 1 − a2 (d) b2 = 1 + a Ans: (c)

44. A square matrix which is its own inverse is a/an


⎡ 0 2b c ⎤ _______ matrix.
⎢ ⎥
37. If A = ⎢ a b −c ⎥ is orthogonal then
(a) Orthogonal (b) Involutory
⎢⎣ a −b c ⎥⎦
(c) Indempotent (d) Periodic Ans: (c)
(|a|, |b|, |c|) =
45. The determinant of an orthogonal matrix is
⎛ 1 1 1 ⎞ ⎛ 1 1 1 ⎞
(a) ⎜ , , (b) ⎜ , , (a) 1 (b) −1 (c) ±1 (d) 0 Ans: (c)
⎝ 3 2 6 ⎟⎠ ⎝ 2 6 3 ⎟⎠
⎛ 1 1 1 ⎞ 46. The determinant of an Involutory matrix is
(c) ⎜ , , (d) (1, 1, 1) Ans: (b)
⎝ 6 3 2 ⎟⎠ (a) 1 (b) −1 (c) ±1 (d) 0 Ans: (a)
A-10  !   Engineering Mathematics-II

⎡ p p ⎤ 53. The diagonal elements of _______ matrix are


⎢ 0 2sin
6
sin
6 ⎥ imaginary or zero.
⎢ ⎥
p p p
47. The matrix ⎢⎢sin − sin ⎥ is a/an
sin (a) Hermitian (b) Skew-Hermitian
4 6 6⎥ (c) Unitary (d) Symmetric Ans: (b)
⎢ ⎥
⎢sin p p p
− sin sin ⎥
⎣⎢ 4 6 6 ⎥⎦ 54. If A and B are Unitary matrices of same size
______ matrix. then AB is _____ matrix.

(a) Orthogonal (b) Idempotent (a) Hermitian (b) Skew-Hermitian


(c) Involutory (d) Nilpotent Ans: (a) (c) Unitary (d) Skew-symmetric Ans: (c)

55. Row or column vectors 〈 Xi|i = 1, 2 … n 〉


⎡1 1 1⎤
p⎢ ⎥ of a/an ______ matrix satisfy the condition
48. The matrix sin ⎢1 w w 2 ⎥ where w is a Xi · Xj = dij.
6⎢ ⎥
2
⎣1 w w⎦ (a) Hermitian (b) Skew-Hermitian
cube root of unity if a/an _______ matrix. (c) Unitary (d) Symmetric Ans: (c)
(a) Orthogonal (b) Idempotent
(c) Unitary (d) Nilpotent Ans: (c) ⎡ 0 1 + 2i ⎤
56. If A = ⎢ then (I −A) (I + A)−1 is
⎣ −1 + 2i 0 ⎥⎦
⎡0 −1⎤ a/an ______ matrix.
49. The matrix ⎢ ⎥ is a/an ______ matrix.
⎣1 0 ⎦ (a) Unitary (b) Idempotent
(c) Involutory (d) Periodic Ans: (a)
(a) Idempotent (b) Periodic
(c) Nilpotent (d) Orthogonal Ans: (b)
CHAPTER 4 QUADRATIC FORMS
50. If A and B are orthogonal matrices of the same 1. A real quadratic form in n variables is a
type then AB and BA are such that _______ expression in n variables with ______
coeffcients.
(a) AB is orthogonal but BA is not
(b) BA is orthogonal but AB is not (a) Second degree; real
(c) Both AB and BA are orthogonal (b) Second degree; complex
(d) Neither AB nor BA is orthogonal Ans: (c) (c) Second degree homogeneous; real
(d) Second degree homogeneous; complex
Ans: (c)
⎡ 2 3 + 4i ⎤
51. The eigenvalues of ⎢ are
⎣3 − 4i 2 ⎥⎦ 2. The quadratic form in 2 variables among the
(a) Imaginary (b) Complex following
(c) Complex conjugates (d) Real or zero (a) 3x + 5xy − 2y2 (b) 8x2 − 5y + 3y2
Ans: (d) (c) x2 − y3 + 2xy (d) 2x2 − 4xy +5y2
Ans: (d)
⎡ 2i −4 + 3i ⎤
52. The eigenvalues of ⎢ are
⎣ 4 + 3i 2i ⎥⎦ 3. The symmetric matrix of the quadratic form XT
AX = x2 + 6xy +5y2 is A =
(a) Imaginary (b) Complex
(c) Complex conjugates (d) Real or zero ⎡1 3⎤ ⎡ 1 6⎤
(a) ⎢ ⎥ (b) ⎢ ⎥
Ans: (c) ⎣3 5⎦ ⎣ 0 5⎦
Multiple Choice Questions  !   A-11

⎡1 2⎤ ⎡5 1⎤ 8. If the matrix of the quadratic form y2 + 2z2 +


(c) ⎢ ⎥ (d) ⎢ ⎥ Ans: (a) ⎡ 0 a 0⎤
⎣ 3 5⎦ ⎣5 1⎦
10xy + 12yz is ⎢⎢ a 1 b ⎥⎥ then (a, b) =
4. The symmetric matrix of the quadratic form XT ⎢⎣ 0 b 2⎥⎦
AX = 2x1x2 + 8x1x3 − x2x3 is A =
(a) (5, 4) (b) (3, 4) (c) (4, 5) (d) (5, 6)
⎡ 0 −1 2⎤ ⎡ 0 1 −4⎤ Ans: (d)
⎢ ⎥
(a) ⎢ −1 0 3⎥ (b) ⎢⎢ 1 0 2 ⎥⎥
9. The rank of the quadratic form x2 + 3y2 + 3z2 −
⎢⎣ 2 3 0⎥⎦ ⎢⎣ −4 2 0 ⎥⎦
2yz is
⎡ ⎤
⎢0 1 4 ⎥ (a) 1 (b) 2 (c) 3 (d) 0 Ans: (c)
⎢ ⎥ ⎡0 2 8 ⎤
(c) ⎢ 1 0 − 1 ⎥ (d) ⎢ 2 0 −1⎥ Ans: (c) 10. The nature of the quadratic form 2xy + 2yz +
⎢ 2⎥ ⎢ ⎥
⎢ ⎥ ⎢⎣8 −1 0 ⎥⎦ 2zx is
1
⎢4 − 0 ⎥
⎣ 2 ⎦ (a) Positive definite (b) Negative definite
(c) Indefinite (d) Positive semidefinite
5. The matrix of the quadratic form x12 + 3x22 + Ans: (c)
3x32 − 2 x2 x3 is A =
11. The index of the quadratic form x2 + 3y2 + 4z2 is
⎡ −1 0 0 ⎤ ⎡ 1 −1 0⎤ (a) 3 (b) 2 (c) 1 (d) 0 Ans: (a)
(a) ⎢⎢ 0 3 −1⎥⎥ (b) ⎢⎢ −1 3 1⎥⎥
⎢⎣ 0 −1 3 ⎥⎦ ⎢⎣ 0 1 3⎥⎦ 12. The singnature of x2 + 2xz is
⎡1 0 0⎤ ⎡ 1 −1 0⎤ (a) 1 (b) 0 (c) 2 (d) 3 Ans: (b)
(c) ⎢⎢0 3 0⎥⎥ (d) ⎢ −1 3 0⎥ Ans: (a)
⎢ ⎥
⎢⎣0 0 3⎥⎦ ⎢⎣ 0 0 3⎥⎦ 13. The rank of the quadratic form x2 + 2yz is
(a) 1 (b) 2 (c) 3 (d) 0 Ans: (c)
6. The quardratic form XT AX with XT = [X1, X2]
⎡ 1 −1⎤ 14. The index of a quadratic form is the number of
whose matrix A = ⎢ ⎥ is
⎣ −1 1 ⎦ the _____ terms in its canonical form.

(a) x12 − 4 x1 , x2 + x22 (b) x12 − 2 x1 x2 + x22 (a) Positive (b) Negative
(c) Nonzero (d) Zero Ans: (a)
(c) x12 + 2 x1 x2 + x32 (d) x12 + x22 Ans: (b)
15. The signature of a quadratic form is the number
7. The quadratic form XT AX with XT = [X1, X2, X3] of the _____terms in its canonical form
⎡ 1 0 −4⎤
whose matrix A = ⎢⎢ 0 2 0 ⎥⎥ is (a) Positive
(b) Negative
⎢⎣ −4 0 −2⎥⎦ (c) Excess of positive terms over negative
(d) Excess of the nonzero terms over the zero
(a) 2 x12 + x22 + 3x32 + 8 x2 x3 Ans: (c)
(b) −2 x12 + x22 + 2 x32 + 8 x2 x3
(c) x12 + x22 − x32 − 2 x1 x3 16. The rank of the quadratic form 2x2 + 4xy + z2 is
(d) x12 + 2 x22 − 2 x32 − 8 x1 x3 Ans: (d) (a) 1 (b) 0 (c) 2 (d) 3 Ans: (d)
A-12  !   Engineering Mathematics-II

17. The column vectors X1, X2 and X3 are pairwise ⎡1 1 ⎤


orthogonal if ⎡1 1 ⎤ ⎢2 2 ⎥
(a) ⎢ ⎥ (b) ⎢ ⎥
(a) X i X Tj = 0 (i ≠ j ) (b) X1 X 2T = 0 ⎣1 −1⎦ ⎢1 − 1⎥
⎢⎣ 2 2 ⎥⎦
(c) X 2 X 3T = 0
⎡ 1 1 ⎤ ⎡ 1 1 ⎤
(d) X 3 X1T = 0 for all i, j = 1, 2, 3 ⎢ 3 ⎥ ⎢ 2 2 ⎥
Ans: (a) 3
(c) ⎢ ⎥ (d) ⎢ ⎥ Ans: (d)
⎢ 1 1 ⎥ ⎢ 1 1 ⎥
18. If X1, X2 and X3 are three linearly independent −
eigenvectors of a symmetric matrix A and ||Xi|| =

⎣ 3 3 ⎥⎦ ⎢ 2 − 2⎥
⎣ ⎦
ai (i = 1, 2, 3) then the model matrix of AP =
⎡2 4⎤ ⎡ −4 1⎤
⎡X X X ⎤ ⎡X X X ⎤ 23. The modal matrix of A = ⎢ ⎥ is P = ⎢ ⎥.
(a) ⎢ 3 , 2 , 1 ⎥ (b) ⎢ 1 , 2 , 3 ⎥ ⎣1 5⎦ ⎣ 1 1⎦
a
⎣ 3 a 2 a1⎦ a
⎣ 1 a 2 a 3⎦
Then the normalised modal matrix is
⎡X X X ⎤ ⎡X X X ⎤
(c) ⎢ 2 , 3 , 1 ⎥ (d) ⎢ 1 , 3 , 2 ⎥
⎣ a 2 a 3 a1 ⎦ ⎣ a1 a 3 a 2 ⎦ ⎡ −4 1 ⎤ ⎡ −4 1 ⎤
⎢ 17 2⎥ ⎢ 5 2⎥
Ans: (b) (a) ⎢ ⎥ (b) ⎢ ⎥
⎢ 1 1 ⎥ ⎢ 1 1 ⎥
19. The method of reduction of a quadratic form ⎢ 17 2 ⎥⎦ ⎢ 5 2 ⎥⎦
⎣ ⎣
q = XT AX to its canonical form YT DY by apply-
⎡ 1⎤ ⎡ −4 1 ⎤
ing a linear transformation X = PY where P =
⎢ −4 2 ⎥ ⎢ 13 2 ⎥
[X1, X2, X3], X1, X2, X3 being normalised eigen- (c) ⎢ ⎥ (d) ⎢ ⎥ Ans: (a)
vectors of A is called ______ ⎢ 1 1⎥ ⎢ 1 1⎥
⎣⎢ 2 ⎥⎦ ⎢ 13 2 ⎥
(a) Diagonalisation ⎣ ⎦
(b) Orthogonalisation
(c) Lagrange’s reduction Ans: (b) ⎡0 1 0⎤
24. The modal matrix of A = ⎢⎢1 0 1⎥⎥ is
20. The method of reduction of a quadratic form ⎡ −1 1 −1 ⎤ ⎢⎣0 1 0⎥⎦
q = XT AX to its canonical form YT DY by apply- ⎢ ⎥ Then the normalised model
ing row and column operations simultaneously ⎢0 2 2⎥
⎢1 1 −1 ⎥⎦
on A = I3AI3 to convert A into a diagonal matrix ⎣
is called ________ matrix P =
(a) Diagonalisation ⎡ 1 1 1⎤ ⎡ 1 1 1⎤
⎢− 2 − ⎥
2 ⎢− 2 − ⎥
(b) Orthogonalisation
⎢ 2 ⎥ 2 2
(c) Lagrange’s reduction Ans: (a) ⎢ ⎥
⎢ 1 1 ⎥ 1 1 ⎥
(a) ⎢ 0 (b) ⎢ 0
2 2 ⎥ ⎢ 2 2 ⎥
21. The method of reduction of the quadratic form ⎢ ⎥ ⎢ 1
q = XT AX to canonical form by grouping the ⎢ 1 1 1⎥ ⎢ 1 1⎥
⎢ 2 − ⎥ − ⎥
terms and completing the squares is called ⎣ 2 2⎦ ⎢⎣ 2 2 2 ⎥⎦
________ ⎡ 1 1 1⎤ ⎡ 1 1 1⎤
⎢− 2
− ⎥
2 ⎢− 2 2
− ⎥
2
(a) Diagonalisation ⎢ 2 ⎥ ⎢ ⎥
(b) Orthogonalisation ⎢ 1 1 ⎥ ⎢ 2 2⎥
(c) ⎢ 0 (d) ⎢ 0
(c) Lagrange’s reduction Ans: (c)
⎢ 2 2 ⎥⎥ ⎢
2 2 ⎥

⎢ 1 1 1⎥ ⎢− 1 1 −1 ⎥
22. The normalised modal matrix P corresponding ⎢ 2 − ⎥ ⎢ 2
⎡ 1 2⎤ ⎣ 2 2⎦ ⎣ 2 2 ⎥⎦
to the symmetric matrix A = ⎢ ⎥ is
⎣ 2 1⎦ Ans: (c)
Multiple Choice Questions  !   A-13

Real quadratic form in canonical form Its signature is


25. 2 y12 + y22 − 3 y22 . Its rank is (a) 0 (b) 1 (c) 2 (d) −1 Ans: (c)
(a) 1 (b) 3 (c) 2 (d) 0 Ans: (b) Its nature is
Its index is (a) Positive semidefinite (b) Positive definite
(c) Negative definite (d) Negative semidefinite
(a) 1 (b) 0 (c) 2 (d) 3 Ans: (c)
Ans: (a)
Its signature is
29. −2y12 − y22 − y32; Its rank is
(a) 3 (b) 2 (c) 1 (d) 0 Ans: (c)
(a) 3 (b) 1 (c) 2 (d) 0 Ans: (a)
Its nature is
Its index is
(a) +ve Semidefinite (b) −ve Definite
(c) +ve Definite (d) Indefinite Ans: (d) (a) 3 (b) 2 (c) 0 (d) 1 Ans: (c)
Its signature is
26. y12 + 2 y22 + 3 y32 . Its rank is
(a) 2 (b) 1 (c) 3 (d) 0 Ans: (c)
(a) 1 (b) 2 (c) 3 (d) 0 Ans: (c)
Its nature is
Its index is
(a) Positive definite
(a) 2 (b) 3 (c) 1 (d) 0 Ans: (b) (b) Negative definite
Its signature is (c) Positive semidefinite
(d) Negative semidefinite Ans: (b)
(a) 1 (b) 2 (c) 3 (d) −1 Ans: (c)
Its nature is 30. Quadratic form 2x1x2 + 2x1x3 + 2x2x3; Its
rank is
(a) Positive definite (b) Positive semidefinite
(c) Indefinite (d) Negative definite Ans: (a) (a) 1 (b) 3 (c) 2 (d) 0 Ans: (b)
Its index is
27. y12 − 2 y32 . Its rank is
(a) 1 (b) 2 (c) 3 (d) 0 Ans: (a)
(a) 2 (b) 1 (c) 3 (d) 0 Ans: (a)
Its signature is
Its index is
(a) 0 (b) 1 (c) −1 (d) 2 Ans: (c)
(a) 3 (b) 2 (c) 1 (d) 0 Ans: (c)
Its nature is
Its signature is
(a) Positive definite (b) Positive indefinite
(a) 2 (b) 0 (c) 2 (d) 1 Ans: (b) (c) Indefinite (d) Positive semidefinite
Its nature is Ans: (c)

(a) Positive definite (b) −ve Definite 31. x2 + y2 + z2 + 2yz; Its rank is
(c) Indefinite (d) +ve Semidefinite Ans: (c)
(a) 2 (b) 3 (c) 1 (d) 0 Ans: (a)
28. 3y12 + y32 . Its rank is Its index is
(a) 0 (b) 1 (c) 2 (d) 3 Ans: (d) (a) 1 (b) 2 (c) 3 (d) 0 Ans: (b)
Its index is Its signature is
(a) 2 (b) 1 (c) 0 (d) 3 Ans: (a) (a) 2 (b) 1 (c) 0 (d) −1 Ans: (a)
A-14  !   Engineering Mathematics-II

Its nature is ⎧ − x ( −p , 0)
(c) f ( x ) = ⎨
(a) Positive definite (b) Negative definite ⎩x (0, p )
(c) Positive semidefinite (d) Indefinite
⎧ − k , ( −p , 0)
Ans: (c) (d) f ( x ) = ⎨ Ans: (c)
⎩ k (0, p )
CHAPTER 5 FOURIER SERIES
Classify the following functions defined in the 11. The graph of a function is as shown in the
interval mentioned against them as figure; then the function is
(a) Even (b) Odd (c) Neither even nor odd (a) Even
(d) Nonclassifiable and more the correct letter in the (b) Odd
brackets. (c) Neither even nor odd
Function Interval (d) Nonclassifiable Ans: (b)
1. sin x + cos x (−∞, ∞) Ans: (c) y

2. ex
(−∞, ∞) Ans: (c)

3. log x (0, ∞) Ans: (d)


x
1+ x
4. log (−1, 1) Ans: (b)
1− x
5. sin x sinh x (−∞, ∞) Ans: (a)
1+ x
6. x log (−1, 1) Ans: (a)
1− x 12. The graph of a function is as shown in the
figure; then the function is
1+ x
7. sin −1 x + x 2 log (−1, 1) Ans: (b)
1− x (a) Even
(b) Odd
8. |x| (−∞, ∞) Ans: (a)
(c) Neither even nor odd
(d) Nonclassifiable Ans: (a)
9. The odd function among the following is

⎧ − k , ( −p , 0)
(a) f ( x ) = ⎨
⎩ k, (0, p )
⎧ − x, ( −p , 0)
(b) f ( x ) = ⎨
⎩ x, (0, p )
(c) f ( x ) = x (0, ∞)
(d) f (x) = log(−x) (−∞, 0) Ans: (a)

13. The leading term in the Fourier series expan-


10. The even function among the following is
sion of f (x) = x cos x in (−p, p) is
(a) f ( x ) = x (0, ∞) 1 1
(a) −sin x (b) sin x (c) − sin x
(b) f ( x ) = log x (0, ∞) 2 2
(d) sin x Ans: (c)
Multiple Choice Questions  !   A-15

14. The coeffcients b2n = 0 in the Fourier series 3p p


expansion in (−p, p) of the function f (x) = (a) p (b) (c) (d) p Ans: (d)
2 2 4
⎧ − k ( −p , 0)
(a) | sin x| (b) ⎨ (c) x2
⎩ k (0, p ) ⎧ 2p
(d) cos x Ans: (b) ⎪⎪1 + p , ( −p , 0)
22. If the Fourier series of f ( x ) ⎨
⎧ −p ( −p , 0) ⎪1 − 2p , (p , 0)
15. If f ( x ) = ⎨ then the value of f (x) ⎪⎩ p
⎩ x (0, p ) 8 ⎛ cos x cos 3x cos x ⎞
at x = 0, which is a point of discontinuity, is is f ( x ) = ⎜ + 2 + 2 +! ⎟
p 2 ⎝ 12 3 5 ⎠
p p 1 1 1
(a) (b) − (c) 0 (d) Not defined then 2 + 2 + 2 + ! =
2 2 Ans: (b) 1 3 5
2 2 2 2
16. If f (x) = |x| in (−p, p) then the Fourier coeffcient (a) p (b) p (c) p (d) p Ans: (c)
a0 = 12 6 8 4
p 3p
(a) p (b) (c) 2p (d) Ans: (a)
2 2 23. If the Fourier series of sin ax in (−p, p) is
1 + sin x 2sin ap
⎡ sin x 2sin 2 x 3sin 3x ⎤
17. The function f ( x ) = log in ( −∞, ∞), sin ax = ⎢⎣ 12 − a2 + 22 − a2 + 32 − a2 − ! ⎥⎦
1 − sin x p
p 1 3 5
x ≠ (2n + 1) , is________ function where a ∉ Z, then 2 2 − 2 2 + 2 2 − ! =
2 1 −a 3 −a 5 −a

(a) Nonclassifiable (b) Even p ap p ap


(a) sec (b) sec
(c) Odd (d) Neither even nor odd Ans: (c) 4 2 2 2
p ap p ap
18. The function which is an odd function in (−∞, ∞) (c) cos (d) sin Ans: (a)
2 2 4 2
among the following is
(a) cos x (b) 1+ x (c) e−x (d) x Ans: (d) 24. If the Fourier series of sin ax in (−p, p) is
2sin ap ⎡ sin x 2sin 2 x 3sin 3 x ⎤
19. The function which is an even function in sin ax = ⎢⎣ 12 − a2 − 22 − a2 + 32 − a2 − ⎥⎦
p
(−∞, ∞) among the following is 1 3
−x
where a ∉ z, then 2 2
− 2 2
+
(a) cos x (b) 1+ x (c) e (d) x Ans: (a) 5 1 − (0.1) 3 − (0.1)
−! =
52 − (0.1)2
20. The function which is neither even nor odd
function in (−∞, ∞) among the following p p p p
(a) sec (b) sec
(a) cos x + sin x (b) ex + e−x 4 10 2 10
(c) ex − e−x (d) |x| + x2 Ans: (a) p p p p
(c) sec (d) sin Ans: (c)
4 20 4 20
21. If the Fourier series of f ( x ) = x / p in (−p, p)
x 2 ∞ ( −1)n −1 2p 2 4 ∞

is = ∑ sin nx then 25. If 2px − x 2 =


3
− 2
p
∑ cos nx in (0, 2p)
p p n =1 n n =1
1 1 1
1 1 1 1 1 then + + +! =
− + − + −! = 12 22 32
1 3 5 7 9
A-16  !   Engineering Mathematics-II

2 2 2 2 1 p −2 p +2 p −4
(a) p (b) p (c) p (d) p (a) (b) (c) (d)
16 12 8 6 2 2 4 4
Ans: (d) Ans: (a)

⎧0 [ −p , 0]
26. If f ( x) = ⎨ has Fourier series 31. If the half-range sine series of ex in (0, p) is
⎩ x [0, p ] 2 ⎡1 + e x 2(1 − e x ) 3(1 + e x )
p 2 ∞ cos(2n − 1) x ex = ⎢ 2 sin x + 2 sin 2 x + 2
expansion f ( x) = − ∑ + p ⎣1 +1 2 +1 3 +1
4 p n =1 (2n − 1)2
⎤ 1 3 5

( −1)n −1 sin nx 1 1 1 sin 3x + ! ⎥ then − 2 + 2 −! =
∑ n
then 1 + + + + ! =
32
52
72 ⎦ 1 2
+ 1 3 + 1 5 +1
n =1
p p p p
(a) sech (b) sech
p 2
p 2
p2
p2 2 2 4 2
(a) (b) (c) (d) Ans: (b) p p p p
16 8 6 12 (c) cosh (d) cosech Ans: (b)
4 2 4 2
p2 ∞
( −1)n cos nx
27. If x =
2
− 4∑ in (−p, p) then 32. If the half-range cosine series in (0, p) for
3 n =1 n2
1 2 2
1 1 1 1 1 x sin x = 1 − cos x − + cos 2 x + cos 3x −
1− 2 + 2 − 2 + 2 − 2 +! = 2 1.3 2.4
2 3 4 5 6 2 1 1 1 1
cos 4 x +! then − + − +! =
p2 p2 p2 p2 3.5 1.3 3.5 5.7 7.9
(a) (b) (c) (d) Ans: (a)
12 8 6 16
p 1 p −1 p −1 p +2
(a) − (b) (c) (d)
2 4 2 4 2 4
1 1 1 p 1 1
28. If 1 − −+ +! = and 1 + 2 + 2 + Ans: (a)
22
32 42 12 2 3
1 p2 1 1 1 1
2
+! = then 2 + 2 + 2 + 2 + ! = 33. If
1

1
+
1

1 p 1
+ ! = − and
1
+
1
+
4 6 1 3 5 7 1.3 3.5 5.7 7.9 4 2 1.3 3.5
1 1 1 1 1 1
p2 p2 p2 p2 + + ! = then + + +! =
(a) (b) (c) (d) Ans: (c) 5.7 7.9 2 1.3 5.7 9.11
24 16 8 4 p
p p p
29. If the half-range sine series in (0, p) for (a) (b) (c) (d) Ans: (c)
4 6 8 12
8 ∞ sin(2n − 1) x
f ( x ) = x(p − x ) = ∑ then
p n =1 (2n − 1)3 2sinh ap ⎡⎛ 1 a cos x a cos 2 x
1 1 1 34. If e − ax = ⎢⎜⎝ 2a − 2 + −
1− 3 + 3 − 3 +! = p ⎣ 1 + a2 22 + a2
3 5 7
a cos 3x ⎞ ⎛ sin x 2sin 2 x 3sin 3x ⎞⎤
+! ⎟ − ⎜ 2 − + +! ⎟⎠ ⎥
p2 p2 p2 p2 3 + a2
2 ⎠ ⎝ 1 + a2 22 + a2 32 + a2 ⎦
(a) (b) (c) (d) Ans: (d)
24 8 16 32 1 1 1
in (−p, p) then − + ! =
30. If the half-range cosine series in (0, p) for 22 + 1 32 + 1 42 + 1
2 4 ∞ cos 2nx 1 1 2p p
f ( x ) = sin x = − ∑ 2 then + − (a) (b)
p p n =1 4n − 1 1.3 3.5 sinh p sinh p
1 1 p p
+ ! = (c) (d) sinh p Ans: (c)
5.7 7.9 2sinh p 2
Multiple Choice Questions  !   A-17

35. If the Fourier series expansion of f (x) = x2 in 40. If the Fourier series for ex (0, 2p) is given by
l 2 4l 2 ∞ ( −1)n −1 npx e 2p − 1 ⎡ 1 ∞ cos nx ∞ n sin nx ⎤
(−l, l) is x = − 2 ∑ ex = ⎢ +∑ −∑
2
cos then ⎥ then
3 p n =1 n 2
l p ⎣ 2 n =1 n2 + 1 n =1 n2 + 1 ⎦
1 1 1 1 1 ∞ ( −1)n −1
2
− 2
+ 2
− 2
+! = −∑ =
1 2 3 4 2 n =1 4n2 + 1
p2 p2 p2 p2 pe −p /2 e −p /2
(a) (b) (c) (d) Ans: (b) (a) (b)
4 12 6 2 2sinh p 2sinh p
pe −p /2 pep / 2
36. If the Fourier series expansion of cos ax in (−p, p) (c) (d) Ans: (a)
sinh p 2sinh p
2a sin ap 1 ∞
( −1)n −1 cos nx
is cos ax =
p
+ ∑
2a2 n =1 n2 − a2
,
[Hint: Put x = p /2; n even.]
∞ n −1
1 ( −1)
(a ∉ z) then +∑ =" ⎧ 0, [ −p , 0]
2a 2 n =1 n
2
− a2 41. If the Fourier series of f ( x ) = ⎨
⎩sin x [0, p ]
p p
(a) (b) 1 2 ∞ cos 2nx 1
a sin ap 2sin ap is f ( x) = − ∑ + sin x is then
p p p p n =1 4n2 − 1 2
(c) (d) Ans: (c)
2a sin ap 2a sin 2ap 1
+
1

1
+! =
1.3 5.7 9.11
37. If n is an integer and ‘a’ is not an integer then
sin( n + a)p (a) p (b) p (c) 3p (d) p Ans: (d)
= 4 2 8 8
sin ap

(a) (−1)n−1 (b) (−1)n (c) (−1) (d) 0 ⎢Hint: Put x = 0 and p /2 and obtain,
Ans: (b) ⎣
1 1 1 1
respectively, + + +  = (i) and
38. If n is an integer and ‘a’ is not an integer then 1.3 3.5 5.7 2
sin( n − a)p 1 1 1 p 1
= − + ! = − (ii)
sin ap 1.3 3.5 5.7 4 2
(a) (−1)n−1 (b) (−1)n (c) (−1) (d) 0 1 1 1 (i) + (ii) p ⎤
Then + + +! = = .⎥
Ans: (a) 1.3 5.7 9.11 2 8 ⎦

39. If the Fourier series for ex (0, 2p) is given by ⎧ 0, [ −p , 0]


42. If the Fourier series of f ( x ) = ⎨
e 2p − 1 ⎡ 1 ∞ cos nx ∞ n sin nx ⎤ ⎩sin x, [0, p ]
ex = ⎢ +∑ −∑ ⎥ then
p ⎣ 2 n =1 n2 + 1 n =1 1 + n2 ⎦ 1 2 ∞ cos 2nx 1
is f ( x ) = − ∑ 2 + sin x then
1 ∞ ( −1)n p p n =1 4n − 1 2
+∑ = 1 1 1
2 n =1 1 + n2 − + +! =
3.5 7.9 11.13
p
(a) sech p (b) p cosech p 1 p
2 (a) 1 − p (b) +
p 4 8 2 8
(c) cosech p (d) p cosech p Ans: (c) 1 p 1 p
2 (c) − (d) + Ans: (c)
[Hint: Put x = p.] 2 8 4 8
A-18  !   Engineering Mathematics-II

⎡ 1 2l npx
⎢Hint: In above question (a)
l ∫0
f ( x ) cos
l
dx

1 1 1 (i) − (ii) 1 p .⎤ 2 2l npx
+ + + = = − ⎥
l ∫0
3.5 7.9 11.13 2 2 8 ⎦ (b) f ( x ) cos dx
l
1 l np x
l ∫0
43. If n is an even number then cos np /2 = (c) f ( x ) cos dx
l

(a) ( −1)
n /2
(b) ( −1)
( n −1/2)
(c) −1 1 l np x
(d) 1
Ans: (a)
(d)
l ∫ − l
f ( x ) cos
l
dx Ans: (a)

44. sin np = 50. If f (x) = 1− cos x is expanded as a Fourier


series in (−p, p) then the value of a0 =
(a) (−1)n (b) −1 (c) 0 (d) 1 Ans: (c)
p 2 2 2 p 2
(a) (b) (c) (d) Ans: (c)
⎧⎪ 1 ( x − x ), 0< x<l2 2 p p 4
2
45. The function f ( x ) ⎨ 1 2 is
⎪⎩ − 2 ( x + x ), − l < x < 0 51. The leading term in the Fourier series expan-
_______ function is (−l, l). sion of f (x) = ex in (−p, p) is

(a) Nonclassifiable (b) Even sinhp


(a) sinh p (b) (c) p sinh p
(c) Odd (d) Neither even nor odd Ans: (b) p
p
(d) Ans: (b)
sinh p
46. If f (x) = |x| in (−p, p) then the Fourier coeffcient
a0 = CHAPTER 6 PARTIAL DIFFERENTIAL
p 2 EQUATIONS
(a) p (b) (c) (d) 2p Ans: (a)
2 p 1. The partial differential equation obtained by
47. If f (x) = 2x is represented by Fourier series in eliminating a and b from (x − a)2 + (y − b)2 = z2
(−p, p) then the value of a0 = cot2a is
p (a) p2 − q2 = tan2a (b) p2 + q2 = tan2a
(a) 0 (b) p (c) (d) 2p Ans: (a)
2 (c) p2 + q2 = tana (d) p2 − q2 = cot2a
48. The Euler’s integral formula for determining Ans: (b)
the constants bn for a function f (x) defined in
(−p, p) is bn = 2. The partial differential equation obtained by
1 p eliminating a and b from x2 + y2 + (z −a)2 = b2 is
(b) 2 ∫ f ( x ) dx
p

p ∫0
(a) f ( x )sin np x dx
p 0 (a) py = qx (b) py + qx = 0
(c) px = qy (d) px + qy = 0 Ans: (a)
2 p 2 p
(c) ∫0 f ( x )sin np dx (d) ∫ f ( x )sin nx dx
p p 0 3. The partial differential equation obtained by
Ans: (d) a−b
eliminating a and b from z = ax + by + is
a
49. The Euler’s integral formula for the constants
p−q
an for the function f (x) defined in the interval (a) px + qy + =0 (b) p = q
(0, 2l) is an = p
Multiple Choice Questions  !   A-19

p−q (c) z = bx + ay + c (d) None of these


(c) z = px + qy + (d) p + q = z
p Ans: (c) Ans: (a)

4. The partial differential equation obtained 11. The partial differential equation obtained by
by eliminating the arbitrary function f from eliminating a and b from z = ax + by is
z = f (x2 − y2) is
(a) px = qy (b) py = qx
(a) py + qx = 0 (b) py = qx (c) z = px + qy (d) px + qy = 0 Ans: (c)
(c) px = qy (d) px + qy = 0 Ans: (a)
12. The partial differential equation obtained by
5. Solving the partial differential equation p = xy eliminating a and b from z = ax + by + ab is
we obtain
(a) z = px + qy (b) z = px + qy + pq
x2
(a) y=z (b) xy = z (c) z = px − qy (d) px = qy Ans: (b)
2
x2 x2 13. The partial differential equation obtained by elim-
(c) + f ( y) = z (d) y + f ( y) = z
2 2 inating a and b from z = (x2 + a2) (y2 + b2) is
Ans: (d)
(a) 4xy = z (p + q) (b) pq = 4xy
6. The general solution of px + qy = z is (c) pq = 4xyz (d) pq = 2xyz Ans: (c)
⎛ x y⎞
(a) F ⎜⎝ , ⎟⎠ = 0 (b) F (x, y) = 0 14. The result of eliminating the constants h and k
z z
from (x − h)2 + (y − k)2 + z = a2 is the partial
(c) F (x − z, y − z) = 0 (d) F (x, y, z) = 0 differential equation
Ans: (a)
(a) z2 (p2 + q2) = a2 (b) z2 (p2 + q2 + 1) = a2
7. The general solution of x(y − z)p + y(z − x)q = (c) z p + z q + z = a (d) None of these
2 2 2 2 2
z(x − y) is Ans: (b)
(a) F (x + y + z, xyz) = 0
(b) F (xy + yz + zx, xyz) = 0 15. Eliminating a and b from z = aebt sin bx we
⎛1 1 1 ⎞ obtain the partial differential equation
(c) F ⎜ + + , xyz ⎟ (d) none of these
⎝x y z ⎠ ∂2 z ∂2 z 2 2
(b) ∂ z = ∂ z
Ans: (a)
(a) + =0
∂x 2 ∂t 2 ∂x 2 ∂t 2
8. The general solution of (y − z)p + (z − x)q =
2
∂z ∂z
x − y is (c) ∂ z = ∂z (d) = Ans: (a)
(a) F (x + y + z, (x + y + z)2) = 0 ∂x 2 ∂t ∂x ∂t
(b) F (xy + yz + zx, xyz) = 0
(c) F (x + y + z, x2+ y2+ z2) = 0 16. Eliminating a and b from z = ax + (1 − a)y + b
(d) F (x + y + z, x2 + y2 − z2) = 0 Ans: (c) we obtain the partial differential equation

9. The complete integral of pq = k is (a) py = qx (b) px = qy


k k (c) p = q (d) p + q = 1 Ans: (d)
(a) z = az + y+ c (b) z = az + y
a a
(c) z = ax + c (d) Z = byte Ans: (a) 17. The partial differential equation of all planes
whose x and y intercepts are always equal is
10. The complete integral of z = px + qy + pq is
(a) p = q (b) p + q = 0
(a) z = ax + by + ab (b) z = ax + by (c) px = qy (d) py = qx Ans: (a)
A-20  !   Engineering Mathematics-II

18. By eliminating the arbitrary function from CHAPTER 7 FOURIER INTEGRAL TRANSFORMS
⎛ xy ⎞ ∞ − ax
z = x n f ⎜ ⎟ we obtain the partial differential 1. ∫0 e sin bx dx =
⎝ z⎠
equation
b a
(a) px + qy = z (b) px + qy = zn (a) − (b)
(c) px + qy = nz (d) py + qx = nz Ans: (c)
2
a +b 2
a + b2
2

b −a
(c) (d) 2 Ans: (c)
19. By eliminating the arbitrary function f from a2 + b2 a + b2
⎛ xy ⎞ ∞
z = f ⎜ ⎟ we obtain the partial differential 2. ∫ e − ax cos bx dx =
⎝ z⎠ 0

equation a a
(a) − 2 2
(b)
(a) py = qx (b) px = qy a +b a + b2
2

(c) px + qy = z (d) py + qx = z Ans: (b) b b


(c) (d) − 2 Ans: (b)
20. The complex solution of z = px + qy + log pq is a2 + b2 a + b2

(a) z = ax + by + log ab (b) z = x + y + ab


∞ − x2
(c) z = ax − by + eab (d) z = ax + by
Ans: (a)
3. ∫−∞ e dx =

21. The complete integral of yp + xq + pq = 0 is p p p


(a) p (b) (c) (d)
given by 2 2 2
Ans: (a)
x2 y2
(a) 2 z = +b−
a2 − 1 a ∞ e ax − e − ax
x2 y2
4. ∫0 ep x − e − p x
dx =
(b) z = 2 − +b
a −1 a 1 p 1 a
x2 y2 (a) tan (b) tanh
(c) z = 2 + +b 2 2 2 2
a −1 a a 1
x2 y2 (c) tan (d) 2 tan a Ans: (a)
(d) z = − +b Ans: (a) 2
2
1− a a
22. The complete integral of 4z = p + q is ∞ e ax + e − ax
5. ∫0 ep x − e − p x
dx =
(a) z = (x + ay + b) (b) az = (x + ay + b)2
(c) z = (x + ay + b) (d) z = x + ay + b 1 a 1 a
Ans: (b) (a) cos (b) tan
2 2 2 2
23. The general solution of p + q = 1 is
1 a 1 a
(a) f (x, z) = 0 (b) f (x − y − z, x + y) = 0 (c) sec (d) cot Ans: (c)
2 2 2 2
(c) f (x − y, y − z) = 0 (d) f (x + y, y + z) = 0
Ans: (c)
∞ e − ax
24. The general integral of p + q = z is 6. ∫0 x
sin bx dx = ( a > 0, b > 0)

⎛ ey ⎞ ⎛ ey ⎞ 1 −1 b b
(a) F ⎜ x − y, ⎟ = 0 (b) F ⎜ x + y, ⎟ = 0 (a) cot (b) tan −1
⎝ z⎠ ⎝ z⎠ 2 a a
(c) F (x − y, zey) = 0 1 −1 b b
(d) None of these (c) tan (d) cot −1 Ans: (b)
Ans: (a) 2 a a
Multiple Choice Questions  !   A-21

∞ sin ax ⎧⎪ ⎛ 1 ⎞ n ⎫⎪
7. ∫0 x
dx, ( a > 0) = 4. Z ⎨ ⎜ − ⎟
⎝ 2⎠ ⎬=
⎩⎪ ⎭⎪
p p pa
(a) p (b) (c) (d) Ans: (b) z 2z z 2z
2 4 2 (a) (b) (c) (d)
z −1 2z + 1 2z + 1 z +1
8. The Fourier transform of Ans: (b)
⎧1
⎪ , x ≤a 5. Z {〈 n 〉} =
f ( x ) = ⎨ 2a is
⎪0, >
⎩ x a z2 z2 z
(a) 2
(b) (c)
sin(aa) sin(aa) ( z − 1) z −1 ( z − 1)2
(a) (b)
pa paa z
(d) Ans: (c)
sin(aa) sin a ( z + 1)2
(c) (d) Ans: (c)
2paa 2paa
6. Z {〈 −1, 0, 1 〉} =
9. The Fourier cosine transform of f (x) = e−ax,
(x ≥ 0, a ≥ 0) is 1 1 1 1
(a) −1 (b) +1 (c) 1 − (d) 1 +
a 1 z2 z 2
z2 z
(a) 2 2
(b) 2 2 Ans: (a)
a +a a +a
a 2a
(c) (d) Ans: (a) 7. Z {〈 n2 〉} =
2 2 2 2
a -a a -a
z2 − z z2 + z
(a) (b)
∞ cos sx ( z − 1)3 ( z − 1)3
10. ∫0 2
s +1
ds =
z2 + z z2 + z
(c) (d) Ans: (b)
−x
p −x p x p −x ( z + 1)3 (z − 1)2
(a) pe (b) e (c) 2 e (d) 4 e
2
Ans: (b) ⎧ 1 ⎫
8. Z ⎨ ⎬=
CHAPTER 8 Z-TRANSFORMS AND SOLUTION ⎩ n +1 ⎭
OF DIFFERENCE EQUATIONS (a) ze1/ z + 1 (b) ze1/ z − 1
1. Z {〈 a 〉} =
n
(c) z (e1/ z − 1) (d) z (e1/ z + 1) Ans: (c)
z z z z
(a) (b) (c) (d)
z −1 z+a a−z z−a 9. Z {〈 cos nt 〉} =
Ans: (d)
z 2 − cos t z ( z − cos t )
2. Z {〈 ln 〉} = (a) (b)
2
z − 2 z cos t + 1 z 2 − 2 z cos t + 1
z z z z2
(a) (b) (c) (d) z ( z + cos t ) z ( z − cos t )
z −1 z +1 1− z 1− z (c) (d)
2 2
Ans: (a) z − 2 z cos t + 1 z + 2 z cos t + 1
Ans: (b)
3. Z {〈 (−1)n 〉} =
10. Z {〈 sin nt 〉} =
z z z z2
(a) (b) (c) (d) z sin t z sin t
z −1 z +1 1− z 1− z (a) 2
(b)
Ans: (b) z + 2 z cos t + 1 z 2 − 2 z sin t + 1
A-22  !   Engineering Mathematics-II

z sin t z sin t the vector space made up of these functions. A


(c) (d) basis for this vector space is
z 2 − 2 z cos t + 1 z 2 − z cos t + 1
Ans: (a) (a) {f (t), y1,0 (t), y1,1 (t)}
(b) {y (t), y1,0 (t), y1,1 (t)}
11. Z ⎧⎨ 1 ⎫⎬ = (c) {f (t), y (t), y1,0 (t), y1,1 (t)}
⎩ n! ⎭ (d) {f (t), y (t), y1,0 (t) Ans: (c)

1 Let f(t) ∈ V2 and f(t) = a1f(t) + a2y(t) + a3y1,0(t)


(a) e1/2 (b) e −1/2 (c) ez (d) Ans: (a) 5.
1/2
e + a4y1,1(t). The constants ar (r = 1, 2, 3, 4) are
called the _______ coefficients
12. Z ⎧⎨ 1 ⎫⎬ = (a) Wavelet (b) Fourier
⎩ n +1 ⎭ (c) Invariant (d) Variable Ans: (a)
⎛ z ⎞ ⎛ z ⎞
(a) log ⎜ (b) z log ⎜
⎝ z − 1⎟⎠ ⎝ z − 1⎟⎠ 6. The set of functions {..., f(t + 2), f(t + 1), f(t),
f(t − 1), f(t − 2),...} is a basis for _____
⎛ z − 1⎞ ⎛ z + 1⎞
(c) z log ⎜ (d) z log ⎜
⎝ z ⎟⎠ ⎝ z ⎟⎠
Ans: (b) (a) v1 (b) v2 (c) v0 (d) v3 Ans: (c)

7. The set of functions {..., f(t + 2), f(t + 1), f(t),


CHAPTER 9 WAVELETS f(t − 1), f(t − 2),...} is called the set of _____
translates of f.
1. Vector space is a mathematical structure involving
the operation of (a) Integer (c) Real
(b) Complex (d) Primitive Ans: (a)
(a) Addition and multiplication
(b) Addition and scalar multiplication
8. The scaling function f(t) has energy
(c) Multiplication and scalar multiplication ∞
∫−∞ f (t ) dt = ____
2
(d) Multiplication and division Ans: (b)

2. Wavelets are generated from the mother wavelet (a) 0 (b) 1 (c) −1 (d) ∞ Ans: (b)
through the use of
9. Any two distinct members of the basis set {...,
(a) Scalings only f(t + 2), f(t + 1), f(t), f(t − 1), f(t − 2),...} are
(b) Translations only ______
(c) Scalings or translations exclusively
(d) Scaling and translations only Ans: (d) (a) Identical (b) Equivalent
(c) Orthogonal (d) Invariant Ans: (c)
3. The Haar wavelet function χt(t) is defined on
I = (0, I ] by χt(t) = 10. Let f(t) = a1f(t) + a2y (t) + a3y1,0(t) + a4y1,1(t)
1, for all t ∈I
(a) 0 for all t ∈ I (b) ⎧⎨ 19, 0 < t <1 4
⎩ , otherwise
0 4, 1 4 < t <1 2
=
⎧0, for all t ∈I 5, 1 2<t <3 4
(c) ⎨ (d) 0 for all t ∈ I Ans: (b)
⎩1, otherwise 3, 3 4 < t <1
4. Let (0, 1] = I1 ∪ I2 ∪ I3 ∪ I4 where f(t) = cr, on Then a1 + a2 + a3 =
1 1
Ir = (r − 1) < t ≤ r (r = 1, 2, 3, 4) and V2 be
4 4 (a) 19 (b) 4 (c) 5 (d) 3 Ans: (a)
FILL IN THE BLANKS

CHAPTER 1 MATRICES AND LINEAR ⎡0 0 1⎤


SYSTEMS OF EQUATIONS 10. The matrix A = ⎢⎢0 0 0⎥⎥ is a/an ______
⎡2 1 ⎤ ⎡1 −2⎤ ⎣⎢0 1 0⎥⎦
1. If A = ⎢ ⎥ and B = ⎢ ⎥ then det (AB) =
⎣ 3 −1⎦ ⎣5 2 ⎦ matrix of ______. Ans: Nilpotent, index 3
_______. Ans: −60
⎡1 1 3⎤ ⎡ −4 −7 −5⎤

2. The cofactor of ‘−2’ in A = ⎢ 1 3 −3⎥⎥ is 11. The matrix A = ⎢ 2 3 3 ⎥⎥ is a/an _____

⎢⎣ −2 −4 −4⎥⎦ ⎢⎣ 1 2 1 ⎥⎦
_______. Ans: −12 matrix of ______. Ans: Nilpotent, index 3

3. The minor of a32 element in matrix A in Question [Hint: Characteristic equation l3 = 0.]
2 is _______. Ans: −6
4. For matrix A in Question 2 det A = _______. ⎡ 2 1⎤
12. If A = ⎢ ⎥ then the characteristic equation
⎣ 3 2⎦
Ans: –8
satisfied by A−1 is ______. Ans: l2 − 4l + 1 = 0
5. (Adj I ) = _______. Ans: I

6. If A is an nth order square matrix, then Adj (kA) ⎡ 3 −1⎤ ⎡ x ⎤ ⎡ 4 ⎤


13. The solution of the system ⎢ ⎥⎢ ⎥ = ⎢ ⎥
is _______. Ans: kn−1(Adj A) ⎣2 5 ⎦ ⎣ y ⎦ ⎣ −3⎦
7. If the system AX = B is consistent then for is (x, y)T = ______. Ans: 1, −1
unique solution, A must be a _______ matrix.
Ans: Nonsingular ⎡ a −1 0 ⎤
14. If A = ⎢⎢ 0 a −1⎥⎥ and r (A) = 2 then a =
⎡ −1 0 4⎤
⎢⎣ −1 0 a ⎥⎦
8. The matrix ⎢⎢ 5 2 3⎥⎥ is a/an _______ matrix.
⎢⎣ 0 7 2⎥⎦ ______. Ans: 1
Ans: Nonsingular
15. The set of vectors (1, 2, 4) (2, −1, 3) (0, 1, 2) is
⎡ b −1 + c −1 a −1 1⎤
⎢ ⎥ ______. Ans: Linearly independent
9. If A = ⎢ c −1 + a −1 b −1 1⎥ then det A =
⎢ −1 −1 ⎥
⎣⎢ a + b c −1 1⎦⎥ 16. Set of vectors (1, 1, 1) (2, 3, 3) (1, 2, 2) is
_______. Ans: 0 ______. Ans: Linearly dependent
A-24  !   Engineering Mathematics-II

17. If the augmented matrix of a system of NH ⎡ 2 −2 −4⎤


⎡1 1 1 6 ⎤ 3. By diagonalisation of A = ⎢⎢ −1 3 4 ⎥⎥ we
⎢ ⎥
equations is [ AB] ! ⎢0 0 2 4 ⎥ then ⎢⎣ 1 −2 −3⎥⎦
⎢⎣0 0 a − 3 b − 10⎥⎦
⎡0 0 0⎤
the system has no solution if ______. obtain D = ⎢⎢0 1 0⎥⎥. From this we conclude
Ans: a = 3, b ≠ 10 ⎢⎣0 0 1⎥⎦

18. The system in Question 17 has a unique solution that A is ______. Ans: Singular
if ______. Ans: a ≠ 3, b ≠ 10
4. A is an n-square matrix with tr. A = 0 then the
coefficient of ln−1 in its characteristic equation
19. The system in Question 17 has an infinite number
is ______. Ans: 0
of solutions if ______. Ans: a = 3, b = 10
5. If the characteristic equation of matrix A of
20. If the augmented matrix of a system of NH order 2 is ax2 + bx + c = 0 then the characteris-
⎡1 1 1 1 ⎤ tic equation of A−1 is ______.
⎢ ⎥ Ans: cx2 + bx + a = 0
equations is [ AB] ! ⎢0 1 3 l −1 ⎥
⎢⎣0 0 0 ( l − 1)( l − 2)⎥⎦ 6. If the eigenvalues of A are 3, −1, 7 then the
eigenvalues of (A + I) are ______. Ans: 4, 0, 8
then the system is consistent if l = ______ or
______. Ans: 1 or 2 7. The eigenvalues a, b, c of A are in A.P and those
of A2 are in A.P too. Then b = ______.
21. In Question 20 for l = 1 the solution set is 2 2
Ans: a + c
(x, y, z)T = (______, ______, ______). a+c
Ans: 1 + 2k, −3k, k
8. If the product of the eigenvalues of A is zero
then A is a _______ matrix. Ans: Singular
22. In Question 20 for l = 2 the solution set is
(x, y, z)T = (______, ______, ______). 9. If the product of the eigenvalues of A is positive
Ans: 2k, 1 − 3k, k then A is a ______ matrix. Ans: Nonsingular
1
10. If 2 and are the eigenvalues of a 2-square
2
CHAPTER 2 EIGENVALUES matrix A then those of A−1 are ______.
AND EIGENVECTORS
1
Ans: 2,
2
1. The sum of the characteristic roots of
11. If the characteristic equation of a 2-square
⎡0 1⎤ matrix A is ax2 + bx + c = 0 then that of 3A
A= ⎢ ⎥ is ______. Ans: 0
⎣1 0⎦ is ______. Ans: ax2 + 3bx + 9c = 0

2. If the sum and the product of the eigenvalues


⎡ 3 −9 −12⎤
⎡ a + 1 −1 ⎤
of ⎢ are s1 and s2, respectively, then 12. If 2 is an eigenvalue of ⎢⎢1 a 4 ⎥⎥
⎣ 1 a − 1⎥⎦
⎣⎢0 0 1 ⎦⎥
s21 = ______. Ans: 4s2 then a = ______. Ans: −7
Fill in the Blanks  !   A-25

⎡ 3 − 2 −9 CHAPTER 3 REAL AND COMPLEX


⎢ Hint : A − 2 I = 0 ⇒ ( −1) MATRICES
⎢⎣ 1 a−2
⎤ ⎡ −1 1 −1⎤
⇒ a = −7.⎥
⎦ 1. The matrix ⎢⎢ 3 −3 3 ⎥⎥ is a /an ______
⎢⎣ 5 −5 5 ⎥⎦
⎡1 1 −1⎤
matrix. Ans: Idempotent
13. The latent roots of ⎢⎢0 2 −1⎥⎥ are ______.
⎢⎣0 0 −2⎥⎦
2. A square matrix A is periodic if ______.
Ans: 1, 2, −2 Ans: Am+1 = A

14. A is similar to D = diag(1, −1, 3) then the proper 3. If A is idempotent then its transpose is ______.
values of A2 are ______. Ans: 1, 1, 9 Ans: Idempotent

⎡1 2 2 ⎤ ⎡1 0 0⎤
15. If ‘0’ is a latent root of ⎢0 a 1 ⎥ 4. The matrix ⎢⎢0 1 0⎥⎥ is a/an _____ matrix.
⎢ ⎥
⎢⎣0 1 −1⎥⎦ ⎢⎣1 0 0⎥⎦
then a = ______. Ans: −1 Ans: Idempotent
⎡ a 1 ⎤
⎢ Hint : 0 = ⇒ a = −1.⎥
⎡ 2 −2 −4⎤
⎣ 1 −1 ⎦
5. The matrix ⎢⎢ −1 3 4 ⎥⎥ is a/an ______
⎡ 2 1⎤ ⎢⎣ 1 −2 −3⎥⎦
16. The eigenvalues of ⎢ ⎥ are the roots of the
⎣1 5⎦ matrix. Ans: Idempotent

equation ______. Ans: x2 − 7x + 9 = 0 ⎡1 b⎤


6. The matrix ⎢ ⎥ is a/an _______ matrix for
⎡ 2 2 1 ⎤ ⎣0 0⎦
⎢ Hint : x − (2 + 5) x + = 0.⎥
⎣ 1 5 ⎦ all real b. Ans: Idempotent

⎡ 0 2⎤
17. If X1 and X2 are the eigenvectors of A = ⎢ ⎥ ⎡ −1 3 5 ⎤
⎣ 2 0⎦ 7. The matrix ⎢ 1 −3 −5⎥ is a/an _______
then X1 and X2 are ______. Ans: Orthogonal ⎢ ⎥
⎢⎣ −1 3 5 ⎥⎦
18. The algebraic multiplicity of the eigenvalue l = 2 matrix. Ans: Idempotent
⎡ 1 0 2⎤
of the matrix ⎢ −1 2 2⎥ is ______. Ans: 2 8. A square matrix A such that An = 0 is called a nil-
⎢ ⎥
⎢⎣ −1 0 4⎥⎦ potent matrix, and the least n for which this holds
is called the index of nilpotent matrix. The

⎢ Hint : Expanding by C2 , A − lI . ⎡1 1 3⎤
⎣ index for A = ⎢⎢ 5 2 6 ⎥⎥ is _______.
1− l 2 ⎤ ⎢⎣ −2 −1 −3⎥⎦
= (2 − l ) = −( l − 2)2 ( l − 3).⎥
−1 4 − l ⎦ Ans: 3
A-26  !   Engineering Mathematics-II

⎡1 2 3⎤ 17. If A is an idempotent matrix then An = A


provided n = ______.
9. The matrix ⎢ 1 2 3 ⎥ is _______ matrix
Ans: (2, 1)
⎢ ⎥
⎣⎢ −1 −2 −3⎦⎥
⎡ −1 0 ⎤
with _______. Ans: Nilpotent, index 2 18. The matrix ⎢ ⎥ is a/an _____ matrix.
⎣ 0 −1⎦
1− l 2 3
⎡ Ans: Involutory
⎢Hint: In A − l I = 1 2 − l 3 if
⎣ −1 −2 −3 − l
⎡0 −i ⎤
we put l = 0, three rows become L.D. so that 19. The matrix A = ⎢ is a/an _______
⎣ −i 0 ⎥⎦

l 2 = 0 ⇒ A2 = 0 by CHT.⎥ matrix. Ans: Hermitian

⎡ a 0 b⎤
⎡ ab b2 ⎤ 20. The matrix ⎢ 0 1 0 ⎥ is orthogonal if
10. The matrix ⎢ ⎥ is _______ matrix ⎢ ⎥
2
⎣⎢ − a − ab ⎦⎥ ⎣⎢ −b 0 a ⎥⎦
with ______. Ans: Nilpotent, index 2 _______. Ans: a2 + b2 = 1

⎡ a − b −( a + b) ⎤ ⎡1 0 0⎤
11. The matrix ⎢ − a b ( a + b) ⎥⎥ is ______ 21. The matrix ⎢⎢0 b a ⎥⎥ is orthogonal if b2 =

⎢⎣ a −b −( a + b)⎥⎦ ⎢⎣0 − a b ⎥⎦

matrix with _______. Ans: Nilpotent, index 2 _______. Ans: 1 − a2

[Hint: Please see hint under Question 9.] 22. A square matrix which is its own inverse is a/an
_______ matrix. Ans: Idempotent
12. A square matrix A is involutory if _______.
Ans: A2 = I 23. The value of the determinant of an orthogonal
matrix is _______. Ans: ±1
13. A square matrix A is orthogonal if _______.
Ans: AAT = I 24. The determinant of an involutory matrix is
_______. Ans: 1
⎡1 0 0 ⎤

14. The matrix A = ⎢0 sin q cos q ⎥⎥ is a/an ⎡ p p ⎤
⎢ 0 2sin
6
sin
6 ⎥
⎢⎣0 − cos q sin q ⎥⎦ ⎢ ⎥
p p p⎥
______ matrix. Ans: Orthogonal 25. The matrix ⎢⎢sin sin − sin is a/an
4 6 6⎥
⎢ ⎥
⎡ 0 2b c ⎤ ⎢sin p − sin
p p
sin ⎥
⎣⎢ 4 6 6 ⎥⎦
15. If A = ⎢⎢ a b −c ⎥⎥ is orthogonal then
______ matrix. Ans: Orthogonal
⎣⎢ a −b c ⎥⎦
⎛ 1 1 1 ⎞ ⎡1 1 1⎤
(|a|, |b|, |c|) = ______. Ans: ⎜ , ,
⎝ 2 6 3 ⎟⎠ ⎢ ⎥
26. The matrix ⎢1 w w 2 ⎥ where w is a cube root
⎢ 2 ⎥
16. If A is an idempotent matrix then An = I provided ⎣1 w w⎦
n = ______. Ans: 2, 0 of unity is a/an _______ matrix. Ans: Unitary
Fill in the Blanks  !   A-27

⎡0 −1⎤ 5. The method of reduction of a quadratic


27. The matrix ⎢ ⎥ is a/an ______ matrix. form q = XT AX to its canonical form YT DY
⎣1 0 ⎦ by applying row and column operations
Ans: Periodic simultaneously on A = I3AI3 to convert A into
a diagonal matrix is called _______.
28. If A and B are orthogonal matrices of the same Ans: Diagonalisation by congruent row/column
type then AB and BA are both ______. operations
Ans: Orthogonal
6. The method of reduction of the quadratic form
q = XT AX to canonical form by grouping
⎡ 2 3 + 4i ⎤
29. The eigenvalues of ⎢ are ______. the terms and completing the squares is
⎣3 − 4i 2 ⎥⎦ called _______. Ans: Lagrange’s reduction
Ans: Real or zero
7. The normalised modal matrix P corresponding
⎡ 2i −4 + 3i ⎤ ⎡ 1 2⎤
30. The eigenvalues of ⎢ 4 + 3i are to the symmetric matrix A = ⎢ ⎥ is _______.
⎣ 2i ⎥⎦ ⎣ 2 1⎦
______. Ans: Complex conjugates ⎡ 1 1 ⎤
⎢ 2 2 ⎥
Ans: ⎢ ⎥
31. The diagonal elements of _______ matrix are 1 1 ⎥

imaginary or zero. Ans: Skew-Hermitian ⎢ −
⎣ 2 2 ⎥⎦
32. If A and B are Unitary matrices of same size
⎡2 4⎤ ⎡ −4 1⎤
then AB is _____ matrix. Ans: Unitary 8. The modal matrix of A = ⎢ ⎥ is P = ⎢ ⎥.
⎣1 5 ⎦ ⎣ 1 1⎦
CHAPTER 4 QUADRATIC FORMS Then the normalised modal matrix is _______.
⎡ −4 1 ⎤
1. The rank of the quadratic form 2x2 + 4xy + z2 is ⎢ 17
Ans: ⎢ 2⎥

_______. Ans: 3 ⎢ 1 1 ⎥
⎢ 17 2 ⎥⎦

2. The column vectors X1, X2 and X3 are pairwise
orthogonal if, for all i, j = 1, 2, 3 _______. ⎡0 1 0⎤
Ans: X i X Tj = 0 (i ≠ j ) 9. The modal matrix of A = ⎢⎢1 0 1⎥⎥ is
⎢⎣0 1 0⎥⎦
3. If X1, X2 and X3 are three linearly indepen- ⎡ −1 1 −1 ⎤
dent eigenvectors of a symmetric matrix A and ⎢ ⎥
||Xi|| = ai (i = 1, 2, 3) then the modal matrix ⎢0 2 2 ⎥ . Then the normalised modal
⎢1 1 −1 ⎥⎦

of A is = _______. Ans: ⎡⎢ X1 , X 2 , X 3 ⎤⎥ ⎡ 1 1 1⎤
⎣ a1 a 2 a 3 ⎦ ⎢− 2
− ⎥
2
⎢ 2 ⎥
⎢ 1 1 ⎥
4. The method of reduction of a quadratic matrix P = _______. Ans: ⎢ 0
form q = XT AX to its canonical form YT DY ⎢ 2 2 ⎥⎥
by applying a linear transformation X = ⎢ 1 1 1⎥
⎢ 2 − ⎥
PY where P = [X1, X2, X3], X1, X2, X3 being ⎣ 2 2⎦
normalised eigenvectors of A is called _______. Real quadratic form in canonical form
Ans: Orthogonalisation (Questions 10–14)
A-28  !   Engineering Mathematics-II

10. 2 y12 + y22 − 3 y22 . Its rank is _______. p2 ∞


( −1)n cos nx
Ans: 3 2. If x 2 = − 4∑ in (−p, p) then
Its index is _______. Ans: 2 3 n =1 n2
Its signature is _______. Ans: 1 1 1 1 1 1
1− 2
+ 2
− 2
+ 2
− + ! = _______.
Its nature is _______. Ans: Indefinite 2 3 4 5 62
p2
Ans:
11. y12 + 2 y22 + 3 y32 . Its rank is _______. Ans: 3 12
Its index is _______. Ans: 3
Its signature is _______. Ans: 3 1 1 p2 1 1 1
3. If 1 − 2
+ 2
− +! =
and 1 + 2 + 2 +
2
Its nature is _______. Ans: Positive definite 2 3 4 12 2 3
1 p2 1 1 1 1
12. y12 − 2 y32 . Its rank is _______. Ans: 2 +! = then 2 + 2 + 2 + 2 + ! =
42 6 1 3 5 7
Its index is _______. Ans: 1 p2
Its signature is _______. Ans: 0 _______. Ans:
8
Its nature is _______. Ans: Indefinite
4. If the half-range sine series in (0, p) for
13. 3y12 + y32 . Its rank is _______. Ans: 3
8 ∞ sin(2n − 1) x
Its index is _______. Ans: 2 f ( x ) = x(p − x ) = ∑ then
p n =1 (2n − 1)3
Its signature is _______. Ans: 2
Its nature is _______. 1 1 1 p2
1− + − + ! = _______. Ans: 32
Ans: Positive semidefinite 33 53 73
14. −2y12 − y22 − y32. Its rank is _______. Ans: 3
5. If the half-range cosine series in (0, p) for
Its index is _______. Ans: 0
2 4 ∞ cos 2nx
Its signature is _______. Ans: 3 f ( x ) = sin x = − ∑ then
Its nature is _______. Ans: Negative definite p p n =1 4n2 − 1
15. Quadratic form 2x1x2 + 2x1x3 + 2x2x3; Its rank 1 1 1 1 1
+ − + ! = _______. Ans:
is _______. Ans: 3 1.3 3.5 5.7 7.9 2
Its index is _______. Ans: 1
Its signature is _______. Ans: −1 6. If the half-range sine series of ex in (0, p) is
Its nature is _______. Ans: Indefinite 2 ⎡1 + e x 2(1 − e x ) 3(1 + e x )
ex = ⎢ 2 sin x + 2 sin 2 x + 2
16. x2 + y2 + z2 + 2yz. Its rank is _______. Ans: 2 p ⎣1 +1 2 +1 3 +1
Its index is _______. Ans: 2 ⎤ 1 3 5
Its signature is _______. Ans: 2 sin 3x + ! ⎥ then − 2 + 2 −! =
2
Its nature is _______. Ans: Positive semidefinite ⎦ 1 +1 3 +1 5 +1
p p
_______. Ans: 4 sech 2
CHAPTER 5 FOURIER SERIES
⎧0 [ −p , 0] 7. If the half-range cosine series in (0, p) for
1. If f ( x ) = ⎨ has Fourier series
⎩ x [0, p ] 1 2 2
expansion x sin x = 1 − cos x − + cos 2 x + cos 3x
2 1.3 2.4
p 2 ∞ cos(2n − 1) x ∞ ( −1)n −1 sin nx 2 1 1 1 1
f ( x) = − ∑ +∑ − cos 4 x +! then − + − +! =
4 p n =1 (2n − 1)2 n =1 n 3.5 1.3 3.5 5.7 7.9
1 1 1 p2 p 1
then 1 + 2 + 2 + 2 + ! = _______. Ans: _______. Ans: −
3 5 7 8 4 2
Fill in the Blanks  !   A-29

1 1 1 1 p 1 14. If the Fourier series for ex (0, 2p) is given by


8. If 1.3 − 3.5 + 5.7 − 7.9 + ! = 4 − 2 and e 2p − 1 ⎡ 1 ∞ cos nx ∞ n sin nx ⎤ then
ex = ⎢ +∑ −∑ ⎥
1 1 1 1 1 1 1 p ⎣ 2 n =1 n2 + 1 n =1 1 + n2 ⎦
+ + + + ! = then + +
1.3 3.5 5.7 7.9 2 1.3 5.7
1 ∞ ( −1)n p
1 p +∑ = _______. Ans: cosech π
+ ! = _______. Ans: 2 n =1 1 + n2 2
9.11 8
[Hint: Put x = p.]

2sinh ap ⎡⎛ 1 a cos x a cos 2 x 15. If the Fourier series for ex (0, 2p) is given by
9. If e − ax = ⎢⎜⎝ 2a − 2 + −
p ⎣ 1 + a2 22 + a2
e 2p − 1 ⎡ 1 ∞ cos nx ∞ n sin nx ⎤ then
a cos 3x ⎞ ⎛ sin x
+! ⎟ − ⎜ 2 −
2sin 2 x ex = ⎢ +∑ −∑ ⎥
⎠ ⎝ 1 + a2 22 + a2 p ⎣ 2 n =1 n2 + 1 n =1 n2 + 1 ⎦
32 + a2
3sin 3x ⎞⎤ 1 1 1 ∞ ( −1)n −1 ⎛ pe −p /2
+ + ! ⎟ ⎥ in (−p, p) then 2 − 2 −∑ = _______. Ans:
2
3 +a 2 ⎠ ⎦ 2 +1 3 +1 2 n =1 4n2 + 1 ⎝ 2sinh p
1 p p
+ 2 ! = _______. Ans: [Hint: Put x = , n even.]
4 +1 2sinh p 2

10. If the Fourier series expansion of f (x) = x2 in ⎧ 0, [ −p , 0]


16. If the Fourier series of f ( x ) = ⎨
l 2 4l 2 ∞
( −1)n −1 npx ⎩sin x [0, p ]
(−l, l) is x 2 = −
3 p2
∑ n 2
cos
l
then
1 2 ∞ cos 2nx 1
n =1 is f ( x) = − ∑ + sin x then
1 1 1 1 p2 p p n =1 4n2 − 1 2
− + − + ! = _______. Ans:
12 22 32 42 12 1 1 1 p
+ − + ! = _______. Ans:
1.3 5.7 9.11 8
11. If the Fourier series expansion of cos ax in ⎡ p
⎢Hint: Put x = 0 and and obtain
2a sin ap 1 ⎣ 2
(−p, p) is cos ax = +
p 2a 2 1 1 1 1
respectively + + + =
1.3 3.5 5.7 2

( −1)n −1 cos nx 1
∑ n2 − a 2
, (a ∉ Z) then
2a2
+
(i) and
1

1
+
1 p 1
! = −
n =1
1.3 3.5 5.7 4 2

( −1)n −1 p 1 1 1 (i) + (ii) p ⎤
∑ n2 − a 2 = _______. Ans:
2a sin ap (ii) Then + +
1.3 5.7 9.11
+! =
2
= .⎥
8 ⎦
n =1

1 1 1
17. From Question 16, + + + =
12. If n is an integer and ‘a’ is not an integer then 3.5 7.9 11.13
sin( n + a)p 1 p
= _______. Ans: (−1)n _______. Ans: −
sin ap 2 8
⎡ 1 1 1
13. If n is an integer and ‘a’ is not an integer then ⎢Hint: In Question 16, + + +!
⎣ 3.5 7.9 11.13
sin( n − a)p n−1
= _______. Ans: (−1) (ii) − (i) 1 p .⎤
sin ap = = − ⎥
2 2 8 ⎦
A-30  !   Engineering Mathematics-II

np CHAPTER 6 PARTIAL DIFFERENTIAL


18. If n is an even number then cos = _______ EQUATIONS
2
Ans: ( −1)n /2 1. The partial differential equation obtained by
eliminating a and b from (x − a)2 + (y − b)2 = z2
cos ( n + a)p cot2a is _______.
19. , ( n ∈ Z, a ∉ Z), = _______. Ans: p2 + q2 = tan2a
cos ap
Ans: (−1)n
2. The partial differential equation obtained by
eliminating a and b from x2 + y2 + (z − a)2 = b2
20. sin np = _______. Ans: 0
is _______. Ans: py = qx

⎧⎪ 1 ( x − x 2 ), 0< x<l 3. The partial differential equation obtained by


21. The function f ( x ) ⎨ 2 2
is a−b
1
⎪⎩ − 2 ( x + x ), − l < x < 0 eliminating a and b from z = ax + by + is
a
_______ function in (−l, l). Ans: Even p−q
______. Ans: z = px + qy +
p
22. If f (x) = |x| in (−p, p) then the Fourier coeffi-
p 4. The partial differential equation obtained
cient a0 = _______. Ans:
2 by eliminating the arbitrary function f from
z = f (x2 − y2) is ______.
23. If f (x) = 2x is represented by Fourier series in Ans: py + qx = 0
(−p, p) then the value of a0 = _______. Ans: 0
5. Solving the partial differential equation p = xy
24. The Euler’s integral formula for determining the 1
constants bn for a function f (x) defined in (−p, p) we obtain _______. Ans: z = x 2 y + f ( y )
2
2 p
p ∫0
is bn = _______. Ans: f ( x )sin nx dx 6. The general solution of px + qy = z is _______.
⎛ x y⎞
Ans: F ⎜ , ⎟ = 0
25. The Euler’s integral formula for the constants ⎝ z z⎠
an for the function f (x) defined in the interval
7. The general solution of x(y − z)p + y(z − x)q =
(0, 2l) is an = _______. z(x − y) is _______. Ans: F (x + y + z, xyz) = 0
1 2l npx
l ∫0
Ans: f ( x ) cos dx 8. The general solution of (y − z)p + (z − x)q = x − y
l
is _______. Ans: F (x + y + z, x2+ y2+ z2) = 0

26. If f (x) = | sin x | is expanded as a Fourier series 9. The complete integral of pq = k is _______.
in (−p, p) then the value of a0 = _______. k
Ans: z = ax + y + c
4 a
Ans:
p
10. The complete integral of z = px + qy + pq is
27. The leading term in the Fourier series expan- _______. Ans: z = ax + by + ab

sion of f (x) = ex in (−p, p) is _______. 11. The partial differential equation obtained by
sinh p ⎞ eliminating a and b from z = ax + by is _______.
Ans:
p ⎠ Ans: z = px + qy
Fill in the Blanks  !   A-31

12. The partial differential equation obtained by 22. The complete integral of 4z = p + q is _______.
eliminating a and b from z = ax + by + ab is ( x + ay + b)2
_______. Ans: z = px + qy + pq Ans:
a

13. The partial differential equation obtained by 23. The general solution of p + q = 1 is _______.
eliminating a and b from z = (x2 + a2) (y2 + b2) Ans: f (x − y, y − z) = 0
is _______. Ans: pq = 4xyz
24. The general integral of p + q = z is _______.
14. The result of eliminating the constants h and k
Ans: F ( x − y, z −1e y ) = 0
from (x − h)2 + (y − k)2 + z = a2 is the partial
differential equation _______.
Ans: z (p2 + q2 + 1) = a2 CHAPTER 7 FOURIER INTEGRAL
TRANSFORMS
15. Eliminating a and b from z = aebt sin bx we 1. An integral transform of a function f(x) is
b
obtain the partial differential equation _______. defined by I { f ( x )} = ∫ f ( x ) k ( s, x ) dx = f ( s).
a
∂2 z ∂2 z
Ans: + =0 Here k(s, x) is called the _____ of the transform.
∂x 2 ∂t 2 Ans: Kernel
16. Eliminating a and b from z = ax + (1 − a)y + b 2. In the case of Fourier transform k(s, x) = _____.
we obtain the partial differential equation Ans: eisx
_______. Ans: p + q = 1
3. In the case of Fourier transform the limits are
17. The partial differential equation of all planes (a, b) = (_____, _____). Ans: −∞, ∞
whose x and y intercepts are always equal is
_______. Ans: p = q 4. The Fourier transform of a function f (x) is

∫−∞ e
isx
F ( s) = _____. Ans: f ( x )dx
18. By eliminating the arbitrary function from
⎛ y⎞
z = x n f ⎜ ⎟ we obtain the partial differential
⎝ z⎠ 5. The Fourier sine transform of a function f (x) is

Ans: px + qy = nz
equation _______. _____. Ans: ∫0 f ( x )sin sx dx

19. By eliminating the arbitrary function f from 6. The Fourier cosine transform of a function f (x)
⎛ xy ⎞ ∞
z = f ⎜ ⎟ we obtain the partial differential
⎝ z⎠
is Fc ( s) = _____. Ans: ∫0 f ( x ) cos sx dx

equation _______. Ans: px = qy


7. The Fourier inverse transform of F ( s) is f (x) =
∞ − isx
20. The complete solution of z = px + qy + log pq is _____. Ans: ∫−∞ e Fc ( s) ds
_______. Ans: z = ax + by + log ab

21. The complete integral of yp + xq + pq = 0 is given 8. The inverse Fourier sine transform of Fs ( s) is

by _______. Ans: 2 z = ( a2 − 1) −1 x 2 − a −1 y 2 + b f(x) = _____. Ans: ∫0 Fs ( s)sin sx ds
A-32  !   Engineering Mathematics-II

9. The inverse Fourier cosine transform of Fc ( s) 9. The particular integral of the difference equa-
tion un+2 − 4un+1 + 3un = 5n is _____.

is f(x) = _____. Ans: ∫0 Fc ( s) cos sx ds 1 n
Ans: 〈5 〉
8
10. If the complex Fourier transform of f(x) is F ( s)
10. Initial Value Theorem: If Z {< un >} = u ( z ) then
then that of f(ax) is F{f(ax)} = _____.
1 ⎛ s⎞ u0 = _____. Ans: lim u ( z )
Ans: F ⎜ ⎟ z →∞
a ⎝ a⎠

11. The complex Fourier transform of f(x – a) is CHAPTER 9 WAVELETS


F{f(x – a)} = _______. Ans: eisa F ( s)
1. A set V = {f, g, h, …} of elements f, g, h … that
CHAPTER 8 Z-TRANSFORMS AND SOLUTION forms
OF DIFFERENCE EQUATIONS 1. a commutative group w.r.t. addition (+)
and
1. Z{〈 cos nt 〉} = _____.
2. satisfies the anxious
z ( z − cos t )
Ans: 2 (a) a f ∈ v (b) (ab) f = a (bf) (c) (a + b) f = af
z − 2 z cos t + 1
+ bf a(f + g) = af + bg (d) 1. f = f, where a,
b are scalars is called a __________.
z sin t Ans: vector (linear) space
2. Z{〈 sin nt 〉} = _____. Ans: 2
z − 2 z cos t + 1
2. A set of vectors in the vector space V that are
linearly independent and span the vector space
n
⎛ 1⎞ V is called a __________. Ans: basis
3. Z{〈 nCk | 0 ≤ k ≤ n 〉} = _____. Ans: ⎜1 + ⎟
⎝ z⎠
3. A function defined on real time is said to
4. Z{〈 4.2n + 5(−1)n 〉} = _____.
have _________ on the interval t1 < t ≤ t2 if
⎛ z ⎞ ⎛ z ⎞ t2
Ans: 4 ⎜ ⎟ + 5⎜
∫t [ f (t )]2 dt < ∞
⎝ z − 2⎠ ⎝ z + 1⎟⎠ ______. Ans: finite energy
1

2z − z2 ⎞ 4. The set of functions with finite energy on (t1, t2) is


5. Z{〈 n − 1 〉} = _____. Ans: usually denoted by _________. Ans: t2 (t1, t2)
( z − 1)2 ⎠

6. Z{〈 d (n) 〉} = _____. Ans: 1 5. The set of functions with finite energy form a
mathematical structure called a _______.
Ans: vector (linear) space
⎪⎧ z2 ⎪⎫
7. Z −1 ⎨ ⎬ = _____.
⎩⎪ ( z − 2)( z − 3) ⎭⎪ 6. If B = {b1 , b 2 ! b k } is a basis of a vector space V
n m
⎛ 2⎞ then u ∈V can be uniquely represented as a
Ans: 3 ∑ ⎜⎝ ⎟⎠
n

m= 0 3 ________ of vectors in B. Then there exist sca-


lars {a1, a2 … ak such that u = ________.
8. The complementary function of the difference Ans: linear combination,
equation un+2 − 7un+1 − 8un = 2n n2 is _____.
Ans: A〈8n〉 + B〈(−1)n〉 a1 b1 + a 2 b 2 + ! + a k b k
Fill in the Blanks  !   A-33

7. The characteristic function XI(t) on I = (0, 1] is 12. _________ are functions that form a
defined by ___________. ________ for a vector space of functions.
Ans: Wavelets, basis
⎧1, if t ∈I
Ans: X I (t ) = ⎨
⎩0, otherwise
13. The _________ function of (0, 1] is called the
__________ function.
8. The Fourier series expansion of f(t) in −l < t < l Ans: characteristic, Haar wavelet
is given by f(t) = ________.

1
Ans: = ao + ∑ [an cos (npt l ) + bn sin (npt l )] 14. The _________ wavelet function is defined by
2 n =1 the dilation equation y (t) = _________.
Ans: f(2t) − f(2 − t)
9. The constants ao, an and bn in the Fourier series
expansion of a function f(t) are called the
___________. Ans: Fourier coefficients 15. The functions generated through the use of
scalings and translations of y(t), namely,
10. The set of continuous functions on the interval yn,k (t) = y (2nt - k) where n, k are integers are
(−l, l) form a mathematical structure called a called the _________.
____________. Ans: vector (linear) space Ans: daughter wavelets

11. The basis elements of the vector space defined


by a set of continuous functions on (−l, l) are
____________, ___________.
Ans: cos (npt/l), sin (npt/l)
MATCH THE FOLLOWING

CHAPTER 1 MATRICES AND LINEAR SYSTEMS OF EQUATIONS

1. A is an orthogonal matrix a. 1
2. A is an invertible matrix b. 3
⎡1 1 0⎤
3. P(A) where A = ⎢⎢1 0 1⎥⎥ c. A = AT where A is a square matrix
⎢⎣0 1 1⎥⎦
⎡1 1 1⎤
4. P(A) where A = ⎢⎢1 1 1⎥⎥ d. AB = BA = I for some B
⎢⎣1 1 1⎥⎦
5. Symmetric matrix e. AAT = ATA = I
System of equations AX = B
⎡1 1 1 6⎤

where [ A, B] = ⎢0 1 2 4⎥⎥
⎣⎢0 0 a − 3 b − 10⎦⎥ f. No solution
6. a ≠ 3, b any number g. Finitely many solutions
7. a = 3, b ≠ 10 h. Unique solution
8. a = 3, b = 10 i. Infinite number of solutions

Ans: 1— e; 2 — d; 3 — b; 4 — a; 5 — c; 6 — h; 7— f; 8 — i

CHAPTER 2 EIGENVALUES AND EIGENVECTORS

1. li are eigenvalues of A and |A| ≠ 0 a. A is singular


2. One of the eigenvalues of A is zero b. 1 / li are also eigenvalues of A
3. li are eigenvalues of A and li ≠ 0 for all i c. A and B are similar
4. li are eigenvalues of orthogonal matrix A d. |A| / li are eigenvalues of Adj A
5. A, B, P are n-square matrices |P| ≠ 0 and B = P−1 AP e. A is nonsingular
6. D = [dij] is an n-square matrix with dij = 0 for i ≠ j f. Characteristic polynomials of A and B are same
A-36  !   Engineering Mathematics-II

7. Characteristic polynomials of A and A2 are same g. dii are eigenvalues of D


8. Characteristic equation of A is lm = 0 h. Characteristic equation of A is a reciprocal
equation
9. A is orthogonal i. A is idempotent
j. A is nilpotent with index m

Ans: 1 — d; 2 — a; 3 — e; 4 — b; 5 — c; 6 — g; 7 — i; 8 — j; 9 — h

CHAPTER 3 REAL AND COMPLEX MATRICES

1. A is a Hermitian matrix a. Q.F. is indefinite


2. A is a Skew-Hermitian matrix b. Q.F. is negative definite
3. If A and B are Hermitian then (AB − BA) is c. Re li = 0
4. li are eigenvalues of a Hermitian matrix A d. |li| = 1
5. li are eigenvalues of a Skew-Hermitian matrix A e. m li = 0
6. li are eigenvalues of a Unitary/Orthogonal matrix A f. Q.F. is positive definite
7. The characteristic equation of the matrix A of a g. Skew-Hermitian
Q.F. is x3 − 6x2 + 11x − 6 = 0
8. The characteristic equation of the matrix A h. A* = −A
of a Q.F. is x3 + 6x2 + 11x + 6 = 0
9. The characteristic equation of the matric A i. Q.F. is positive semidefinite
of a Q.F. is x3 − 3x2 + 2x = 0
j. A* = A

Ans: 1 — j; 2 — h; 3 — g; 4 — e; 5 — c; 6 — d; 7 — f; 8 — b; 9 — i

CHAPTER 4 QUADRATIC FORMS


A. Quadratic Form B. Matrix of the Quadratic Form

⎡1 0 0 ⎤
1. x − 2xy + 2y
2 2
a. ⎢0 3 −1⎥
⎢ ⎥
⎢⎣0 −1 3 ⎥⎦

⎡0 1 1⎤
2. x2 + 3y2 + 3z2 − 2yz b. ⎢1 0 1⎥
⎢ ⎥
⎢⎣1 1 0⎥⎦

3. 2xy + 2yz + 2zx c. ⎡ 1 −1⎤


⎢ −1 2 ⎥
⎣ ⎦
⎡1 2 3⎤
4. x2 − 18xy + 5y2 d. ⎢2 0 3⎥
⎢ ⎥
⎢⎣ 3 3 1⎥⎦
Match the Following  !   A-37

⎡ 0 2 2⎤
5. x2 + z2 + 4xy + 6xz + 6yz e. ⎢ 2 0 2⎥
⎢ ⎥
⎢⎣ 2 2 0⎥⎦

6. x2 + 2y2 + 3z2 + 7xy + xz f. ⎡ 1 −9⎤


⎢ −9 5 ⎥
⎣ ⎦
⎡ 1 7 2 1 2⎤
g. ⎢
⎢7 2 2 0 ⎥⎥
⎢⎣1 2 0 3 ⎥⎦

Ans: 1 — c; 2 — a; 3 — b; 4 — f; 5 — d; 6 — g

CHAPTER 5 FOURIER SERIES


A. Numerical Series B. Sum

1 1 1 ⎛ ∞ ( −1)n −1 ⎞
1. 1 − + − + ! ⎜ ∑ ⎟ a. p 2 8
3 5 7 ⎝ n =1 2n − 1 ⎠
1 1 1 ⎛ 1 ⎞ 1 p
2. + + +! ⎜ ∑ b. −
1.3 3.5 5.7 ⎝ (2n − 1)(2n + 1) ⎟⎠ 2 8
⎛ ( −1)n −1 ⎞ 1
3. 1 − 1 + 1 − 1 + !
1.3 3.5 5.7 7.9 ⎜

∑ (2 n − 1)(2 n + 1) ⎟

c.
2

4. 1 + 1 + 1 + ! ⎛ ∑ 1 ⎞ d. p 4
1.3 5.7 9.11 ⎜⎝ (4n − 3)(4n − 1) ⎟⎠

p −2
5. 1 + 1 + 1 + ! ⎛ 1 ⎞
⎜⎝ ∑ (4n − 1)(4n + 1) ⎠⎟
e.
3.5 7.9 11.13 4
p
6. 1 + 1 + 1 + 1 + ! ⎛ 1⎞
⎜⎝ ∑ 2 ⎟⎠
f.
2
1 2 2
32
4 2
n 8

7. 1 + 1 + 1 + 1 + ! ⎛ 1 ⎞ g. p 2 6
2 2 2 2 ⎜∑ 2⎟
1 3 5 7 ⎝ (2n − 1) ⎠
⎛ ( −1)n −1 ⎞
8. 1 − 1 + 1 − 1 + ! h. p 2 9
⎜∑ ⎟
12 22 32 42 ⎝ n2 ⎠
i. p 2 12

Ans: 1— d; 2— c; 3— e; 4— f; 5— b; 6— g; 7— a; 8— i

CHAPTER 6 PARTIAL DIFFERENTIAL EQUATIONS


A. Relation B. Partial Differential Equation
1. z = ax + by + ab a. 2z = px + qy
A-38  !   Engineering Mathematics-II

2. z = ax2 + by2 b. z = pq
3. z = (x + a) (y + b) c. z = px + qy + pq
4. z = x 3 f ( y x ) d. px + qy = 0
5. z = f (x2 − y2) e. 3z = px + qy
6. z = xy + f (x + y )2 2
f. py − qx = y2 – x2
g. px2 + qy2 =1

Ans: 1— c; 2— a; 3— b; 4— e; 5— d; 6— f
A. Partial Differential Equation B. General Solution
1. px + qy = z a. lx + my + nz = f (x2 + y2 + z2)
2. p tan x + q tan y = tan z b. x + y + z = f (x2 + y2 + z2)
x ⎛ y⎞
3. (mz − ny)p + (nx − lz)q = ly − mx c. = f⎜ ⎟
y ⎝ z⎠
⎛ sin x sin z ⎞
4. x(y − z)p + y(z − x)q = z(x − y) d. f ⎜ , =0
⎝ sin y sin y ⎟⎠
5. (y − z)p + (z − x)q = x − y e. x + y + z = f (xyz)
6. p + q = 1 f. x + y = f (y + z)
g. x − y = f (y – z)
Ans: 1— c; 2— d; 3— a; 4— e; 5— b; 6— g

CHAPTER 7 FOURIER INTEGRAL TRANSFORMS


∞ sin x p −a
1. ∫ dx a. e
0 x 2
∞ x cos x − sin x x
2. ∫ 3
cos dx b. −p 4
0 x 2
∞ x cos x − sin x
3. ∫ dx c. −3p 16
0 x3
2
∞ ⎛ sin x ⎞ p − ax
4. ∫ ⎜ ⎟ dx d. e
0 ⎝ 2x ⎠ 2
∞ s
5. ∫ 2 2 sin s x ds e. p 8
0 a +s

∞ x sin ax
6. ∫ dx f. p 2
0 1 + x2

g. p e−a

Ans: 1— f; 2— c; 3— b; 4— e; 5— d; 6— a
Match the Following  !   A-39

CHAPTER 8 Z-TRANSFORMS AND SOLUTION OF DIFFERENCE EQUATIONS


A. Sequence 〈un〉 B. Z-transform
1. 〈an〉 a. z ( z − 1)3

2. 〈np〉 b. e1 z
d
3. 〈n〉 c. − z ( z 〈 n p −1 〉)
dz
1
4. d. z ( z − a)
n +1
1 ⎛ z − 1⎞
e. − z log ⎜
⎝ z ⎟⎠
5.
n!
z
6. 〈cos n θ〉 f.
( z − 1)2
z sin q
7. 〈sin n θ〉 g. 2
z − 2 z cos q + 1
z ( z − cos q )
h.
z 2 − 2 z cos q + 1

Ans: 1— d; 2— c; 3— f; 4— e; 5— b; 6— h; 7— g

CHAPTER 9 WAVELETS

Let I = I1 ∪ I2 ∪ I3 ∪ I4 where Ir = r − 1 < t ≤ r , r = 1, 2, 3, 4; suppose that f(t) ∈ V2 such that


4 4
f(t) = a1f(t) + a2y(t) + a3y1,0(t) + a4y1,1(t)
⎧ 19 in I1
⎪ 4 in I2

=⎨
⎪ 5 in I3
⎪⎩ 3 in I4
Then
A. Wavelets coefficients B. Values
1. a1 a. 15/2

2. a2 b. 31/4

3. a3 c. 1

4. a4 d. 15/4

e. 0

Ans: 1— b; 2— d; 3— a; 4— c
TRUE OR FALSE STATEMENTS

CHAPTER 1 MATRICES AND LINEAR SYSTEMS 11. Cayley–Hamilton Theorem is applicable to


OF EQUATIONS every matrix. Ans: F

1. In a diagonal matrix some of the diagonal 12. A square matrix with repeated eigenvalues is
elements may be zero. Ans: T not diagonalizable. Ans: F

2. The product of two nonzero matrices is always 13. Powers of a square matrix can be found using
a nonzero matrix. Ans: F diagonalization. Ans: T

3. The rank of a singular matrix is one less than its 14. Every real square matrix can be uniquely
order. Ans: F expressed as the sum of a symmetric and a
skew-symmetric matrix. Ans: T
4. The sum of two nonsingular matrices is always
nonsingular. Ans: F 15. If A and B are symmetric then AB is symmetric.
Ans: F
5. The product of two matrices is singular if either
of them is singular. Ans: T CHAPTER 3 REAL AND COMPLEX
MATRICES
6. The system x − y = 2, x + y = 0 has a unique
solution. Ans: T
16. If l is an eigenvalue of a Hermitian matrix then
7. The system x + y = 2, 2x + 2y = 4 has an infinite Re l = 0. Ans: F
set of solutions. Ans: T
17. If l is an eigenvalue of a Skew-Hermitian
8. The system x + 2y = 1, 3x + 2y = 4 has no solu- matrix then Re l = 0. Ans: T
tion. Ans: F
18. If A and B are Hermitian matrices then
CHAPTER 2 EIGENVALUES AND (AB − BA) is also a Hermitian matrix. Ans: F
EIGENVECTORS
19. The eigenvalues of Unitary matrix are of unit
modulus. Ans: T
9. A matrix and its inverse have the same eigen-
values if A is orthogonal. Ans: T
⎡ i 0 0⎤
20. ⎢0 0 i ⎥ is a Unitary matrix. Ans: T
10. Two linearly independent eigenvectors may ⎢ ⎥
correspond to an eigenvalue l of a matrix. Ans: T ⎢⎣0 i 0⎥⎦
A-42  !   Engineering Mathematics-II

CHAPTER 4 QUADRATIC FORMS 33. The general solution of p + q = 0 is


f (x − y, z) = 0. Ans: T
21. If A is a symmetric matrix with distinct eigenvalues
then its eigenvectors are orthogonal. Ans: T
34. p2 = a2 is a linear partial differential equation.
22. A square matrix with two repeated eigenvalues Ans: F
is not diagonalizable. Ans: F
35. The complete solution of pq = 1 is
23. A quadratic form is semidefinite if some of its
1
eigenvalues are positive and the others negative. z = ax + y + c. Ans: T
Ans: F a

24. A quadratic form is positive definite if the 36. For solving the equation Pp + Qq − R = 0 the
nonzero coefficients in its canonical form are Lagrange’s auxiliary equations are
positive. Ans: F dx dy dz
= = . Ans: F
P Q −R
CHAPTER 5 FOURIER SERIES

25. Fourier series expansion is possible for a func- CHAPTER 7 FOURIER INTEGRAL
tion which is discontinuous. Ans: T TRANSFORMS

⎧⎪1 + 2 x in [ −p , 0] s
26. The function f ( x ) = ⎨ 2px is an 37. Fs {e − ax } = . Ans: T
⎪⎩1 − p in [0, p ] a + s2
2

odd function in [−p, p]. Ans: F


∞ s p − ax
⎧−k in [ −p , 0]
38. ∫0 2
a +s 2
sin sx ds =
2
e . Ans: T
27. The function f ( x ) = ⎨ is an even
⎩k in [0, p ]
function. Ans: F ∞ sin x
39. ∫0 x
dx = p . Ans: F
28. Half-range Fourier series expansions are
obtained by setting an = 0 or bn = 0. Ans: F
40. If a function f (x) and its transform F(s) are same
29. f (x) = 1 has a Fourier sine series expansion in then it is called self-reciprocal. Ans: T
[0, p]. Ans: T

30. A function f (x) defined in an interval [0, l] has


only sine or cosine series expansion. Ans: F CHAPTER 8 Z-TRANSFORMS AND SOLUTION
OF DIFFERENCE EQUATIONS
CHAPTER 6 PARTIAL DIFFERENTIAL
EQUATIONS 41. Z-transform is to a function of natural number
n as a Laplace transform is to a piecewise con-
31. The partial differential equation whose solution tinuous function of t. Ans: T
is Z = ax + by + a b is Z = px + qy + p q. n z
Ans: T 42. Z 〈(−1) 〉 = . Ans: T
z +1
32. The partial differential equation of all unit
z ( z − 1)
spheres with their centres at the variable point 43. Z 〈 n2 〉 = . Ans: F
at (h, k, o) is z2(p2 + q2 + 1) = 1. Ans: T ( z + 1)3
True or False Statements  !   A-43

⎪⎧ z2 ⎪⎫ 1 n +1 48. The fact that the collection of functions defined


44. Z −1 ⎨ ⎬ = (3 − 1). Ans: F on I = (t1, t2) has finite energy is indicated by
⎪⎩ ( z − 1)( z − 3) ⎪⎭ 3
t 2
45. The solution of yn+1 − 2yn = 0; y0 = 1 is ∫t1[ f (t )] dt < ∞ . Ans: T

yn = 〈 2n 〉. Ans: T
49. Daughter wavelets are generated from the mother
CHAPTER 9 WAVELETS wavelet through the use of scalings and transla-
tions of y (t). Ans: T
46. The collection of functions with finite energy is
denoted by L(t1, t2). Ans: F 50. The simplest wavelets to use are the members of
the Haar wavelet family. Ans: T
47. The set of functions with finite energy forms a
vector space. Ans: T
SOLVED QUESTION PAPERS

Code No. 33115 Set No. 1

II B.Tech. I Semester Supplementary Examination May/June 2009

Mathematics-II

(Common to Civil Engineering, Mechanical Engineering, Chemical Engineering,


Mechatronics, Metallurgy & Material Technology, Production Engineering,
Aeronautical Engineering and Automobile Engineering)

Time: 3 hours Max Marks: 80

Answer any FIVE questions

All questions carry equal marks

1. (a) Find the rank of the matrix by reducing it to 4. (a) Expand f(x) = cos ax as a Fourier series in
⎡4 3 2 1 ⎤ (−p, p) where a is not an integer. Hence,
⎢ 5 1 −1 2 ⎥ 1 2q 2q
the normal form. ⎢ ⎥ prove that cotq = q + q 2 − p 2 + q 2 − 4p 2 + ...
⎢0 1 2 3 ⎥
⎢ ⎥
⎣⎢1 −1 3 −2 ⎦⎥
p
(b) Find whether the following set of equations (b) If f(x) = x, 0 < x <
2
is consistent if so solve them:
p
2x − y + z = 5 = p − x, < x<p
3x + y − 2z = −2 2
x − 3y − z = 2. [8+8] (c) Show that f(x)
4⎡ 1 1 ⎤
= ⎢sin x − 2 sin 3 x + 2 sin 5 x − ...⎥ .
2. (a) Find the eigenvalues and the corresponding p⎣ 3 5 ⎦
⎡2 2 0⎤ [8+8]
eigenvectors of ⎢ 2 5 0 ⎥
⎢ ⎥ 5. (a) Form the partial differential equation by
⎢⎣ 0 0 3 ⎥⎦
eliminating the arbitrary constants from
(b) Prove that the eigenvalues of P−1AP are (x − a)2 + (y − b)2 + z2 = r2.
same as that of A. [10+6] (b) Solve the partial differential equation
x(y − z)p + y(z − x)q = z(x − y).
3. (a) Prove that the product of two orthogonal (c) Solve the partial differential equation
matrices is orthogonal. y2 z p + x2 z q = xy2. [5+6+5]
(b) Reduce the quadratic form 8x2 +7y2 +
6. Solve the boundary value problem ut = a2 uxx;
3z2 − 12xy − 8yz + 4xz to the canonical form.
0 < x < p, t > 0 with u (0, t) = 0 = u (p, t) and
[6+10]
u (x, 0) = px − x2. [16]
A-46 ! Engineering Mathematics-II

7. (a) Find f(x), if the Fourier cosine transform is 8. (a) Find Z [a|n|].
sin as . (b) If Z (〈un〉) = u (z), then prove that Z (〈nun〉)
s d ⎛u ⎞
= −z .
dz ⎜⎝ z ⎟⎠
(b) Find the finite cosine and sine transform of
f ( x) = 1, 0 < x < p / 2
(c) Solve the difference equation un+2 − 5un+1 +
= −1, p / 2 < x < p [8+8]
6un = 4n, uo = 0, u1 = 1. [5+5+6]
Solved Question Papers ! A-47

Code No. 33115 Set No. 2

II B.Tech. I Semester Supplementary Examination May/June 2009

Mathematics-II

(Common to Civil Engineering, Mechanical Engineering, Chemical Engineering,


Mechatronics, Metallurgy & Material Technology, Production Engineering,
Aeronautical Engineering and Automobile Engineering)

Time: 3 hours Max Marks: 80

Answer any FIVE questions

All questions carry equal marks

1. (a) Reduce the matrix 4. Expand f(x) = x sin x, 0 < x < 2p as a Fourier
⎛ 1 −1 2 −3 ⎞ series. [16]
⎜ ⎟
4 1 0 2⎟
A= ⎜ to the normal form 5. (a) Form the partial differential equation by
⎜0 3 0 4 ⎟
eliminating the arbitrary functions z = f(x) +
⎜⎜ 0 1 0 2 ⎟⎟
⎝ ⎠ ey g(x).
and hence determine its rank. (b) Solve the partial differential equation
(b) Determine whether the following equa- p x +q y = z.
tions will have a nontrivial solution, if so (c) Solve the partial differential equation x2p
solve them. (y−z) + y2q (z−x) = z2 (x−y). [5+5+6]
x + y − 2z + 3w = 0, x − 2y + z − w = 0
4x + y − 5z + 8w = 0, 5x − 7y + 2z − w = 0.
6. Solve the boundary value problem ut = uxx;
[8+8]
0 < x < l, t > 0 with u(0, t) = 0; ux(l, t) = 0 and
u(x, 0) = x. [16]
2. (a) Find the eigenvalues and the corresponding
eigenvectors of the matrix
7. (a) Find the cosine transform of e − ax x
⎡ 3 10 5 ⎤
A = ⎢⎢ −2 −3 −4⎥⎥ (b) Find the finite Fourier sine and cosine trans-
⎣⎢ 3 5 7 ⎦⎥ form of eax in (0, l). [8+8]
(b) Prove that the product of eigenvalues of a
matrix is equal to its determinant. [10+6] 8. (a) State and prove the final value theorem.
(b) Using Z-transform, solve 4un − un+2 = 0
3. (a) Prove that the eigenvalues of a real symmet- given that u0 = 0, u1 = 2. [6+10]
ric matrix are real.
(b) Reduce the quadratic form 7x2 + 6y2 + 5z2
− 4xy − 4yz to the canonical form. [6+10]
A-48 ! Engineering Mathematics-II

Code No. 33115 Set No. 3

II B.Tech. I Semester Supplementary Examination May/June 2009

Mathematics-II

(Common to Civil Engineering, Mechanical Engineering, Chemical Engineering,


Mechatronics, Metallurgy & Material Technology, Production Engineering,
Aeronautical Engineering and Automobile Engineering)

Time: 3 hours Max Marks: 80

Answer any FIVE questions

All questions carry equal marks

1. (a) Determine the rank of the matrix (b) If


⎡ 1 2 −1 5 4 ⎤ ⎧ p p
A = ⎢⎢ 1 0 2 −3 5 ⎥⎥ by reducing ⎪ 3 , for 0 ≤ x < 3

⎪ p 2p
⎣⎢ −1 2 3 4 −2⎥⎦ f ( x) = ⎨ 0, for ≤ x < show that
⎪ 3 3
it to the normal form.
⎪ −p 2p
(b) Find whether the following set of equations ⎪ 3 , for 3 ≤ x < p
is consistent, if so solve them. ⎩
x+y+z=8 2 ⎡ cos 5 x cos 7 x ⎤
f ( x) = ⎢ cos x − + − ...⎥ .
2x + 3y + 2z = 19 3 ⎣ 5 7 ⎦
4x + 2y + 3z = 23. [8+8] [8+8]
2. Verify the Cayley–Hamilton theorem and hence 5. (a) Form the partial differential equation by
⎡1 3 7 ⎤ eliminating the arbitrary functions
z = f(x − it) + g(x + it).
evaluate A−1, A = ⎢⎢1 2 3⎥⎥ . [16]
(b) Solve the partial differential equation
⎢⎣1 2 1 ⎥⎦ pyx + qz = xy.
(c) Solve the partial differential equation
3. (a) Prove that the eigenvalues of a real symmetric (z2 − 2yz − y2)p + (xy + zx)q = xy − zx.
matrix are real. [5+6+5]
(b) Reduce the quadratic form 7x2 + 6y2 + 5z2 −
4xy − 4yz to the canonical form. [6+10] 6. Solve the boundary value problem utt = a2 uxx;
0 < x L; t > 0 with u(0, t) = 0; u(L, t) = 0 and
4. (a) Express f(x) = |x|, −p < x < p as u(x, 0) = 0; ut(x, 0) = sin3(px/L). [16]
Fourier series. Hence, show that
7. (a) Find the finite Fourier sine and cosine trans-
1 1 1 1 p2
+ + + ....... = forms of
12 32 52 7 2 8 i. f(x) = x in (0, 1).
Solved Question Papers ! A-49

(b) Find the finite sine transform of f(x) = cos kx 8. (a) Find the Z-transform of sin (3n + 5).
in 0 < x < p. [8+8] ⎡ z ⎤
(b) Find Z−1 ⎢ 2 ⎥. [8+8]
⎣ z + 11z + 24 ⎦
A-50 ! Engineering Mathematics-II

Code No. 33115 Set No. 4

II B.Tech. I Semester Supplementary Examination May/June 2009

Mathematics-II

(Common to Civil Engineering, Mechanical Engineering, Chemical Engineering,


Mechatronics, Metallurgy & Material Technology, Production Engineering,
Aeronautical Engineering and Automobile Engineering)

Time: 3 hours Max Marks: 80

Answer any FIVE questions

All questions carry equal marks

1. (a) Find the rank of the matrix by reducing it to (b) Identify the nature of the quadratic form
the normal form −3 x12 − 3 x22 − 3 x32 − 2 x1 x2 − 2 x1 x3 + 2 x2 x3 .
⎡6 1 3 8⎤ Find index and signature. [8+8]
⎢ 4 2 6 −1⎥
A=⎢ ⎥
4. (a) Given that f(x) = x + x2 for −p < x < −p find
⎢10 3 9 7 ⎥
⎢ ⎥ the Fourier expansion of f(x). Deduce that
⎢⎣16 4 12 15 ⎥⎦ p2 1 1 1
(b) Find the values of λ for which the following = 1 + 2 + 2 + 2 + ...
6 2 3 4
set of equations may posses nontrivial solution
(b) Find the half-range sine series for f(x) =
and solve them in each case.
x(p − x), in 0 < x < p. Deduce that
3x1 + 2x2 − lx3 = 0
1 1 1 1 p3
4x1 − 2x2 − 3x3 = 0 − + − + ........ = .
2lx1 + 4x2 + lx3 = 0. [8+8] 13 33 53 73 32 [10+6]

2. (a) Find the eigenvalues and the corresponding 5. (a) Form the partial differential equation by
eigenvectors of the matrix eliminating the arbitrary constants from
⎡ 3 10 5 ⎤ (x − a)2 + (y − b) 2 + z2 = r2.
A = ⎢⎢ −2 −3 −4 ⎥⎥ (b) Solve the partial differential equation
x(y − z)p + y(z − x)q = z(x − y).
⎢⎣ 3 5 7 ⎥⎦
(c) Solve the partial differential equation
(b) Prove that the product of eigenvalues of a y2 z p + x2 z q = xy2. [5+6+5]
matrix in an equation is determinant. [10+6]
⎡ 2 3 + 2i 4 ⎤ 6. (a) Solve 4ux + uy = 3u given u = 3e−y − e−5y

3. (a) If 3 − 2i 5 6i ⎥⎥ show that A is when x = 0.

(b) Find the general of one-dimensional heat
⎣⎢ −4 6i 3 ⎦⎥
equation. [8+8]
Hermitian and iA is skew-Hermitian
matrices.
Solved Question Papers ! A-51

7. (a) Find the Fourier cosine transform of 8. (a) State and prove the damping rule.
e−ax cos ax. (b) Find Z (cosh at sin bt).
z
(b) Prove that the Fourier transform of the (c) Find the inverse Z-transform of 2 .
convolution of f(x) and g(x) is the product of z + 7 z + 10
their Fourier transforms. [8+8] [5+6+5]
A-52 ! Engineering Mathematics-II

ANSWERS

Set No. 1 (b) In matrix notation, the system is A X = B where


⎡ 2 −1 1 ⎤ ⎡5⎤
1. (a) Let
A = ⎢ 3 1 −2⎥ , B = ⎢⎢ −2⎥⎥ and X = [ x y z ] .
⎢ ⎥ T
⎡4 3 2 1 ⎤
⎢ 5 1 −1 2 ⎥
⎢⎣1 −3 −1⎥⎦ ⎢⎣ 2 ⎥⎦
A=⎢ ⎥
⎢0 1 2 3 ⎥ Applying elementary transformations on the
⎢ ⎥
⎢⎣1 −1 3 −2⎥⎦ augmented matrix [A, B], we have
By application of elementary ⎡ 2 −1 1 5 ⎤ ⎡1 −3 −1 2 ⎤
transformations, we reduce A
[ A, B ] = ⎢⎢ 3 ⎥ ⎢
1 −2 −2⎥ R13 ⎢ 3 1 −2 −2⎥

⎡ I 0⎤ ⎢⎣1 −3 −1 2 ⎥⎦ ⎢⎣ 2 −1 1 5 ⎥⎦
to the normal form ⎢ r ⎥
⎣ 0 0⎦
R2 − 3R1 ⎡1 −3 −1 2 ⎤ 1 ⎡1 −3 −1 2 ⎤
⎢ ⎥ R3 − R2 ⎢ ⎥
⎡1 −1 3 −2⎤ ⎡1 −1 3 −2⎤ R3 − 2 R1 ⎢0 10 1 −8⎥ 2 ⎢0 10 1 −8⎥
R 5 1 −1 2 ⎥ R2 − 5 R 1 ⎢⎢0 6
A 14 ⎢
⎢ ⎥ −16 12 ⎥⎥ ! ⎢⎣0 5 3 1 ⎥⎦ ! ⎢⎣0 0 5 2 5 ⎥⎦
! ⎢0 1

2 3 ⎥ R4 − 4 R1 ⎢0 1
⎥ ⎢
2 3 ⎥

r(A) = r(A, B) = 3 number of unknowns.
⎢⎣ 4 3 2 1 ⎥⎦ ! ⎢⎣0 7 −10 9 ⎥⎦ ∴ The system is consistent and has a unique
⎡1 −1 3 −2⎤ solution.
R23 ⎢⎢0 1 2 3 ⎥⎥ By back substitution, we get z = 2, 10y + z = −8
! ⎢0 6 −16 12 ⎥
⎢ ⎥
⇒ y = −1
x − 3y −z = 2 ⇒ x = 3(−1) + 2 + 2 = 1.
⎢⎣0 7 −10 9 ⎥⎦
⎡ 2 2 0⎤
⎡1 −1 3 −2 ⎤ ⎡1 −1 3 −2 ⎤
R3 − 6 R2 ⎢ 2. (a) The characteristic equation of A = ⎢⎢ 2 5 0⎥⎥
0 1 2 3 ⎥ R34 ⎢⎢0 1
⎥ 2 3 ⎥⎥
R4 − 7 R2 ⎢ is ⎢A − lI ⎢= 0 ⎢⎣ 0 0 3⎥⎦
! ⎢0 0 −28 −6 ⎥
⎢ ⎥
!⎢0 0

−24 −12⎥

⎣⎢0 0 −24 −12⎥⎦ ⎢⎣0 0 −28 −6 ⎥⎦ = (3 − l ) (l 2 − 7 + 6) = 0
1 ⎡1 −1 3 −2⎤ exp anding by R3
− R3 ⎢0 1 2 3 ⎥ 2−l 2 0
12 = (3 − l ) (l − 1) (l - 6) = 0
⎢ ⎥ ⇒ 2 5−l 0
"1 ⎢0 0 2 1 ⎥ ⇒ l = 1, 3, 6
− R4 ⎢ ⎥ 0 0 3− l
2 ⎢⎣0 0 14 3 ⎥⎦ are the eigenvalues of A.
⎡1 −1 3 −2⎤ 1 ⎡1 −1 3 −2 ⎤ The eigenvector X ≠ 0 corresponding to the eigen-
− R4 ⎢
R4 − 7 R3 ⎢⎢0 1 2 3 ⎥⎥ 4 ⎢0 1 2 3 ⎥⎥ value l is obtained by solving (A−lI) X = 0.
⎢0 0 2 1⎥ "1 ⎢0 0 1 1 2⎥ For l = 1, we have to solve
⎢ ⎥ R3 ⎢ ⎥
⎢⎣0 0 0 −4⎥⎦ 2 ⎣⎢0 0 0 1 ⎥⎦ ⎡1 2 0⎤ ⎡ x1 ⎤ Equivalent system is
3 ⎢ 2 4 0⎥ ⎢ x ⎥ = 0 x1 + 2 x2 = 0
R1 + R2 − 5 R3 + R4 ⎢ ⎥⎢ 2⎥
2 ⎡1 0 0 0⎤ ⎢⎣ 0 0 2⎥⎦ ⎢⎣ x3 ⎥⎦ 2 x3 = 0
R2 − 2 R3 − 2 R4 ⎢0 1 0 0⎥⎥
⎢ =I Solving
1 ⎢0 0 1 0⎥ 4
R3 − R4 ⎢ ⎥ x1 x x
2 ⎢⎣0 0 0 1⎥⎦ = 2 = 3
! 2 −1 0
x1 = [2 − 1 0]T
∴ r ( A) = 4
Solved Question Papers ! A-53

For l = 3, we have to solve = (8 − l ) (l 2 − 10l + 5)


⎡ −1 2 0⎤ ⎡ x1 ⎤ + 6 (6l − 10) + 2 (2l + 10)
⎢ 2 2 0⎥ ⎢ x ⎥ = 0
⎢ ⎥⎢ 2⎥ = − l 3 + l 2 (8 + 10) + l ( −80 − 5 + 36 + 4)
⎢⎣ 0 0 0⎥⎦ ⎢⎣ x3 ⎥⎦ + 40 − 60 + 20
Equivalent system is
x1 x2 x = − l 3 + 18l 2 − 45l = 0 ⇒ l = 0, 3, 15 are the
− x1 + 2 x2 = 0 = = 3
0 0 1 eigenvalues of A.
x1 + x2 = 0
X 2 = [0 0 1]T The eigenvector X ≠ 0 corresponding to an
0.x3 = 0
eigenvalue l is found by solving (A − lI) X = 0.
For l = 6, we have to solve
For l = 0, we have to solve
⎡ −4 2 0 ⎤ ⎡ x1 ⎤
⎢ 2 −1 0 ⎥ ⎢ x ⎥ = 0 ⎡ 8 −6 2 ⎤ ⎡ x1 ⎤
⎢ ⎥⎢ 2⎥ ⎢ −6 7 −4⎥ ⎢ x ⎥ = 0
⎢⎣ 0 0 −3⎥⎦ ⎢⎣ x3 ⎥⎦ ⎢ ⎥⎢ 2⎥
⎢⎣ 2 −4 3 ⎥⎦ ⎢⎣ x3 ⎥⎦
Equivalent system iss x1 x2 x Solving
= = 3 Equivalent system iss x1 x2 x
−4 x1 + 2 x2 = 0 1 2 0 = = 3
X 3 = [1 2 0]T R1 − 4 R3 : 10 x2 − 10 x3 = 0 1 2 2
− 3 x3 = 0
2 R1 + R2 : 10 x1 − 5 x2 = 0 X1 = [1 2 2]T
X1 X2 X3 are the eigenvectors corresponding to
the eigenvalues, l = 1, 3, 6, respectively. For l = 3 we have to solve
b) Refer to Section 2.6.3 on similarity of matrics ⎡ 5 −6 2 ⎤ ⎡ x1 ⎤
and Theorem 2.3 on p. 2-14 Chapter 2. ⎢ −6 4 −4⎥ ⎢ x ⎥ = 0
⎢ ⎥⎢ 2⎥
⎢⎣ 2 −4 0 ⎥⎦ ⎢⎣ x3 ⎥⎦
3. (a) Let A and B be orthogonal matrices of the Solving
same order n. Then AT = A−1, BT = B−1. AB and Equivalent system iss x1 x2 x
= = 3
BA are square matrics of order n. Now, (AB)T = R2 + R3 : − 4 x1 − 4 x3 = 0 2 1 −2
BTAT = B−1 A−1 = (AB)−1.
R3 : 2 x1 − 4 x2 = 0 X 2 = [2 1 − 2]T
Thus, AB is also orthogonal.
(b) The matrix of the quadratic form For l = 15 we have to solve
8x2 + 7y2 + 3z2−12xy − 8yz + 4xz ⎡ −7 −6 2 ⎤ ⎡ x1 ⎤
⎢ −6 −8 −4 ⎥ ⎢ x ⎥ = 0
⎡ 8 −6 2 ⎤ ⎡ x ⎤ ⎢ ⎥⎢ 2⎥
= X AX = [x y z] ⎢⎢ −6 7 −4⎥⎥ ⎢⎢ y ⎥⎥
T
⎢⎣ 2 −4 −12⎥⎦ ⎢⎣ x3 ⎥⎦
Solving
⎢⎣ 2 −4 3 ⎥⎦ ⎢⎣ z ⎥⎦ Equivalent systtem is x1 x x
= 2 = 3
2 R1 + R2 : − 20 x1 − 20 x2 = 0 2 −2 1
⎡ 8 −6 2 ⎤
is A = ⎢⎢ −6 7 −4⎥⎥ . The characteristic equation R2 + 3R3 : − 20 x2 − 40 x3 = 0 X 3 = [2 − 2 1]T

⎣⎢ 2 −4 3 ⎥⎦
⎡1 2 2⎤
of A is ⎢ A − l I ⎢= 0 Normalized modal matrix P = 2 1 −2⎥
1 ⎢
3⎢ ⎥
8−l −6 2 ⎢⎣ 2 −2 1 ⎥⎦
⇒ −6 7 − l −4
Diagonal matrix D = P−1 AP = PT A P
2 −4 3 − l ∴
[ P−1 = PT, P being orthogonal]
A-54 ! Engineering Mathematics-II

⎡1 2 2⎤⎡8 −6 2 ⎤ ⎡sin (n + a ) p = sin np cos ap + cos np sin ap ⎤


1 −2⎥⎥ ⎢⎢ −6 7 −4⎥⎥
1⎢ ⎢ ⎥
PT A = 2 = ( −1) n sin ap
3⎢ ⎢ ⎥
⎢⎣ 2 −2 1 ⎥⎦ ⎢⎣ 2 −4 3 ⎥⎦ ⎢sin (n − a ) p = sin np cos ap − cos np sin ap ⎥
⎢ ⎥
⎡0 0 0 ⎤ ⎡1 2 2⎤ ⎢⎣ = − ( −1) n sin ap ⎥⎦
3 −6⎥⎥ ⎢⎢ 2 1 −2⎥⎥
T 1⎢
( P A) P = ⎢ 6 ∞
9 sin a p sin a p ⎛ 1 1 ⎞
⎣⎢30 −30 15 ⎦⎥ ⎣⎢ 2 −2 1 ⎥⎦ ∴cos ax =
ap
+
p
∑ (−1)n ⎜⎝ n + a − n − a ⎟⎠
n =1
⎡0 0 0 ⎤
sin a p 2 a ∞
( −1) n
= ⎢⎢0 3 0 ⎥⎥ = − sin ap ∑ cos nx
ap p n2 − a 2
⎢⎣0 0 15⎥⎦ n =1
(3)
The canonical form of the quadratic form is 2a sin ap ⎛ 1 cos x
or cos ax = ⎜⎝ 2 + 2
p 2a 1 − a2
⎡0 0 0 ⎤ ⎡ y1 ⎤
⎢y ⎥ cos 2 x cos 3 x ⎞
T
Y DY = [ y1 y2 y3 ] ⎢⎢0 3 0 ⎥⎥ ⎢ 2⎥ − 2 + 2 −" ⎟
2 −a 2
3 − a2 ⎠
⎢⎣0 0 15⎥⎦ ⎢⎣ y3 ⎥⎦
(4)
= 0 y12 + 3 y22 + 15 y32 .
Value of f at x = p is [ f(p) + f(−p)]/2 =
= cos ap
4. (a) The Fourier series of f(x) = cos ax in (−p, p) 2a sin ap ⎛ 1 1 1 ⎞
is given by = ⎜⎝ 2 + 2 2 + 2 + ....⎟
p 2a a −1 a − 2 2


1 sin ap
cos ax = ao +
2
∑ an cos nx (1) = −∑ 2
2
sin ap
n =1 ap n − a2
Since cos ax is an even function in (−p, p), we 1 2q 2q
have ⇒ cot q = + + + ...
q q 2 − p 2 q 2 − 4p 2
2 p 2 p where q = ap
ao =
p ∫0
f ( x) dx =
p ∫0 cos ax dx
p
(2)
2 ⎛ sin ax ⎞ 2 5. (a) Example 6.1, p. 3, Ch. 6 Engg. Maths-II.
= ⎜ ⎟⎠ = sin ap (b) Example 6.36, p. 11, Engg. Maths-II.
p ⎝ a 0 pa
(c) Lagrange’s auxiliary equations are
and dx dy dz
= =
2 p 2 p y 2 z x 2 z xy 2
an =
p ∫0
f ( x) cos nx dx = ∫ cos ax cos nx dx
p 0
From the first reference, two ratios on cross-
1 p
= ∫ [cos (n + a ) x + cos (n − a ) x] dx multiplication and cancellation of z.
p 0
p x 2 dx − y 2 dy = 0 ⇒ x3 − y 3 = c1 on integration.
1 ⎡ sin (n + a ) x sin (n − a ) x ⎤
= ⎢ + ⎥
p ⎣ n+a n − a ⎦0 From the first and last terms, on cross-multipli-
cation, and cancellation of y2
1 ⎡ sin (n + a )p sin (n − a )p ⎤
= ⎢ + ⎥ x dx − z dz = 0 ⇒ x 2 − z 2 = c2 on integration.
p ⎣ n+a n−a ⎦
! sin x = 0 at x = 0 ∴The general solution is F(x3 − y3, x2 − z2) = 0.
Solved Question Papers ! A-55

6. Example 6.59 in Ch. 6 (pp. 30–31) Engg. p p 2


Maths-II. Take l = p, α = l2 = p2. (i) Fc (n) = ∫ f ( x) cos nx dx = ∫ 1.cos nx dx
0 0
p
The solution is +∫ ( −1) cos nx dx
p 2
8 sin nx
u ( x, t ) = ∑
p n =1,3..5 n3
exp ( − n 2 a 2 t )
sin nx
p 2
sin nx
p
= + ( −1)
n 0 n p 2

sin as np np
7. (a) Let Fc ( s ) = sin sin
s = 2 − ( −1) 2 = 2 sin np .
n n n 2
The inverse Fourier cosine transform of Fc(s) is
p p 2
2 ∞ 2 ∞ sin as (ii) Fs (n) = ∫ f ( x) sin nx dx = ∫ 1sin nx dx
f ( x) = ∫ Fc ( s ) cos sx ds = ∫ cos sx ds 0 0
p 0 p 0 s p

1 ∞1
+ ∫ ( −1) sin nx dx
= ∫ [sin(a + x) s + sin(a − x) s ] ds p 2
p 0 s p 2 p
− cos nx ⎛ − cos nx ⎞
= −⎜ ⎟
1 ∞ sin(a + x) s 1 ∞ sin( a − x) s n 0
⎝ n ⎠p
p ∫0 ∫0
2
= ds + ds
s p s np
cos n p − cos
1⎛ np ⎞ 2
⎧ 1 ⎛p p⎞ = ⎜1 − cos ⎟ +
⎪ p ⎜⎝ 2 + 2 ⎟⎠ = 1 if x > a n⎝ 2⎠ n

=⎨ 1 + ( −1 ) 2n
np
⎪ 1 ⎛ p − p ⎞ = 0 if x < a = − cos .
⎪⎩ p ⎜⎝ 2 2 ⎟⎠ n n 2

sin ax p
∫ x
dx =
2
if a > 0
⎧ n
n ⎪ a , if n > 0
8. un = a ⎨ − n
0

⎧p ⎪⎩ a , if n < 0

sin(a − x) s ⎪⎪ 2 if a − x > 0 ⇔ x < a
⇒∫ dx = ⎨

( )
s ∞ −1 ∞ n
⎪ − p if a − x < 0 ⇔ x > a n 1 ⎛ a⎞
∑ un z − n = ∑ (a z )n + ∑ ⎜⎝ z ⎟⎠
0
⎪⎩ 2 Z a =
n = −∞ n = −∞ n=0
2
1 1 a ⎛ a⎞
(b) Finite (i) cosine and (ii) sine transforms of =! + + + 1 + + ⎜ ⎟ ...
f(x) in 0 < x < l are given, respectively, by (a z )2 az z ⎝ z⎠
⎛ 1 ⎞ 1 az z
=⎜ −1 + = +
l npx ⎝ 1 − az ⎟⎠ a 1 − az z − a
(i) Fc (n) = ∫ f ( x) cos dx 1−
0 l z
The first geometric series converges absolutely
(ii) Fs (n) = f ( x) sin npx dx
l
∫0 l for z <
1
and the second geometric series
a
converges absolutely for ⏐z⏐>⏐a⏐. So, the
⎧ 1, 0 < x>p 2 1
Here f ( x) = ⎨ and l = p region of convergence is a < z < .
⎩ − 1, p 2< x>p a
A-56 ! Engineering Mathematics-II

(b) Refer Section 8.5.4 on multiplication by n; u ( z) ( z − 4) + 1


Theorem 8.6, p. 8, Engg. Maths-II. =
z ( z − 2) ( z − 3) ( z − 4)
(c) Applying Z-transform Z(〈un〉) = u (z), u0 = 0, z −3
=
u1 = 1 we obtain ( z − 2) ( z − 3) ( z − 4)
1 1⎛ 1 1 ⎞
Z un + 2 − 5 Z un + 1 + 6 Z un = Z 4 n = = −
( z − 2) ( z − 4) 2 ⎝ z − 4 z − 2 ⎟⎠

⎛ 1⎞
(
⇒ z ⎜ u ( z ) − u0 − u1 ⎟ − 5 z u ( z ) − u0

2
z⎠
) 1 z
u ( z) = −. .
1 z
2 z−4 2 z−2
z
+ 6 u (zz ) = Taking the inverse Z-ttransform
z−4
1 n 1 n
z
( )
⇒ u ( z ) z 2 − 5z + 6 = z +
z−4
un =
2
4 − 2
2
z
! u0 = 0, u1 = 1 ∵ Z an =
z−a
Solved Question Papers ! A-57

Set No. 2 For the H system to have a nontrivial solution


A must be singular. Applying elementary trans-
!1 # 1 2 # 3 "
$ % formations on A, we have
$4 1 0 2%
1. (a) Let A = $
$ % A ⎡1 1 −2 3 ⎤
0 3 0 4 % ⎢0 − 3 3 − 4 ⎥
$
$0 1 0 2%
% R2 − R1 ⎢
"

$ %
& ’ R3 − 4 R1 ⎢ 0 − 3 3 − 4 ⎥
By applying elementary transformations, we ⎢ ⎥
R4 − 5 R1 ⎢⎣ 0 − 12 12 − 16⎦⎥
⎡I 0⎤
reduce A to the normal form N = ⎢ r ⎥. ⎡1 1 − 2 3 ⎤
⎣0 0 ⎦ "R3 − R2 ⎢0 − 3 3 − 4 ⎥
⎢ ⎥
1 −1 2 −3⎤
R2 _ 4 R 1 ⎡⎢ R4 − 4 R2 ⎢0 0 0 0⎥
− 14⎥⎥ ⎢ ⎥
A ! ⎢ 0 5
⎢0 1 0 2 ⎥
8 ⎢⎣0 0 0 0⎥⎦
R34 ⎢ ⎥ Equivalent syystem is
⎢⎣0 3 0 4 ⎥⎦ x + y − 2z + 3 w = 0
1 −1 2 −3 ⎤
R4 − 3R3 ⎡⎢ ⎥
−3 y+3z −4w = 0
! ⎢0 0 − 8 4 ⎥ r (A) = r = 2 < no. of unknowns = n = 4.
⎢0 1 0 2 ⎥ ∴The H system has a nontrivial solution. Since
R2 − 5 R3 ⎢ ⎥
⎢⎣0 0 0 − 2⎥⎦ n − r = 4 − 2, we can choose two variables, z, w
(say). Let z = k1, w = k2. Then
⎡1 − 1 2 − 3 ⎤
R23 ⎢ ⎥ 1
y = (3 k1 − 4 k2 ) and x = − y + 2 z − 3w
! ⎢0 1 0 2 ⎥
⎢0 0 − 8 4 ⎥ 3
⎢ ⎥ 4 5
⎢⎣0 0 0 − 2⎥⎦ = − k1 + k2 + 2 k1 − 3 k2 = k1 − k2
3 3
⎡1 − 1 2 − 3⎤
R3 + 2 R4 ⎢ 5
⎢0 1 0 2 ⎥⎥ The solution is x = k1 − k2
"1 3
− R4 ⎢0 0 − 8 0 ⎥ 4
2 ⎢ ⎥ y = k1 − k2
⎢⎣0 0 0 1 ⎥⎦ 3
1 z = k1
R1 + R2 + R3 + R4 ⎡1 0 0 0⎤
4 ⎢0 1 0 0⎥⎥ = I 4 ( Normal w = k2
! R2 − 2 R4 ⎢
⎢0 0 1 0⎥ form) where k1, k2 are arbitrary real numbers.
1 ⎢ ⎥
− R3 ⎢⎣0 0 0 1⎥⎦ 2. (a) The characteristic equation of A is ⏐A−lI⏐= 0
8
(b) In matrix notation, the homogeneous 3− l 10 5
system (H) is AX = 0, the coefficient matrix ⇒ −2 −3 − l −4
⎡1 1 − 2 3 ⎤ 3 5 7−l
⎢1 − 2 1 − 1 ⎥
where is A = ⎢ ⎥ ( )
= (3 − l ) l 2 − 4l − 1 − 10 ( 2l − 2) + 5 (3l − 1)
⎢4 1 − 5 8 ⎥ = − l + l (3 + 4) + l ( −12 + 1 − 20 + 15)
3 2
⎢ ⎥
⎢⎣5 − 7 2 − 1⎥⎦
+ ( −3) + 20 − 5
A-58 ! Engineering Mathematics-II

(
= − l 3 − 7 l 2 + 16 l − 12 = 0 ) ( )
= ( 7 − l ) l 2 − 11 l + 26 + 2 ( 2l − 10)
⇒ l = 3, 2 , 2 (eigenvalues of A)
= − l 3 + l 2 ( 7 + 11) + l ( −77 − 26 + 4) + 182 − 20
3 1 −7 16 −12
(
= − l 3 − 18 l 2 + 99 l − 162 = 0 )
3 −12 12
⇒ l = 3, 6, 9 are the eigenvalues of A.
1 −4 4 0
3 1 −18 99 −162
The eigenvector X ≠ 0 corresponding to an
eigenvalue l is found by solving (A − lI) X = 0. 3 −45 162
For l = 2 we have to solve 1 −15 54 0
⎡ 1 10 5 ⎤ ⎡ x1 ⎤ The eigenvector X ≠ 0 corresponding to an
⎢ −2 − 5 − 4⎥ ⎢ x ⎥ = 0 eigenvalue l is found by solving (A − lI) X = 0.
⎢ ⎥ ⎢ 2⎥
⎢⎣ 3 5 5 ⎥⎦ ⎢⎣ x3 ⎥⎦ For l = 3 we have to solve
Solving
Equivalent system iss x1 x2 x ⎡ 4 − 2 0⎤ ⎡ x1 ⎤
= = 3 ⎢ −2 ⎢ ⎥
R1 − R3 : − 2 x1 + 5 x2 = 0 5 2 −5 ⎢ 3 − 2 ⎥⎥ ⎢ x2 ⎥ = 0
x1 + x3 = 0 X1 = [5 2 − 5] = X 2
T
R2 + R3 : ⎢⎣ 0 − 2 2 ⎥⎦ ⎢⎣ x3 ⎥⎦ Solving
Only one eigenvector corresponds to the Equivalent system is x1 x2 x3
= =
repeated eigenvalue l = 2. R1 : 2 x1 − x2 = 0 1 2 2
R3 : − x2 + x3 = 0 X1 = [1 2 2]
T
For l = 3 we have to solve
⎡ 0 10 5 ⎤ ⎡ x1 ⎤ For l = 6 we have to solve
⎢ −2 − 6 − 4⎥ ⎢ x ⎥ = 0
⎢ ⎥ ⎢ 2⎥ ⎡ 1 − 2 0 ⎤ ⎡ x1 ⎤
⎢ −2 ⎢ ⎥
⎢⎣ 3 5 4 ⎥⎦ ⎢⎣ x3 ⎥⎦
Solving ⎢ 0 − 2 ⎥⎥ ⎢ x2 ⎥ = 0
Equivalent system iss x1 x2 x3 ⎢⎣ 0 − 2 − 1⎥⎦ ⎢⎣ x3 ⎥⎦
= = Solving
R1 : 2 x2 + x3 = 0 1 1 −2 Equivalent system m is x1 x2 x3
= =
X 3 = [1 1 − 2]
T
R2 + R3 : x1 − x2 = 0 R1 : x1 − 2 x2 = 0 2 1 −2
R3 : 2 x2 + x3 = 0 X 2 = [ 2 1 − 2]
T
(b) Refer to Property 3 in Section 2.4. Refer to
Section 2.4 (p. 2-7), Engg. Maths-II.
For l = 9 we have to solve
3. (a) Refer to Theorem 3.19, Section 3.5 of Engg.
Maths-II. ⎡ − 2 − 2 0 ⎤ ⎡ x1 ⎤
The matrix A of the quadratic form is ⎢ −2 − 3 − 2 ⎥ ⎢ x ⎥ = 0
⎢ ⎥⎢ 2⎥
7 x 2 + 6 y 2 + 5 z 2 = −4 xy − 4 yz ⎢⎣ 0 − 2 − 4⎥⎦ ⎢⎣ x3 ⎥⎦ Solving
⎡ 7 −2 0⎤ Equivalent systtem is x1 x2 x
= = 3
A = ⎢⎢ −2 6 − 2⎥⎥ R1 : x1 + x2 = 0 2 −2 1
⎢⎣ 0 − 2 5 ⎥⎦ R3 : x2 + 2 x3 = 0 X 3 = [ 2 − 2 1] T

The characteristic equation of A is ⏐A−lI⏐= 0 ⎡1 2 2 ⎤


1⎢
7−l −2 0 Normalized modal matrix P = 2 1 −2⎥⎥ ,
3⎢
⇒ −2 6 − l −2 ⎢⎣ 2 −2 1 ⎥⎦
0 −2 5 − l since the norm of each vector is 3.
Solved Question Papers ! A-59

The diagonal matrix D is given by Eliminating g from (2) and (3) we obtain
D = P−1 AP = PTAP the required partial differential equation as
⎡1 2 2 ⎤ ⎡ 7 −2 0 ⎤ !z ! 2 z
!
= 1 ⎢ 2 1 −2⎥ ⎢ −2 6 −2⎥ P !y !y 2
3⎢ ⎥⎢ ⎥
⎢⎣ 2 −2 1 ⎥⎦ ⎢⎣ 0 −2 5 ⎥⎦ (b) Refer Ex. 6.35 on p. 16 of Ch. 6, Engg.
Maths-II (Take n = 1/2).
(! P −1 = PT , P beeing orthogonal)
(c) Refer Ex. 6.24 on p. 12 of Ch. 6, Engg.
⎡3 6 6 ⎤ ⎡1 2 2⎤ Maths-II.
1⎢
= 12 6 −12⎥⎥ ⎢⎢ 2 1 −2⎥⎥
9⎢
⎢⎣18 −18 9 ⎥⎦ ⎢⎣ 2 −2 1 ⎥⎦ 6. Refer Ex. 4 of Section 6.13.4 of Ch. 6, Engg.
⎡ 27 0 0 ⎤ ⎡ 3 0 0⎤ Maths-II
1⎢
= ⎢ 0 54 0 ⎥⎥ = ⎢⎢0 6 0⎥⎥ with the initial condition
9
⎣⎢ 0 0 81⎦⎥ ⎣⎢0 0 9 ⎦⎥ u(x,0) = f(x) = x (1)

The canonical form of the quadratic form is The solution given at eq. (7) is

⎛ np ⎞
⎡ 3 0 0⎤ ⎡ y1 ⎤
⎢ ⎥
u(x, t) = ∑ Bn sin pn x exp (−a n pn2t ) ⎜⎝ pn = ⎟ (2)
l ⎠
Y(PAP) Y = YDY = [y1 y2 y3 ] ⎢⎢0 6 0⎥⎥ ⎢ y2 ⎥ n =1

⎢⎣0 0 9 ⎥⎦ ⎢⎣ y3 ⎥⎦ Imposing the initial condition (1) on (2) we get



= 3 y12 + 6 y22 + 9 y32
u(x, 0) = x = ∑ Bn sin pn x
The orthogonal transformation that transforms n =1
the Q. F. to canonical form is given by X = PY
which is the half-range Fourier sine series in (0,
⎡ x1 ⎤ ⎡1 3 2 3 2 3 ⎤ ⎡ y1 ⎤ l) for f(x) = x. The constants Bn are given by
⎢ x ⎥ = ⎢ 2 3 1 3 −2 3⎥ ⎢ y ⎥
⎢ 2⎥ ⎢ ⎥⎢ 2⎥ 2 l 2 l
⎢⎣ x3 ⎥⎦ ⎢⎣ 2 3 −2 3 1 3 ⎥⎦ ⎢⎣ y3 ⎥⎦
Bn =
l ∫0 f ( x) sin pn x dx = l ∫0 x sin pn x dx
⎡1 2 2 ⎤ l
⎢ 3 y1 + 3 y2 + 3 y3 ⎥ 2 ⎡ ⎛ − cos pn x ⎞ ⎛ − sin pn x ⎞ ⎤
⎢ ⎥ = ⎢x ⎜ ⎟⎠ − 1 ⎜⎜ ⎟⎥
l⎢ ⎝ pn ⎝ p n2 ⎟⎠ ⎥
= ⎢ y1 + y2 − y3 ⎥ .
2 1 2 ⎣ ⎦0
⎢3 3 3 ⎥
⎢ ⎥ − cos pn l 2l ( −1) n −1
= 2. =
⎢2 y − 2 y + 1 y ⎥ pn np
⎣⎢ 3 3 ⎦⎥
1 2 3
3
Hence the temperature distribution in the bar is
4. Refer to Ex. 5.4 on p. 10 of Ch. 5 in Engg
Maths II. 2l ∞ ( −1) n −1 np x
u(x, t) = ∑
p n =1 n
sin
l
exp ( − n 2p 2 a 2 t )
5. (a) It is given that z = f(x) + ey g(x)
Where f, g are arbitrary functions. Differen-
tiating (1) partially w.r.t. y twice we get 7. (a) By definition, the Fourier cosine transform
of f(x) is
∂z ∂2 z
= 0 + e y g ( x) = e y g ( x) ∞
∂y ∂y 2
Fc{f(x)} = Fc(s) = ∫0 f ( x) cos sx dx
A-60 ! Engineering Mathematics-II

Here f(x) = e − ax ⇒ 8 (a) Refer Property 7 on p. 9 of Section 7, p. 7-4,


x Art 8.5 of Engg. Maths-II.
− ax ⎧ e − ax
⎪⎧ e ⎪⎫ ∞⎪ ⎪⎫ (b) Applying Z-transform defined by Z〈un〉 =
Fc ⎨ ⎬ = Fc ( s ) = ∫0 ⎨ cos sx dx ⎬
⎩⎪ x ⎭⎪ ⎩⎪ x ⎭⎪ u ( z ) we obtain
Differentiating both sides w.r.t. ‘s’ 4Z 〈un〉 − Z 〈un+2〉 = 0
d ∞e
− ax

1
Fc ( s ) = ∫ (cos sx) dx = − ∫ e − ax sin sx dx ⇒ z2 ( u ( z ) − u0 − u1. ) − 4 u ( z) = 0
ds 0 x 0 z


⇒ (z2 − 4) u ( z ) = 2z
− ax
= ⎡− e ⎤
( − a sin sx − s cos sx) ⎥ = 2
−s
! u0 ! 0, u1 ! 2
⎢ 2
⎣ a +s ⎦0 a + s
2 2

Integrating now w.r.t. s we obtain u( z) 2 2⎛ 1 1 ⎞


⇒ = 2 = ⎜ − ⎟
1
z z − 4 4 ⎝ z − 2 z + 2⎠
Fc ( s ) = − log (a 2 + s 2 ) + c 1 1 1 1
2 = . − .
2 z−2 2 z+2
When s = 0 we have Fc(0) =
1 z 1 1
1
! log a 2 ! c " 0 " c " log a ⇒ u( z) = . − .
2 2 z − 2 2 z − ( −2)

Hence Fc ( s ) = log a2 Taking inverse Z-transform we obtain the


.
a + s2 2
solution as
(b) Refer to finite Fourier sine/cosine trans-
1 n 1
forms in Engg. Maths-II. un = . 2 − ( −2) n .
2 2
Solved Question Papers ! A-61

Set No. 3 (b) In matrix notation the system can be written as


⎡1 1 1 ⎤ ⎡8⎤
1. (a) By applying elementary transformations we
AX = B where A = ⎢ 2 3 2⎥ , B = ⎢⎢19 ⎥⎥ and
⎢ ⎥
I 0⎤
reduce A to the normal form ⎡ r ⎢⎣ 4 2 3⎥⎦ ⎢⎣ 23⎥⎦
⎢ 0 0⎥ .
⎣ ⎦ X = [x y z]T
Applying elementary transformations on the
⎡ 1 2 −1 5 4⎤
augmented matrix we obtain
A = ⎢⎢ 1 0 2 −3 5 ⎥⎥
⎢⎣ −1 2 3 4 ⎡1 1 1 8 ⎤
−2⎥⎦
R2 − R1 ⎡1 2 −1 5 4⎤
[ A, B ] = ⎢⎢ 2 3 2 19 ⎥⎥
⎢⎣ 4 2 3 23⎥⎦
R3 + R2 ⎢⎢0 −2 3 −8 1 ⎥⎥
! ⎢⎣0 2 5 1 3⎥⎦
we obtain
R2 − 2 R1 ⎡1 1 1 8 ⎤
⎡1 2 −1 5 4⎤ ⎢ ⎥
R3 − 4 R1 ⎢0 1 0 3 ⎥
R3 + R2 ⎢0 −2 3 −8 1 ⎥⎥
⎢ ! ⎢⎣0 −2 −1 −9⎥⎦
! ⎣⎢0 0 8 −7 4⎥⎦
⎡1 1 1 8 ⎤
C2 − 2C1
" R3 + 2 R2 ⎢ ⎥
⎡1 0 0 0 0⎤
C3 + C1 ⎢ ! ⎢0 1 0 3 ⎥
0 −2 3 −8 1 ⎥⎥ ⎢⎣ 0 0 −1 −3⎥⎦
C4 − 5C1 ⎢
⎢⎣0 0 8 −7 4⎥⎦
C5 − 4C1 ⎡ 1 1 1 8⎤
− R3 ⎢ ⎥
1 ⎡1 0 0 0 0⎤ ! ⎢0 1 0 3⎥
− C2 ⎢ ⎥ ⎢⎣0 0 1 3⎥⎦
2 ⎢0 1 3 −8 1 ⎥
! ⎢⎣0 0 8 −7 4⎥⎦ r(A) = r(A, B) = 3 = number of unknowns.
C3 − 3C2 ∴The system is consistent and has a unique
⎡1 0 0 0 0 ⎤ solution: x = 2, y = z = 3.
C4 + 8C2 ⎢
0 1 0 0 0⎥⎥
C5 − C2 ⎢ ⎡1 3 7 ⎤
⎢⎣0 0 8 −7 4⎥⎦
! ⎢ ⎥
2. A = ⎢1 2 3⎥ . The characteristic equation of
⎡1 0 0 0 0 ⎤ ⎢⎣1 2 1 ⎥⎦
1" ⎢0 1 0 0 0⎥
C3 A is ⏐A − lI⏐ = 0
8 ⎢ ⎥
⎢⎣0 0 1 −7 4⎥⎦
1− l 3 7
C4 + 7C3 ⎡1 0 0 0 0⎤ ⇒ 1 2−l 3
C5 − 4C3 ⎢⎢0 1 0 0 0⎥⎥ 1 2 1− l
! ⎢⎣0 0 1 0 0⎥⎦
= (1 − l ) (l 2 − 3l - 4) − 3( −2 − l ) + 7 l
= [ I 3 0] normal form ∴ r ( A) = 3 = − l 3 + l 2 (1 + 3) + l (-3 + 4 + 3 + 7) − 4 + 6 = 0
⇒ l 3 − 4l 2 − 11l - 2 = 0 (1)
A-62 ! Engineering Mathematics-II

By Cayley–Hamilton theorem, every square 4 (a) Refer Ex. 5.19 on pp. 24–25, Ch. 5 in Engg.
matrix satisfies its own charateristic equation. Maths-II. Take a = 0, b = −1.
So, we must have A3 − 4A2 − 11A − 2I = 0
(b) f(x) is defined as
This can be written as BA = 2I (2)
where B = A2 − 4A − 11I = (A − 4I)A − 11I (3) ⎧ p 3 for 0 ≤ x < p 3

f ( x) = ⎨ 0 for p 3 ≤ x < 2 p 3
⎡ −3 3 7 ⎤
⎪ -p 3 for p 3 ≤ x < p (1)
A − 4 I = ⎢⎢ 1 −2 3 ⎥⎥ ; ⎩
⎢⎣ 1 2 −3⎥⎦ The funtion is defined in the posi-
tive interval (0, p). We have to show that
⎡ −3 3 7 ⎤ ⎡1 3 7 ⎤ 2 ⎛ cos 5 x cos 7 x ⎞
f(x) = ⎜⎝ cos x − + − .....⎟ (2)
( A − 4 I ) A = ⎢⎢ 1 −2 3 ⎥⎥ ⎢⎢1 2 3⎥⎥ 3 5 7 ⎠
⎢⎣ 1 2 −3⎥⎦ ⎢⎣1 2 1 ⎥⎦ We have, therefore, to expand f(x) in a half-
range cosine series in (0, p). Half-range Fourier
⎡7 11 −5⎤ cosine series of f(x) in [0, l] is given by
= ⎢⎢ 2 5 4 ⎥⎥ 1 ∞
npx
f ( x) = a0 + ∑ an cos where
⎢⎣ 0 1 10 ⎥⎦ 2 n =1 l

⎡ −4 11 −5⎤ 2 l 2 l npx
a0 = ∫ f ( x)dx and an = ∫ f ( x) cos dx
B = ( A − 4 I ) A − 11I = ⎢⎢ 2 −6 4 ⎥⎥ and l 0 l 0 l
⎢⎣ 0 1 −1⎥⎦
Here l = p, and f(x) is defined as (1).

⎡ −4 11 −5⎤ ⎡1 3 7 ⎤ 2 l 2 p
BA = ⎢⎢ 2 −6 4 ⎥⎥ ⎢⎢1 2 3⎥⎥
a0 =
l ∫0
f ( x)dx = ∫ f ( x)dx
p 0
⎢⎣ 0 1 −1⎥⎦ ⎢⎣1 2 1 ⎥⎦ 2 p/ 3 p 2 p ⎛ p⎞
= ∫ dx + 0 + ∫ ⎜ − ⎟ dx
⎡2 0 0⎤ p 0 3 p 2p 3 ⎝ 3 ⎠
= ⎢0 2 0⎥⎥ = 2I =
2 p p 3 2 ⎛ p⎞ p
( x ) + ⎜⎝ − ⎟⎠ ( x )2p

⎢⎣ 0 0 2⎥⎦ p 3 0 p 3 3

2p 2⎛ 2p ⎞ 2p 2p
= − ⎜p - ⎟= − =0
Hence Caryley–Hamilton theorem is verified. p 3 p⎝ 3⎠ 9 9
|A| = −2, putting l = 0 in (1) ⇒ |A| ≠ 0 and
2 l npx 2 p
hence A −1 exists. an = ∫ f ( x) cos dx = ∫ f ( x) cos nx dx
l 0 l p 0
From (2) we obtain 2 p/ 3 p
= ∫ cos nx dx + 0
⎡ −4 11 −5⎤ p 0 3
2 A = B = ⎢⎢ 2 −6 4 ⎥⎥
−1 2 p ⎛ p⎞
+ ∫ ⎜ − ⎟ cos nx dx
⎢⎣ 0 1 −1⎥⎦ p 2p/ 3 ⎝ 3 ⎠
p/ 3 p
⎡ −4 11 −5⎤ 2 p ⎛ sin nx ⎞ 2 p ⎛ sin nx ⎞
= . ⎜ ⎟ − . ⎜ ⎟
⇒ A = ⎢ 2 −6 4 ⎥⎥
−1 1 ⎢ p 3 ⎝ n ⎠0 p 3 ⎝ n ⎠ 2p
3
2
⎢⎣ 0 1 −1⎥⎦ 2 np 2 ⎛ 2np ⎞
= sin − ⎜ sin np - sin ⎟
3. (a), (b), refer to 3 (a), (b), in set 2. 3n 3 3n ⎝ 3 ⎠
Solved Question Papers ! A-63

2 np 2 2np From the first and last terms we get dx = dz,


= sin + sin cancelling xy
3n 3 3n 3
2 ⎛ np 2np ⎞ ⇒ x − z = c1 (2)
= ⎜ sin + sin ⎟
3n ⎝ 3 3 ⎠ Again, from the last equation we get
dy dz
Clearly a3 = a6 = a9 = . . . = 0 = , using (2) (3)
z y (c1 + z )
2⎛ p 2p ⎞ 2 3 ⎛ 2 ⎞ 2
a1 = ⎜⎝ sin + sin ⎟⎠ = . ⎜⎝ 2. ⎟⎠ = z ⎛ c ⎞
3 3 3 3 2 3 3 ⇒ y dy = dz = ⎜1 − 1 ⎟ dz
c1 + z ⎝ c1 + z ⎠
2 ⎛ 2p 4p ⎞ 1 3
a2 = ⎜⎝ sin + sin ⎟ = .
3× 2 3 3⎠ 3 2 Integrating 1 y 2 = z − c log(c + z ) + c (4)
1 1 2
2
1 3
− . =0 General solution is
3 2
⎛ 1 2 ⎞
⎜ x − z , y − z + ( x − z ) log x ⎟ = 0 (5)
2 ⎛ 4p 8p ⎞ 1⎛ 3 3⎞ ⎝ 2 ⎠
a4 = ⎜ sin + sin ⎟= ⎜⎜ + ⎟=0
3× 4 ⎝ 3 3 ⎠ 6⎝ 2 2 ⎟⎠ (c) Refer Ex. 6.34 on p. 15, Ch. 6 of Engg.
Maths-II.
2 ⎛ 5p 10p ⎞
a5 = ⎜⎝ sin + sin ⎟ 6. Refer Section 6.13.7 of p. 6-38, Engg. Maths-II.
3×5 3 3 ⎠ Take u in place of y.
2⎛ p 2p ⎞
= ⎜ − sin + sin ⎟⎠ In eq. (6.140) if we take
15 ⎝ 3 3
3 px 3 px 1 3px
g(x) = sin = sin − sin
2 ⎛ −2 3⎞ 2 . l 4 l 4 l
= ⎟=
15 ⎜⎝ 2 ⎠ 5 3 We obtain

∂y npa npx
2 ⎛ 7p 14p ⎞ ( x, 0) = ∑ Cn ⎛⎜⎝ .sin ⎞⎟⎠ = g ( x)
a7 = ⎜⎝ sin + sin ⎟ ∂t n =1 l l
3× 7 3 3 ⎠
2 ⎡ ⎛ p⎞ p⎞⎤ pa px 2pa 2px 3pa 3px
⎛ ⇒ C1 sin + C2 sin + C3 sin
= ⎢ sin ⎜ 2p + ⎟ + sin ⎜ 5p - ⎟ ⎥ l l l l l l
21 ⎣ ⎝ 3 ⎠ ⎝ 3⎠⎦
3 px 1 3px
2 + ..... = sin − sin
= ,.... 4 l 4 l
7 3
Equating the coefficient of like terms on either
Substituting these values we get the expansion as side, we get
2 ⎛ cos 5 x cos 7 x ⎞ pa 3 2pa 3pa 1
f ( x) = ⎜ cos x − + − .... ⎟ C1 . = , C2 . = 0, C3 . =− ,
3⎝ 5 7 ⎠ l 4 l l 4
and Cn = 0 for n > 3.
5. (a) Refer Ex. 6. 13 on pp. 6-7 of Ch. 6, Engg.
Maths-II (take a = −i). The solution of the problem is
(b) Lagrange’s auxiliary equations are 3l px pat
u ( x, t ) = sin sin
dx dy dz 4pa l l
= = (1)
xy z xy l 3px 3pat
− sin sin .
12pa l l
A-64 ! Engineering Mathematics-II

7 (a) f(x) = x in (0, 1) 8. (a) Z sin(3n + 5) = cos 5 Z sin 3n


The finite Fourier sine and cosine transforms of + sin 5 Z (cos 3n)
f(x) are ! sin(3n + 5) = sin 3n(cos 5)
l npx + cos 3n(sin 5)
Fs { f ( x)} = Fs (n) = ∫ f ( x) sin dx and
0 l
l npx z sin 3 z ( z − cos 3)
Fc { f ( x)} = Fc (n) = ∫ f ( x) cos dx = cos 5. + sin 5. 2
0 l z 2 − 2 z cos 3 + 1 z − 2 z cos 3 + 1
Taking f(x) = x and l = 1 we obtain z 2 sin 5 − z (sin 5 cos 3 − cos 5 sin 3)
=
1 npx z 2 − 2 z cos 3 + 1
Fs { x} = Fs (n) = ∫ x sin dx
0 1 z 2 sin 5 − z sin 2
1
= 2 .
⎡ ⎛ 1 ⎞ ⎛ 1 ⎞⎤ z − 2 z cos 3 + 1
= ⎢x ⎜ − .cos npx⎟ − 1 ⎜ − 2 2 .sin npx⎟ ⎥ z sin q − z sin 2
⎣ ⎝ np ⎠ ⎝ n p ⎠ ⎦0 ! Z sin nq = 2 ;
= 0 at 1, 0 z − 2 z cos 0 + 1
z 2 − z cos 0
cos np ( −1) n −1 Z cos nq = 2
=− = z − 2 z cos 0 + 1
np np
1 npx
Fc { x} = Fc ( x) = ∫ x cos dx (b)
0 1
⎧ z ⎫ −1 ⎧ z ⎫
1
⎡ ⎛ sin npx ⎞ ⎛ cos npx ⎞ ⎤ cos np − 1 Z −1 ⎨ 2 ⎬=Z ⎨ ⎬
= ⎢x ⎜ − ⎟⎠ − 1 ⎜⎝ − 2 2 ⎟⎠ ⎥ =
⎣ ⎝ np n p ⎦0 n 2p 2 ⎩ z + 11z + 24 ⎭ ⎩ ( z + 8)( z + 3) ⎭
z
= 0 at 1, 0 Let u ( z ) =
( z + 3)( z + 8)
⎧ 0 if n is even
⎪ u( z) 1 1⎛ 1 1 ⎞
=⎨ 2 ⇒ = = ⎜ − ⎟
⎪ − n 2p 2 if n is odd. z ( z + 3)( z + 8) 5 ⎝ z + 3 z + 8 ⎠

1 z 1 z
npx ⇒ u( z) . − .
(b) Fs { f ( x)} = Fs (n) = ∫ f ( x) sin
1
dx 5 z − ( −3) 5 z − ( −8)
0 1
Now
Take l = π and f(x) = cos kx
⎧ z ⎫ 1 −1 ⎧ z ⎫
p Z −1 ⎨ 2 ⎬= Z ⎨ ⎬
Fs {cos kx} = Fs (n) = ∫ cos kx sin nx dy ⎩ z + 11z + 24 ⎭ 5 ⎩ ( z − ( −3) ⎭
0
1 ⎧ z ⎫
1 p
= ∫ [sin(n + k ) x + sin(n − k ) x ] dx − Z −1 ⎨ ⎬
2 0 5 ⎩ ( z − ( −8) ⎭
p
1 ⎡ cos(n + k ) x cos(n − k ) x ⎤ 1 1
= ⎢− − = ( −3) n − ( −8) n .
2⎣ n+k n − k ⎥⎦ 0 5 5
⎧ z ⎫
1 ⎡1 − ( −1) n + k 1 − ( −1) n − k ⎤ ! Z −1 ⎨ n
⎬= a .
= ⎢ + ⎥ ⎩ ( z − a ⎭
2⎣ n+k n+k ⎦
Solved Question Papers ! A-65

Set No. 4
⎡1 3 −l ⎤
C12 ⎢
1. (a) Denoting the given matrix by A and apply- ⎢ 0 10 -2l - 3⎥⎥
ing elementary transformations
! ⎢⎣0 -2l - 12 5l ⎥⎦
⎡ I r 0⎤ = 50l + (2l + 3)(2l − 12)
we reduce A to the normal form ⎢ ⎥.
⎣ 0 0⎦ = 50l + 4l 2 + 24l + 6l - 36
⎡6 1 3 8⎤ ⎡1 6 3 8 ⎤ = 4l 2 + 32l - 36
⎢4 2 6 −1⎥⎥ C12 ⎢⎢ 2 4 6 −1⎥⎥
A= ⎢ = 4(l − 1)(l + 9)
⎢10

3 9 7⎥
⎥ ⎢
!
⎢ 3 10 9 7 ⎥
⎥ ⎡1 3 −l ⎤
⎢⎣16 4 12 15 ⎥⎦ ⎢⎣ 4 16 12 15 ⎥⎦
" R − (2l + 2) R ⎢0 10
10 −2 l − 3 ⎥
3 2 ⎢ ⎥
⎢⎣0 0 4(l − 1)(l + 9) ⎥⎦
C2 − 6C1 ⎡⎢
1 0 0 0 ⎤
2 −8 0 −17 ⎥⎥ For the H system to have non-trivial solu-
C3 − 3C1 ⎢
⎢ 3 −8 0 −17 ⎥ tion, the coefficient matrix A must be singular
C
"− 8C ⎢
1 4 −8 ⎥ ⇒ l = 1 or −9
4
⎢⎣ 0 −17 ⎥⎦
For l = 1 an equivalent system is
1
− C2 ⎡ 1 0 0 0⎤
C4 − C2
⎡1 0 0 0⎤
x1 + 3 x2 − x3 = 0 ⎫ x1 x2 x3
8 ⎢2 1 ⎥⎥ ⎢2 1 0 0⎥⎥ ⎬⇒ = =
1 ⎢
− C4 ⎢ 3
1
1
0
0 1⎥
! ⎢
⎢3 1 0 0⎥
10 x2 − 5 x3 = 0⎭ −1 1 2
17 ⎢ ⎥ ⎢ ⎥
! ⎢⎣ 4 1 0 1 ⎥⎦ ⎢⎣ 4 1 0 0⎥⎦ Solution vector X 1 = [ −1 1 2]
T

⎡1 0 0 0⎤ For λ = 9 an equivalent system is


R2 − 2 R1 ⎢0 1 0 0⎥⎥ x1 + 3 x2 + 9 x3 = 0 ⎫ x1 x2 x3
R3 − R1 − R2 ⎢ ⎬⇒ = =
⎢0 0 0 0⎥ 10 x2 + 15 x3 = 0 ⎭ 9 3 −2
R
"
2 − 2 R1 − R2 ⎢ ⎥
⎢⎣0 0 0 0⎥⎦
Solution vector X1 = [9 3 − 2]
T

⎡I 0⎤
=⎢ 2 , normal form
⎣0 0⎥⎦ 2. (a), (b) Refer question 2 (a), (b), set 2.
∴ r ( A) = 2 3 + 2i −4⎤
3. (a) ⎡ 2
(b) In matrix notation the homogeneous system A = ⎢⎢3 − 2i 5 6i ⎥⎥
of equations (H) can be written as AX = 0 ⎢⎣ −4 −6i 3 ⎥⎦
where the coefficient matrix −T
By definition A is Hermitian ⇔Aq = A = A
⎧3 1 −l ⎫ −T
A is skew i Hermitian ⇔Aq = A = − A
⎪ ⎪
A=⎨ 4 −2 −3 ⎬ Now
⎪ 2l 4 l ⎪⎭
⎩ ⎡ 2 3 − 2i −4 ⎤ ⎡ 2 3 + 2i −4⎤
R2 + 2 R1 ⎡ 3 ⎢
A = ⎢3 + 2i ⎥ T ⎢
−6i ⎥ ; A = ⎢3 − 2i 6i ⎥⎥
1 −l ⎤ 5 5
R3 − 4 R1 ⎢ 10 0 -2l - 3⎥⎥ ⎢⎣ −4 6i 3 ⎥⎦ ⎢⎣ −4 − 6i 3 ⎥⎦

! ⎢⎣ 2l - 12 0 5l ⎥⎦ = A ⇒ A is Hermitian
A-66 ! Engineering Mathematics-II

Let ⎡ 2i −2 + 3i −4i ⎤ 6. (a) Refer Ex. 6.53 (Ch. 6), p. 23 of Engg.



B = iA = ⎢ 2 + 3i 5i −6 ⎥⎥ ;
Maths-II.
(b) Refer Ex. 6.13.4 (Ch. 6) of Engg. Maths-II
⎣⎢ −4i 6 3i ⎥⎦
(one-dimensional heat equation).
⎡ −2i −2 − 3i 4i ⎤
B = ⎢⎢ 2 − 3i −5i −6 ⎥⎥ 7. (a) Fourier cosine transform of f(x) is
⎢⎣ 4i 6 −3i ⎥⎦ ∞
Fc { f ( x)} = ∫ f ( x) cos sx dx (1)
0
⎡ −2i 2 − 3i 4i ⎤
B = B = ⎢⎢ − 2 − 3i −5i 6 ⎥⎥
q T
Taking f(x) = e − ax cos ax we have
⎢⎣ 4i −6 −3i ⎥⎦

⎡ 2i −2 + 3i −4i ⎤ Fc {e − ax cos ax} = ∫ e − ax cos ax cos sx dx
0

= − ⎢ 2 + 3i 5ii −6 ⎥⎥
1 ∞ − ax 1 ∞
⎢⎣ −4i 6 3i ⎥⎦ = ∫ e cos(a + s ) x dx + ∫ e − ax cos(a − s) x dx
2 0 2 0
= − B ⇒ B is skew Hermitian.

1⎡ e − ax ⎤
(b) Refer Ex. 4.13, Ch. 4 of Engg. Maths-II, = ⎢ 2 {− a cos(a + s) x + (a + s) sin(a + s) x}⎥
2 ⎣ a + (a + s)2 ⎦0
p. 5.
= 0 at ∞ = 0 at ∞, 0

4. (a) Refer Ex. 5.7 (Ch. 5) p. 13 of Engg. Maths-II. 1⎡ e − ax ⎤
Take a = b = 1 + ⎢ 2 {− a cos(a − s) x + (a − s) sin(a − s) x}⎥
2 ⎣ a + (a − s) 2
⎦0
p2 ∞
( −1) n ∞
( −1) n = 0 at ∞ = 0 at ∞, 0
x + x2 = + 4∑ 2 cos nx − 2∑ 2 sin nx
3 n =1 n n =1 n 1 a 1 a
= . 2 + . 2 .
2 a + (a + s) 2
2 a + (a − s) 2
At x = −p = p, the Fourier series is convergent
to
(b) Refer Art. 7.7 (Convolution Theorem) on
f (p ) + f (−p ) p2 ∞
1 p. 5 of Ch. 7, Engg. Maths-II.
= p2 = + 4∑ 2
2 3 n =1 n

1 1⎛ 2 p2 ⎞ p2 8. (a) Refer Art 8.5.2 (Damping Rule) on p. 6 of


⇒∑ = ⎜p − ⎟=
n2 4 ⎝ 3 ⎠ 6 Engg. Maths-II.

(b) Refer Ex. 5.22, Ch. 5 of Engg. Maths-II, (b) Z transform is applicable for functions of
p. 28. ‘n’ and not of ‘t’ (continuous variable). The
problem is understandably a misprint for
Z 〈cosh an sin bn〉.
5. (a) Refer Ex. 6.1 (Ch. 6), p. 3 of Engg. Maths-II.
(Take r2 in place of 1 on the RHS)
z sin q
Since Z sin nq = and
(b) Refer Ex. 6.36 (Ch. 6), p. 16 of Engg. z 2 − 2 z cos q + 1 (1), (2)
Maths-II.
z cos q
(a), (b) or (c) Refer Question 5 (a), (b), (c) of Z cos nq =
z 2 − 2 z cos q + 1
set 1.
Solved Question Papers ! A-67

Replacing q by b in (1) we get since


(z2e2a − 2zea cos b +1) (z2e−2a − 2ze−a cos b +1)
z sin b
Z sin bn = (3)
z − 2 z cos b + 1
2 = z4 − 4z3 cosha cosb + 2z2 (cosh 2a + 2 cos2b)
− 4z cosha cosb + 1
By damping rule, we have
(c) Let
z sin b z z
Z e − an sin bn = u ( z) = 2 =
z − 2 z cos b + 1 z → zea
2
z + 7 z + 10 ( z + 2) ( z + 5)
(4)
z e a sin b u ( z) 1 1⎛ 1 1 ⎞
= ⇒ = = ⎜ − ⎟
z e
2 2a a
− 2 z e cos b + 1 z ( z + 2) ( z + 5) 3 z + 2 z + 5 ⎠

1 z 1 z
⇒ u( z) = ⋅ − ⋅
z sin b 3 z+2 3 z+5
Z e an sin bn =
z − 2 z cos b + 1 z → z
2
Taking inverse Z-transform on both sides
ea (5)
−a
ze sin b 1 ⎛ z ⎞ 1 −1 ⎛ z ⎞
= Z −1 (u ( z ) = Z −1 ⎜ − Z ⎜
z 2 e −2 a − 2 z e − a cos b + 1 3 ⎝ z + 2 ⎟⎠ 3 ⎝ z + 5 ⎟⎠

(4) + (5) 1 −1 ⎛ z ⎞ 1 −1 ⎛ z ⎞
= Z cosh an sin bn = Z ⎜ − Z ⎜
2 3 ⎝ z − ( −2) ⎟⎠ 3 ⎝ z − ( −5) ⎟⎠
a 2 −2 a −a
z sin b [e ( z e − 2 ze cos b + 1) 1 1 z
= ⇒ un = ( −2) n = ( −5) n ⋅ ! Z a n =
2 + e − a ( z 2 e 2 a − 2 ze a cos b + 1)] 3 3 z−a

( z 2 e 2 a − 2 ze a cos b + 1) ( z 2 e −2 a − 2 ze −2 a cos b + 1)

Z cosh an sin bn
z sin b ( z 2 + 1cosh a − 2 z cos b)
=
z 4 − 4 z 3 cosh a cos b + 2 z 2 (cosh 2a + 2 cos 2 b)
− 4 z cosh a cos b + 1
Bibliography
Churchill, RV and Brown, JW, Fourier Series and Pipes, LA and Harvill, LR, Applied Mathematics for
Boundary Value Problems, McGraw-Hill, 1987. Engineers and Physicists, McGraw-Hill, 1970.
Jain, RK and Iyengar, SRK, Advanced Engineering Sneddon, Ian, Elements of Partial Differential Equations,
Mathematics, Narosa Publishing House Pvt. Ltd, McGraw-Hill, 1985.
2006.
Kreyszig, E, Advanced Engineering Mathematics,
8th Ed., John Wiley, 2000.
Index
A complete integral 6-8
addition of matrices 1-3 general integral 6-8
negative of a matrix 1-4 particular integral 6-9
subtraction of B from A 1-4 singular integral 6-9
adjoint of a square matrix 1-9 classification of first-order partial differential
equations 6-7
adjoint method 1-21, 1-36
linear equation 6-7
Alfred Haar 9-1
nonlinear equation 6-8
algebra of matrices 1-3
quasi-linear equation 6-8
application of Z-transforms: solution of a difference
equation; by Z-transform 8-17 semi-linear equation 6-8
complementary function and particular closed subspace S 9-8
integral 8-17 closure of S 9-8
difference equation 8-17 complex matrices 3-7
general solution (complete solution) 8-17 complex-valued function 9-1
linear difference equation 8-17 conjugate of a matrix 3-7
order of a difference equation 8-17 properties 3-7
particular solution (particular integral) 8-17 contour integral method or method
of residues 8-14
B convolution theorem 7-5, 8-15
Basis of a vector space 9-2 convolution 7-5

C D
canonical form (or) sum of the squares form 4-3 determinant of a square matrix 1-5
Cayley–Hamilton theorem 2-9 cofactor of an element 1-6
change of interval: Fourier series in interval determinant-related matrices 1-11
(a, a + 2l) 5-19 determination of Fourier coefficients 5-3
characteristic equation of matrix A 2-2 diagonalisation 2-14, 2-15
characteristic function of an interval I 9-2 conditions for diagonalisability
characteristic value problem 2-1 of a matrix A 2-15
classifiable functions—even and odd functions 5-2 powers of a square matrix A 2-14
even function 5-2 simultaneous application of row
nonclassifiable functions 5-2 and column transformations 4-6
odd function 5-2 diffusion equation 6-26
classification of solutions of first-order partial differential Dirichlet’s conditions 5-4
equation 6-8 functions having points of discontinuity 5-5
I-2  !   Engineering Mathematics-II

functions which have no Fourier series Fourier sine transform (FST) and Fourier cosine
expansions 5-5 transform (FCT) 7-3
disadvantages with the Fourier series 9-1 Fourier transform of f (x) 7-3
existence of Fourier transform 7-3

E Fourier transform 7-3


inverse Fourier transform 7-3
elementary matrices 1-17, 1-22
fundamental frequency 9-1
elementary row and column operations
(transformations) 1-17
enumeration method 1-26 G
equality of matrices 1-3 Gauss–Jordan elimination method 1-38
equations solvable by direct integration 6-9 Gauss–Jordan method 1-22
equivalence of matrices 1-18 Gauss’s elimination method 1-36
Euler’s method of separation of variables 6-22
expansion of a determinant of third order 1-6 H
expansion of the determinant of a matrix Haar scaling function f(t) 9-4
of any order n 1-7 Haar wavelet functions 9-5
Haar wavelets 9-1
F half-range Fourier sine/cosine series: odd and even
FT, FST and FCT alternative definitions 7-4 periodic continuations 5-26
Hermitian matrix 3-8
field 9-3
homogeneous system of equations (H) 1-39
finite Fourier sine transform and finite Fourier cosine
transform (FFCT) 7-4
formation of partial differential equation by elimination I
of two arbitrary constants 6-3 idempotent matrix 1-12
formation of partial differential equations by elimination index and signature of a real
of arbitrary functions 6-5 quadratic form 4-3
Fourier integral representation of a function 7-3 index s 4-3
Fourier integral theorem 7-1 signature 4-3
Fourier sine and cosine integrals 7-2 integral transforms 7-1
Fourier integral in complex form 7-2 Fourier transform 7-1
Fourier series 5-3 Laplace transform 7-1
Fourier series expansion 9-2 inverse Z-transform 8-1, 8-11
Fourier series expansions of even and odd inverse of a matrix by
functions in (−l, l) 5-24 Cayley–Hamilton theorem 2-9
Fourier series expansions: even/odd synthetic method 2-9, 2-11
functions 5-5 inversion of a nonsingular matrix 1-21
Fourier series of odd and even functions invertible matrix 1-10
in the interval (−l, l ) 5-6
involutory matrix 1-13
Fourier series of odd and even functions
in the interval (−p, p) 5-7
Fourier series in any arbitrary K
interval (a, b) 5-19 Kronecker delta 1-3
Index  !   I-3

L use of the table of transforms 8-11


Lagrange’s method of reduction 4-8 method of matrix inversion (or adjoint method) 1-36
Laplace’s equation 6-42 methods of reduction of a quadratic form
to a canonical form 4-6
L2(I) 9-2
modal matrix and spectral matrix of a square
linear systems of equations 1-16 matrix A 2-14
homogeneous system 1-16 mother wavelet 9-7
nonhomogeneous system 1-16 multiresolution analysis 9-8
linear transformation 2-1 multiresolution analysis with Haar wavelets 9-8

M multiply-defined or piecewise-defined functions 5-7

matrix 1-1
column matrix or column vector 1-2 N
complex matrix 1-2 nature of real quadratic forms 4-3
definition 1-1 indefinite 4-4
diagonal matrix 1-3 negative definite 4-3
lower triangular matrix 1-2 negative semi-definite 4-4
principal or main diagonal 1-2 positive definite 4-3
real matrix 1-2 positive semi-definite 4-4
rectangular matrix 1-2 nilpotent matrix 1-13
row matrix or row vector 1-2 nonlinear equations of first order 6-18
scalar matrix 1-3 standard form I: pq-equation 6-18
square matrix 1-2 standard form II: zpq-equation 6-20
triangular matrix 1-2 standard form III: separable equation
f (x, p) = g (y, q) 6-20
unit or identity matrix 1-3
standard form IV: Clairaut’s equation
upper triangular matrix 1-2
z = px + qy + f (p, q) 6-21
zero or null matrix 1-2
nonsingular matrix 1-11
matrix multiplication 1-4
norm of a vector 3-1, 9-3
power of square matrix A 1-5
n-vector space 9-3
matrix polynomial 2-1
maximum number of linearly
independent rows 1-26 O
method for solving a linear difference equation with one-dimensional heat equation 6-26
constant coefficients 8-18 solution of 6-26
complementary function 8-18 one-dimensional wave equation 6-34
particular integral 8-18 order, linearity and homogeneity of a partial differential
method of determinants (Cramer’s rule) 1-35 equation 6-1
methods for evaluation of inverse Z-transforms 8-11 homogeneity 6-1
convolution method 8-15 linearity 6-1
inverse integral method 8-14 order 6-1
long division method 8-13 origin of partial differential equation 6-2
method of partial fractions 8-12 orthogonal matrix 2-2, 3-3
power series method 8-15 properties 3-3
I-4  !   Engineering Mathematics-II

orthogonality/orthonormality 9-3 initial value theorem 8-8


orthogonal/orthonormal vectors 9-3 linearity 8-6
orthogonal/orthonormal system of vectors 3-1 multiplication by n 8-8
orthogonalisation of a symmetric matrix 2-16 shifting property 8-7
orthogonalisation 4-7
Q
P quadratic form 4-1
Parseval’s Formula 5-34 definition 4-1
Parseval’s identity for Fourier transforms 7-6 quasi-linear equations of first order 6-11
partial differential equations 6-1
periodic function 5-1, 9-1 R
properties 5-1 rank of a matrix 1-19, 1-25
periodic matrix 1-14 real matrices 3-1
powers of a square matrix A 2-18 recurrence formula for the sequence of a power
of natural numbers 8-5
finding of modal matrix P 2-18
reduction of a quadratic form to canonical form 4-5
inverse matrix A−1 2-18
reduction of matrix A to Echelon form 1-31
predictor–corrector methods 9-2, 9-17
Echelon form 1-31
procedure for finding eigenvalues and
eigenvectors 2-2 nonzero row 1-31
properties of determinant of matrix A 1-7 zero row 1-31
properties of eigenvalues and reduction to normal or canonical form 1-27
eigenvectors 2-7 relation between Laplace and Fourier transforms 7-5
characteristic polynomial Pn(l) 2-7 root mean square (RMS) value of a function 5-34
orthogonal matrix 2-8
orthonormal set of vectors 2-8 S
orthogonal vectors 2-8 scalar multiplication 1-4
spectral mapping theorem 2-8 associative law 1-4
spectral shift 2-8 distributive law 1-4
properties of Fourier transform 7-6 scaling and translation functions 9-3
change of scale property m
scaling factors of the form 2 9-7
or damping rule 7-6 scaling of f(t) 9-5
linearity property 7-6 similarity of matrices 2-14
modulation theorem 7-6 simply-defined and multiply-(Piecewise) defined
shifting property 7-6 functions 5-7
properties of Hermitian, skew-Hermitian and singular matrix 1-11
unitary matrices 3-8 skew-Hermitian matrix 3-8
properties of trace of A 1-3 skew-symmetric matrix 3-1
properties of Z-transforms 8-6 properties of symmetric and skew-symmetric
change of scale or damping rule 8-6 matrices 3-2
convolution theorem 8-9 Shannon wavelets 9-10
division by n 8-8 solution for NH system of
final value theorem 8-9 n equations in n unknowns 1-33
Index  !   I-5

solution of linear, semi-linear and quasi-linear


equations 6-11
U
unit impulse sequence 8-2
all the variables are separable 6-12
unit step sequence 8-2
method of multipliers 6-12
unitary matrix 3-8
two variables are separable 6-12
solutions of a system
of linear equations 1-33 V
solution over the complex field—eigenvectors of a real vectors: linear dependence and independence 1-18
matrix over complex field 2-23 vector space of functions with finite energy 9-2
special matrices 1-12 vibrating string with zero initial velocity 6-34
spectrum of A 2-2
square integrable 9-1 W
steady-state heat flow equation 6-42 wavelets 9-1
Neumann’s problem 6-42 wavelet expansion 9-7
the Dirichlet problem 6-42
two-dimensional 6-42
submatrix of a matrix 1-11
Z
Z-transform 8-1
subspace and span 9-3
Z-transforms of unit step and unit impulse
subspaces of L2(R) 9-8
sequences 8-2
Sylvestor’s law of inertia 4-6
Z-transforms of some standard functions
symmetric matrix 3-1 (special sequences) 8-4
system of NH equations: consistency and alternating unit constant sequence 8-4
inconsistency 1-33
binomial coefficient sequence 8-5
geometric sequence 8-4
T natural number sequence 8-4
translation of f(t) 9-5
power-cum-reciprocal factorial sequence 8-5
transpose of a matrix: properties 1-8
power-cum-reciprocal factorial sequence with a
transposed conjugate (tranjugate) of a matrix 3-8 multiple 8-5
properties 3-8 reciprocal factorial sequence 8-4
tridiagonal system 1-44 unit constant sequence 8-4

You might also like